CP3 medicine/surgery

Réussis tes devoirs et examens dès maintenant avec Quizwiz!

Describe the acute management of the subarachnoid haemorrhage, including referral to the appropriate specialty, and management of electrolytes, glucose and blood pressure.

1) 1st line - cardiopulmonary support: a) pts admitted to ICU b) GCS to assess consciousness c) assess need for endotracheal intubation and mechanical ventilation d) BP, HR, respiratory function, and glucose should be closely monitored 2) + surgical clipping or coil embolisation - surgical clipping: more suitable for aneurysms associated with large parenchymal haematomas; endovascular coil embolisation: more suitable for older people or patients in poor medical condition 3) + calcium-channel blockers eg/ nimodipine: 60 mg for vasospasm prophylaxis 4) + anticonvulsants eg/ phenytoin or levetiracetam 5) + stool softeners - prevent straining & dec risk of rebleeding eg/ docusate and senna 6) cough - +/- antitussives eg/ codeine phosphate 7) headache - +/- analgesia eg/ oxycodone: 5-10 mg, or morphine sulfate: 2-4 mg IV, or fentanyl 8) altered coagulation - +/- coagulopathy correction eg/ FFP, or prothrombin complex concentrate; + phytomenadione 9) hyponatraemia - +/- sodium replacement eg/ 1.25% saline, 1.5% or 2% or 3%; +/- fludrocortisone; also need to manage glucose levels

Outline a plan of treatment in a patient with small bowel obstruction including a consideration of fluid and electrolyte therapy, antibiotic therapy, intestinal intubation and operative therapy

1) 1st line - emergency laparotomy + fluid resuscitation (IV Ringer's lactate or normal saline) + catheter 2) + preoperative abx prophylaxis: ampicillin + gentamicin or cefoxitin 3) + nasogastric decompression 4) + analgesia: morphine sulfate 5) + correction of the underlying cause eg/ appendectomy for appendicitis, Ladd procedure for malrotation (infants), tumour resection for obstructing tumour, and hernia repair for inguinal hernia should be performed when diagnosed 6) If surgery contraindicated: nasogastric decompression + fluid resuscitation + analgesics +/- anti-emetics +/- antispasmodics

Outline the current results (survival and recurrence rates) of treated breast cancer according to clinical stage

1) 5-yr survival by stage - a) stage 0 & 1: 100% b) stage 2: 93% c) stage 3: 72% d) stage 4: 22% e) overall 5-yr survival for breast cancer = 90% 2) recurrence rates within 5-yrs - a) local recurrence = 1.7% b) chest wall = 0.8% c) regional LN (within 10yrs) = 1%

Discuss the clinical and social complications of obesity and understand how BMI is calculated

1) BMI = weight (kg)/((height in m)^2) 2) clinical complications : a) mechanical: osteoarthritis and lower back pain, stress, incontinence b) metabolic syndrome: hypertension, MI, stroke, DVT, PE, T2DM c) malignancy: eg/ breast, uterus, CRC - due to steroid conversion in peripheral adipose tissue d) humoral: menstrual abnormality/erectile dysfunction/infertility e) respiratory restriction: sleep apnoea, asthma f) abdominal: non-alcoholic fatty liver disease, GORD, hiatus hernia, gall stones g) inability to maintain hygiene: skin infections, foot ulcers with diabetes h) psychological consequences: stress, low self-esteem, inability to exercise/fit-in etc 3) obesity and low socio-economic status are linked

List investigations for gastric cancers

1) FBC, U&E, LFT, CRP, coag 2) upper GI endoscopy with biopsy (if biopsy is -ve then repeat) 3) endoscopic ultrasound 4) CT CAP 5) CXR 6) consider - a) PET scan b) laparoscopy (if -ve radiography)

Differences between an upper and lower motor neurone 7th nerve lesion

1) LMN - causes weakness of all the muscles of facial expression: angle of the mouth falls, weakness of frontalis occurs, and eye closure is weak 2) UMN - frontalis is spared, normal furrowing of the brow is preserved, and eye closure and blinking are not affected

Compare and contrast a large bowel obstruction and a small bowel obstruction

1) LBO - a) s/s - I) colicky abdominal pain - inc constant pain and pain on movement, coughing, or deep breathing may imply perforation or impending perforation II) abdominal distention III) tympanic abdomen IV) change in bowel habits - either failure to pass faeces (complete obstruction) or successful passing of some flatus or faeces (partial obstruction) V) hard faeces - may indicate faecal impaction on DRE, soft stool may indicate partial obstruction on DRE VI) empty rectum - implies proximal obstruction on DRE VII) others - recent weight loss, rectal bleeding, abnormal bowel sounds (may be normal initially, then inc in frequency with absent sounds in advanced stages of obstruction), palpable rectal or abdominal mass (if rectal carcinoma or malignant disease or diverticular mass), positive faecal occult blood test b) ix - I) FBC - inc WBC may indicate an infective or inflammatory cause or complication such as perforation or impending perforation; anaemia may be found in malignancy II) serum electrolytes - May be deranged from dehydration, fluid shifts, or sepsis III) renal function - inc urea or creatinine IV) serum amylase/lipase - INC V) coagulation studies coagulopathy may be present in sepsis from perforation: prolonged INR, PTT, and PT VI) erect CXR - unremarkable or free sub-diaphragmatic air VII) plain abdominal x-ray - shows dilation (obstruction) & level of obstruction VIII) consider - contrast enema (shows Level, degree, and type of obstruction), CT abdomen and pelvis (gaseous distension of large bowel), flexible/rigid endoscopy, biopsy (confirms malignancy) 2) SBO - a) s/s - 1) risk factors eg/ previous abdominal surgery, hernias, Crohn's, foreign body ingestion, appendicitis, intussusception, malrotation, volvulus, and intestinal atresia in newborns/infants 2) failure to pass flatus or stool 3) abdominal pain - crampy and intermittent, and can be severe 4) vomiting - may be faceulant 5) abdominal distention & tenderness 6) absolute constipation 7) peritonitis 8) others - nausea, fever, tachycardia, severe lethargy, hypotension b) ix - I) abdominal x-rays - Partial SBO: gas throughout the abdomen and into the rectum; complete SBO: no distal gas, and staggered air-fluid levels; complicated SBO: free air under the diaphragm suggestive of perforation; thumb-printing of the bowel suggestive of ischaemia II) FBC - inc WCC May indicate severe intestinal obstruction with necrosis; anaemia may indicate blood loss into obstructed bowel III) urea - inc shows severity of dehydration/renal failure in complicated SBO IV) electrolyte panel - Hyponatraemia, hypokalaemia, metabolic alkalosis in dehydration V) abdominal CT scan - shows cause, Extent and location of obstruction when x-rays are inconclusive VI) consider - upper gastrointestinal (GI) x-ray with small bowel follow-through (SBFT), laparotomy or laparoscopy, abdominal ultrasound, abdominal MRI

List the investigations used in ascertaining a diagnosis in MS. Outline the diagnostic utility of magnetic resonance imaging (MRI), evoked potentials and CSF examination

1) MRI brain - with contrast, see hyperintensities in the periventricular white matter, most sensitive images are sagittal FLAIR 2) MRI spinal cord - see demyelinating lesions in the spinal cord, particularly cervical spinal cord; detection of alternate diagnosis, such as cervical spondylosis 3) FBC, TSH, comprehensive metabolic panel, vit B12 - should all be normal 4) cerebrospinal fluid (CSF) evaluation - invasive so all other non-invasive tests should be pursued first, shows - glucose, protein, and cell count normal; oligoclonal bands and elevated CSF immunoglobulin G (IgG) and IgG synthesis rates in 80% MS cases 5) evoked potentials - visual evoked potentials are most commonly abnormal, with somatosensory and auditory evoked potentials less so, useful to assist in establishing diagnosis but not helpful for monitoring response to treatment, shows - prolongation of conduction, particularly asymmetrical prolongation in visual evoked potentials 6) others - anti-NMO antibody

Describe the difference in prognosis between STEMI, non-STEMI, and UA with respect to mortality and morbidity

1) NSTEMI - short term mortality = 3-5%, 2yr mortality ~30%, GRACE registry = 6 month mortality of 13% 2) STEMI - short term mortality = 10-15%, 2yr mortality ~30%. 1 month mortality in community = 50%, with 50% occurring in 2hrs. Early death may be due to arrhythmia. Of those who reach hospital, 80% survive up to 28 days. Prognosis worse for anterior vs inferior infarcts. Morbidity related to level of ischaemia and myocardial damage sustained 3) unstable angina - 1yr mortality = 10%, GRACE registry = 6 month mortality of 8%

Describe the risk factors, clinical features and investigations for DVT

1) Risk factors - a) hospitalisation within the past 2 months, major surgery within 3 months b) active cancer c) lower-extremity trauma d) inc age e) pregnancy f) obesity g) factor V Leiden, prothrombin gene mutation G20210A, protein C or S deficiency, antithrombin deficiency, antiphospholipid antibody syndrome, h) medical comorbidity (eg/ vascular inflammation or stasis, heart failure, respiratory disease, and acute infections) i) recent long-distance air travel j) use of specific drugs (COC, tamoxifen and raloxifene, thalidomide, erythropoietin) k) fx 2) s/s - calf swelling, localised pain along deep venous system, tenderness with dorsiflexion of the foot (Homans' sign) or calf pain on palpation (Pratt's sign) during the physical examination may be present, can also have - asymmetric oedema, collateral superficial veins (dilated superficial veins over foot and leg, not varicose veins, are a sign of DVT) 3) ix - a) Wells' clinical probability tool - if Wells' score 2+, condition is likely (absolute risk is approximately 40%). Criteria include - active cancer, calf circumference >3 cm more than other calf (measured 10 cm below tibial tuberosity), prominent superficial veins (non-varicose), pitting oedema (only symptomatic leg), whole leg swollen, localised pain along distribution of deep venous system, paralysis, paresis, or recent cast immobilisation of lower extremities, recent bed rest for >3 days or major surgery requiring regional or general anaesthetic within past 12 weeks, hx of DVT or PE NB/ if alternative diagnosis at least as probable: subtract 2 points b) quantitative D-dimer level - if D-dimer is normal, DVT is excluded in low-probability patients, if inc a venous duplex ultrasound is indicated. Not a definitive test - inc levels highly sensitive but non-specific. Other causes of inc D-dimer - inc age, illness, hepatic disease, infection, pregnant c) venous duplex ultrasound (DUS) - 1st line in all high-probability patients (Wells' score 2+) or in low-probability patients (Wells' score <2) with inc D-dimer level, assesses popliteal, deep femoral, femoral, and common femoral veins, if positive shows - inability to fully compress lumen of vein using ultrasound transducer; dec or absent spontaneous flow; lack of respiratory variation; intraluminal echoes; colour flow patency abnormalities d) INR and activated partial thromboplastin time (aPTT), and U&E (all baselines) e) LFTs - may detect abnormalities associated with underling provoking factor (eg/ cancer) f) FBC - may detect abnormalities eg/ haematological malignancy (anaemia, leucopenia), high platelet count (essential thrombocytosis or a myeloproliferative disorder) g) Others - doppler venous flow testing, CT AP (with contrast), thrombophilia screen

Discuss current theories of the aetiology of anal fissure and describe the principles of management

1) aetiology - a) hard or painful bowel movement, which damages the surrounding skin b) straining on toilet c) diarrhoea d) IBD e) infection in skin around anus eg/ STI f) taking certain meds eg/ painkillers that contain opiate g) trauma eg/ through having anal sex h) anal cancer i) giving birth 2) mx - may heal in 1-2/52, otherwise - a) changes to diet & fluids to avoid constipation b) laxative eg/ lactulose syrup, or other laxatives (including other types) b) paracetamol or ibuprofen for the pain c) meds to relieve anal spasm I) local anaesthetic eg/ lidocaine - pain relief II) GTN ointment - helps internal sphincter muscle to relax & inc blood flow to anus to help the fissure heal III) calcium-channel blockers eg/ diltiazem cream - relax part of sphincter to dec spasm & pressure in anus d) injections to relieve anal spasm - botulinum A toxin e) surgery - to relax sphincter muscle or remove damaged skin

Outline the clinical features of encephalitis and list the common causes

1) aetiology - a) viruses are the main cause of encephalitis, with herpes virus the most common group of viruses eg/ I) HSV-1, HSV-2, VZV, CMV, EBV, human herpes virus (HHV)-6, herpes B virus II) enterovirus-71, coxsackievirus, poliovirus III) flaviviruses: West Nile, Japanese encephalitis virus, tick-borne encephalitis virus, Murray Valley encephalitis virus, Saint Louis encephalitis virus, Powassan virus, dengue virus IV) others: MM (not R), HIV, rabies, adenovirus, influenza, parainfluenza, hep C, rotavirus, parvovirus B19, BK virus, JC virus, cycloviruses b) bacterial infections - I) Neisseria meningitides II) lungs - TB, Listeria, Mycoplasma & Strep pneumonia III) Syphilis IV) Bartonella (cat-scratch disease), Borrelia burgdorferi (Lyme disease), Brucellosis V) Typhoid fever VI) Leptospirosis VII) Klebsiella VIII) Staphylococcus aureus IX) Streptococcus viridans X) Group C beta-haemolytic streptococci XI) Treponema pallidum c) fungal infections - Cryptococcus, Coccidioides, Histoplasma, Blastomycosis, Candida d) parasitic infections - Toxoplasma gondii, Amoebic, Entamoeba histolytica, Plasmodium falciparum, African trypanosomiasis, Schistosomiasis. 2) s/s - a) risk factors - <1 or >65, immunodeficiency, viral infections, body fluid exposure, organ transplantation, animal or insect bites, location, and season b) fever - frequently seen in infectious causes of encephalitis c) rash I) vesicular eruption - enterovirus, HSV, VZV II) maculopapular eruption - EBV (after ampicillin), measles, HHV-6, Colorado tick fever, West Nile virus III) malar rash - SLE IV) erythema migrans - Lyme disease V) erythema nodosum - tuberculosis and histoplasmosis, sarcoidosis VI) erythema multiforme - HSV, Mycoplasma VII) mucous membrane lesions - herpes virus, Behcet's VIII) genital lesions - HSV-2, Behcet's d) altered mental state - ranges from mild somnolence to coma, cognitive dysfunction with acute memory disturbances and psychiatric and behavioural manifestations (eg/ withdrawal, apathy, abulia, akinetic mutism, personality changes, psychotic behaviour, disorientation, and hallucinations) can be seen e) focal neurological deficit - aphasia, hemianopia, hemiparesis, ataxia, brisk tendon reflexes, Babinski's sign, cranial nerve deficits (HHV-6, TN, syphilis, brucellosis, etc); tremors (arboviruses); myoclonus (subacute sclerosing panencephalitis); paraesthesias (Colorado tick fever, rabies); generalised weakness (West Nile virus, rabies) f) meningismus - some have meningeal inflammation with headache, photophobia, neck stiffness g) cough - URTI/LTRI s/s can occur in HSV-1, influenza, parainfluenza, Mycoplasma pneumoniae , Q fever, Coccidioides , Histoplasma , blastomycosis, or rabies h) GI infection - enteroviruses, rotavirus, Whipple's disease. i) seizures - generalised tonic-clonic seizures, partial complex seizures, and focal seizures (with secondary generalisation) are common, mostly with measles causing subacute sclerosing panencephalitis, HHV-6 infection, and HSV-1 infection

Describe the aetiology and pathology of pancreatitis

1) aetiology - acute pancreatitis occurs when factors involved in maintaining cellular homeostasis are out of balance. Initiating event = anything that injures acinar cell & impairs secretion of zymogen granules eg/ alcohol, gallstones, certain drugs. Unclear exactly what event triggers onset of acute pancreatitis, but believed both extracellular factors (eg, neural and vascular response) and intracellular factors (eg, intracellular digestive enzyme activation, increased calcium signaling, and heat shock protein activation) play a role. Acute pancreatitis can develop when ductal cell injury leads to delayed or absent enzymatic secretion, as seen in pts with CFTR gene mutation: a) lysosomal and zymogen granule compartments fuse, enabling activation of trypsinogen to trypsin b) intracellular trypsin triggers the entire zymogen activation cascade c) secretory vesicles extruded across basolateral membrane into interstitium, where molecular fragments act as chemoattractants for inflammatory cells d) activated neutrophils release superoxide or proteolytic enzymes (cathepsins B, D, and G; collagenase; and elastase). Macrophages release cytokines that further mediate local inflammatory responses. Early mediators = TNF-α, IL-6+8 e) mediators of inflammation cause inc pancreatic vascular permeability = haemorrhage, oedema, and pancreatic necrosis. As mediators are excreted into circulation, systemic complications can arise eg/ bacteremia (gut flora translocation), ARDS, pleural effusions, GI haemorrhage, and renal failure f) SIRS can develop, leading to systemic shock. Mediators of inflammation can become so overwhelming that haemodynamic instability and death ensue 2) pathology - in acute pancreatitis, oedema and fat necrosis occur first - acute oedematous pancreatitis. When necrosis involves the parenchyma, accompanied by haemorrhage and dysfunction of the gland, inflammation evolves into haemorrhagic or necrotising pancreatitis. Pseudocysts and pancreatic abscesses can result NB/ damage to pancreas leads to release of proteolytic enzymes = necrosis of acini + thrombosis in vessels, leading to further damage. Early on necrosis is limited to periductal area (centre of each lobule). Damage to epithelium lining the duct allows enzyme diffusion into surrounding tissue + small areas of necrosis appear radiating outwards from ducts. Changes are associated with alcohol and gallstones Unless damage is inhibited by proteases eg/ a-1 antitrypsin, it progresses to panlobular pancreatitis. Diffuse necrosis, haemorrhage (resulting from vessel destruction) and venous thrombosis. Haemorrhage may lead to Grey-Turner's and Cullen's signs. Release of enzymes past the pancreas can lead to peripancreatic & omental fat digestion. 2 most common causes of pancreatitis are alcohol and gallstones. Gallstones - bile refluxes up pancreatic duct (irritant). Alcoholic - ampulla spasms, ducts plugged by viscous secretion + alcohol directly toxic to acinar cells, chronic pancreatitis in alcoholics following repeated bouts of acute pancreatitis Protein rich plugs deposited in smaller ducts = dilation of acini distally. Epithelium degenerates & disappears, tissue adjacent to obstruction becomes necrotic. Chronic inflammatory infiltrate around dying acinus, and eventually glandular tissue replaced by fibrosis. With progression calcification occurs, duct cysts rupture into surrounding tissue (mesenteries (obstruction), bile duct (jaundice), portal system (GI haemorrhage)). Insufficiency of pancreatic secretion leads to malabsorption and diabetes

Describe the typical clinical presentation of EAA and list common causes

1) aetiology - multiple causative agents have been identified a) bacteria eg/ thermophilic Actinomycetes b) animal proteins eg/ avian proteins, excrement, exposure to large farm animals c) certain fungi d) reactive chemicals such as acid anhydrides (epoxy resin lung disease), diisocyanates, and agents used in metal working e) drugs eg/ nitrofurantoin, methotrexate, roxithromycin, and rituximab f) mould, mouldy hay, grain, wood, wood dust, or sugar cane g) occupational exposure to chemicals eg/ epoxy resins, polyurethane foam, plastic coatings, and spray paint for vehicles h) occupational exposure to metal-working fluid 2) s/s - dyspnoea, fevers, non-productive or productive cough, and malaise, clubbing, rales (diffuse or bibasilar), weight loss

List the electrolyte values in serum

1) ammonia: 15-50 µmol/l 2) ceruloplasmin: 15-60 mg/dl 3) chloride: 95-105 mmol/l 4) copper: 70-150 µg/dl 5) creatinine: 0.8-1.3 mg/dl 6) blood urea nitrogen: 8-21 mg/dl 7) ferritin: 12-300 ng/ml (men), 12-150 ng/ml (women) 8) glucose: 65-110 mg/dl 9) inorganic phosphorous: 1-1.5 mmol/l 10) ionized calcium: 1.03-1.23 mmol/l 11) magnesium: 1.5-2 meq/l 12) phosphate: 0.8-1.5 mmol/l 13) potassium: 3.5-5 mmol/l 14) pyruvate: 300-900 µg/dl 15) sodium: 135-145 mmol/l 16) total calcium: 2-2.6 mmol/l (8.5-10.2 mg/dl) 17) total iron-binding capacity: 45-85 µmol/l 18) total serum iron: 65-180 µg/dl (men), 30-170 µg/dl (women) 19) transferrin: 200-350 mg/dl 20) urea: 1.2-3 mmol/l 21) uric acid: 0.18-0.48 mmol/l 22) zinc: 70-100 µmol/l

Describe anatomical mechanisms responsible for producing thoracic outlet compression syndromes and list the investigations for thoracic outlet syndrome. Describe the surgical principles for its correction

1) anatomy - anatomy in thoracic outlet provides potential areas of pressure leading to neurovascular-vascular compression eg/ interscalene triangle, costoclavicular space, subcoracoid space, and, more rarely, the sternal-costovertebral bony circle a) many factors can cause compression, elongation, or angulation of the neurovascular bundle at the thoracic outlet, but the basic aetiological factor is deranged anatomy eg/ dynamic, static, congenital, traumatic, and occasionally atherosclerotic factors I) dynamic - unusually wide latitude of motion in the components of the shoulder joint, which may result in compression or impingement on vessels and/or nerves eg/ when arm is in full hyperabduction above the head, the axillary artery is bent 180° from its position when the arm is at the side. This motion pulls the vessel across the coracoid process and head of the humerus, as across a pulley, causing arterial compression II) Static factors - vigorous work or exercise may create relative muscle hypertrophy, creating a relative narrowing of the spaces in the thoracic outlet through which neurovascular bundles pass (eg/ interscalene triangle, costoclavicular space, or subcoracoid space) eg/ job with repetitive overhead stresses, prolonged periods sitting at a keyboard, and repetitive overhead sport like tennis and swimming III) Congenital factors - can contribute to narrowing of all 4 spaces in the thoracic outlet eg/ anomalies of the first rib can create stretching, angulation, or compression of the neurovascular bundle as it passes over the first rib in the sternal-costovertebral bony circle, interscalene triangle narrowing with presence of a cervical rib, long cervical transverse processes, first rib abnormality, congenital fibrous bands, abnormal insertions of the scalene muscles onto the first rib, and supernumerary scalene muscles, costoclavicular space narrowing if prominent costoclavicular membrane or through lower shoulder girdle positioning IV) Traumatic factors - hyperextension injuries of the neck (eg/ whiplash or falls) are well known contributing factors in neurological TOS, creating compression of the brachial plexus in the interscalene triangle, costoclavicular space and/or subcoracoid space; clavicle fracture V) Atherosclerotic factors 2) Ix - a) cervical spine and chest x-rays - may see bony abnormalities eg/ cervical rib, articulated or bifid first rib, fusion of first and second ribs, clavicular deformities, or previous thoracoplasties b) EMG/nerve conduction velocity - if suspected true neurological TOS only to rule out alternative aetiologies (eg/ carpal tunnel syndrome), if neurological - dec motor amplitude of the median nerve and reduced sensory amplitudes of the medial antebrachial cutaneous and ulnar nerves (true neurological TOS); normal in other types of TOS unless patient has concurrent condition c) CTA - recommended if arterial TOS is suspected, evaluates for presence and location of arterial thrombus, gives anatomical detail regarding adjacent osseous structures that may be contributing to thrombus, can show - arterial thrombus, stenosis, or compression of the subclavian or axillary artery of the affected upper extremity; can also show aneurysm formation d) Doppler ultrasonography - recommended if venous TOS is suspected, may show venous thrombus, stenosis, or compression of the subclavian or axillary vein of the affected upper extremity; will show venous patency after treatment e) Others - MRI neck/clavicle/shoulder, muscle block, conventional arteriography, MRA, MRV, contrast venography, FBC and coagulation studies 3) Tx - a) neurological TOS (NTOS) - 1st line - surgery: surgical decompression of the thoracic outlet (eg/ removal of cervical rib, neurolysis of supraclavicular brachial plexus, first rib resection, removal or sectioning of the anterior and middle scalene muscles, release of the pectoralis minor muscle) is recommended I) presents with objective signs of nerve compression: specifically, motor deficits such as weakness and atrophy of the involved muscle groups b) venous TOS (vTOS) - surgical candidate 1st line - catheter-directed thrombolysis + surgery: surgical decompression of thoracic outlet determined by the area of compression (eg/ first rib resection, anterior scalenectomy, and venoplasty or open venous reconstruction) is recommended c) arterial TOS (ATOS) - 1st line - surgery: immediate surgical intervention is required if there is concern for ischaemia, transaxillary approach is often recommended. A supraclavicular or infraclavicular approach can sometimes be employed if bypass is necessary (if embolectomy cannot be achieved or an aneurysm is present)

List causes of a mass in the right iliac fossa and outline the assessment, investigation and management

1) appendicular mass - inflamed appendix with an adherent covering of omentum and small bowel. History similar to appendicitis with a longer duration since onset. Exam reveals a mass in the right iliac fossa. CT scan to investigate. Conservative management - NBM, Abx, size of the mass marked out and surgery indicated if either: mass enlarges or small bowel obstruction supervenes due to adhesions or pt becomes more toxic eg/ inc pulse, WCC, pain, temp 2) ileocaecal tuberculosis - infection with M. TB, most commonly due to abundance of lymphoid tissue. Low grade fever associated with loss of appetite and weight loss. Barium meal, ultrasound, CT. treat with anti-TB drugs 3) intussusception - part of intestine folds into the section next to it. Typically involves the small bowel and less commonly the large bowel. Abdominal pain which may come and go, vomiting, abdominal bloating, and bloody stool. History and physical exam, including observation of Dance's sign. A DRE is particularly helpful in children. Ultrasound. Treated with either a barium or water-soluble contrast enema or an air-contrast enema, which both confirms the diagnosis of intussusception, and in most cases successfully reduces it, otherwise surgical reduction 4) carcinoma caecum - unexplained pain in the RIF + general symptoms such as anaemia, malaise and weakness. Pts >40, mostly 70-80. Affects more women. FBC, blood film, DRE, sigmoidoscopy and proctoscopy, stool sample, barium enema. Surgery 5) ectopic kidney - kidney fails to ascend from true pelvis or from a superiorly ascended kidney located in the thorax. Often asymptomatic. Ultrasound & CT. Treatment based on whether there are symptoms or complications. Obstruction - surgery may be required for correction of position of the kidney to allow better drainage of urine. Extensive renal damage - nephrectomy indicated 6) psoas abscess - tuberculous abscess of lumbar vertebra that tracks from the spine inside sheath of the psoas muscle. Exam will show psaos inflammation. FBC etc. Abx and drainage

Describe the essential components of the annual review in diabetes care

1) assess long term blood glucose control - HbA1C, as well as day-to-day glucose level 2) assess cholesterol levels 3) check BP 4) check BMI 5) check eyes - fundoscopy 6) check feet for ulcers & neuropathy 7) check kidney function - urine dip 8) if inject insulin need injection or infusion sites checked 9) screening for depression or sexual dysfunction

Causes of clubbing (resp)

1) bronchiectasis, CF 2) lung abscess 3) lung cancer (most common) 4) fibrosing alveolitis/interstitial lung disease

CCF - s/s

1) cardiac symptoms - chest pain/pressure and palpitations 2) respiratory symptoms - rest/exertional dyspnoea, orthopnoea, paroxysmal nocturnal dyspnea, pulmonary oedema 3) noncardiac - anorexia, nausea, weight loss, bloating, fatigue, weakness, oliguria, nocturia, and cerebral symptoms (anxiety to memory impairment & confusion) 4) weight loss (cardiac cachexia) due to anorexia, poor tissue perfusion, dec exercise tolerance (wasting of disuse) NB/ New York Heart Association Classification of heart failure ranks patients into 4 groups: 1 = no limitation, 2 = mild limitation, normal physical activity causes fatigue, 3 = marked limitation, but still comfortable at rest, and 4 = symptoms of heart failure at rest

Describe the mechanism of action of aspirin and outline its role in cardiovascular disease prevention

1) causes irreversible inactivation of COX enzyme which is responsible for prostaglandin + thromboxane synthesis (from arachidonic acid - to cyclic endoperoxidase) - lead to dec inflammation, analgesia, prevention of clotting and dec fever 2) prostaglandins are powerful acting vasodilators + inhibit aggregation of blood platelets + involved in inflammation 3) thromboxane (TXA2) stimulates activation of new platelets + inc platelet aggregation 4) role in CV disease - helps prevent MI, thinks blood so can dec angina etc

Describe the syndromes that would arise from a lesion in; Cerebral hemisphere; Brainstem; Cerebellum; Basal ganglia

1) cerebral hemisphere - a) s/s - headaches, neck pain or stiffness, N&V & anorexia, vision changes or eye pain, changes in mood, personality, behaviour, mental ability, and concentration, memory loss or confusion, seizures, fever, difficulty moving b) abscesses - areas of infection with pus and inflamed tissue c) arteriovenous malformations (AVMs) - brain lesion that occurs during early development. Arteries and veins in the brain grow in a tangle and become connected by tube-like structures called fistulae. The arteries are not as strong as normal arteries. The veins are often enlarged because of the constant flow of blood directly from the arteries through the fistulae to the veins. These fragile vessels may rupture, leaking blood into the brain. In addition, the brain tissue may not receive enough blood to function properly. Damage to the brain may cause seizures as the first symptoms of an AVM d) cerebral infarction e) cerebral palsy - affects movement, can also make communication and related skills difficult. However, many children with cerebral palsy have normal intellectual functioning f) multiple sclerosis (MS) - immune system attacks and damages the nerve linings (myelin) in the brain and spinal cord g) tumours 2) brainstem - a) motor pathway (corticospinal tract) = contralateral weakness b) medial lemniscus = contralateral proprioception/vibration loss c) medial longitudinal fasciculus = ipsilateral internuclear ophthalmoplegia d) motor nucleus & nerve = ipsilateral CN function loss e) spinocerebellar pathway = ipsilateral ataxia f) spinothalamuc = contralateral pain & temperature & sensory loss g) sensory nucleus of CN5 = ipsilateral pain & temperature loss in face h) sympathetic pathway = ipsilateral Horner's syndrome i) CN 9 (glossopharyngeal) = ipsilateral pharyngeal sensory loss j) CN 10 (vagus) = ispilatral palatal weakness k) CN 11 (spinal accessory) = ipsilateral shoulder weakness l) CN 12 (hypoglossal) = ipsilateral eakness of tongue m) CN 5 (trigeminal) = ipsilateral facial sensory loss n) CN 6 (abducent) = ipsilateral eye abduction weakness o) CN 7 (facial) = ipsilateral facal weakness p) CN 8 (vestibular-cochlear) = ipsilateral deafness q) CN 3 (coulomotor) = eye turned down & out r) CN 4 (trochlear) = eye unable to look down towards nose 3) cerebellum - a) cerebellar ataxia - non-progressive congenital ataxia (NPCA) is a classical presentation of cerebral ataxias I) can occur as a result of many diseases and presents with symptoms of an inability to coordinate balance, gait, extremity and eye movements b) lesions to the cerebellum can cause dyssynergia, dysmetria, dysdiadochokinesia, dysarthria and ataxia of stance and gait. Deficits with movements are ipsilateral 4) basal ganglia - a) dystonia (muscle tone problems) b) Huntington disease (disorder in which nerve cells in certain parts of the brain waste away, or degenerate) c) multiple system atrophy (widespread nervous system disorder) d) Parkinson disease e) progressive supranuclear palsy (movement disorder from damage to certain nerve cells in the brain) f) Wilson disease (disorder causing too much copper in the body's tissues)

Outline the clinical features investigation and treatment of immune thrombocytopaenia

1) clinical features - a) usually no symptoms - picked up on routine FBC b) some have - nosebleeds, bleeding gums, heavier/longer periods, bruising, petechia, purpura c) general - malaise, fatigue, weakness d) bleeding not common with platelet counts >50, and severe spontaneous bleeding unusual >20 2) ix - a) FBC and peripheral blood smear - platelet count <100 x 10^9/L (<100 x 10^3/microlitre), normal cells b) consider - I) HIV serology, Helicobacter pylori breath test or stool antigen test, hepatitis C serology - all negative II) TFTs - may be hyper- or hypothyroid if other s/s III) quantitative immunoglobulins - may reveal common variable immunodeficiency (CVID) or selective IgA deficiency IV) bone marrow biopsy/aspiration - increased megakaryocytes; no evidence of malignancy; no flow cytometry or cytogenetic abnormalities 3) mx - a) all patients (child or adult): with life- or organ-threatening bleeding - I) 1st line - IVIG + corticosteroid + platelet transfusion eg/ normal immunoglobulin human + prednisolone or methylprednisolone or dexamethasone + platelet transfusion, emergency treatment may take 1-5 days to have an effect, and usually lasts for 2 to 4 weeks II) without haematuria - +/- antifibrinolytic eg/ aminocaproic acid or tranexamic acid b) newly diagnosed child who is asymptomatic or with minor bleeding symptoms - I) 1st line - observation c) newly diagnosed child with major bleeding symptoms (eg/ mucosal bleeding) - I) 1st line - corticosteroid or IVIG or anti-D Ig d) newly diagnosed adult (pregnant or non-pregnant) - I) platelet count ≥30 x 10^9/L (≥30 x10^3/microlitre): asymptomatic without additional risk factors - observation risk factors eg/ receiving antithrombotic tx, undergoing an intervention that may cause bleeding, employed in a high-injury risk profession, or participating in high-injury risk activities II) platelet count ≥30 x 10^9/L (≥30 x 10^3/microlitre): with bleeding symptoms and/or additional risk factors - corticosteroid or anti-D +/- IVIG - anti-D if Rh +ve and non-splenectomised e) children with ongoing disease - I) 1st line - mycophenolate or rituximab or eltrombopag: if continue to have ITP (i.e., platelet count ≤100 x 10^9/L [≤100 x 10^3/microlitre]) >3 months after diagnosis, if persistent or chronic disease treated with mycophenolate if intolerant of, or have failed, first-line treatments (e.g., corticosteroids). II) 2nd line - splenectomy (plus treatment to achieve target platelet level) eg/ normal immunoglobulin human or prednisolone: 1-2 mg/kg/day orally, children may be candidates for splenectomy if they have persistent or chronic disease and are unresponsive to, or intolerant of, pharmacological interventions. However, this should be delayed for at least 12 months unless immediate treatment is required (e.g., to improve quality of life), prior to splenectomy, patients should receive polyvalent pneumococcal vaccine, Hib vaccine, and quadrivalent meningococcal polysaccharide vaccine f) adult non-pregnant with persistent or chronic disease - I) 1st line - low-dose corticosteroid (eg/ prednosilone) or repeat IVIG infusions II) 2nd line - mycophenolate or thrombopoietin receptor agonist (eg/ romiplostim or eltrombopag) or rituximab or splenectomy

Identify the common symptoms and signs of carcinoma of the colon, rectum and anus

1) colorectal - a) LHS = I) bleeding/mucus PR II) altered bowel habit or obstruction III) tenesmus & nocturnal need to open bowels = red flag symptoms IV) mass PR b) RHS = I) dec weight + Hb II) abdominal pain III) obstruction less likely c) either = I) abdominal mass II) perforation III) haemorrhage IV) fistula 2) anal (generally squamous cell - strongly associated with HPV)- a) bleeding b) pain c) bowel habit change d) pruritis ani e) masses f) stricture g) spread to inguinal LN NB/ ~30% present as emergency with obstruction (80%), perforation (15%), haemorrhage (5%)

Suggest additional agents which may be added in Suspected viral meningo-encephalitis

1) confirmed viral agent, other than HSV, varicella zoster, or CMV - a) 1st line - supportive care eg/ adequate hydration, antipyretics for fever, anti-emetics if vomiting, and analgesia for headaches 2) HSV or varicella zoster confirmed causative agent - a) 1st line - antiviral therapy + supportive care: aciclovir: 10 mg/kg IV every 8 hours, or valaciclovir: 1 g orally every 8 hours; for 7-10 days 3) CMV confirmed causative agent - a) 1st line - antiviral therapy + supportive care: ganciclovir: 5 mg/kg IV every 12 hours or valganciclovir: 900 mg orally every 12 hours; for 7-10 days

Causes of nystagmus

1) congenital nystagmus/infantile nystagmus syndrome (INS) - inherited genetic condition, appears within first 6 weeks to 3 months of life, nystagmus usually mild & not caused by an underlying health problem 2) acquired nystagmus - a) stroke b) meds eg/ sedatives and AEDs like phenytoin c) excessive alcohol consumption d) head injury or trauma e) disease of the eye f) diseases of the inner ear g) B-12 or thiamine deficiencies h) brain tumors i) diseases of CNS, including MS

Describe the important components of abdominal wound closure to avoid dehiscence and herniation

1) continuous closure suturing using small bite/small stich-technique 2) prophylactic mesh implantation 3) prophylactic abx to prevent wound infection, and HAP

Surgical treatment of constrictive pericarditis

1) definitive care is primarily surgical (pericardiectomy), operative therapy leads to rapid hemodynamic and symptomatic improvements 2) medical mx eg/ careful observation or symptomatic tx, has been suggested in less severe cases eg/ diuretics 3) complete pericardiectomy - definitive therapy & potential cure a) pericardiectomy can be a long and technically complex procedure b) 2 standard approaches - via an anterolateral thoracotomy and via a median sternotomy

List the acute clinical situations where a lumbar puncture would be indicated

1) diagnosis of central nervous system (CNS) infection 2) diagnosis of subarachnoid hemorrhage (SAH) 3) infusion of anesthetic, chemotherapy, or contrast agents into the spinal canal 4) tx of idiopathic intracranial hypertension 5) evaluation and diagnosis of demyelinating or inflammatory CNS processes

Discuss the presentation, differential diagnosis, investigations and management of complications of colonic diverticulae including diverticulitis, perforation, bleeding, stricture, abscess and fistula

1) diverticular disease - a) DD - colorectal cancer, IBD, appendicitis, UTI/pyelonephritis, ischaemic colitis, PID, IBS b) ix - I) FBC with differential - polymorphonuclear leukocytosis b) consider - abdo x-ray (pneumoperitoneum, ileus, soft tissue densities; free air in bowel perforation), CT abdo (thickening of bowel wall, mass, abscess, streaky mesenteric fat; may show gas in the bladder in cases of fistula), abdo US (signs of abscess, perforation, obstruction), CXR (normal, or free air under diaphragm if perforation), contrast enema (diverticuli, abscess, perforation, obstruction, fistula), colonoscopy (diverticula, with or without acute mucosal inflammation), sigmoidoscopy (mucosal pathology such as ischaemia, IBD, and neoplasm), angiogram, isotope-labelled red blood cell nuclear scan, diagnostic laparoscopy/exploratory laparotomy, blood culture c) mx - asymptomatic diverticulosis - no tx required symptomatic diverticular disease - I) 1st line - dietary modification + fibre supplementation: gradually inc fibre content over weeks and increasing hydration II) +/- oral abx: amoxicillin/clavulanate if evidence of infection and/or suspicion of bacterial overgrowth symptomatic diverticulitis - I) 1st line - analgesia: paracetamol, tramadol II) +/- oral abx: amoxicillin/clavulanate III) +/- low-residue diet: causes dec frequency and volume of stools while prolonging intestinal transit time - restricts foods that inc bowel activity and is low in fibres eg/ refined bread, cereals, white rice, vegetable and fruit juice without pulp, dairy products IV) 2nd line - IV abx: ceftriaxone + metronidazole or piperacillin/tazobactam V) with acute rectal bleeding - endoscopic haemostasis/angiographic embolisation (if this fails may need surgery) + supportive therapy (fluids, blood etc) + abx (amoxicillin/clavulanate) + analgesia + low residue diet VI) unresponsive to IV abx or abscess >3 cm diameter, perforation, fistulae, or obstruction - radiological drainage/surgery + IV abx (ceftriaxone + metronidazole or piperacillin/tazobactam) + analgesia = low residue diet recurrent diverticulitis - elective surgery: laparoscopic colonic resection 2) diverticulitis s/s - thought to be caused by perforation of one of the sacs a) abdo pain - typically occurs in LLQ b) others - fever, N&V, urinary symptoms (eg/ frequency, urgency), change in bowel habits c) blood in the stool, as well as bleeding from the rectum, can occur in both diverticulosis and diverticulitis 2) perforation s/s - a) abdo pain - typically LLQ, may radiate to the suprapubic area, left groin, or back b) can have alterations in bowel habits (constipation and less commonly diarrhea) c) others - fevers, chills, and urinary symptoms d) on exam - pts may have isolated tenderness in LLQ or diffuse peritoneal signs, depending on the severity of the perforation. A tender mass in the left lower quadrant is suggestive of a phlegmon or abscess, or the patient may be distended as a result of a small bowel ileus, or from a large bowel obstruction due to a colonic stricture 3) bleeding s/s - occurs with chronic injury to the small blood vessels next to the diverticula. Amount can vary depending on extent of injury 4) stricture s/s - may lead to obstructive symptoms eg/ N&V and distension 5) abscess s/s - N&V, fever, and severe tenderness in abdomen 6) fistula s/s - hx: recurrent UTIs, pneumaturia, faecaluria, hx of hysterectomy

Classify abnormalities of speech

1) dysphasia - expressive/receptive/nominal 2) dysarthria - difficult or unclear articulation of speech that is otherwise linguistically normal 3) dysphonia - difficulty in speaking due to a physical disorder of the mouth, tongue, throat, or vocal cords

Define and discuss explanatory/management trial, different trial designs, randomisation and its effects, types of blinding, "intention to treat" versus "per protocol" analyses, and Type I and Type II errors

1) explanatory/management trial - a randomized controlled trial (RCT) can at one extreme investigate whether a treatment could work in ideal circumstances (explanatory), or at the other extreme, whether it would work in everyday practice (pragmatic) 2) different trial designs - different study designs provide information of different quality a) first, distinguish between observational and experimental studies - observational: researcher observes and systematically collects information, but does not try to change the people being observed; experiment: researcher intervenes to change something (e.g., gives some patients a drug) and then observes what happens b) observational designs - I) cross-sectional surveys eg/ what is the prevalence of diabetes in this community? Here, you draw a random sample of people and record information about their health in a systematic manner II) cohort studies - like surveys, but extend over time so you can study changes & establish the time-sequence in which things occur III) case-control study - retrospective study so find people with disease, then look for causes c) experimental designs - I) Randomized Controlled Trial ("RCT") - a sample of patients with the condition, and who meet other selection criteria, are randomly allocated to receive either the experimental treatment, or the control treatment (commonly the standard treatment for the condition) 3) randomisation and its effects - a) advantages - eliminates bias in treatment assignment eg/ selection bias and confounding; facilitates blinding of identity of treatments from investigators, participants, and assessors 4) types of blinding - a) unblinded or open - all parties know which tx they receive b) single blind or single masked - only participant if unaware of tx c) double blind or double masked - participant and clinicians/data collectors are unaware of tx participant receives d) triple blind - participant, clinicians/data collectors and outcome adjudicators/data analysts are all unaware of tx participant receives 5) "intention to treat" versus "per protocol" analyses - a) intention-to-treat analysis - comparison of the treatment groups that includes all patients as originally allocated after randomization. This is the recommended method in superiority trials to avoid any bias b) per-protocol analysis - comparison of treatment groups that includes only those patients who completed the treatment originally allocated. If done alone it leads to bias 6) Type I and Type II errors - a) Type I error - occurs when the null hypothesis (H0) is true, but is rejected. It is asserting something that is absent, a false hit. A type I error may be likened to a so-called false positive (result indicates that a given condition is present when isn't) Type II error - occurs when the null hypothesis is false, but erroneously fails to be rejected. It is failing to assert what is present, a miss. A type II error may be compared with a false negative (where an actual 'hit' was disregarded by the test and seen as a 'miss') in a test checking for a single condition with a definitive result of true or false. A Type II error is committed when we fail to believe a true alternative hypothesis

List typical symptoms of a patient with anaemia

1) fatigue 2) dyspnoea 3) faintness 4) palpitations 5) tinnitus 6) anorexia 7) headache 8) intermittent claudication NB/ angina if pre-existing coronary artery disease NB2/ signs - pallor, tachycardia, systolic flow murmur and cardiac failure (oedema, hypotension). Specific anaemias may produce koilonychia, jaundice, bone deformities, leg ulcers, angular stomatitis, brittle hair and nails, Plummer-Vinson syndrome (dysphagia and glossitis)

Describe the symptoms and signs of patients with femoral hernia. Perform the physical examination of patients with femoral hernia

1) femoral hernias occur just below the inguinal ligament, when abdominal contents pass through the femoral canal, anatomy of the femoral canal: anterior border = inguinal ligament, posterior border = pectineal ligament, medial border = lacunar ligament, lateral border = femoral vein 2) almost all develop in women due to inc width of female pelvis 3) s/s - a) groin lump or bulge lateral & inferior to pubic tubercle - may get smaller or disappear completely in prone position b) may be associated with pain c) may strain on coughing d) bulk of a femoral hernia lies below an imaginary line drawn between the ASIS and pubic tubercle whereas an inguinal hernia starts above this line d) may be lower abdo pain if incarceration occurs

List the potential complications of a lumbar puncture

1) headache from a persistent spinal fluid leak - happens in 25% pts with LP 2) brain herniation 3) bleeding (spinal haematoma) 4) infection 5) back pain 6) epidermoid tumor implantation

Recognise infected granulation tissue and discuss its clinical significance

1) healthy granulation tissue is pink in colour and is an indicator of healing 2) unhealthy granulation is dark red in colour, often bleeds on contact, and may indicate the presence of wound infection - need culture & tx in the light of microbiological results. Can be a sign of infection, ischemia, or poor perfusion - tissue won't heal properly and may lead to chronic injury 3) excess granulation or overgranulation may also be associated with infection or non-healing wounds. Appear dark red and devitalised (due to poor oxygenation) or pale due to lack of oxygen. Hyper-granulation tissue inhibits the migration of epithelial cells across the wound surface and increases the risk of scar tissue formation by preventing the wound edges from closing. May occur from prolonged inflammation due to infection or the presence of an irritant or foreign body; overuse of occlusive dressings; constant rubbing of dressings or tubes against the skin causing an inflammatory response (eg/ peg tube or supra pubic catheter); allergy to dressings; or imbalance of cellular activities that regulate the production of healthy tissue. These often respond to simple cautery with silver nitrate or with topically applied steroid preparations NB/ Recognise and differentiate wounds that have resulted in loss of tissue and those that have not NB2/ Identify the development of inflammatory changes in a wound and around a suture

Describe portal venous anatomy

1) hepatic portal vein = vessel that moves blood from spleen & GI tract to liver - SMV + IMV + SpV 2) ~3-4 inches in length & usually formed by merging of SMV (intestines) + SpV (pancreas + spleen) behind the upper edge of the head of the pancreas. In some, IMV (intestines) may enter this intersection instead 3) portal vein splits into L&R veins before entering liver, R vein then branches off into anterior and superior veins 4) HPV runs into liver alongside proper hepatic artery. Venous drainage of liver via hepatic veins, which drain to the IVC 5) portal vein supplies ~75% of blood flow to liver. Not a true vein, which means it does not drain into the heart, instead brings nutrient-rich blood to liver from GI tract & spleen. Once there, the liver can process the nutrients from the blood and filter out toxic substances it contains before the blood goes back into general circulation 6) abnormally high BP in portal vein = portal hypertension - may cause growth of new blood vessels that bypass liver, which can = circulation of unfiltered blood throughout the body - potential serious complication of liver cirrhosis

Describe the features suggesting a high risk of suicide in a patients presenting with self harm or overdose

1) high unresolved suicidal intent 2) multiple attempts 3) depressive disorder 4) chronic alcohol and drug misuse 5) social isolation 6) current physical illness 7) socioeconomic deprivation

Discuss the diagnosis and investigation of a patient with jaundice

1) history - blood transfusions, IV drug use, body piercings, tattoos, sexual activity, travel abroad, jaundiced contacts, family history, alcohol use, all medications 2) blood tests - FBC, clotting, film, reticulocyte count, Coomb's test and haptoglobins for haemolysis, malaria parasites, Paul Bunnell (EBV) 3) tests for viral load, Ig (paraproteinaemia) + electrolytes (Cu, Fe) if suspected 4) radiology - ultrasound: are bile ducts dilated? Gallstones, hepatic metastases, pancreatic mass? 5) biopsy is able to histologically confirm diagnoses of intra-hepatic pathology

Discuss additional cardiovascular risk factors in diabetic patients and outline their assessment and management

1) hypertension - BP < 135/85 mmHg recommended, control with diet & lifestyle, and ACEI or ARB etc 2) hyperglycaemia - strict glucose control using medication as instructed, control of diet & adequate exercise, measuring blood sugars if using insulin 3) increased LDL cholesterol, low levels of HDL - recommended pts should have LDL cholesterol <100 mg/dl 4) smoking - encourage pts to stop smoking, refer to services that can help and they can prescribe NRT

Describe the immediate management of an unconscious patient, including the protection of the patient's airway and maintenance of patient's circulatory pressure

1) if hx or suspicion of trauma, the cervical spine should be immobilised 2) whilst ABC assessment is taking place, others need to establish IV access, connecting cardiac and oxygen saturation monitoring 3) commencing appropriate oxygen therapy if indicated 4) intubation if GCS <9 or those who cannot protect their own airway or have ineffective respiratory drive and poor oxygenation 5) hypotension is initially managed with IV fluid resuscitation; early vasopressor support is considered when BP does not respond 6) if raised ICP suspected have pt in 30°head tilt position 7) if suspect toxicity - IV: naloxone in opiate toxicity, glucose 20% in hypoglycaemia, thiamine in Wernicke's encephalopathy, flumazenil in benzodiazepine overdose

List the main causes of peptic ulcer disease

1) inc age 2) H. pylori infection 3) smoking 4) NSAIDs 5) reflux of duodenal contents 6) delayed gastric emptying 7) stress eg/ neurosurgery or burns 8) Zollinger-Ellison syndrome 9) acid/coffee consumption

List the indications for amputation and discuss the selection of amputation site

1) indications - a) trauma b) gangrene c) diabetes d) malignant tumours e) neuroma f) frostbite g) infection 2) when a lower extremity amputation has to be performed, the surgeon is challenged to select the lowest possible level of amputation to preserve amputee mobility, but at the same time the risk of failed amputation wound healing has to be avoided - need to be able to feel palpable pulses above level of amputation

List secondary causes of NS and outline investigations necessary to confirm the diagnosis

1) infection eg/ HIV, hep B & C, mycoplasma, syphilis, malaria, schistosomiasis, filariasis, toxoplasmosis 2) collagen vascular diseases - eg/ SLE, RA, polyarteritis nodosa, HSP, vasculitides 3) metabolic diseases eg/ DM, amyloidosis 4) inherited disease eg/ Alport's syndrome, hereditary nephritis, sickle cell disease 5) malignant disease eg/ multiple myeloma, leukaemia, lymphoma, carcinoma of breast, carcinoma of lung, carcinoma of colon and carcinoma of stomach 6) drugs eg/ NSAIDs, captopril, lithium, gold, diamorphine, interferon alfa, penicillamine, probenecid etc 7) toxins eg/ bee sting, snake bites, phytotoxins 8) pregnancy eg/ pre-eclampsia 9) transplant rejection

Outline management of an obstructing colonic cancer

1) initial tx - 2) fluid resuscitation of the patient 3) correcting any electrolyte abnormalities 4) GI decompression with a nasogastric tube placed to suction 5) close monitoring of urine production, usually with insertion of urinary catheter 6) obstructive process extrinsic to the colon eg/ hernia - surgical correction of underlying problem 7) obstruction intrinsic to the colon - choices include both surgical and endoscopic techniques a) colostomy - if acutely ill with signs of peritonitis or sepsis will likely need an urgent operation, normally either an end colostomy, with resection of the distal obstruction if possible, or more proximal loop colostomy, can also do segmental colectomy and subtotal colectomy, with or without on-table decompression of the proximal bowel b) segmental colectomy - RHS obstruction = R hemicolectomy with ileocolic anastomosis, not customarily used in LHS obstruction c) subtotal colectomy - LHS obstruction: subtotal colectomy with ileosigmoid or ileorectal anastomosis d) self-expanding metal stent placement - endoscopic stenting for colorectal obstruction

Discuss appropriate investigations to confirm that a patient has iron deficiency

1) iron levels (dec) 2) TIBC (inc) 3) Ferritin (dec) - iron store levels 4) MCV (dec) NB/ blood count & film with serum ferritin = 1st line. Cells are microcytic + hypochromic, and vary in size and shape. Target & pencil cells may be seen NB2/ if clinical picture is still vague, bone marrow sampling may be used

Outline the investigations and treatment options for a patient with suspected interstitial lung disease

1) ix - a) CXR- basilar, peripheral, bilateral, asymmetrical, reticular opacities b) HRCT - basilar- and subpleural-predominant areas of increased reticulation, honeycombing, and possible traction bronchiectasis c) peak flow - restrictive: reduced FVC, reduced total lung capacity d) ANAs & rheumatoid factor - titres of >1:160 suggest the possibility that a collagen vascular disease might be responsible for the interstitial lung disease e) others - surgical lung biopsy, bronchoalveolar lavage (BAL), trans-bronchial biopsy 2) tx - a) acute exacerbation - hospital + high dose corticosteroids eg/ prednisolone I) +/- cytotoxic therapy b) ongoing - I) 1st line - antifibrotic therapy eg/ pirfenidone or nintedanib II) + smoking cessation + pulmonary rehabilitation III) +/- O2 therapy IV) +/- opioids if severe dyspnoea & advanced disease V) +/- PPI eg/ lansoprazole: 15 mg, omeprazole, 20 mg VI) 2nd line - lung transplantation

Describe the laboratory features of haemolysis. Outline the causes of haemolytic anaemia and their treatment

1) ix - a) FBC - low Hb b) MCHC - inc, may indicate presence of spherocytes and reticulocytes c) reticulocyte count - inc reticulocyte percentage (>1.5%), absolute reticulocyte count, and reticulocyte index d) peripheral smear - if: schistocytes (microangiopathic process from physical destruction within the vasculature), spherocytes (hereditary spherocytosis), elliptocytes (elliptocytosis), spur cells (liver problem), blister or bite cells (oxidant damage to the cell), RBC inclusions may occur with malaria, babesiosis, and Bartonella infections e) unconjugated (indirect) bilirubin - inc but <85 micromol/L unless liver function is impaired f) LDH & haptoglobin - inc LDH + dec haptoglobin = haemolytic anaemia g) urinalysis - haemoglobinuria is present in intravascular haemolysis h) consider - direct antiglobulin test (DAT or Coombs'), creatinine, urea, LFTs, Donath-Landsteiner antibody, Hb electrophoresis, flow cytometry for CD55/CD59, G6PD fluorescent spot test and spectrophotometry, ANA 2) causes - a) hereditary causes fall into 3 broad categories: I) inherited RBC defects (membrane) - hereditary spherocytosis, elliptocytosis, pyropoikilocytosis II) enzyme deficiencies - G6PD deficiency, pyruvate kinase deficiency III) abnormal Hb production - sickle cell anaemia, thalassaemia b) acquired haemolytic anaemia can be subdivided into immune and non-immune: I) autoantibodies cause immune-mediated haemolytic anaemias, most often as part of other autoimmune conditions (e.g., SLE, RA, scleroderma) or related to a lymphoproliferative disorder (non-Hodgkin's lymphoma, chronic lymphocytic leukaemia) II) immune haemolytic anaemias can be divided into warm- or cold-reacting antibodies, depending on temperature at which antibody binds most avidly to RBCs III) alloimmune haemolytic anaemias include haemolytic disease of the newborn or transfusion reaction IV) many drugs are associated with haemolysis V) other non-immune-mediated causes of haemolysis - infection, trauma (microangiopathic haemolysis such as DIC, thrombotic thrombocytopenic purpura, HUS, malignant HTN, eclampsia, HELLP syndrome, mechanical prosthetic heart valves, march haemolysis, thermal injury, osmotic lysis), hypersplenism, and liver disease 3) mx - a) acquired: Coombs' positive (autoimmune haemolytic anaemia) - 1st line - removal of insult or treatment of underlying condition + supportive care I) removal of insult or tx underlying conditions + supportive care eg/ folic acid, tx infection in warm AIHA or lymphoma in cold AIHA, packed RBC transfusion can be used for all subtypes but should be restricted to symptomatic anaemia. II) + corticosteroid eg/ prednisolone: 1 mg/kg/day orally until response, then slow taper (causes dec antibody production) III) +/- whole-body plasmapheresis IV) 2nd line - splenectomy + supportive care, splenectomy considered upon failure to respond to corticosteroids (removes the site of significant antibody production and the predominant site of RBC destruction) V) +/- rituximab before splenectomy VI) 3rd line - rituximab +/- corticosteroid b) drug-induced (immune-mediated) - I) 1st line - stop offending medication + supportive care eg/ folic acid, packed RBC transfusion II) +/- corticosteroid (if doesn't resolve within 3 weeks of discontinuation of the provoking medication) c) acquired: Coombs' negative - I) 1st line - removal of any offending agents + supportive care II) with worsening prosthetic valve-related haemolysis - + cardiology evaluation III) with thrombotic thrombocytopenic purpura - + plasma exchange IV) with paroxysmal nocturnal haemoglobinuria - + corticosteroid or eculizumab for corticosteroid-resistant disease V) with paroxysmal nocturnal haemoglobinuria - +/- anticoagulation (warfarin or LMWH) VI) with hypersplenism not caused by portal hypertension - +/- splenectomy d) inherited disorders - I) red cell membrane disorders - splenectomy + supportive care eg/ hereditary spherocytosis, elliptocytosis, and pyropoikilocytosis II) G6PD deficiency - avoidance of triggers + supportive care III) pyruvate kinase deficiency - supportive care +/- splenectomy IV) haemoglobinopathies - supportive + disease-specific specialist care eg/ sickle cell anaemia and thalassaemia, sickle cell crises - oxygen, pain control, aggressive hydration, and treatment of any concurrent stressors (e.g., infection)

Identify and characterise, through enquiry and regional examination, the s/s signs of arthropathy

1) joint inflammation +/- damage at the sternoclavicular joint, acromioclavicular joint, shoulder, elbow, wrist, hand, hip, sacro-iliac joint, knee, ankle/hindfoot, midfoot and forefoot 2) signs may include inc heat, soft-tissue swelling, effusion, stress pain, joint-line tenderness, crepitus, bony swelling, dec movement, deformity and instability

Outline the general principles of curative and palliative surgical procedures for gastric cancers and discuss the role of adjuvant and palliative therapy

1) localised & surgical candidate - a) 1st line: surgery b) + perioperative or postoperative chemoradiation 2) localised & non-surgical candidate - a) 1st line: chemoradiation 3) advanced & metastatic disease - a) 1st line: chemoradiation or chemotherapy - improves QOL & may inc survival b) + palliative gastrectomy - may improve bleeding & obstruction

List the features which reflect severe head injury

1) loss of or change in consciousness anywhere from a few seconds to a few hours 2) decreased level of consciousness, i.e., hard to awaken 3) convulsions or seizures 4) unequal dilation in the pupils of the eyes or double vision 5) clear fluids draining from the nose or ears 6) nausea and vomiting 7) new neurologic deficit, i.e., slurred speech; weakness of arms, legs, or face; loss of balance

Describe the indications for Magnetic resonance angiography, Duplex ultrasound, CT angiography and invasive investigations of the arterial and venous system and list the common insertion sites for arterial catheter studies

1) magnetic resonance angiography - a) identify abnormalities eg/ aneurysms, in the aorta, chest and abdomen etc b) detect atherosclerotic disease in the carotid artery of the neck - can cause stroke c) identify a small aneurysm or arteriovenous malformation inside the brain or elsewhere d) detect atherosclerotic disease in legs to prepare for endovascular intervention or surgery e) detect disease in the arteries to the kidneys f) guide interventional radiologists and surgeons making repairs to diseased blood vessels, such as implanting stents or evaluating a stent after implantation g) detect injury to arteries in the neck, CAP or extremities in patients after trauma h) evaluate arteries feeding a tumor prior to surgery or chemoembolization etc i) identify dissection or splitting in the aorta in the chest or abdomen j) show the extent and severity of the effects of coronary artery disease and plan for a surgical operation, such as a coronary bypass and stenting k) examine pulmonary arteries in the lungs to detect pulmonary embolism l) look at congenital abnormalities in blood vessels, especially arteries in children (e.g., malformations in the heart or other blood vessels due to congenital heart disease) 2) duplex ultrasound - a) abdominal aneurysm b) arterial occlusion c) blood clot d) carotid occlusive disease e) renal vascular disease f) varicose veins g) venous insufficiency 3) CT angiography - a) stenosis or blockage in the coronary arteries b) heart problems eg/ pericarditis, damage or injury to the heart valves c) aneurysm or dissection in the aorta d) PE e) peripheral arterial disease f) abnormal pattern of blood vessels that may be a sign of a tumour 4) invasive investigations of the arterial and venous system - a) Ankle Brachial Index Test b) cardiac catheterization c) carotid artery duplex scan - evaluates blood flows through the carotid arteries d) electrophysiological studies - small, thin wire electrodes placed directly on heart to evaluate for abnormal heart rhythm e) myocardial perfusion scan, resting or stress - imaging test to show how well blood flows through the heart muscle f) pulmonary artery catheterization - catheter is inserted into a pulmonary artery to diagnose and manage a wide variety of health problems g) radionuclide angiogram, resting and exercise - type of nuclear medicine procedure to help show the tissue under study h) transesophageal echocardiogram - transducer (like a microphone) is lowered into the oesophagus, it sends out US waves at a frequency too high to be heard. When the transducer is placed at certain locations and angles, the ultrasonic sound waves move through the skin and other body tissues to the heart tissues, where the waves bounce or "echo" off of the heart structures i) venogram - special dye is injected that then shows veins on X-ray 5) common insertion sites for arterial catheter studies - radial or femoral artery

List non-motor features of Parkinson disease, with specific reference to disorders of sleep, mood and cognition

1) masked facies - loss of spontaneous facial movement and expressivity, often noted only by spouse 2) hypophonia - dec volume of voice 3) hypokinetic dysarthria - related to bradykinesia and rigidity of orobuccolingual and laryngeal musculature 4) micrographia 5) stooped posture - related to rigidity 6) shuffling gait - related to rigidity and bradykinesia 7) conjugate gaze disorders - saccadic (jerky) pursuit and hypometric saccades (falling short of intended target) 8) fatigue 9) constipation - autonomic dysfunction. 10) depression 11) dementia - common in mid- to late-stage disease

Differentiate by pt enquiry & exam common mechanical neck/back pain (+ root entrapment), inflammatory back pain (eg/ spondylitis), destructive back pain (malignancy, sepsis) and pain from vertebral fracture

1) mechanical neck/back pain - a) back or neck sprain and strain - sprain: spinal overstretched or torn; strain: involves a muscle and/or tendon attachment. Causes soft tissues swelling, pain, can have muscle spasm & stiffness b) disc herniation - nucleus pulposus breaks through annulus fibrosus, can cause nerve compression, irritation, inflammation, and pain which can radiate into an arm or leg. Numbness, weakness, and tingling may occur c) lumbar spinal stenosis - when nerve structures are compressed the dominate symptom is pain which can radiate into one or both legs (sciatica). Usually affects older adults and can be associated with degenerative changes that cause bone and/or tissue to grow into nerve pathways or compress the spinal canal d) spondylosis - degenerative spinal osteoarthritis; common in older adults. Causes inflammation, stiffness, pain. Spondylosis is part of other degenerative changes that cause spinal stenosis and disc herniation e) spondylolisthesis - when one vertebral body moves forward over the vertebra beneath. Mostly lumbar spine. Besides pain, that can be severe, muscle spasms, and sciatic-type symptoms may develop 2) root entrapment - a) positive sensory symptoms are earliest to occur eg/ tingling and neuropathic pain such as burning, then dec sensation or complete numbness. Muscle weakness & atrophy occur later. May occur directly following insult or several hours or days afterwards b) causes - nerve may be compressed by prolonged or repeated external force eg/ sitting with arm over back of chair (radial nerve), frequently resting elbows on table (ulnar nerve), or an ill-fitting cast or brace on the leg (peroneal nerve). Part of the patient's body can cause the compression eg/ tumour, ganglion cyst or haematoma. Alternatively, may be expansion of the tissues around a nerve in a space where there is little room eg/ carpal tunnel syndrome 3) inflammatory back pain eg/ ankylosing spondylitis - a) pain and stiffness in low back, buttocks, and hips that continue > 3 months. Spondylitis often starts around sacroiliac joints. Bony fusion affecting bones of the neck, back, or hips may impair ability to perform routine activities. Fusion of the ribs to the spine or breastbone may limit a person's ability to expand chest when taking a deep breath. Pain in ligaments and tendons - tendonitis (inflammation of tendon) may cause pain and stiffness in the area behind or beneath the heel, such as the Achilles tendon at the back of the ankle. Systemic disease - can have fever, fatigue, loss of appetite, eye inflammation, and rarely lung and heart 4) destructive back pain eg/ malignancy, sepsis a) spinal metastases - persistent, unremitting back pain, not alleviated by rest. Often the pain is worse at night, waking the patient from sleep. Back pain is typically focal to the level of the lesion and may be associated with belt-like thoracic pain or radicular symptoms of pain or weakness in the legs. A spinal mass can cause neurologic signs or symptoms by directly compressing the spinal cord or nerve roots, mimicking disk herniation or stenosis. Pathologic fractures from vertebral destruction may be the first presentation of a tumor. Systemic disease - may have fatigue, weight loss, and changes in bowel habits. Hemoptysis, lymphadenopathy, subcutaneous or breast masses, nipple discharge, atypical vaginal bleeding, or blood in the stool suggest malignancy (check for signs from breast, lungs, abdomen, thyroid, and prostate) 5) pain from vertebral fracture - vertebral compression fractures can be caused by osteoporosis, trauma, and diseases affecting bone (pathologic fracture) a) pain - mostly lower back, numbness, tingling, and weakness may occur if compression of the nerves at the fracture site, may have incontinence of stool or urine can occur if fracture is pushing on the spinal cord itself

Outline the commonest causes of disability in people with impaired mobility

1) mobility impairments range in severity from limitations of stamina to paralysis. Some mobility impairments are caused by conditions present at birth while others are the result of illness or physical injury 2) spinal cord injury - injuries to the spinal cord cause different types of mobility impairments, depending on the areas of the spine affected eg/ Quadriplegia vs Paraplegia 3) amputation 4) arthritis 5) back disorders eg/ degenerative disk disease, scoliosis, and herniated disks 6) cerebral palsy 7) neuromuscular disorders eg/ muscular dystrophy, multiple sclerosis, and ataxia 8) fibromyalgia

Describe the morphology and pathological consequences of diverticulosis of the colon

1) morphology - colonic diverticula typically form in parallel rows between taeniae coli because of weakness of the muscle wall at sites of penetration of the vasa recta supplying the mucosa. Diverticula vary from solitary findings to many hundreds. Typically 5-10 mm in diameter but can exceed 2 cm. have peritoneal surrounding 2) pathological consequences - diverticulitis (infection) which leads to: abscess, which occurs when pus collects in the pouch, blockage in colon or SI caused by scarring, abnormal passageway (fistula) between sections of bowel or the bowel and bladder, peritonitis, which can occur if the infected or inflamed pouch ruptures, spilling intestinal contents into abdominal cavity. Medical emergency and requires immediate care

Describe the epidemiology, classification, morphology and natural history of gastric cancers

1) morphology: a) gastric adenocarcinoma - malignant epithelial tumour originating from glandular epithelium of gastric mucosa. 90% stomach cancers = adenocarcinomas b) histologically - 2 major types of gastric adenocarcinoma: intestinal type or diffuse type. Adenocarcinomas tend to aggressively invade gastric wall, infiltrating muscularis mucosae, submucosa and thence muscularis propria c) intestinal type adenocarcinoma tumour cells - irregular tubular (glandular) structures, harbouring pluristratification, multiple lumens, reduced stroma. Often it associates intestinal metaplasia in neighbouring mucosa. May present 3 degrees of differentiation: well, moderate and poorly differentiated. Often associated with H. Pylori, and may be polypoid (presents earlier with bleeding and easier to resect) or ulcerative (most common form, has a raised edge & necrotic base) d) diffuse type adenocarcinoma tumour cells - discohesive and secrete mucus, which is delivered in the interstitium, producing large pools of mucus/colloid. It is poorly differentiated. If the mucus remains inside the tumour cell, it pushes the nucleus to the periphery: "signet-ring cell". Linitus plastic (leather bottle stomach), infiltration (worse prognosis). Tightening & thickening of mucosa causes dec stomach capacity 2) natural history: a) tumour spreads by direct invasion into surrounding tissues & blood vessels. Lymphatic spread occurs to regional LNs. Blood borne spread carries tumour to distant sites in the body, most commonly the liver and lungs, and less commonly bones and brain b) majority of adenocarcinomas are antral c) invasion through peritoneum = malignant ascites d) metastasis may occur to ovary = "Krukenberg" tumour, or to umbilicus = "Sister Mary Joseph's nodule". Other tumours include GIST, primary gastric lymphoma (10% of non-Hodgkins lymphomas) and gastric polyps (usually benign). As TNM stage inc, 5 yr survival dec 3) epidemiology -

Discuss the management of acute pancreatitis and potential early complications of acute pancreatitis

1) mx - 2) complications - a) shock/ARDs b) renal failure (give lots of fluid) c) DIC d) sepsis e) dec calcium f) inc glucose (transient - 5% need insulin) g) pain/distension without palpable mass h) retroperitoneal bleed i) pancreatic necrosis j) pseudo-cyst k) infection/abscess

Describe the main principles of treatment including physiotherapy, antibiotics, pancreatic enzymes DNAse, and lung transplantation

1) need MDTs in CF tertiary centre 2) respiratory problems - a) mucus clearance, preventing infection and maintaining good lung function are the main aims b) chest physio 2x/day, inc with infective exacerbations c) additional exercise should be encouraged d) regular sputum samples sent for bacterial culture e) prophylactic abx used to dec S. aureus in children & prevent secondary bacterial infections when a patient has a presumed acute viral respiratory infection f) bronchodilators by metered dose inhaler (MDI) or by nebuliser. (SABA and LABA) g) high-dose ibuprofen & azithromycin may slow progression of lung disease, especially in children h) hypertonic saline by =nebuliser i) mannitol dry powder for inhalation j) no role for corticosteroids except in treating allergic bronchopulmonary aspergillosis k) lung or heart and lung transplantation listing considered where there is respiratory failure 3) nasal polyps - nasal steroids, then polypectomy 4) pancreatic insufficiency - take enteric-coated enzyme preparations before meals with the dose adjusted to achieve normal stools a) presents with neonatal meconium ileus or failure to thrive, steatorrhoea and malnutrition, can cause rectal prolapse, intussusception, volvulus, obstruction 5) maintaining adequate weight - 2x normal protein amount needed, 130% normal calories, if patient can't maintain weight, enteral feeding via gastrostomy may be required 6) drugs may be needed to dec acid secretion, as well as vitamin supplements for fat-soluble vitamins A, D and E 7) liver disease - LFTs are abnormal and should be treated with caution a) ursodeoxycholic acid improves bile flow and produces some improvement but does not alter the course of chronic liver disease b) liver transplantation should be offered to CF patients with progressive liver failure +/- life-threatening sequelae of portal HTN 8) diabetes and glucose intolerance - screening for diabetes is performed at regular intervals, insulin replacement is usually required with the dose adjusted to match high dietary intake 9) reproductive health and fertility - nearly all males with CF have obstructive azoospermia with sexual function that is otherwise normal (normal spermatogenesis but no vas deferens), IVF needed 10) psychological problems - counselling may be needed, including genetic counselling 11) osteoporosis - pts to take calcium, vitamin D and bisphosphonates as appropriate, need regular DEXA scans 12) vaccines - influenza & pneumococcal vaccines

Describe the investigation of a patient presenting with a community-acquired pneumonia and interpret investigations

1) need to: establish diagnosis, identify pathogen, assess severity 2) CXR (most useful) - lobar/multi-lobar infiltrates, cavitation or pleural effusion, consolidation - CXR should return to normal in 6 weeks 3) O2 sats & BP 4) blood tests - FBC, U&Es, LFT, CRP, WCC, ESR, blood culture (before Abx started) 5) sputum - microscopy, culture - severe cases check for Legionella (sputum + urine) 6) pleural fluid - aspirated for culture 7) consider bronchoscopy + bronchoalveolar lavage if pt immunocompromised or on ITU

Interpret a blood count showing a leucocytosis and list common causes for neutrophilia and neutropaenia, lymphocytosis and lymphopaenia

1) neutrophilia (>10)- a) bacterial infections b) inflammation eg/ MI, polyarteritis nodosa c) myeloproliferative disorders d) drugs eg/ steroids e) disseminated malignancy 2) neutropenia (<1.5)- a) viral infections (+ severe bacteria (typhoid)) b) drugs eg/ post-chemotherapy, cytotoxic agents, carbimazole, sulphonamides c) severe sepsis d) neutrophil antibodies (inc destruction) eg/ SLE, haemolytic anaemia (autoimmunity) e) bone marrow failure (dec production) 3) lymphocytosis (>5)- a) acute viral infections (particularly EBV, CMV and early HIV) b) chronic infections eg/ TB, Brucella, hepatitis, syphilis c) leukaemia + lymphomas, especially CLL d) large amounts of atypical lymphocytes in EBV e) post-splenectomy 4) lymphopenia - a) steroid therapy eg/ SLE b) uraemia/renal failure c) Legionnarie's + HIV d) marrow infiltration e) post-chemotherapy or radiotherapy f) inflammation (connective tissue disease) g) transient following recent infection

Compare the CSF findings in bacterial, fungal and viral meningitis/encephalitis

1) normal - opening pressure: 80-200, WCC: 0.5 lymphocytes, glucose: 50-75, protein: 15-40 2) bacterial - opening pressure: inc (200-300), WCC: v. inc (up to 5000 polymorphonuclear leukocyte), glucose: dec (<40), protein: inc (>100) inc - OP, WCC (v), protein; dec glucose 3) viral - opening pressure: normal, WCC: inc (10-300 lymphocytes), glucose: normal, protein: normal normal - protein + glucose + OP; inc WCC 4) fungus - opening pressure: inc (180-300), WCC: inc (10-200 lymphocytes), glucose: dec, protein: inc (50-200) inc - OP + WCC + protein; dec glucose

Discuss the ethical issues associated with artificial nutritional support

1) nutrition can be a highly emotive topic, with staff and relatives using phrases such as 'starving to death' on the one hand and 'force feeding' on the other 2) the provision of adequate nutrition and hydration is a basic human right when patients are able to take this independently and that this should be offered as part of basic care 3) law regards artificial nutrition as a medical treatment 4) crucial difference between providing nutrition as a medical treatment and force feeding is the patient's consent to treatment. If the patient is unable to give this, any treatment given must be in their best interests. However, each patient is different eg/ terminal illness, vegetative state etc - becomes more challenging to know what's best for the patient

Describe the clinical features of complications of cirrhosis and portal hypertension....... and outline their management

1) oesophageal varices - blood flow to liver is blocked by scar tissue in liver, so blood flow begins to back up, increasing pressure within portal vein. Portal hypertension forces the blood to seek other pathways through smaller veins, such as those in the lowest part of the esophagus. These thin-walled veins balloon with the added blood. Sometimes the veins can rupture and bleed a) mx - I) resuscitation II) + vasoactive drugs eg/ octreotide or terlipressin III) + endoscopic therapy IV) + prophylactic abx eg/ norfloxacin or ceftriaxone V) trans-jugular intrahepatic porto-systemic shunt (TIPS) VI) +/- balloon tamponade or self-expanding metal mesh stent 2) ascites - inc pressure in portal blood vessels may cause protein-containing (ascitic) fluid to leak from the surface of the liver & intestine and accumulate within the abdomen a) mx - I) diet - dec sodium II) meds - diuretics (spironolactone + furosemide) III) therapeutic paracentesis - if not enough response to above, frequent therapeutic paracentesis can be performed to remove large amounts of fluid (4-5L per time) IV) surgery - for more refractory cases eg/ transjugular intrahepatic portosystemic shunts (TIPS), liver transplant 3) encephalopathy - when substances that are normally removed from the liver pass into the general circulation and reach the brain, they may cause confusion or drowsiness a) mx - I) supportive care + reversal of precipitating factors - frequently monitoring pt's neurological and mental status, if comatose may need endotracheal intubation reverse precipitating factors eg/ GI bleeding, infections, electrolyte disturbances and renal failure, sedative or narcotic ingestion, excessive dietary protein intake, acute hepatic or portal vein thrombosis, and recent placement of a transjugular intra-hepatic portosystemic shunt (TIPS II) + lactulose III) +/- abx eg/ rifaximin or metronidazole or neomycin or vancomycin

Discuss the presentation of post-primary tuberculosis from reactivation of infection

1) often asymptomatic or have only minor symptoms eg/ chronic dry cough 2) symptomatic pts = B symptoms - fever, malaise, night sweats & weight loss, productive cough which is often blood-stained may be present 3) occasionally present with massive haemoptysis due to erosion of a bronchial artery 4) pleuritic chest pain will accompany effusion 5) foci in reactivation appear in apices of lung & hilar often involved

Describe the clinical features, symptoms and signs of diverticulitis

1) often incidental finding on colonoscopy (95% asymptomatic) - just dietary advice 2) may have: a) abdominal pain on LIF (colic) relieved by defecation b) altered bowel habit (eratic) c) nausea d) flatulence e) severe disease - lumen of sigmoid colon may narrow = severe pain & constipation 3) diverticulitis - a) features of diverticulosis, alongside: b) pyrexia + constipation c) inc WCC d) inc CRP/ESR e) tender colon f) localised or generalised peritonism - abdominal tenderness + guarding g) CT of abdomen - thickening & diverticula (USS available & cheaper but less sensitive) NB/ may seem similar to appendicitis, but on LHS

List the potentially treatable causes of cognitive decline. Outline an investigation plan to exclude a treatable cause of dementia

1) other causes - a) Parkinson's b) substance abuse c) hypothyroidism d) space-occupying intracranial lesions e) normal pressure hydrocephalus f) syphilis g) vit B12 or folate deficiency h) pellagra 2) ix - FBC, ESR or CRP, MSU, U&E, LFT, glucose, Ca2+, TFT, B12 and folate

Describe the use of oxygen & analgesia in acute coronary syndrome

1) oxygen - recommended if pt has SaO2 <95%, are breathless, or in acute LVF 2) analgesia - oral opiates eg/ morphine 5-10mg, give with an antiemetic on 1st dose eg/ metoclopramide 10mg, cyclizine 50mg. Can also have oral/sublingual/IV nitrates NB/ pts require oxygen, aspirin, clopidogrel and LMWH. B-blocker may be given alongside

Describe the symptoms and physical examination of patients with fissure-in-ano

1) pain, sometimes severe, during bowel movements 2) pain after bowel movements that can last up to several hours 3) bright red blood on stool or toilet paper after a bowel movement 4) itching or irritation around anus 5) visible crack in skin around the anus 6) small lump or skin tag on skin near anal fissure 7) perianal abscesses & fistulae may occur NB/ can be an isolated primary problem in young to middle-aged adults or occur due to IBD NB2/ rectal examination not usually possible because of pain and sphincter spasm -spasm not only causes pain but impairs healing

Signs of chronic liver disease

1) palmar erythema 2) leukonychia 3) clubbing 4) bruising 5) spider naevi 6) parotid enlargement 7) gynaecomastia 8) testicular atrophy 9) peripheral oedema

List the antidotes available to treat specific poisons

1) paracetamol = n-acetylcysteine 2) opiates = naloxone 3) benzodiazepines = flumazenil 4) B-blockers = glucagons 5) TCAs = sodium bicarbonate 6) anticholinergics = physostigmine 7) warfarin = vit K 8) digoxin = digibind 9) heparin = protamine sulfate 10) methotrexate = leucovorin

Discuss the clinical presenting features, staging, pathology, natural history and clinical management of carcinoma of the prostate including a description of hormonal manipulation

1) pathology - region of prostate gland where the adenocarcinoma is most common = peripheral zone. Initially, small clumps of cancer cells remain confined to otherwise normal prostate glands (carcinoma in situ or prostatic intraepithelial neoplasia (PIN)). Over time, these cancer cells begin to multiply and spread to surrounding prostate tissue (stroma) forming a tumour. Eventually tumour may grow large enough to invade nearby organs eg/ seminal vesicles or rectum, or tumour cells may develop ability to travel in blood & lymphatic system. Malignant tumour as mass of cells can invade other parts of the body (metastasis). Most commonly metastasizes to bones, LNs, and may invade rectum, bladder and lower ureters after local progression. Route of metastasis to bone is thought to be venous as prostatic venous plexus draining the prostate connects with the vertebral veins 2) natural history - adenocarcinoma of the prostate gland generally occurs in the posterior, outer, glandular portion of tissue. Progression will result in an elevation in PSA levels. Symptoms include progressive, lower urinary tract obstruction, back pain, weight loss and anaemia. Malignant growth in prostate is very common in elderly men. Typically, it is a cancer that men die WITH and not FROM 3) s/s - a) risk factors eg/ >50 yo, black, fmx of prostate cancer b) inc prostate-specific antigen (PSA) c) nocturia, urine frequency, urinary hesitancy, dysuria - indicates higher T-stage or benign prostatic hyperplasia d) abnormal DRE - asymmetrical, nodular prostate 4) staging - a) primary tumour (T category): T1: clinically tumour not palpable; T2: tumour is palpable and confined within prostate (a+b = 1 side, c = both sides); T3: extraprostatic tumour that is not fixed or does not invade adjacent structures; T4: tumour is fixed or invades adjacent structures other than seminal vesicles eg/ wxternal sphincter, rectum, bladder, levator muscles, and/or pelvic wall b) regional lymph nodes (N): N0: no regional LN metástasis; N1: metastasis in regional LN(s) c) distant metastasis (M): M0: no distant metastasis; M1: distant metastasis (a = non-regional LN, b = bone, c = other site) 5) mx - a) intermediate-risk disease - I) 1st line - observation: definitive treatments (e.g., brachytherapy or external-beam radiotherapy) might be delayed and patients undergo observation instead, depending on the patient's projected survival (e.g., <10 years) and wish to avoid treatment-related side effects II) or brachytherapy - permanent transperineal implantation of radioactive sources into the prostate gland III) or external-beam radiotherapy - daily for 7 to 8 weeks IV) or radical prostatectomy ± lymph node dissection - if confined to prostate V) if unfavourable may add androgen deprivation therapy to radiotherapy: bicalutamide or flutamide, + leuprorelin or goserelin I) +/- brachytherapy boost b) high-risk or very high-risk disease - I) radical prostatectomy plus pelvic lymph node dissection II) or external-beam radiotherapy plus brachytherapy boost III) or external-beam radiotherapy plus androgen deprivation therapy c) metastatic disease - I) androgen deprivation therapy ± docetaxel or abiraterone II) + denosumab or bisphosphonate or toremifene III) +/- systemic radiotherapy IV) 2nd line - hormonal therapy or chemotherapy: docetaxel + prednisolone or abiraterone acetate or ketoconazole V) +/- denosumab or bisphosphonate or toremifene

Be aware of other therapies which may increase GI bleed risk

1) peptic ulcer disease 2) gastroduodenal erosions 3) oesophagitis 4) varices 5) Mallory-Weiss tears 6) diverticular disease 7) Meckel's diverticulum 8) colonic angiodysplasia 9) ischaemic colitis 10) radiotherapy for pelvic or abdominal cancers 11) other vascular causes eg/ aorto-enteric fistula, vasculitis, hereditary haemorrhagic telangiectasia, blue rubber bleb nevus syndrome 12) IBD 13) infection causing infectious colitis eg/ Escherichia , Salmonella , Histoplasma , and Cytomegalovirus colitis 14) colorectal cancer or polyps 15) haemorrhoids 16) rectal ulcer or varices 17) anal fissures

Describe the management of dementia, with particular reference to the multi-disciplinary approach and (where appropriate) control of risk factors

1) person-centred care (work with pt & carers) 2) memory assessment for all pts with suspected dementia 3) early discussions to allow advance planning - discuss advance statements or decisions, lasting power of attorney and preferred place of care 4) valid consent for tx when possible (give info in an appropriate way) 5) carers offered individual or group psycho-education & psychological therapy, peer-support groups & training courses 6) following diagnosis give info on: s/s of dementia, course & prognosis, tx, local care & support services, support groups, sources of financial & legal advice, and advocacy, local info sources eg/ voluntary organisations 7) promote independence where possible. Strategies to cope with disabilities eg/ modifications to home & simplification of daily activities 8) pts legally obliged to inform DVLA - some can continue driving an ordinary car, subject to medical reports or assessment & annual review 9) non-pharmacological mx - cognitive stimulation, multisensory stimulation, music or art or animal therapy, massage, structured exercise 10) pts cared for in community when possible 11) factors which may exacerbate violent or aggressive behaviour: overcrowding, lack of privacy, boredom, poor communication, conflict 12) Acetylcholinesterase (AChE) inhibitors eg/ donepezil, galantamine or rivastigmine considered in mild or moderate Alzheimer's a) cholinergic S/E: N+V+D, loss of appetite, muscle cramps, tiredness & insomnia 13) N-methyl-D-aspartate (NMDA) antagonists eg/ memantine = 2nd line 14) if antidepressants avoid TCAs & anticholinergics, no antipsychotics if Alzheimer's, vascular or mixed dementias - risperidone = 1st line 15) if aggression etc pose a threat to safety & non-pharmacological measures failed, benzos or antipsychotics eg/ lorazepam, haloperidol or olanzapine IM 16) palliative & EOL care - physical, psychological, social & spiritual support. Oral nutrition encouraged. CPR unlikely to succeed in severe dementia 17) screening & prevention - pts & relatives with suspected genetic cause offered genetic counselling, focus for prevention = modification of behaviour (dec smoking, alcohol, obesity, & cerebrovascular disease risk factors eg/ HTN, hypercholesterolaemia)

Describe the pathology, presentation and management of: phimosis, paraphimosis; priapism; Peyronie's disease; carcinoma of the penis; varicocoele, hydrocoele, epididymal cyst

1) phimosis - condition in which foreskin of penis cannot be pulled back past the glans. Inc risk of inflammation of the glans (balanitis). Either - tip of foreskin is too narrow to pass over the glans penis; inner surface of foreskin is fused with glans penis; frenulum is too short to allow complete retraction of the foreskin (frenulum breve) a) s/s - penile pain (esp during erection), band of retracted foreskin tissue beneath the glans, swollen glans penis (esp with urination) b) mx - I) watch & wait to see if will resolve naturally II) use of steroid cream to help stretch the foreskin III) surgery - to partially or completely remove the foreskin eg/ dorsal slit, circumcision 2) paraphyimosis - uncommon, foreskin becomes trapped behind the glans penis, and cannot be reduced (pulled back to its normal flaccid position covering the glans). If persists for several hours or signs of a lack of blood flow should be treated as a medical emergency, as can result in gangrene. Associated with not replacing the foreskin following activity eg/ catheterisation or sex a) s/s - penile pain, band of retracted foreskin tissue beneath the glans, swollen glans penis distal to lesion, and flaccidity proximal b) mx - I) 1st line - emergency surgery if evidence of significant glans penis ischaemia, with necrosis and sloughing of the foreskin and/or glans II) 1st line - manual manipulation: to dec swelling & replace foreskin over the glans penis III) 2nd line - puncture technique: after failed attempts at reduction using minimally invasive procedures, perforation of the foreskin at multiple locations allows exudation of the oedematous fluid and reduction in swelling IV) +/- hyaluronidase V) 3rd line - surgical reduction followed by circumcision: 1st line if chronic problem 3) varicocele - abnormal enlargement of pampiniform venous plexus in scrotum (drains testicles). Incompetency of venous valves or compression of the venous drainage of testes a) s/s - visible or palpable enlarged vein - scrotal mass, aching pain within scrotum, feeling of heaviness in the testicle, atrophy of testicle(s), normally LHS s/s (due to renal vein) b) mx - I) sub-clinical or grade I varicocele: 1st line - reassurance & no tx needed II) grade II or III varicocele: 1st line - observation with serial examinations III) grade II or III varicocele: 1st line - surgery: open repair or laparoscopic repair 4) hydrocele - accumulation of fluids around a testicle, often caused by fluid secreted from a remnant piece of peritoneum wrapped around the testicle (tunica vaginalis). Primary hydroceles may develop in adulthood, particularly in elderly + hot countries, by slow accumulation of serous fluid (impaired reabsorption. Causes - trauma, tumour, infection, peritoneal dialysis. Can also be due to plugged inguinal lymphatic system - chronic infection of Wuchereria bancrofti or Brugia malay a) s/s - scrotal mass, transillumination +ve, fluctuation +ve, impulse on coughing -ve, reducibility absent, testis cannot be palpated separately, enlargement of scrotal mass following activity, variation in scrotal mass during the day (smaller in the morning than in the evening and after lying down) b) mx - I) without discomfort or infection: 1st line - observation, reassurance and scrotal support II) with discomfort or infection: 1st line - surgery or aspiration and sclerotherapy 5) epididymal cyst/spermatocele - collection of spermatic fluid within epididymis. Not associated with tumours a) s/s - usually painless, may present as a scrotal swelling 6) priapism - pathological condition of persistent (>4 hours) penile erection in the absence of sexual excitation a) s/s - hx of current systemic disease (eg/ sickle cell disease, thalassaemia, haematological dyscrasias etc), hx of vasoactive medication or drug use (eg/ for erectile dysfunction, antihypertensive, antipsychotics, antidepressants, alcohol and cocaine), prolonged erection of >4 hours' duration, painful rigid penis b) mx of ischaemic priapism - I) duration <4 hours: 1st line - observation II) duration <4 hours: 1st line - aspiration ± irrigation III) + intracavernosal injection of sympathomimetic agent: phenylephrine injection IV) duration >4 hours: 1st line - aspiration ± irrigation V) + intracavernosal injection of sympathomimetic agent VI) duration >4 hours: 2nd line - penile shunt surgery VII) duration >4 hours: 3rd line - penile prosthesis c) mx of non-ischaemic priapism - I) 1st line - observation II) +/- counselling and cavernosal artery embolisation: if pt requests tx III) +/- surgery: last resort 7) Peyronie's disease - scar tissue, called plaque, forms inside the penis. It can result in a bent, rather than straight, erect penis a) s/s - when penis is soft can't see a problem, if severe the hardened plaque hampers flexibility, causing pain and forcing the penis to bend or arc when erect, in most pain eases over time, but the bend in the penis can worsen b) mx - I) 1st line - observation: can improve with time, may wait up to 2 yrs II) 2nd line - medication: pentoxifylline or potassium para-aminobenzoate III) 3rd line - surgery: usually only for men who can't have sex because of their Peyronie's disease 8) carcinoma of the penis - rare type of cancer on the skin of the penis or within the penis. It most commonly affects men over the age of 50. Several types of penile cancer, depending on the type of cell the cancer developed from eg/ squamous cell (90%), carcinoma in situ (CIS), adenocarcinoma, melanoma of the penis a) s/s - a growth or sore on the penis that doesn't heal within 4 weeks, bleeding from the penis or from under the foreskin, foul-smelling discharge, phimosis, change in the colour of the skin of the penis or foreskin, rash on the penis b) mx - depends on grade and stage and tyoe of cancer. Examples include: I) chemotherapy cream II) surgery to remove the affected area of skin III) chemotherapy +/- radiotherapy IV) +/- penile reconstructive surgery V) +/- LN dissection and removal

Describe and classify tremor

1) postural tremor - occurs when a person maintains a position against gravity, such as holding the arms outstretched 2) resting tremor - occurs when the muscle is relaxed, such as when the hands are resting on the lap 3) intention tremor (cerebellar tremor) - dyskinetic disorder characterized by a broad, coarse tremor, amplitude inc as an extremity approaches the endpoint of deliberate and visually guided movement 4) flapping tremor (asterixis/liver disease) - tremor of the hand when the wrist is extended

Describe the laboratory tests to assess the clotting system and recognise and interpret patterns of abnormality

1) prothrombin time - thromboplastin added to test extrinsic system, expressed as ratio (INR) a) tests abnormalities in factors - I, II, V, VII, X b) prolonged by - warfarin, vitK + factor deficiency, liver disease, DIC 2) activated partial thromboplastin time - kaolin added to test intrinsic system a) tests abnormalities in factors - I, II, V, VIII-XII b) prolonged by - heparin treatment, DIC, liver disease, factor deficiency (Haemophilia, vWF defc), vit K deficiency 3) thrombin time - thrombin added to plasma to convert fibrinogen to fibrin (final common pathway) a) prolonged by - heparin treatment, DIC, dysfibrinogenaemia 4) D-dimers - fibrin degradation product (fibrinolysis) a) occurs during DIC, DVT, PE, inflammation eg/ malignancy, infection 5) bleeding time - tests haemostasis, rarely done

Describe the morphology and pathological consequences of pyelonephritis, interstitial nephritis, polycystic kidney disease, hypertensive renal damage and obstructive uropathy to the kidney

1) pyelonephritis - a) in most, renal damage occurs slowly over time in response to a chronic inflammatory process or infections. This results in thinning of the renal cortex and deep, segmental, coarse cortical scarring. Club-shaped deformity of the renal calyces occurs as the papilla retract into the scar. Can have multiple scars in one or both kidneys. Most scars develop in the upper and lower poles because of the frequency of reflux in these sites. Parenchyma within the scarred areas often contains atrophic tubules with no glomeruli b) obstruction predisposes the kidney to infection, and chronic obstruction contributes to parenchymal atrophy c) vesicoureteral reflux is the most common mechanism of renal scarring in chronic pyelonephritis. Renal changes begin early in childhood from chronic UTI superimposed on congenital vesicoureteral reflux and intra-renal reflux. Scarring and atrophy lead to loss of tubular function, especially concentrating ability 2) interstitial nephritis - a) exact pathogenesis is unknown. Renal biopsy reveals an inflammatory infiltrate with variable numbers of eosinophils, lymphocytes, and plasma cells. If nephrotic syndrome is present, the pattern is usually minimal change disease b) may have an allergic-hypersensitivity mechanism 3) polycystic kidney disease - a) renal cysts develop and grow over time, leading to compression of the normal renal architecture and intrarenal vasculature, inc renal size, interstitial fibrosis and tubular atrophy, and progressive renal impairment. Cysts develop from cells in the tubular portion of the nephron and collecting ducts. 4) hypertensive renal damage - a) vascular damage (due to chronic hypertension, arteritis, coagulopathy) inc permeability of small vessels to fibrinogen and other plasma proteins, causing endothelial injury and platelet deposition, which causes fibrinoid necrosis. Kidneys become ischemic, which stimulates renin-angiotensin system to produce angiotensin II, which causes renal vasoconstriction, as well as inc aldosterone secretion and salt retention, which elevate BP further b) fibrinoid necrosis of arterioles, hyperplastic arteriolitis (onion skinning) due to concentric layering of collagen, may see necrotizing glomerulitis, wrinkling and collapse of capillary walls and small crescents 5) obstructive uropathy to the kidney - a) causes back pressure on the kidney by preventing urinary flow, causing dec renal blood flow, dec glomerular filtration rate, and up-regulation of the renin-angiotensin system. This causes atrophy and apoptosis of the renal tubules and interstitial fibrosis with infiltration of the interstitial spaces by macrophages, leading to dec re-absorption of solutes and water, inability to concentrate the urine, and impaired excretion of hydrogen and potassium b) if left untreated, obstructive nephropathy can cause irreversible renal damage. Obstruction can ultimately cause tubulointerstitial fibrosis, tubular atrophy, and interstitial inflammation

Outline the antimalarial spectra and modes of action of commonly used anti-microbial agents

1) quinine and related agents - alkaloid accumulated in the food vacuoles of Plasmodium species & inhibits the hemozoin biocrystallization, facilitating an aggregation of cytotoxic heme 2) chloroquine - 4-aminoquinolone compound. It is believed to reach high concentrations in the vacuoles of the parasite, which, due to its alkaline nature, raises the internal pH. It controls the conversion of toxic heme to hemozoin by inhibiting the biocrystallization of hemozoin, thus poisoning the parasite through excess levels of toxicity 3) pyrimethamine - inhibits dihydrofolate reductase in the parasite, preventing the biosynthesis of purines and pyrimidines, & halting DNA replication, cell division and reproduction 4) proguanil - biguanide; a synthetic derivative of pyrimidine. Converted to the active metabolite cycloguanil which inhibits the malarial dihydrofolate reductase enzyme NB/ atovaquone is available in combination with proguanil (Malarone) 5) sulphonamides - inhibitors of the enzyme dihydropteroate synthetase in the tetrahydrofolate synthesis pathway of malaria parasites. Compete with PABA to block its conversion to dihydrofolic acid 6) mefloquine - forms toxic heme complexes that damage parasitic food vacuoles 7) primaquine - 8-aminoquinolone acts on gametocytes & merozoites in the bloodstream & on hypnozoites by blocking oxidative metabolism in Plasmodia 8) artemisinin - acts primarily on the trophozite phase, preventing progression of the disease 9) halofantrine - chemically related to Quinine and acts as a blood schizonticide effective against all Plasmodium parasites. Cytotoxic complexes formed with ferritoporphyrin XI that cause plasmodial membrane damage 10) doxycycline - tetracycline compound that is bacteriostatic & acts to inhibit the process of protein synthesis by binding to the 30S ribosomal subunit thus preventing the 50s and 30s units from bonding 11) clindamycin - derivative of lincomycin, with a slow action against blood schizonticides

Define hernia and the descriptive terms reducible, irreducible, obstructed, strangulated and sliding

1) reducible - bowel & hernia sac can be pushed back into place by surgery 2) irreducible - bowel that can't be pushed back into right place 3) incarcerated - contents of hernia sac are stuck inside by adhesions - still receives blood supply, but lumen is occluded, denying contents' passage - s/s of obstruction 4) strangulated - hernia loses blood supply = ischaemia, pt becomes toxic & requires urgent surgery 5) sliding - contain a partially extraperitoneal structure (eg/ caecum on R, sigmoid on L), sac doesn't completely surround contents (part of wall of hernia sac formed by another viscus, ie the bladder or colon. characterised by mvmt of GOJ through diaphragmatic orifice)

Discuss the prognosis of pancreatic neoplasms with regard to histology

1) resectable disease (stage I, II) - a) at time of diagnosis, 15% to 20% pts have resectable disease b) median survival = 15-19 months c) 5-year survival rate of about 20% 2) unresectable locally advanced disease (stage III) - although chemo +/- radio may give a modest improvement in survival and palliation, it rarely controls the cancer - within months of finishing tx pts frequently have evidence of local tumour progression or new metastatic disease 3) metastatic disease (stage IV) - a) majority of patients (~50%) b) median survival of 3-6 months

Discuss the diagnosis of bladder infection outlining the importance of confirming significant bacteriuria (>100,000 organisms/ml) and the importance of white cells in the urine

1) s/s + physical exam 2) urine analysis - MSU to determine whether bacteria, blood or pus in urine. High WCC 3) cystoscopy - allows visualisation of urinary tract for signs of disease, can take biopsies 4) imaging eg/ X-ray or US may help determine cause eg/ tumor or structural abnormality NB/ UTI = presence of bacteriuria >100,000 organisms/ml. Polyclonal samples suggest contamination. Presence of inflammatory cells differentiates between asymptomatic bacturia and UTI

Outline the clinical presentation of bacterial meningitis and describe the appearance of the typical rash of meningococcal septicaemia

1) s/s - 2) rash - petechial, non-blotchy rash that is non-blanching (glass test) & can occur anywhere on body

Define the Eisenmenger syndrome including the clinical features and underlying pathophysiology

1) s/s - a) cyanosis b) inc RCC, iron deficiency c) clubbing d) syncope e) heart failure & arrhythmia f) bleeding disorders g) haemoptysis h) infections (endocarditis and pneumonia) i) kidney problems (inc uric acid = gout) j) gallstones 2) pathophysiology (presents later in life) - a) if an anatomic defect (hole or breach) exists between 2 sides of heart, a shunt will occur, so blood flows from LHS to RHS b) L to R shunt = abnormally high blood flow and pressure directed to R heart circulation, leading to maladaptive changes = pulmonary HTN - damage to delicate pulmonary capillaries so replaced with scar tissue which doesn't allow oxygen transfer & less flexible & less compliant c) weakened heart must pump harder to continue supplying lungs, leading to damage of more capillaries, irreversible damage d) eventually inc pulmonary pressures = myocardium of RHS hypertrophies (RVH), onset of Eisenmenger's syndrome when RVH causes right heart pressures > left heart = reversal of blood flow through shunt (R to L) so deoxygenated blood returns from body & bypasses lungs through reversed shunt to systemic circulation = cyanosis and resultant organ damage e) defect (now R to L) = dec O2 sats in arterial blood as mixing of oxygenated blood returning from lungs with deoxygenated blood from systemic circulation f) dec sats sensed by kidneys = compensatory inc in erythropoietin & inc RBC to inc O2 delivery. As bone marrow inc erythropoiesis, systemic reticulocyte count & risk for hyperviscosity syndrome inc. Reticulocytes are less efficient at carrying oxygen as mature red cells, and less deformable, causing impaired transit through capillary beds = death of pulmonary capillary beds g) pt with Eisenmenger's syndrome is paradoxically subject to possibility of both uncontrolled bleeding due to damaged capillaries and high pressure, as well as spontaneous clots due to hyperviscosity and stasis of blood.

Describe symptoms and signs of perianal haematoma and outline management

1) s/s - a) looks like a blue bruise under the skin or a dark-purple collection of blood near the anus b) may feel a small lump, ranging in size from about a small raisin to a tennis ball c) may have bubbling or bulging skin near the anus d) mild to severe pain, depending on the size e) bloody stools 2) mx - a) most perianal hematomas resolve on their own within 1/52 b) to dec pain - use a cool compress on the site, take a sitz bath 2x/day, sitting on a donut pillow to relieve pressure, adding more fiber to diet, avoid strenuous activity c) depending on size may need draining d) may need removal of blood clot

Discuss the clinical presentation, diagnosis and management of Guillain-Barre syndrome

1) s/s - a) risk factors - preceding viral or bacterial infection, hep E virus b) muscle weakness - I) progressive symmetrical muscle weakness usually lower extremities before upper extremities and proximal muscles before distal muscles II) + paraesthesias in the feet and hands - paralysis is typically flaccid with areflexia (ankle jerk, knee jerk) and progresses acutely with 50% reaching a lowest point within 1 week and 98% by 4 weeks III) weakness evolving for >4 to 8 weeks is more consistent with chronic inflammatory demyelinating polyradiculoneuropathy c) respiratory distress - dyspnoea on exertion and SOB, can need mechanical ventilation d) speech problems - facial weakness and oropharyngeal weakness creates slurring of speech e) back/leg pain f) bulbar dysfunction causing oropharyngeal weakness - can affect swallow g) others - extra-ocular muscle weakness, facial droop, diplopia, dysarthria, dysphagia, dysautonomia (sinus tachycardia, HTN, and postural hypotension, urinary retention and ileus) 2) ix - a) nerve conduction studies - slowing of nerve conduction velocities b) lumbar puncture (LP) - elevated cerebrospinal fluid (CSF) protein, normal/slightly high lymphocytes (<50 cells/mm³) c) LFTs - inc AST & ALT; bilirubin may be transiently inc but rarely high enough to cause jaundice d) spirometry - may show reduced vital capacity, maximal inspiratory pressure, or maximal expiratory pressure e) antiganglioside antibody - presence of subtype-specific antiganglioside antibodies can differentiate between subtypes and may be useful when the diagnosis remains unclear despite clinical examination, CSF analysis, and electrodiagnostic testsInvestigations to consider f) consider - I) serology, stool culture, HIV antibodies, Borrelia serology (Lyme disease), CSF meningococcal PCR, CSF cytology, CXR (sarcoidosis) for aetiology II) spinal MRI - may help identify GBS by showing enhancement of cauda equina nerve roots with gadolinium 3) mx (ambulatory within 2 weeks of onset or non-ambulatory within 4 weeks of onset) - a) 1st line - plasma exchange or IVIG: both shown to be equally efficacious, plasma exchange is given through a central venous catheter b) + supportive treatment - I) pulse & BP monitored until they are off ventilator support and have begun to recover II) subcut heparin or enoxaparin and support stockings to prevent DVT if non-ambulatory III) intubation and ventilation as needed IV) gabapentin or carbamazepine for acute pain, TCAs, tramadol, gabapentin, carbamazepine, or mexiletine for long-term management of neuropathic pain V) hypotension - fluid boluses VI) HTN - labetalol, esmolol, or nitroprusside c) rehabilitation - individual programme of rehabilitation in the acute phase

Define screening and distinguish between mass proactive screening and opportunistic case finding

1) screening - process of detecting disease early in apparently well individuals, with the intention of intervening to halt its progression 2) mass proactive screening - involves screening a large population (eg/ CXR for TB) 3) opportunistic case finding - form of screening restricted to patients who consult a health practitioner for some other purpose (eg/ GP may take BP when you come for your 'flu shot)

Outline the clinical features and laboratory diagnosis of sickle cell anaemia

1) screening - a) neonatal screening programmes identify children with sickle cell disease before they present with potentially fatal sepsis - heel prick blood spots collected 3-10 days after birth b) preconceptual testing for haemoglobinopathies in at-risk groups c) pre-operative screening for sickle cell disease carried out in pts from ethnic groups in which there is a significant prevalence of the condition d) diagnosed antenatally through amniocentesis, CV sampling & fetal blood 2) diagnosis - s/s begin between 3-6 months of age when HbF levels falling a) anaemia, jaundice, pallor, lethargy, dactylitis, growth restriction and general weakness b) inc susceptibility to infections eg/ pneumococcus c) splenomegaly may be present d) delayed puberty 3) ix - a) DNA-based assays - replacement of both B Hg subunits with HbS is diagnostic for sickle cell anaemia (HbSS); presence of 1 normal beta haemoglobin subunit and 1 HbS is diagnostic for sickle cell trait b) haemoglobin isoelectric focusing (Hb IEF) - used in newborn screening, shows percentage of HbS in a blood sample c) high-performance liquid chromatography (HPLC) - used +/- DNA assay after abnormal Hb identified, like DNA assays allows distinction between heterozygotes and homozygotes d) +/- cellulose acetate electrophoresis, haemoglobin solubility testing, FBC & reticulocyte count (both done in older pts) e) peripheral blood smear - sickle-shaped cells, Howell-Jolly bodies, nucleated RBC, and cell fragments can be seen on the blood smear. Oat cells and targets can be seen in HbSC disease. Target cells can be seen in thalassaemias, liver disease, and haemoglobin C f) iron studies - distinguishes haemolytic anaemia from iron-deficiency anaemia g) +/- pulse oximetry (dec can mean onset of acute chest syndrome), plain x-rays of long bones (may have bone infarctions), CXR (if respiratory symptoms, fever, or chest pain as may mean acute chest syndrome)

Calculate from suitable data the sensitivity, specificity, predictive value and likelihood ratio of a test

1) sensitivity -refers to the test's ability to correctly detect ill patients who have the condition eg/ proportion of people who test +ve for the disease among those who have the disease: a) no of true positives/ number of true positives + number of false negatives = no of true positives/total no of all sick individuals in a population b) -ve result in a test with high sensitivity is useful for ruling out disease; +ve result in a test with high sensitivity is not useful for ruling in disease 2) specificity -relates to the test's ability to correctly reject healthy patients without a condition eg/ proportion of healthy patients known not to have the disease, who will test negative for it: a) no of true negatives/ no of true negatives + no of false positives = no of true negatives/ no of well individuals in population b) +ve result in a test with high specificity is useful for ruling in disease 3) predictive value - a) positive predictive value - probability that subjects with a positive screening test truly have the disease eg/ 1,115 subjects whose screening test was positive, but only 132 of these actually had the disease: so if a subject's screening test was positive, the probability of disease was 132/1,115 = 11.8%; therefore, among those who had a positive screening test, the probability of disease was 11.8% b) negative predictive value - probability that subjects with a negative screening test truly don't have the disease eg/ 63,895 subjects whose screening test was negative, and 63,650 of these were, in fact, free of disease. Consequently, the negative predictive value of the test was 63,650/63,695 = 99.9%; therefore probability of being disease-free was 99.9% 4) likelihood ratio - 2 versions of the likelihood ratio exist, one for positive and one for negative test results a) positive likelihood ratio is calculated as: sensitivity/1-specificity, or "the probability of a person who has the disease testing positive divided by the probability of a person who does not have the disease testing positive." negative likelihood ratio is calculated as: 1-sensitivity/specificity, or "the probability of a person who has the disease testing negative divided by the probability of a person who does not have the disease testing negative."

List the contraindications to a lumbar puncture

1) skin infection near site of LP 2) suspicion of inc intracranial pressure due to a cerebral mass - brain tumor, cerebral hemorrhage, cavernous sinus thrombosis, brain abscesses, epidural or subdural hematomas, pts at inc risk of deteriorating neurologically with LP (with LP a low-pressure shunt is formed at the site of LP where CSF can escape, as CSF pressure drops in the spinal column, CSF and brain mass may then shift towards the LP site = herniation) 3) uncorrected coagulopathy 4) acute spinal cord trauma

Differentiate between mechanical small bowel obstruction and paralytic ileus

1) small bowel obstruction - mechanical disruption in the patency of the GI tract a) s/s - emesis (that may be bilious), absolute constipation, and abdominal pain 2) paralytic ileus - slowing of GI motility accompanied by distention, in the absence of a mechanical intestinal obstruction a) diagnosis of exclusion after bowel obstruction has been ruled out b) usually occurs in response to physiological stress, including surgery, sepsis, metabolic derangements, and Gl diseases c) s/s - N&V, abdominal distension, no features of mechanical obstruction or peritoneal inflammation (eg/ abdominal hernia, peritoneal signs)

Describe features of the clinical presentation of a headache that might raise concern about a more sinister pathology, listing, in each case the relevant differentials and appropriate investigations including headache of Subarachnoid haemorrhage, meningitis/Encephalitis, raised Intracranial Pressure

1) subarachnoid haemorrhage - a) s/s - I) risk factors eg/ HTN, smoking, fmx, autosomal dominant polycystic kidney disease (ADPKD), >50, black, female II) headache - described as sudden, severe or 'worst ever' III) photophobia IV) LOC +/- altered mental status V) N&V b) DD - arterial dissection, cerebral and cervical arteriovenous malformation (AVM), dural arteriovenous fistulae (AVF), vasculitis, cardiac myxoma, septic (mycotic) aneurysm, cocaine, anticoagulants, sickle cell c) ix - I) CT head - hyperdense areas in the basal cisterns, major fissures, and sulci II) FBC - leukocytosis III) clotting profile - elevated INR, prolonged PTT IV) serum electrolytes - electrolyte abnormalities V) troponin I - inc VI) ECG - may have arrhythmias, prolonged QT, ST segment, or T wave abnormalities VII) consider - LP (if CT is unrevealing, CSF opening pressure should be measured and CSF should be visually inspected for gross blood and xanthochromia - measure after 12hrs), digital subtraction angiography or computed tomography angiography or magnetic resonance angiography (may show aneurysm) 2) meningitis (bacterial) - a) s/s - I) risk factors eg/ age ≤5 years or ≥65 years, crowding, exposure to pathogens, non-immunised infants, immunodeficiency, asplenia or hyposplenia, cranial anatomical defects, ventriculoperitoneal shunt, cancer, HIV/AIDS, cochlear implants, and sickle cell disease II) headache III) neck stiffness IV) fever V) altered mental status, confusion VI) photophobia VII) vomiting VIII) seizures IX) infants - hypothermia, irritability, lethargy, poor feeding, apnoea b) DD - encephalitis, meningitis - viral, drug-induced, TB, fungal c) ix - I) CSF cell count and differential - polymorphonuclear pleocytosis II) CSF protein - inc III) CSF glucose - dec IV) CSF Gram stain + culture + antigen detection in CSF (eg/ for Neisseria meningitidis) + PCR V) blood culture VI) FBC and differential - leukocytosis, anaemia, thrombocytopenia VII) CRP - inc VIII) electrolytes, calcium (Ca), magnesium (Mg), glucose - acidosis, low Ca/Mg, or hyper/hypoglycaemia IX) coagulation profile (prothrombin time, INR, activated PTT, fibrinogen, fibrin degradation products) - may have evidence of DIC (prolonged thrombin time, elevated fibrin degradation products or D-dimer, low fibrinogen or antithrombin levels) X) CT head - normal or raised intracranial pressure or intracranial lesion if other pathologies present XI) consider - MRI head (normal or intracranial lesion if other pathologies present), serum procalcitonin (normal or inc) 3) encephalitis - a) s/s - I) risk factors eg/ age <1 or > 65 years, immunodeficiency, viral infections, body fluid exposure, organ transplantation, animal or insect bites, location, and season II) fever III) rash IV) altered mental state V) focal neurological deficit eg/ aphasia, hemianopia, hemiparesis, ataxia, brisk tendon reflexes, Babinski's sign, cranial nerve, myoclonus, paraesthesias, generalised weakness VI) meningismus - meningeal inflammation with headache, photophobia, and neck stiffness VII) others - cough, GI infections, seizures b) DD - aseptic meningitis, encephalopathy (toxic/metabolic), status epilepticus, CNS vasculitis, malignant HTN, intracranial tumours and cysts, SLE, intracranial bleed, meningitis c) ix - I) FBC - often elevated WBC II) peripheral blood smear - detection of Plasmodium falciparum and Ehrlichia III) serum electrolytes - may have hyponatraemia IV) LFTs - inc V) blood cultures + throat swab + nasopharyngeal aspirate + sputum culture VI) CXR, CT brain, MRI brain, ECG VII) CSF analysis + culture + serology + PCR - findings depend on aetiology; may have inc WCC, normal/elevated protein, normal/low glucose, normal/elevated RBC VIII) consider - urine culture, stool enteroviral culture, IgG and IgM antibodies, PCR (blood), HIV serology/RNA test, CSF biomarkers/prion protein assay, paraneoplastic antibodies (blood and CSF), whole-body CT or PET (detection of underlying cancers), magnetic resonance spectroscopy, next-generation sequencing of CSF, brain biopsy 4) raised Intracranial Pressure - a) s/s - I) headache II) N&V III) inc BP IV) decreased mental abilities, confusion V) double vision, pupils that don't respond to changes in light VI) shallow breathing VII) seizures VIII) LOC, coma IX) combination of headache + papilloedema + vomiting is generally considered indicative of raised ICP X) pressure headaches described as throbbing or bursting and are exacerbated by any factors that further increase ICP eg/ coughing, sneezing, recumbency or exertion. Classically the headache of raised ICP is worse in the morning b) DD - stroke, brain tumor, head injury. c) ix - I) visual field testing (perimetry) - most common defects are enlargement of the physiological blind spot and loss of inferonasal portions of the visual field, constriction of isopters II) optic disc photographs - Frisén grading of papilloedema III) MRI of brain with or without contrast - negative intracranial and intraorbital pathology; empty sella; flattening of the globe IV) LP at spinal L3/L4 - inc: opening pressure >250 mm H2O V) consider - magnetic resonance venogram of head

Outline the features and management of headaches disorders including Tension Headaches, medication overuse headache, migraine and cluster headaches

1) tension headache - a) s/s - I) risk factors eg/ mental tension, stress, missing meals, and fatigue II) generalised head pain - bilateral pressure-like ('dull' or 'constricting' pain) and non-throbbing pain, often frontal or occipital head pain III) normal neurological examination IV) others - tenderness: pericranial, SCM, trapezius, temporalis, lateral pterygoid, masseter muscles b) mx - I) 1st line - simple analgesics: aspirin or paracetamol, or ibuprofen, or naproxen PRN II) chronic symptoms (>7 to 9 headache days/month) - antidepressants: amitriptyline: 10 mg or doxepin: 10mg III) +/- non-drug therapies: EMG biofeedback or relaxation training or CBT IV) 2nd line - muscle relaxants: tizanidine 2) medication overuse - people who use acute pain-relief medicine >2-3x/week for 10+ days of the month can set off a cycle called 'medication-overuse headaches' (MOH) a) s/s - as each dose of medicine wears off, the pain comes back, leading pts to take even more. This overuse causes medicine to stop helping pain and actually start causing headaches b) dull constant headache which is often worse in the morning. It is present on most days. More likely if pmx or fmx of migraine. Vicious cycle and even if the medication is stopped, withdrawal symptoms are commonly reported including chronic headache c) common drugs: caffeine, ergots, paracetamol, codeine and the triptans d) mx - I) only way of treating this condition is to stop the medication either gradually or cold turkey. Occasionally have very severe withdrawal symptoms needing inpt stay and IV dihydroergotamine 3) migraine - a) s/s - I) risk factors eg/ fmx migraine; childhood motion sickness; caffeine; high altitude; female; menstruation; divorced, widowed, or separated; obesity; habitual snoring; stressful life events; overuse of headache medications; lack of sleep II) prolonged headache - lasts 4-72 hours III) nausea IV) dec ability to function V) headache worse with activity VI) sensitivity to light b) mx - ED I) 1st line - rescue therapy: metoclopramide:10-20 mg IV or prochlorperazine: 10 mg IV/IM + diphenhydramine: 50 mg IV or promethazine: 25 mg IV or sumatriptan: 6 mg subcut II) +/- high-flow oxygen III) +/- IV corticosteroid: dexamethasone: 8-16 mg IV IV) post-discharge: +/- secobarbital before bed acute I) 1st line - NSAIDs or aspirin II) +/- anti-emetics: metoclopramide: 5-10 mg PO/IM/IV III) +/- hydration IV) 2nd line - paracetamol monotherapy V) 3rd line - paracetamol/aspirin/caffeine ongoing I) 1st line - trigger avoidance and behavioural modification: regular meals, good sleep hygiene, avoid volume depletion, regular exercise, and identify and avoid specific migraine triggers II) symptoms linked to menstrual cycle: + cycle control +/- Mg +/- triptans eg/ frovatriptan or zolmitriptan III) without predominant depression, aura, hemiplegic/basilar features: +/- anticonvulsants: topiramate or valproate semisodium; +/- TCAs: amitriptyline; +/- B-blockers: propranolol or timolol 4) cluster headache - a) s/s - I) risk factors eg/ fmx, male, head injury, heavy smoking, heavy drinking II) repeated attacks of unilateral pain - rapid onset of unilateral orbital, retro-orbital, or temporal pain, or maxillary region, which peaks within minutes and lasts for ~15-180 mins. Average 4 per day, with a max of 8. Higher numbers of attacks should raise suspicion of paroxysmal hemicrania III) excruciating pain - worst pain ever experienced, can be described as boring, sharp, piercing, burning, or pulsating IV) lacrimation, rhinorrhoea, and partial Horner's syndrome V) agitation VI) N&V VII) photophobia, phonophobia VIII) migrainous aura b) mx - acute I) 1st line - subcut sumatriptan II) 1st line - high flow O2 III) 2nd line - intranasal zolmitriptan; intranasal sumatriptan or oral zolmitriptan IV) requiring attack suppression: + transitional therapy: prednisolone or dihydroergotamine; +/- greater occipital nerve block episodic/chronic I) 1st line - verapamil II) 2nd line - lithium or topiramate or gabapentin or melatonin or valproate semisodium or surgery

distinguish type 2 (ventilatory) from type 1 (respiratory) failure & understand the implications of having a high arterial pCO2

1) type 1 - PaO2 <8kpa (low), PaCO2 <6kpa (normal or low) 2) type 2 - PaO2 <8kpa (low), PaCO2 >6kpa (high) 3) high arterial PCO2 (hypercapnia) = acidosis, either due to hypoventilation or as respiratory compensation - may eventually lead to unconsciousness & death

Discuss the clinical presenting features, diagnosis and management of urethritis and the urethral syndrome. Discuss clinical implications for those causes which can be sexually transmitted

1) urethritis - a) s/s - I) risk factors eg/ 15-24, female, low socio-economic class, new or multiple sex partners, prior or current STD, inconsistent use of condoms II) urethral discharge - primary symptom of urethritis III) urethral irritation or itching IV) dysuria V) orchalgia - sensation of heaviness in the male genitals, if pain is present, it suggests epididymitis, orchitis, or both VI) absence of: epididymal tenderness and/or swelling, pelvic pain (women), pustular or petechial rash, arthritis, eye inflammation b) ix - I) gram stain of urethral discharge and/or urine sediment - presence of gram-negative diplococci diagnostic for gonorrhoeal infection II) nucleic acid amplification tests (NAAT) - can detect Neisseria gonorrhoeae or Chlamydia trachomatis III) culture of urethral discharge - can show Neisseria gonorrhoeae or Chlamydia trachomatis IV) KOH prep of urethral discharge - useful to rule out candidal infection if the Gram stain shows candidal hyphae V) routine - HIV test, VDRL (for syphilis) VI) consider - urinalysis and culture (shows if UTI or urethritis), wet mount of urethral discharge (may detect Trichomonas vaginalis), NAAT and culture of urine or urethral/vaginal swab for Trichomonas vaginalis, NAAT for non-gonococcal, non-chlamydial organisms c) mx - I) initial Gram stain suggestive of gonorrhoea - dual abx: ceftriaxone IM + azithromycin II) initial Gram stain not suggestive of gonorrhoea (?chlamydia) - empirical abx: azithromycin or doxycycline; if culture or NAAT result subsequently confirms gonorrhoea: + cephalosporin 2) urethral syndrome - a) s/s - I) lower abdominal pain II) a feeling of pressure in the abdomen III) a sense of urgency to urinate IV) more frequent urination V) trouble urinating VI) pain during urination VII) pain during sex VIII) haematruia IX) men - swelling of the testicles, pain while ejaculating, blood in the semen, discharge from the penis X) women - can also cause discomfort in the vulvar area b) ix - I) diagnosis is usually made when more common causes of the symptoms (infection) are ruled out II) urinalysis & culture III) bloods IV) US pelvis V) +/- cystoscopy c) mx - I) lifestyle changes - stop using products or doing activities that can irritate your urethra eg/ caffeine, using scented soaps, long bike rides II) meds - abx (infection), anaesthetics (eg/ lidocaine, dec local pain), antispasmodics (eg.hyoscyamine, oxybutynin), antidepressants (eg/ amitriptyline and nortriptyline - for chronic pain), alpha-blockers (eg/ doxazosin and prazosin - improve blood flow) III) surgery - to dilate urethra. Surgery is done only if the symptoms are thought to be due to constriction of the urethra. Constriction can occur due to injury, inflammation, and scar tissue

Describe the physical examination of patients with perianal infections

1) usually have normal vital signs on initial evaluation 2) physical exam - small, erythematous, well-defined, subcut mass near anal orifice 3) DRE - fluctuant, indurated mass may be encountered 4) if there is an abscess, a mass may be palpated 5) anorectal abscesses = common cause of admission to hospital, 2-3x more common in men, particularly in MSM engaging in penetrating sex. May by first manifestation of IBD (or TB). Present with painful, tender swellings and discharge. Treatment is surgical

Indicate the sites for central venous access

1) veins in the neck - IJV 2) chest - subclavian vein or axillary vein 3) groin - femoral vein 4) veins in the arms - PICC line, or peripherally inserted central catheters

Describe relevant clinical features of chronic stable angina pectoris

1) xanthelasma - cholesterol deposits around eyes 2) tendon xanthoma - cholesterol deposits in hands/skin 3) hypertension 4) anaemia 5) hyperthyroidism 6) aortic stenosis - ejection systolic murmur radiating into neck 7) corneal arcus may be present

List the electrolyte composition of Ringers lactate

130 mEq/L Na 109 mEq/L /Cl 28 mEq/L /lactate 4 mEq/L K+ 1.5mmol/L Ca NB/ lactate metabolised by liver into bicarbonate, which may cure acidosis. pH of 6.5 despite alkalising property

Contrast the effects of each category of shock on the: heart, kidney, brain, lung, gut, immune system

4 stages of shock - continuous condition so no sudden transition from one stage to the next 1) initial - hypoperfusion causes hypoxia. Lack of oxygen so cells perform lactic acid fermentation. Pyruvate converted to lactate by lactate dehydrogenase and hence lactate accumulates 2) compensatory - body employs physiological mechanisms, including neural, hormonal and bio-chemical mechanisms to reverse the condition a) acidosis = person will begin to hyperventilate b) baroreceptors in arteries detect resulting hypotension = release of A + NA, NA = vasoconstriction with a mild inc in HR, A = inc HR + small effect on vascular tone; combined effect results in inc BP c) RAAS activated - ADH released to conserve fluid via kidneys. Vasoconstriction of kidneys, GI tract, and other organs to divert blood to heart, lungs and brain. Lack of blood to renal system causes low urine production 3) progressive - if cause not successfully treated, shock will proceed & compensatory mechanisms begin to fail a) due to dec perfusion of cells, Na+ build up within while K+ leak out. Arteriolar SM + precapillary sphincters relax so blood remains in capillaries b) hydrostatic pressure inc and, combined with histamine release, leads to leakage of fluid and protein into the surrounding tissues c) as this fluid is lost, the blood concn + viscosity inc, causing sludging of micro-circulation d) prolonged vasoconstriction = vital organs compromised due to dec perfusion e) if bowel sufficiently ischemic, bacteria may enter blood stream = endotoxic shock 4) refractory - vital organs have failed & shock can no longer be reversed. Brain damage and cell death are occurring, and death will occur imminently 5) heart - limited capacity to act anaerobically, relatively well perfused until late stages of shock at which point death occurs rapidly. Arteries may vasodilate causing hypotension 6) kidneys - deeper levels of shock are tolerable as kidney can "hibernate" - energy dependent filtration suspended to conserve oxygen. Prolonged ischaemia = necrosis of tubular epithelium 7) lungs - rarely become ischaemic, but can become a hub for immune activity. Immune - due to maintained oxygen supply, amplification of immune response occurs in arteries of lung. Immune complexes & cellular factors aggregate (DIC!) in lung = immune cell recruitment, inc vascular permeability and destruction of lung architecture, producing ARDS. 8) gut - vasculature controlled by autonomic splanchnic system - vasoconstricts early on so gut is one of the first systems to suffer from ischaemia. Ischaemic damage = breakdown in barrier function of wall gut, and bacteraemia results

Discuss the criteria used to predict the prognosis for acute pancreatitis

APACHE II ("Acute Physiology And Chronic Health Evaluation II") an integer score from 0-71 is computed based on several measurements; higher scores correspond to more severe disease and a higher risk of death point score is calculated from a patient's age and 12 routine physiological measurements: AaDO2 or PaO2 (depending on FiO2) Temperature (rectal) Mean arterial pressure pH arterial Heart rate Respiratory rate Sodium (serum) Potassium (serum) Creatinine Hematocrit White blood cell count Glasgow Coma Scale

List the CSF findings that accompany multiple sclerosis

CSF is normal in 10% to 20% of MS cases normal - glucose, protein, and cell count; oligoclonal bands and elevated CSF immunoglobulin G (IgG) and IgG synthesis rates are present in 80% of MS cases

Elicit signs of cerebellar disease

DANISH D - dysdiadochokinesia (ipsilateral to cerebellar lesion) A - ataxia (broad-based) N - nystagmus I - intention tremor S - staccato speech H - hypotonic Other s/s - heel-shin test (can't do it), past pointing, rebound phenomenon, problems turning around (fall to side of lesion), truncal ataxia (if vermis affected), pendula knee jerk Causes of cerebellar lesion - stroke, SOL (kids), trauma, cerebellitis, hypothyroidism

List the electrolyte composition of Albumin solution

Human albumin (5%) = 95% albumin Quantity per/1000ml: Sodium = 130 -160 mmol Potassium = <2 mmol N-acetyl-DL-tryptophan + Caprylic acid = 0.064 -0.096 mmoVg protein NB/ available as 4.5% or 20% solutions. Only used at consultant's express request. Extracellular volume expander, often in excess of the volume administered

Describe the stepped approach to treatment of an acute asthma attack and create a management plan for a patient presenting with acute asthma

O - O2 15L, sats >94% S - salbutamol neb 5mg (QDS) H - hydrocortisone 100mg IV or oral prednisolone 40mg (OD) I - ipratropium neb 500 micrograms (QDS) T - theophylline, Mg

List the electrolyte composition of dextrose saline (0.18%)

Sodium Chloride 0.18% & Glucose 4 % Solution for Infusion BP: Sodium Chloride = 1.80 g/l Glucose (as monohydrate) = 40.00 g/l Therefore each ml contains 1.8 mg sodium chloride and 40 mg glucose (as monohydrate) mmol/l (approx): Na+: 30mmol/L Cl-: 30mmol/L, 160kcal Nb/ crystalloids vs colloids - both are volume expanders, crystalloids (all above but albumin) are aqueous solutions of mineral salts or other water-soluble molecules, colloids (albumin) contain larger insoluble mols, such as gelatin; blood itself is a colloid. No evidence colloids are better than crystalloids in those who have had trauma, burns or surgery and as they are more expensive their use is not recommended

Discuss outlet obstruction of the bladder and list the main causes including mechanical and neurological causes

bladder outlet obstruction (BOO) occurs when there is a blockage at the base or neck of the bladder which reduces or stops the flow of urine into the urethra, which carries urine from the body. Most common in older men and often is linked to prostate problems 1) enlarged prostate/BPH —most common cause in men 2) scarring of urethra or bladder neck from injury or surgery 3) use of certain meds eg/ antihistamines, decongestants or drugs to treat overactive bladder 4) bladder stones 5) pelvic tumors (cervix, prostate, uterus, rectum) 6) cystocele (when the bladder falls into the vagina) 7) foreign objects 8) urethral spasms 9) inguinal (groin) hernia 10) bladder neck elevation 11) urethral stricture

Describe the physical examination of a patient with haemorrhoids, including proctosigmoidoscopy

in all rectal bleeding should perform: 1) abdominal exam to rule out other diseases 2) PR exam - prolapsing piles obvious, internal haemorrhoids not palpable 3) proctoscopy - identifies haemorrhoids 4) sigmoidoscopy - identify rectal pathology higher up (rectosigmoidal junction = highest point) 5) inspection of perineum may show large external haemorrhoids at 3, 7, 11 (left lateral, right posterior, right anterior) and will disclude other DDxs, but anal cancer may look similar NB/ pts over 40 presenting with a bleed need referral to specialist for imaging, similarly those with bleeding alongside lower GI symptoms

Describe the indications for tube thoracostomy and list the necessary steps in performing this procedure

insertion of a tube (chest tube) into the pleural cavity to drain air, blood, bile, pus, or other fluids 1) indications - a) pneumothorax [7] : open or closed; simple or tension [8] b) hemothorax [7] c) hemopneumothorax d) hydrothorax e) chylothorax [9] f) empyema g) pleural effusion [10] h) patients with penetrating chest wall injury who are intubated or about to be intubated i) considered for those about to undergo air transport who are at risk for pneumothorax 2) procedure - a) informed consent from pt or patient's representative except when urgent placement is required b) assemble the drainage system and connect it to the suction source - bubbles in the water chamber is a sign that the chest tube drainage device is functioning properly c) position the pt supine or at a 45° angle, elevating pt lessens the risk of diaphragm elevation and consequent misplacement of the chest tube into the abdominal space, arm on affected side should be abducted and externally rotated, simulating a position in which the palm of the hand is behind the patient's head d) identify 5th ics midaxillary line e) skin incision in between the midaxillary and anterior axillary lines over a rib that is below the intercostal level selected for chest tube insertion f) wear sterile gloves, gown, hair cover, and goggles or face shield, and apply sterile drapes to the area g) administer analgesia, aspiration of air, blood, pus, or a combination into the syringe confirms that the needle entered the pleural cavity h) make a skin incision ~4 cm long overlying the rib below the desired intercostal level of entry i) use a hemostat or a medium kelly clamp to bluntly dissect a tract in the subcut tissue, then down to intercostal muscle j) palpate the tract with a finger, palpation of the selected intercostal space and the superior margin of its inferior rib k) use a closed large kelly clamp to pass through the intercostal muscles and parietal pleura and enter into the pleural space, upon entry into the pleural space, a rush of air or fluid should occur l) the kelly clamp should be opened (while still inside the pleural space) and then withdrawn so that its jaws enlarge the dissected tract through all layers of the chest wall to allow passage of the chest tube when it is inserted m) use a sterile, gloved finger to appreciate the size of the tract and to feel for lung tissue and possible adhesions n) the proximal end of the chest tube is held with a kelly clamp that is used to guide the chest tube through the tract, release clamp and continue to advance the chest tube posteriorly and superiorly o) connect the chest tube to the drainage device p) before securing the tube with stitches, look for a respiration-related swing in the fluid level of the water seal device to confirm correct intrathoracic placement q) secure the chest tube to the skin using 0 or 1-0 silk or nylon stiches r) cover with an occlusive dressing s) strap the emerging chest tube on to the lower trunk with a "mesentry" fold of adhesive tape, to avoid kinking of the tube as it passes through the chest wall t) c-xray to ensure correct placement of the chest tube

Discuss an appropriate antibiotic regimen for treatment of bacterial meningitis

need broad spectrum empirical abx until cause found 1) 1st line - a) vancomycin: 500-750 mg IV every 6 hours + b) ceftriaxone: 2 g IV every 12 hours or cefotaxime: 2 g IV every 4 hours 2) 2nd line - a) vancomycin: 500-750 mg IV every 6 hours + b) meropenem: 1-2 g IV every 8 hours or chloramphenicol: 50-100 mg/kg/day IV NB/ if a cephalosporin cannot be administered (e.g., with an allergy), alternative antibiotics are a carbapenem (e.g., meropenem) or chloramphenicol

Explain the term paraneoplastic syndrome, and describe two paraneoplastic syndromes involving the nervous system

paraneoplastic syndrome is a syndrome (s/s) that is the consequence of cancer in the body, but unlike mass effect, is not due to the local presence of cancer cells, & mediated by humoral factors (eg/ hormones or cytokines) secreted by tumor cells or by an immune response against the tumor typical among middle-aged to older patients, & most commonly present with cancers of the lung, breast, ovaries, or lymphatic system. Following diseases manifest by means of neurological dysfunction: Lambert-Eaton myasthenic syndrome, paraneoplastic cerebellar degeneration, encephalomyelitis, limbic encephalitis, brainstem encephalitis, opsoclonus myoclonus ataxia syndrome, anti-NMDA receptor encephalitis, and polymyositis 1) Paraneoplastic Limbic Encephalitis - inflammation of brain, particularly limbic system (includes the hippocampus, amygdala, hypothalamus). This part of the brain is involved in memory, emotion and behavior, and controls reactions of fear and anger. It can also involve the emotions of sexual behavior. The hypothalamus participates in the functioning of ANS, which regulates bodily functions such as blood pressure control, heart rate, pupil reactivity, endocrine/hormone function, body temperature, food and water intake, and sleeping and wakefulness. These functions can each be adversely affected in the setting of any type of encephalitis a) s/s - mood changes, problems sleeping, and severe, short-term memory deficits, seizures or seizure-like spells b) cancers of the lung and testis and tumors of the thymus (thymoma) although other cancers can also be involved 2) Paraneoplastic Cerebellar Degeneration - a) severe problems in fine motor coordination of the arms, legs, and the muscles that control the eyes, speech and swallowing. All movements become fragmented and a tremor may develop. Due to problems controlling the movement of the eyes, patients develop dilopia +/- opscillopsia b) cancers - gynecologic cancers (eg ovarian), breast, lung, and Hodgkin's lymphoma

Outline a regimen for H. pylori eradication and discuss its implications for ulcer recurrence

significantly dec ulcer recurrence rates triple therapy for 14 days = PPI + amoxicillin/metronidazole + clarithromycin check for eradication of H pylori 1/12 after the end of therapy continuation of PPI after tx not necessary in most cases if not responding to tx can use quadruple therapy = PPi + bismuth subsalicylate + metronidazole + tetracycline

Discuss how to establish the diagnosis of the cause of cirrhosis

test for - 1) hep C antibodies 2) hep B antigen 3) total iron, TIBC, transferrin saturation, and ferritin - inc total iron, reduced TIBC, elevated transferrin saturation, and elevated ferritin in haemochromatosis 4) presence of ANA +/- antismooth muscle antibody - autoimmune hepatitis 5) antimitochondrial antibody - primary biliary cholangitis 6) serum ceruloplasmin - low in Wilson's disease 7) plasma alpha-1 antitrypsin - dec in alpha-1 antitrypsin deficiency 8) serum protein electrophoresis - used to diagnose alpha-1 antitrypsin deficiency (a-1 band not present)

Common anatomical locations of DVTs

thrombi usually form behind valve cusps or at venous branch points, most of which begin in the calf (veins of pelvis + legs). Superficial thrombophlebitis commonly affects saphenous veins; deep vein thrombosis more commonly affects the veins of calf. Axillary vein thrombosis uncommon

Describe the clinical difference between upper and lower motor neuron facial weakness

upper: 1) lower part of face is involved 2) no: Bell's phenomenon, hyperacusis, change in taste, facial wasting/atrophy 3) usually associated with hemiplegia 4) site of lesion: above facial nucleus (below = forehead involved), commonly in internal capsule lower: 1) both upper + lower part of face are involved 2) Bell's phenomenon may be present 3) hyperacusis may occur if nerve to stapedius is involved 4) taste may be affected 5) not associated with hemiplegia 6) site of lesion: in or distal to nucleus 7) may be facial wasting or atrophy

Given a patient's weight, calculate the extracellular, intra-cellular and intravascular volume

water accounts for ~60% of body weight - 2/3rds are intracellular and 1/3 is extracellular of extracellular volume, 25% is intravascular (1/12th of total fluid volume) 0.5L is "transcellular" (quantum mechanics) in women and obese patients, water may only make up 50% of body weight

List the causes of acute and chronic hypocalcaemia

with inc PO4: 1) chronic kidney disease 2) hypoparathyroidism (thyroid & parathyroid surgery, hypomagnesia, DiGeorge's) 3) pseudohypoparathyroidism (resistance to PTH) 4) acute rhabdomyolysis 5) vit D deficiency (osteomalacia/rickets, vitD resistance) 6) phosphate therapy with normal or dec PO4: 1) osteomalacia 2) acute pancreatitis 3) over-hydration 4) respiratory alkalosis (total Ca2+ normal but dec ionised Ca2+) 5) drugs - calcitonin or bisphosphonates

Outline the epidemiology of lung cancer in developed countries

~19% of all cancers, 27% cancer deaths (40,000/yr in UK), incidence inc in women, only 5% cured male:female ratio is 3:1

Hyperosmolar hyperglycaemia syndrome (HHS) - outline the metabolic pathways that underlie it and understand the common reasons for its development

) insulin-sensitive tissues take up glucose during meals - glycemic rise of ingested carbs stimulates insulin secretion - inhibits glucagon release from pancreatic islets, so ratio of plasma insulin:glucagon becomes high (favors storage of glucose as glycogen in liver & muscle + lipogenesis in adipocytes) 2) between meals, insulin secretion is not stimulated - glucagon levels rise = breakdown of glycogen in the liver and muscle and gluconeogenesis by the liver. Dec insulin:glucagon ratio favors lipolysis & formation of ketone bodies by the liver 3) pts with a preexisting lack of or resistance to insulin + physiologic stress eg/ acute illness, MI, stroke = further dec in circulating insulin 4) underlying mechanism of HHS = relative or absolute reduction in effective circulating insulin (relative insulin deficiency leads to a serum glucose >33 mmol/L (600 mg/dL) with a concomitant elevation of counterregulatory hormones (serum osmolarity >320 mOsm) = excessive urination (osmotic diuresis) = volume depletion and hemoconcentration - causes a further increase in blood glucose level Ketosis is absent because the presence of some insulin inhibits hormone-sensitive lipase mediated fat tissue breakdown 5) differentiating DKA and HHS - a) pts with HHS do not develop significant ketoacidosis (insulin in amounts sufficient to inhibit ketogenesis but insufficient to prevent hyperglycemia) b) DKA = T1DM, HHS = T2DM c) HHS = longer history (1 week) + marked dehydration + glucose >35mmol/L

Discuss the relative frequency of indirect, direct and femoral hernia in children, women, young men and elderly men. Perform a physical examination of patients with inguinal hernia and describe signs of incarceration, obstruction and strangulation

1) 'groin hernias'- a) inguinal: ~70,000 surgical interventions per year, equivalent to 0.14% of the overall population, 92% male - mostly elderly, but can be young eg/ if weight lift, 8% female b) inguinal hernia is more common than femoral hernia and other abdominal wall hernias (eg, umbilical, epigastric) with femoral hernias accounting for < 10% of groin hernias c) in general hernias are more common in men than women and in whites than non-whites d) male: female = 8:1 e) lifetime risk of developing a groin hernia is ~25% in men but < 5% in women. Women manifest groin hernias at a later age eg/ 60-79 yo for women compared with 50-69 yo for men. The peak age range at presentation for indirect hernia in women is 40 to 60 years of age 2) incarceration - occurs when can't push the hernia in, and the contents of the hernia get trapped (incarcerated) in the abdominal wall. An incarcerated hernia can become strangulated, which cuts off the blood flow to the tissue that's trapped. A strangulated hernia can be life-threatening if it isn't treated 3) s/s of strangulated hernia: N&V, fever, sudden pain that quickly intensifies, hernia bulge that turns red, purple or dark, inability to move your bowels or pass gas 4) strangulated hernia - blood supply of the hernia contents is cut off, leading to ischemia. The lumen of the intestine may be patent or not 5) obstruction - hernia in which the lumen of the herniated part of intestine is obstructed

Classify heart block and identify the ECG features of each type of heart block

1) 1st degree - long PR interval (>5 small squares/>200ms) at a constant level 2) 2nd degree - some P waves conducted, some are not: a) Mobitz type 1 (Wenckebach) - gradual prolonged PR, then non-conducted beat - QRS missed b) Mobitz type 2 (Mobitz) - no progressive prolonged PR interval, but more P waves than QRS - can be a pattern eg/ 2:1 (2 P waves for each QRS) 3) 3rd degree: complete heart block - a) no relation between P waves & QRS (can occur at same time, be wart of ventricular ectopics) b) escape rhythm - slow, regular, broad, can't comment on ST (can't say if elevation etc) c) can get AF with complete heart block - think if slow AF which is regular, wills see fibrillation of baseline d) risk of asystole

Outline the treatment methods available for bleeding oesophageal varices

1) 1st line - a) vasoactive drug - octreotide IV or terlipressin IV b) resuscitation, assessment of airway, and obtaining peripheral venous access, blood transfusions targeted at Hb of 8 g/dL, can consider volume resuscitation & transfusion of fresh frozen plasma c) endoscopic variceal ligation (EVL) performed ASAP, endoscopic sclerotherapy only considered when EVL cannot be performed d) sclerotherapy - injection of a sclerosing agent eg/ ethanolamine oleate or sodium morrhuate into the varix or adjacent tissue e) + prophylactic abx - norfloxacin or ceftriaxone 2) failed endoscopic/pharmacological therapy - a) 1st line - trans-jugular intrahepatic porto-systemic shunt (TIPS) b) +/- balloon tamponade or self-expanding metal mesh stent

Describe the treatments available to patients with venous disease

1) 1st line - graded compression stockings: for CVI-related oedema, stasis dermatitis, and small venous leg ulcers, put on first thing in the morning and removed only when pt is recumbent (usually just before going to bed). 3 classes of compression stockings: class 1 (light compression) control oedema; class 2 (medium compression) and class 3 (high compression) - for more advanced CVI. Patients with severe CVI or previous ulcers generally require lifelong graded compression stockings of at least 30 to 40 mmHg 2) + moisturiser 3) + pentoxifylline or diosmin 4) with superficial venous reflux - + endovenous ablation or saphenectmy 5) with angiomata and varicosities - +/- endovenous ablation or injection sclerotherapy 6) with perforating vein incompetence - + endovenous ablation 7) with iliac vein obstruction - +/- percutaneous iliac angioplasty and stenting 8) with deep venous reflux - +/- venous valvular reconstruction 9) with leg pain - +/- horse chestnut seed extract

Outline the treatment options for a patient with confirmed lung cancer

1) 1st line - surgery: if sufficient pulmonary reserve, lobectomy or pneumonectomy is preferred. In some patients more complex procedures such as sleeve resection or chest wall resection may be required. More limited surgery eg/ wedge resection or segmentectomy is often necessary in older patients or if comorbidities or impaired lung function, but inc rate of recurrence a) + preop & postop chemo or chemoradiation eg/ cisplatin + vinorelbine or gemcitabine or docetaxel etc b) + supportive care eg/ pulmonary rehabilitation, optimisation of medical comorbidities, endobronchial treatment, treatment of pleural and other disease, and psychological and spiritual interventions and help 2) not suitable for surgery - 1st - radiotherapy: daily for ~6 to 7 weeks a) + adjunct chemo eg/ cisplatin + pemetrexed or vinorelbine or gemcitabine etc b) + supportive care c) +/- bevacizumab

Outline the principles of management of patients with perianal infections including management of fistula in ano

1) 1st line - surgical drainage of abscess, perianal abscesses can frequently be drained in the office or A&E, perirectal abscesses should be drained in the operating room 2) + postoperative care - baths with comfortably warm water 2-3x/day post op to clean the wound until healed, also use warm water baths to cleanse after bowel movements, absorbent dressings can be used to prevent staining of underclothes if there is drainage, need 25 g dietary fibre/day and 60 to 80 ounces/day of fluid to prevent hard stools 3) +/- fistulotomy if anal fistula 4) elderly, immuno-compromised, cardiac valvular disease, diabetes, or significant associated cellulitis - a) + broad-spectrum abx with anaerobic and gram-negative coverage eg/ ampicillin/sulbactam or cefoxitin etc + metronidazole or ciprofloxacin etc, use for 24hrs after surgery b) +/- aminoglycosides eg/ gentamicin or tobramycin if cellulitis

Outline the treatment of a prolactinoma, including the use of dopamine agonists as first line therapy

1) 1st line: dopamine agonists eg/ cabergoline or bromocriptine + COC if don't want to get pregnant 2) 2nd line: trans-sphenoidal surgery - considered if don't tolerate any other meds or meds not working, long-term cure rate is 60% for microprolactinomas a) risk of anterior pituitary failure +/- diabetes insipidus b) dopamine agonist therapy may need to be continued following surgery if symptomatic hyperprolactinaemia persists 3) 3rd line: sellar radiotherapy - rare, if all other tx failed, only 30% prolactin normalisation rate a) risks - hypopituitarism, cerebrovascular accidents, optic nerve damage, cognitive deterioration, and secondary radiation-induced brain malignancies

List the symptoms and physical findings of dehydration

1) 2 early signs - thirst + dark-coloured urine 2) dizziness or light-headedness 3) headache 4) tiredness 5) dry mouth, lips, eyes, mucosal surfaces & dec skin turgor 6) passing small amounts of urine infrequently (< 3-4 times a day) - colour 7) can also lead to a loss of strength and stamina - main cause of heat exhaustion 8) chronic - can affect kidney function and inc risk of kidney stones 9) can also lead to muscle damage and constipation 10) orthostasis (postural hypotension) 11) tachycardia 12) change in consciousness

Describe the pathology of renal complications of diabetes mellitus. Outline the natural history of diabetic nephropathy emphasising the importance of blood pressure control

1) 3 major histologic changes occur in the glomeruli of persons with diabetic nephropathy a) mesangial expansion is directly induced by hyperglycaemia b) thickening of the glomerular basement membrane (GBM) occurs c) glomerular sclerosis from intraglomerular hypertension 2) key change in diabetic glomerulopathy is augmentation of extracellular matrix 3) glomeruli and kidneys are typically normal or increased in size initially, thus distinguishing diabetic nephropathy from most other forms of chronic renal insufficiency, wherein renal size is reduced (except renal amyloidosis and polycystic kidney disease) 3) patients with overt diabetic nephropathy (dipstick-positive proteinuria and decreasing GFR) generally develop systemic hypertension - this then inc progression of diabetic nephropathy 4) central obesity induces hypertension initially by increasing renal tubular reabsorption of sodium and causing a hypertensive shift of renal-pressure natriuresis through multiple mechanisms, including activation of SNS and renin-angiotensin-aldosterone system, as well as physical compression of the kidneys 5) need to control BP to prevent progression of diabetic nephropathy eg/ with ACEI or ARB

Features of 3rd, 4th and 6th nerve palsies

1) 3rd CN palsy - a) affects MR, IR, SR, LPS, pupil b) SO & LR spared c) eye points down and out d) complete ptosis e) dilated pupil (efferent defect) - apparent in bright light f) no afferent defect g) course of CN3 from EWN in midbrain till enters orbit is closely associated with PCA. Painful CN3 palsy with pupil involvement due to compression if oculomotor CN with aneurysm of PCA - urgent Ix with MRI (angiography) for confirmation due to potential risk of rupture of aneurysm. Needs surgery for aneurysm tx to prevent intracranial haemorrhage +/- stroke h) aetiology - I) brain stem: tumour, CVA, demyelination II) skull: PCA aneurysm, extradural haematoma III) cavernous sinus: carotid-cavernous fistula, tumour, inflammation (other CNs also affected) IV) orbit - trauma, inflammation, tumour V) vascular III palsy - diabetes/BP, pupil spared, self-limiting NB/ CN palsy in children always abnormal - either tumour or trauma 2) 4th CN palsy - a) SO affected b) eye unable to look down & in on affected side c) so vertical diplopia - esp looking down & in d) bilateral cases with head injuries e) head tilt test on affected side causes eyeball to look up (also happens when use MR) 3) 6th CN palsy - a) LR/abducens palsy b) inability to abduct c) eye may drift medially on affected side due to pull of MR 4) 7th CN palsy - a) pt unable to close eye & blink properly - cornea dries (exposure keratitis, infection) b) may cause exposure hence: I) test corneal sensation II) test bell's phenomenon (eyeballs roll up when close eye

Differentiate the pathophysiology, findings and treatment in common types of arterial injury

1) 5 major types of arterial injury: a) intimal injuries (e.g. flaps, disruptions, or subintimal/intramural hematomas) b) complete wall defects with pseudoaneurysms or haemorrhage c) complete transections with haemorrhage or occlusion d) arteriovenous fistulas e) spasm 2) mx - a) employ the general approach to management of a trauma patient with significant extremity injuries b) immediate surgical consult c) apply direct pressure and elevation +/- pressure bandaging - direct pressure is best method initially, use 1finger, with interposed gauze, to press directly on the bleeding vessel just proximal to the bleeding point for 10 mins d) consider applying adrenaline soaked gauze or haemostatic dressings if available e) tourniquets may be life saving eg/ inflate BP cuff proximal to the bleeding point f) relocate any dislocations, reduce and splint long bone fractures, apply a pelvic binder for pelvic fractures g) correct coagulopathy and commence haemostatic resuscitation as required h) do not clamp or tie off a vessel in a bleeding wound, unless superficial and clearly visible as can damage a nerve that often runs alongside the artery i) determine if definitive interventions are required for arterial repair j) note that bleeding points proximal to these transition points cannot be controlled by externally applied direct pressure or tourniquets and require urgent surgical intervention: femoral artery at the inguinal ligament, axillary artery as it emerges from under the clavicle k) immediate operation on the injured extremity is usually appropriate if no other life-threatening injuries if: "hard signs''; or presence of extravasation, an acute pulsatile hematoma or early pseudoaneurysm, occlusion, or an arteriovenous fistula l) specific situations: I) CTA if gun injury or multiple fractures to help localise vascular injury before surgery II) hemodynamically stable patients with hard signs with a presumed wall defect or occlusion of a named artery (i.e., doralis pedis pulse is absent, but foot is clearly well perfused) or clinical signs of an arteriovenous fistula should undergo diagnostic imaging and possible therapeutic embolization III) vasospasm can be managed with serial imaging and intra-arterial adminstration of vasodilators (e.g. paparavine, heparin and tolazoline, nitroglycerin and nifedipine IV) CTA may be required if pulses or API cannot be adequately assessed in the haemodynamically stable patient m) injuries to most major 'named' arteries requiring repair or intervention include: extravasation, pulsatile haematoma, occlusion, pseudoaneurysm, fistula formation n) techniques for repair of damaged vessels: I) direct repair — sutures, patch angioplasty, interposition graft or vein patches II) ligation — only small, distal and redundant arteries (most are repaired) III) damage control surgery using temporary intravascular shunts to allow immediate restoration of distal blood flow, with later definitive repair once the patient has been resuscitated and normal physiology has resumed IV) interventional radiology measures eg/ embolisation are also useful in certain arterial injuries o) injuries that do not usually need immediate intervention: intimal defects heal spontaneously; some arteries (profunda femoris, anterior tibial, posterior tibial, or peroneal arteries) do not require surgery but can be re-imaged at 3-5 days to check progress if occluded, or undergo embolisation if the injury involves extravasation or arteriovenous fistula

In a patient with recent trauma, outline the physical findings and diagnostic plan for suspected arterial injury together with indications for radiological investigation in the extremities

1) 50-60% of injuries occur in the femoral or popliteal arteries, 30% in the brachial artery 2) extremity arterial injuries may be the result of blunt or penetrating trauma 3) may be threatening due to exsanguination, result in multi-organ failure due to near-exsanguination or be limb threatening due to ischemia and associated injuries 4) s/s - a) hard signs - I) absent pulses II) bruit or thrill III) active or pulsatile haemorrhage IV) signs of limb ischemia/ compartment syndrome (6 Ps - pale, paralysis, paraesthesia, perishingly cold, pulseless, pain) V) pulsatile or expanding hematoma b) soft signs - I) proximity of injury to vascular structures II) major single nerve deficit (e.g. sciatic, femoral, median, ulna or radial) III) non-expanding hematoma IV) reduced pulses V) posterior knee or anterior elbow dislocation VI) hypotension or moderate blood loss at the scene 5) assessment - a) penetrating extremity injury (e.g. stab or gunshot) or severe blunt trauma (e.g. arterial injury due to associated fracture) b) cold, pale and pulseless distal extremity or a rapidly expanding hematoma suggests arterial compromise — look for 'hard' and 'soft signs' c) check arterial pressure index (API) I) API = injured extremity SBP / uninjured extremity brachial SBP (eg/ R vs L arm, using doppler and BP cuff) II) API > 0.9 is highly unlikely to have a vascular injury and may be observed/ discharged depending on the nature of any other injuries, premorbid and social factors III) API < 0.9 indicates possible vascular injury: requires further evaluation, preferably by computed tomography angiogram (CTA). Doppler ultrasound (50-100% sensitive, 95% specific) can be used as an alternative, and surgeons can perform intraoperative angiograms under fluoroscopy d) assess for haemorrhagic shock e) angiography can be performed only if the patient is hemodynamically stable f) pulses and APIs may be difficult to assess in the obese, shocked, hypothermic or those with pre-existing peripheral vascular disease

Outline the pathological classification of the common tumours of the testis and their biological behaviour. Outline the investigation and management of seminoma and teratoma of the testis

1) > 95% of testicular cancers are germ cell tumours (GCTs). Most of the remaining 5% are sex cord-gonadal stromal tumours derived from Leydig cells or Sertoli cells 2) GCTs - a) seminoma (50%) - peak incidence in pts' 30s. Testis is uniformly enlarged & looks well circumscribed and uniformly grey/white. Spread along spermatic cord through lymphatics and blood vessels. b) 15% of tumours show both seminomatous and teratomatous change c) non-seminomas (35%) eg/ mature teratomas (well differentiated - recognisable structures), teratocarcinomas (intermediately well differentiated - recognisable structures alongside malignant material), choriocarcinomas & yolk sac carcinomas (a-FP and + B-HCG secreting), and embryonal carcinomas (totally undifferentiated) - in order of increasing malignancy. Tumours are irregular in shape and show focal haemorrhage. Spread is via lymph to para-aortic nodes, and through the blood to lungs, bone and liver 3) sex cord/gonadal stromal tumours - Leydig cell tumour, Sertoli cell tumour, lymphoma. Often functional - inc levels of testosterone may result in inc secondary sexual characteristics and aggression 3) ix - a) US (colour Doppler) of testis - testicular mass b) CT scan (abdomen and pelvis) - enlarged retroperitoneal lymph nodes c) serum beta-human chorionic gonadotrophin (beta-hCG)- inc if choriocarcinoma d) serum alpha-fetoprotein (AFP) - inc in embryonal carcinoma, teratoma, yolk sac tumours, or combined tumours. e) serum lactate dehydrogenase (LDH) f) consider - CXR (may have mediastinal and lung mass suggestive of metastasis), serum placenta alkaline phosphatase (inc in advanced disease), serum gamma glutamyl transpeptidase (inc in some seminoma), MRI scan (abdomen and pelvis), CT chest, histological exam of testicular mass post-orchiectomy (confirms histology of testicular mass) 4) mx - early stage seminoma a) stage I or stage II disease - I) 1st line - surveillance post-orchiectomy II) 1st line - external beam radiation post-orchiectomy III) 1st line - carboplatin chemotherapy post-orchiectomy IV) stage I disease (locally invasive) or stage II disease (A/B): 1st line - chemo postorchiectomy (stage IIB disease): bleomycin + etoposide + cisplatin early stage non-seminoma a) stage I disease I) 1st line - surveillance post-orchiectomy II) 1st line - retroperitoneal lymph node dissection (RPLND) post-orchiectomy III) + surveillance or chemotherapy post-retroperitoneal lymph node dissection (RPLND): bleomycin + etoposide + cisplatin

After the assessment of airway, breathing, circulation, describe the assessment of a patient with head injury

1) ABC 2) GCS - if 8 or less, involve anaesthetist or critical care physician to provide appropriate airway management & assist with resuscitation 3) referral for CT head within 1hr if - a) vomited 2+ times b) GCS <13 on initial assessment in the emergency department c) GCS <15 at 2 hours after the injury at ED d) suspected open or depressed skull fracture e) any sign of basal skull fracture (haemotympanum, 'panda' eyes, cerebrospinal fluid leakage from the ear or nose, Battle's sign f) post-traumatic seizure g) focal neurological deficit 4) CT within 8hrs if: some loss of consciousness or amnesia since the injury +: a) 65+ yo b) any history of bleeding or clotting disorders c) dangerous mechanism of injury (a pedestrian or cyclist struck by a motor vehicle, ejected from a motor vehicle or a fall from a height > 1 m or 5 stairs) d) >30 minutes' retrograde amnesia of events immediately before head injury 5) consider CT spine if s/s or x-ray or mechanism of injury suggest may be involved 6) manage pain effectively as can cause inc intracranial pressure eg/ analgesia, splintage of limb fractures 7) catheterisation of a full bladder, where needed 8) discuss with a neurosurgeon if: a) new, surgically significant abnormalities on imaging b) persisting coma (GCS 8 or less) after initial resuscitation c) unexplained confusion > 4 hours d) deterioration in GCS score after admission e) progressive focal neurological signs f) seizure without full recovery g) definite or suspected penetrating injury h) CSF leak

Outline initial management and identify the patient needing urgent resuscitation and operative intervention on the basis of their clinical presentation with acute abdomen

1) ABCDE 2) treat shock 3) crossmatch/group 4) blood culture 5) Abx 6) relive pain 7) IVI (0.9% saline) 8) plan abdominal XR 9) CXR if peritonitic or >50yrs 10) ECG if >50yrs 11) oxygen 12) insert catheter for fluid balance 13) NG tube for drain/feeding (patient should be kept NBM) 14) history and exam should be undertaken and the necessity of theatre assessed 15) urgent resus - 16) operative intervention -

Outline the drugs used in the long-term management of CCF

1) ACEI eg/ Ramipril oral - block angiotension = vasodilation. SE = red, itchy skin rash (serious problem), 1st dose hypotension 2) ARBs eg/ Candesartan oral - block angiotension = vasodilation. SE = cough, hyperkalaemia, hypotension 3) B-blockers eg/ Bisoprolol oral - dec HR + vasodilation. SE = dizziness/lightheadedness, feeling tired, trouble sleeping 4) aldosterone antagonists eg/ Spironolactone oral - dec BP and congestion on heart. SE = can affect kidney function & inc potassium levels - important for pts who also use ACEI or ARBs 5) diuretics eg/ Furosemide IV - causes the body to rid itself of excess fluids and sodium through urination, helping relieve the heart's workload. Also decreases the build up of fluid in the lungs and other parts of the body eg/ ankles. SE = weakness, chest pain, chills

Outline the commonly used anti-epileptic drugs

1) AEDs - monotherapy if possible, started after 2nd seizure a) focal = I) carbamazepine - dizziness, fatigue, vision problems, N+V, headache, dry mouth, swollen feet or ankles, itchy skin rash, inc risk of infection II) lamotrigine - skin rash, headache, back or joint aches, N+V, diarrhoea, dizziness, lethargy, dry mouth, agitated or irritable, difficulty sleeping b) generalised = I) sodium valproate - stop, if seizure free for 2yrs, over 6 weeks - N+V, stomach pain, feeling shaky, hair loss, headache, diarrhoea, feeling sleepy, inc weight, uncontrolled muscle movements, lack of concentration, allergic reactions, problems with hearing, liver problems 2) ketogenic diet - high in fat but low in carbohydrates and protein 3) vagus nerve stimulation (VNS) and deep brain stimulation (DBS) - adjunctive therapy to dec frequency of seizures in children and young people who are refractory to anti-epileptic meds but not suitable for resective surgery 4) surgery - anteromedial temporal resection (medial temporal lobe epilepsy), corpus callosotomy (generalised epilepsy), functional hemispherectomy and multiple subpial transection

List the appropriate laboratory and X-ray tests to be employed in a patient with suspected small intestinal obstruction

1) AXR - a) small bowel obstruction = central gas shadows with valvulae conniventes that completely cross lumen + no gas in large bowel, >3cm = dilated b) large bowel = >5cm = dilated, haustra don't run whole way across large bowel 2) large bowel enema may illustrate site & cause of obstruction 3) CT or barium follow-through may illustrate site & cause in small bowel obstruction 4) PR examination 5) FBC - WCC may be inc in mesenteric ischaemia + U&E & CRP

Outline the common antibiotic regimen used to treat endocarditis and describe the indications and role of antibiotic prophylaxis in patients with pre-existing valve disease

1) Abx for 4-6 weeks - 3-5 sets of blood cultures within 60-90 mins, followed by infusion of appropriate antibiotic regimen. Initial Abx choice is empiric, determined by clinical history and physical exam findings 2) empiric antibiotic therapy chosen based on most likely infecting organisms - a) native valve endocarditis (NVE) treated with penicillin G + gentamicin (streptococci) b) history of IV drug use - treated with nafcillin + gentamicin (methicillin-sensitive staphylococci) c) emergence of methicillin-resistant Staphylococcus aureus (MRSA) + penicillin-resistant streptococci has led to a change in empiric treatment, with substitution of vancomycin in lieu of a penicillin antibiotic NB/ clinical presentation & suspicion coupled with Duke criteria to confirm diagnosis: major: +ve blood culture, endocardial involvement minor: structural predisposition, IVDU, pyrexia, embolic phenomenon, vasculitic phenomenon, suggestive cultures or Echo NB2/ most pts respond within 48 hrs of appropriate Abx - dec fever + inflammatory markers (CRP, ESR). If relief does not occur consider: PE, abscess/extensive infection, drug reaction, other nosocomial infection 3) surgery - if: heart failure, valvular obstruction, repeated/systemic emboli, fungal endocarditis, persistent bacteraemia, myocardial abscess, unstable infected prosthetic valve, certain organisms + vegetation sites, perivalvular infection, worsening disease NB/ needs to be balanced against age + co-morbidities

Discuss the treatment of a deep vein thrombosis, the methods of administering and monitoring appropriate anticoagulants. Outline the indications for primary thrombo-prophylaxis

1) Acute - a) proximal DVT of leg - anticoagulation eg/ rivaroxaban, warfarin + enoxaparin, +/- physical activity, +/- gradient stockings b) Distal DVT of leg - serial imaging of deep veins, anticoagulation eg/ rivaroxaban, warfarin + enoxaparin, + physical activity, +/- gradient stockings c) pregnant - LMWH or SC subcutaneous unfractionated heparin eg/ enoxaparin or dalteparin, + physical activity, +/- gradient stockings d) Active bleeding - IVC filter (can be removed after bleeding resolved), + physical activity, +/- anticoagulation (once bleeding stopped), +/- gradient stockings e) PE-related cardiovascular compromise - individualised, may include anticoagulation, systemic or selective thrombolysis, open embolectomy, or IVC filter NB/ for all anticoagulation - normally 3/12 of tx, if still not treated after this time consider extended anticoagulation (eg/ to 6/12) or aspirin

Outline the indications for medical or surgical treatment of valvular heart disease affecting the aortic and mitral valves

1) Aortic valve - a) Aortic valve regurgitation - blood flows backward through the aortic valve into LV each time the ventricle relaxes rather than in the normal, one-way direction from the ventricle to the aorta I) non-surgical candidate (eg/ severe comobidities) - vasodilator eg/ nifedipine, hydralazine or ACEI (enalapril) or transcatheter aortic valve implantation (TAVI) b) aortic valve stenosis - aortic valve is narrowed or obstructed, which makes it harder for the heart pump blood into the aorta I) no medical treatment shown to improve survival in patients with aortic stenosis. May consider - statins, renin-angiotensin blockers and bisphosphonates etc to target comorbid conditions such as CAD, hyperlipidaemia, hypertension, and heart failure II) if also have heart failure give vasodilators such as ACEI & diuretics c) for some people with mild aortic valve disease without symptoms, careful monitoring under a doctor's supervision may be all that's needed d) in most aortic valve disease and dysfunction progress and get worse despite medical treatment. Most aortic valve conditions are mechanical problems that cannot be adequately treated with medication alone and will eventually require surgery to reduce symptoms and the risk of complications, such as heart failure, heart attack, stroke or death due to sudden cardiac arrest 2) Mitral valve - a) mitral valve regurgitation - leaflets of the mitral valve don't close tightly, causing blood to leak backward into the left atrium eg/ due to mitral valve prolapse I) medical tx - ACEI eg/ captopril, enalapril +/- B-blockers eg/ metoprolol, atenolol +/- diuretic eg/ furosemide or indapamide b) mitral valve stenosis - leaflets become thick or stiff, and they may fuse together, so dec valve opening and blood flow through the valve I) medical tx - often no tx needed, if give diuretic eg/ furosemide c) surgery often needed as medical mx alone may not be enough

Discuss medical therapy for hyperthyroidism. Discuss the indications for surgical treatment and the risks of post-operative complications. Discuss the indications for and complications of radioactive iodine therapy

1) B-blockers eg/ propanolol for symptoms 2) anti-thyroid meds - carbimazole 20-40mg/day PO for 4 weeks, reducing according to TFTs every 1-2 months, or carbimazole + thyroxine, maintain either regime for 12-18 months 3) radioiodine (I131) - most become hypothyroid after treatment a) used both as first-line treatment and salvage therapy after failure of antithyroid medications or surgery. Intention is to ablate thyroid tissue & start thyroxine replacement therapy b) many pts hypothyroid after tx; can have release of thyroid hormones 5-20 days after administration = aggravation of hyperthyroid symptoms and, rarely, thyroid storm; can develop orbitopathy 4) thyroidectomy - a) preferred in: women planning a pregnancy in <6 months provided thyroid hormone levels are normal; symptomatic compression or large goitres; relatively low uptake of radioactive iodine; thyroid malignancy is documented or suspected; large thyroid nodules; coexisting hyperparathyroidism requiring surgery; if moderate-severe active Graves' disease b) risk of damage to recurrent laryngeal nerve & hypoparathyroidism & hypothyroidism

Describe the clinical signs present in a fracture of the base of the skull

1) Battle's sign - bruising of the mastoid process of the temporal bone 2) raccoon eyes - bruising around the eyes, i.e. "black eyes" 3) CSF rhinorrhea 4) cranial nerve palsy 5) bleeding (sometimes profuse) from the nose and ears 6) hemotympanum 7) conductive or perceptive deafness, nystagmus, vomitus 8) in 1-10% of patients, optic nerve entrapment occurs - optic nerve is compressed by the broken skull bones, causing irregularities in vision. 9) serious cases usually result in death

Outline the surgical principles and operative risks involved in the treatment of coronary artery disease and valvular disease including types of prosthetic valve and anticoagulation

1) CAD - may need surgery to unblock vessels - a) Percutaneous Coronary Intervention (angioplasty) - nonsurgical procedure that opens blocked or narrowed coronary arteries. A thin, flexible tube with a balloon or other device on the end is threaded through a blood vessel to the narrowed or blocked coronary artery. Once in place, the balloon is inflated to compress the plaque against the wall of the artery. This restores blood flow through the artery. Can put a stent in the artery to prevent blockages in the artery in the months or years after angioplasty I) Early complications - during or soon after the procedure: bleeding, bruising, haematoma, infection, false aneurysm or arteriovenous fistula, kidney damage, allergic reaction, radiation exposure, blood clot, change in heart rhythm, pericardial effusion, MI stroke, radial artery spasm II) Late complications - stent restenosis, stent thrombosis b) Coronary Artery Bypass Grafting - arteries (chest, arm) or veins (leg) are used to bypass narrowed coronary arteries (one end goes above blockage & other goes into same end vessel). CABG can improve blood flow to heart, relieve chest pain, and possibly prevent a heart attack 2) Prosthetic valves - 3main designs of mechanical valves exist: the caged ball valve, the tilting disc (single leaflet) valve, and the bileaflet valve a) Mechanical valves tend to last longer than biological valves, but they also carry a greater long-term risk of thromboembolism - need lifelong anticoagulation b) With traditional heart valve surgery use a midline sternotomy, onto bypass, heart cooled and stopped temporarily, for some patients, may be able to repair or replace a heart valve through a smaller incision c) Risks - bleeding, thromboembolism, prosthesis-patient mismatch, structural valve dysfunction, endocarditis, and hemolysis

List the common pathogens causing community-acquired and hospital-acquired pneumonia and outline predisposing factors

1) CAP - a) S. pneumoniae b) H. influenza c) M. pneumoniae d) S. aureus e) Legionella f) Chlamydia pneumoniae 2) HAP - a) gram -ve enterobacteria (E. coli, Pseudomonas, Klebsiella) b) S. aureus c) Bacteroides, Clostridia 3) predisposing factors - a) elderly b) males c) immunocompromised d) alcoholics e) smoking f) lung diseases - COPD, asthma, bronchiectasis g) PPIs h) <1 or >65 i) hospitalised and ill j) bronchial obstruction k) GORD

Outline the pathophysiological complications that may occur as a result of adult cyanotic congenital heart disease

1) CCHD is a multisystem disorder - chronic hypoxaemia and secondary erythrocytosis imply many physiologic and pathobiologic changes in all organ systems 2) cerebrovascular complications - inc risk of stroke & TIA 3) Haemorrhagic and thrombotic complications - platelets (thrombocytopenia and thrombasthenia), mural thrombi, haemoptysis 4) Renal dysfunction 5) Coronary circulation - abnormal coronary circulation, including extramural coronary arteries and the coronary microcirculation 6) Infectious disease - cyanotic pts at risk for any type of infection and have a limited cardiovascular reserve to adapt their pulmonary and systemic circulation - worry about cerebral abscess and infective endocarditis 7) Skeletal complications - gouty arthritis and hypertrophic osteoarthropathy are common skeletal complications and must be considered in the presence of atypical skeletal symptoms

Discuss the management of pleural effusion and empyema

1) CHF - a) 1st line - diuretic eg/ furosemide: 40-160 mg, or bumetanide: 0.5 to 2 mg, if refractory volume overload, non-loop diuretics such as hydrochlorothiazide or metolazone may be used in combination with loop diuretics to improve diuresis b) +/- physiotherapy - mobilisation techniques, limb exercises, deep breathing exercises, and incentive spirometry c) symptomatic large effusion - +/- therapeutic thoracocentesis - if causing dyspnoea or pain; +/- O2 2) infective - a) 1st line - empirical IV abx eg/ amoxicillin/clavulanate + metronidazole or clindamycin or vancomycin + piperacillin/tazobactam (need to cover aerobic + anaerobic) b) +/- therapeutic thoracocentesis; +/- physiotherapy c) symptomatic large effusion - +/- O2 3) empyema - + tube thoracostomy

Describe the indications for and interpretation of the following investigations: C-reactive protein, serum immunoglobulins, serum and urine electrophoresis, complement levels, autoantibodies (including autoimmune screen), total and specific IgE levels, lymphocyte phenotyping, skin prick and patch testing

1) CRP - a) indications - if considering any condition where inflammation may be present, to monitor disease progression b) interpretation - I) between 3-10 mg/L is mildly elevated and usually result from chronic conditions eg/ diabetes, HTN, lifestyle factors including tobacco smoking and being sedentary II) 10-100 mg/L is moderately elevated and usually due to more significant inflammation from an infectious or non-infectious cause III) >100 mg/L is severely elevated and almost always a sign of severe bacterial infection 2) serum immunoglobulins - a) indications - used for the evaluation of antibody (humoral) immunodeficiencies, should be ordered if a patient has symptoms suggestive of an immunoglobulin deficiency eg/ fmx of immunodeficiency, recurrent or severe or unusual bacterial infections, lack of response to abx, unusual or recurrent viral infections and/or chronic unexplained diarrhoea. Serum immunoglobulin tests may be used in the assessment of conditions associated with chronic inflammation eg/ RA, SLE, autoimmune liver disease or chronic infections eg/ hep C, HIV. Measurement of IgM in a newborn may assist in the diagnosis of a congenital or neonatal infection b) interpretation - I) low level = 'hypogammaglobulinaemia' II) IgG, IgA, and IgM levels are usually evaluated together III) abnormal test results indicate something is affecting the immune system and may suggest the need for further testing IV) abnormal Igs with an appropriate clinical history and other tests results, can indicate a disease or condition 3) serum and urine electrophoresis - a) indications - to diagnose or monitor multiple myeloma, a monoclonal gammopathy of uncertain significance (MGUS), or further investigate a discrepancy between a low albumin and a relatively high total protein, unexplained bone pain, anaemia, proteinuria, renal insufficiency, and hypercalcemia are also signs of multiple myeloma, and SPE b) interpretation - I) albumin - inc = severe dehydration; dec = malnutrition, cachexia, liver disease, nephrotic syndrome, enteropathies, severe burns II) alpha-1 - inc = inflammation, pregnancy; dec = alpha-1-antritrypsin deficiency III) alpha-2 - inc = inflammation, nephrotic syndrome, OCP, steroids, hyperthyroidism; dec = haemolysis, liver disease IV) beta - inc = hyperlipidaemia, iron-deficiency anaemia; dec = hypo-B-lipoporiteinaemia, malnutrition V) gamma - inc =polyclonal & monoclonal gammopathies, dec = agammaglobulinaemia, hypogammaglobuloinaemia 4) complement levels - a) indications - recurrent (usually bacterial) infections, unexplained swelling, or symptoms related to certain autoimmune disorders. Periodically to help monitor a known long term disease eg/ SLE, that affects the complement system b) interpretation - I) dec CH50, C3, and C4 could result from either hypercatabolism of complement proteins in diseases associated with immune complexes eg/ SLE, vasculitis, hep C-associated cryoglobulinemia, hep B, glomerulonephritis (GN), pneumococcal infection, and gram-negative sepsis or dec complement protein synthesis due to inherited complement deficiencies, severe liver disease, or malnutrition II) inc CH50, C3, and C4 may occur in the context of systemic inflammation eg/ SLE and RA, severe bacterial and viral infections, and cancer, DM, and MI. In addition, hypermetabolic states such as hyperthyroidism and pregnancy may be associated with inc CH50 5) autoantibodies (including autoimmune screen) - a) indications - autoantibody tests may be ordered as part of an investigation of chronic progressive arthritis type symptoms and/or unexplained fevers, fatigue, muscle weakness and rashes. ANA is often ordered first & is a marker of the autoimmune process. If an ANA test is positive, it is often followed up with other tests associated with arthritis and inflammation eg/ RF, ESR, CRP b) interpretation - I) IgG - inc = infection, inflammation, hyperimmunisation, IgG multiple myeloma, liver disease, rheumatic fever, systemic rheumatic disease; dec = agammaglobulinaemia, amyloidosis, leukaemia, myeloma, preeclampsia II) IgM - inc = early HIV, infectious mononucleosis, lymphoma, macroglobulinaemia, myeloma, RA; dec = agammaglobulinaemia, amyloidosis, leukaemia, myeloma III) IgA - inc = chronic infections (esp GIT), IBD, myeloma, rheumatic fever; dec = agammaglobulinaemia, hereditary IgA deficiency, myeloma or protein-losing enteropathy 6) total and specific IgE levels - a) indications - periodic or persistent skin, lung, or digestive symptoms that suggest allergies; when suspect a parasitic infection b) interpretation - inc levels: parasitic diseases, allergic bronchopulmonary aspergillosis, Churg-Strauss/polyarteritis nervosa 7) lymphocyte phenotyping - a) indications - evaluation of peripheral lymphocytosis (absolute lymphocytosis >4,000/µL), unexplained lymphadenopathy, EBV, chronic infections, TB, sarcoidosis, 8) skin prick and patch testing - a) indications -most often used to demonstrate atopy (overactive immune response to environmental factors), results from skin prick tests can be used to guide the management of patients with asthma and hay fever b) interpretation - reactions are assessed by the degree of redness and swelling and the size of the wheal produced. The wheal has a white, raised edge that surrounds the swollen red central area of any skin reaction. It usually takes about 15-20 minutes to reach a maximum size, and thereafter fades over a few hours wheal <4mm = -ve; 5-10mm = mildly sensitive; 10-15 = moderately sensitive; >15 = very sensitive

Investigation of a patient with suspected (bacterial) meningitis. Describe the normal CSF constituents and CSF dynamics

1) CSF cell count and differential - CSF include pleocytosis, with WBC count typically >1.0 x 10⁹/L and predominance of polymorphonuclear leukocytes. 2) CSF protein - inc (>0.5 g/L) 3) CSF glucose - dec (<45 mg/dL (2.5 mmol/L)) 4) CSF Gram stain & culture - positive 5) antigen detection in CSF - can reveal Neisseria meningitidis capsular polysaccharide antigen 6) blood culture - positive 7) FBC and differential - leukocytosis, anaemia, thrombocytopenia 8) CRP - inc 9) electrolytes, calcium (Ca), magnesium (Mg), glucose - acidosis, low Ca/Mg, or hyper/hypoglycaemia 10) coagulation profile (prothrombin time, INR, activated PTT, fibrinogen, fibrin degradation products) - evidence of DIC (prolonged thrombin time, elevated fibrin degradation products or D-dimer, low fibrinogen or antithrombin levels) 11) CT head - cranial CT scan should be considered before LP if focal neurological deficit, new-onset seizures, papilloedema, altered mental state, or immunocompromised state to exclude a brain abscess or generalised cerebral oedema 12) PCR of blood and CSF 13) consider - MRI head, serum procalcitonin 14) normal CSF - total volume of CSF ranges from 140 to 270 ml. CSF from the lumbar region contains 15-45 mg/dl protein and 50-80 mg/dl glucose. Protein concentration in cisternal and ventricular CSF is lower. Normal CSF contains 0-5 mononuclear cells. The CSF pressure, measured at lumbar puncture (LP), is 100-180 mm of H2O (8-15 mm Hg) with the patient lying on the side and 200-300 mm with the patient sitting up

List the complications of a fracture of the base of the skull

1) CSF leak 2) intracranial haemorrhage 3) venous sinus stenosis or thrombosis 4) meningitis 5) cranial nerve deficits - can affect hearing, facial paralysis, facial numbness, nystagmus 6) motor deficits 7) post-traumatic seizure 8) post-surgical anosmia

Explain how one may investigate a suspected subarachnoid haemorrhage within the acute setting

1) CT head - standard diagnostic test for SAH and should be ordered if SAH is suspected: shows hyperdense areas in the basal cisterns, major fissures, and sulci 2) FBC - shows leukocytosis 3) clotting profile - may show coagulopathy eg/ elevated INR, prolonged PTT 4) serum electrolytes - hyponatraemia may occur due to salt wasting. [40] 5) troponin I - inc in ~25% cases during the first 24 hours 6) ECG - 50% pts with SAH have an abnormal ECG on admission eg/ arrhythmias, prolonged QT, ST segment, or T wave abnormalities 7) consider - LP (if CT is unrevealing, shows bloody CSF (xanthochromia) after 12hrs), digital subtraction angiography (DSA) or CTA or MRA (all show aneurysms)

Describe pathological consequences of hypertension as they affect the cardiovascular, cerebrovascular and renal systems

1) CV - a) damaged & narrowed arteries - high BP can damage cells of arteries' inner lining. Fats from diet enter blood & can collect in damaged arteries. Artery walls become less elastic, limiting blood flow throughout body b) aneurysm - constant pressure of blood moving through a weakened artery can cause a section of its wall to enlarge and form a bulge (aneurysm) - can rupture & cause life-threatening internal bleeding. Most common in aorta c) damage to heart - uncontrolled high BP can cause: CAD, LVH, heart failure 2) cerebrovascular - a) TIA - brief, temporary disruption of blood supply to brain, often from atherosclerosis or a blood clot — both can arise from high BP b) stroke - uncontrolled high BP can lead to stroke by damaging & weakening brain's blood vessels - narrow, rupture or leak. High BP can also cause blood clots to form in arteries leading to the brain, blocking blood flow & potentially causing a stroke c) vascular dementia - narrowing and blockage of arteries that supply blood to brain. Can also result from strokes caused by an interruption of blood to the brain. In either case, high BP may be the culprit 3) renal - over time hypertension harms renal blood vessels: a) nephrons supplied with a dense network of blood vessels. Over time, uncontrolled high BP can cause arteries around kidneys to narrow, weaken or harden - damaged arteries can't deliver enough blood to kidney tissue b) damaged kidney arteries do not filter blood well so nephrons do not receive essential oxygen and nutrients — kidneys lose ability to filter blood and regulate the fluid, hormones, acids and salts in the body c) damaged kidneys fail to regulate BP - healthy kidneys produce aldosterone to regulate BP. Kidney damage and uncontrolled high BP contribute to a negative spiral. As more arteries become blocked and stop functioning, the kidneys eventually fail d) kidney scarring (glomerulosclerosis) - can leave kidneys unable to filter waste effectively, leading to kidney failure e) kidney artery aneurysm NB/ essential hypertension accelerates arteriosclerosis and potentiates atheroma. Causes a hyaline thickening of arteriole walls

Compare and contrast the distribution and severity of macrovascular disease in patients with and without diabetes

1) CV disease - adults with T2DM are twice as likely to die of stroke or myocardial infarction (MI) compared with those without diabetes, and they are more than 40 times more likely to die of macrovascular than to die of microvascular complications of diabetes 2) CHF - occurs in up to 10% to 15% of patients with diabetes 3) stroke - related to uncontrolled BP, glucose, and lipids. Lifetime risk is higher in women than in men with diabetes

Discuss the investigation of a unilateral pleural effusion

1) CXR- a) small effusions: blunt costophrenic angles b) larger effusions: seen as water-dense shadows with concave upper borders c) completely flat horizontal upper border implies also a pneumothorax 2) ultrasound - identifies presence of pleural fluid + guides diagnostic or therapeutic aspiration 3) diagnostic aspiration - 1 or 2 ICS below upper border of pleural effusion. pleural fluid sent to lab for: a) clinical chemistry - protein, glucose, pH, LDH, amylase b) bacteriology - microscopy & culture c) cytology d) immunology if indicated - rheumatoid factor, ANA, complement 4) pleural biopsy - Abram's needle, useful for diagnosing malignant effusions or TB if pleural fluid analysis inconclusive, parietal pleural biopsy guided by thorascope or CT (if malignancy or metastases suspected) 5) pleural tap - indicated for diagnostic purposes in all but smallest effusions. Needle attached to syringe inserted through ICS over area of dullness, & thoracocentesis performed - large amounts of fluid withdrawn slowly to relieve SOB

Describe common presenting symptoms of Crohn's disease and ulcerative collitis

1) Crohn's (usually young adults)- a) diarrhoea/urgency b) abdominal pain/mass c) weight loss/failure to thrive d) fever, malaise, anorexia e) aphthous ulcerations f) peripheral abscess/fistulae/skin tags g) anal strictures 2) ulcerative colitis (usually young adults) - a) episodic or chronic diarrhoea (+/- blood & mucus) b) crampy abdominal discomfort c) bowel frequency relates to severity d) disease has a relapsing & remitting course e) severe cases - weight loss + anaemia

Discuss the response to each surgery used in IBD

1) Crohn's - a) 50-80% need 1+ surgery in lifetime - never cures b) indications = drug failure, GI obstruction from stricture, perforation, fistulae, abscess, adhesions, obstruction c) aims - to defunction distal disease eg/ with temporary ileostomy IR resection of worst areas (risk of short bowel syndrome) d) bypass & pouch surgery not done - removal of bowel with an ileorectal anastomosis e) if rectum is involved = panproctocolectomy with ileostomy 2) UC - a) needed in 20% pts b) panproctocolectomy + terminal ileostomy c) colectomy with ileal-anal pouch d) pouchitis can be treated with Abx + immunosuppressant - leaves ileal spout (toxic, raised from skin, RIF) e) indications = perforation, massive haemorrhage, toxic dilation, failed medical therapy, disease length means cancer risk

Discuss the investigation of a patient with suspected inflammatory bowel disease

1) Crohn's - a) blood - FBC, ESR, CRP, U&E, LFT, INR, TIBC, ferritin, B12, folate (anaemia?) b) stool - MC&S + CDT to exclude C. diff, campylobacter + E. coli c) colonoscopy & rectal biopsy even if looks normal d) small bowel enema - detects ileal disease e) capsule endoscopy f) MRI - assess pelvic disease & fistulae 2) UC - a) blood - FBC, ESR, CRP, U&E, LFT, blood culture (anaemia?) b) stool - MC&S + CDT to exclude C. diff, campylobacter, E. coli, salmonella, shigella, amoebae c) AXR - no faecal shadows, mucosal thickening/islands, colonic dilatation, bowel wall oedema, rose-thorn ulcers (full thickness lesions), thumb-printing d) erect CXR - shows perforation e) Ba enema - never during attack or for diagnosis f) colonoscopy - disease extent + allows biopsy - look for inflammatory infiltrate, goblet cell depletion, glandular distortion, mucosal ulcers, crypt abscesses NB/ colonoscopy risky when v. inflamed

List the extra-colonic manifestations of inflammatory bowel disease

1) Crohn's - a) clubbing b) skin problems c) joint problems d) eye problems (conjunctivitis + uveitis) e) aphthous oral ulcers 2) UC - a) as above + b) systemic symptoms in attack - fever, malaise, anorexia, weight loss c) fatty liver + cholango-carcinoma d) nutritional deficits e) amyloidosis

Describe the medical therapy available, including the management of acute flares of colitis and the use of immunosuppressives and biological treatments. Discuss complications of Crohn's disease and ulcerative colitis and indications for surgery

1) Crohn's - a) tx - I) mildly active - observation with monitoring or budesonide or 5-ASA therapy (mesalazine); +/- mx of extra-intestinal manifestations eg/ arthropathies and cutaneous and ocular manifestations II) moderately active 1st line - budesonide and/or 5-ASA therapy, or conventional oral corticosteroids; +/- management of extra-intestinal manifestations; +/- abx (if septic complications suspected) III) moderately active 2nd line - immunomodulator therapy + oral corticosteroid taper: azathioprine or mercaptopurine or methotrexate IV) moderately active 3rd line - biological therapy ± azathioprine + oral corticosteroid taper: infliximab + azathioprine or adalimumab V) moderately active: relapse corticosteroid-dependent/-refractory - surgery may be needed if failure to respond to medical therapy or worsening symptoms VI) severely active 1st line - hospitalisation + oral or IV corticosteroids + consideration of surgery +/- immunomodulator therapy + oral corticosteroid taper (azathioprine or mercaptopurine or methotrexate) VII) severely active 2nd line - biological therapy or surgery (infliximab or adalimumab) +/- gradual tapered dose reduction of corticosteroids b) complications - I) intestinal obstruction II) intra-abdominal sepsis & immunomodulator associated sepsis III) sinuses IV) toxic megacolon V) anaemia VI) short-bowel syndrome - can lead to malabsorption VII) malignancy VIII) kidney stones & metabolic bone disease IX) methotrexate associated: hepatotoxicity, pulmonary fibrosis, myelosuppression X) extra-intestinal complications: cholelithiasis, primary sclerosing cholangitis, hepatic steatosis, liver abscess, granulomatous hepatitis, arthropathy 2) ulcerative colitis - a) tx - fulminant disease 1) 1st line - admission + intravenous corticosteroids: hydrocortisone sodium succinate or methylprednisolone sodium succinate 2) +/- IV fluids 3) +/- ciclosporin or infliximab 4) 2nd line - colectomy severe non-fulminant I) 1st line - topical (rectal) + oral mesalazine II) + oral corticosteroids: prednisolone III) 2nd line - admission + IV corticosteroids: hydrocortisone sodium succinate or methylprednisolone sodium succinate IV) +/- ciclosporin or infliximab V) 3rd line - colectomy mild-to-moderate disease I) distal colitis 1st line - topical mesalazine (rectal) II) distal colitis 2nd line - topical corticosteroids (rectal) or oral mesalazine III) distal colitis 3rd line - oral corticosteroid ± oral tacrolimus: prednisolone or budesonide ± oral tacrolimus IV) extensive colitis 1st line - oral mesalazine V) extensive colitis 2nd line - oral corticosteroid ± oral tacrolimus ongoing I) distal colitis 1st line - oral + topical mesalazine II) distal colitis 2nd line - oral beclometasone b) complications - I) toxic megacolon II) perforation III) infection IV) massive lower GI bleed V) colonic adenocarcinoma, benign stricture, dysplasia associated lesion or mass VI) inflammatory pseudopolyps primary sclerosing cholangitis

Outline the risk of colonic malignancy in inflammatory bowel disease

1) Crohn's - inc risk of colon adenocarcinoma + squamous cell carcinoma of the anus, small bowel cancers, lung cancer, and lymphoma, inc incidence of carcinoma at strictureplasty sites 2) UC - colonic adenocarcinoma develops in 3-5% of patients with UC, risk inc with duration of disease, risk inc with younger age at onset, longer duration of disease, presence of primary sclerosing cholangitis, and greater extent of colonic involvement pts with long-standing colitis have a 5-10-fold higher risk of colorectal cancer than age-matched controls

Summarise the clinical and biochemical features of Cushing's syndrome

1) Cushing's disease = pituitary gland hypersecretion of ACTH 2) Cushing's syndrome - ectopic ACTH secreting tumours (eg/ small cell carcinoma of lung or RCC kidney), adrenal adenomas, glucocorticoid or ACTH administration loss of normal feedback loop (hypothalamo-pituitary-adrenal axis & loss of cortisol secretion circadian rhythms) 3) alcohol & depression may induce pseudo-Cushing's syndrome 4) signs: a) thin skin b) proximal myopathy c) hypertension d) central obesity & wasted limbs e) moon face f) buffalo hump g) supraclavicular fat pads h) striae i) mood change - depression, lethargy, irritability j) gonadal dysfunction k) bruises l) infection-prone m) pathological fractures 5) diagnosis - inappropriate cortisol secretion - low dose DEXA (0.5mg 6 hourly) suppress adrenal tumour, high dose DEXA suppression (2mg 6 hourly) suppresses pituitary source but not ectopic ACTH output

Discuss the management principles underlying the treatment of DKA (fluid, insulin and potassium replacement) and HHS

1) DKA - a) immediate resuscitation as required b) obs - SaO2 monitor, continuous ECG monitor and blood pressure/heart rate monitor c) get IV access or insert central venous catheter d) urinary catheterisation to monitor urine output e) prophylatic LMWH & thromboembolic deterrent (TED) stockings f) in unconscious, drowsy or vomiting patients, consider passing a nasogastric tube g) IV fluids - I) isotonic saline infused at a rate of 1L (or 20 mL/kg body weight) for first hour II) electrolytes checked at least hourly (monitor K+) III) when plasma glucose reaches 11.1 mmol/L change to 5% dextrose with 0.45% NaCl at 150 to 250 mL/hour in order to avoid hypoglycaemia h) insulin - stat bolus of 0.1 units/kg, then fixed-rate IV insulin infusion of 0.1 units/per kg/hour once K+ reaches 3.3mmol/L i) potassium replacement - potassium phosphate: 20-30 mEq/L/hour IV infusion added to IVI j) tx any precipitating illness k) pts should be managed in HDU or ITU l) may also need - vasopressors (dopamine, NA) +/- sodium bicarbonate 2) HHS - a) IV fluids - 0.9% normal saline at 20mL/kg/hour or 1-2 L during first hour, once hypotension is corrected, fluid resuscitation with 0.9% normal saline at 500mL/hour for 4 hours, then 250 mL/hour for another 4 hours b) ICU admission c) tx precipitating factors (e.g., infection or causative medications) d) may need vasopressors - dopamine, NA e) correction of K+ if <3.3mmol/L f) insulin - once K+ >3.3mmol/L, 0.1 units/kg IV bolus, then 0.1 units/kg/hour IV infusion g) may need phosphate therapy

List and outline the pathology of the common causes of chronic renal failure

1) DM - raised blood sugar = rise in the level of some chemicals within the kidney which make the glomeruli more 'leaky' allowing albumin to leak into urine. Raised blood glucose may cause some proteins in the glomeruli to link together - these can trigger a localised scarring process called glomerulosclerosis (takes several years). Scarred tissue gradually replaces healthy kidney tissue so kidneys become less able to do their job of filtering the blood - may gradually progress to ESRF 2) polycystic kidney disease - autosomal dominant polycystic kidney disease = inherited condition that causes small cysts to develop in kidneys. Rarely causes any noticeable problems until cysts grow large enough to affect kidneys' functions - usually between 30-60. Then cause abdominal pain, hypertension, haematuria, UTIs, kidney stones. Kidney function will gradually deteriorate until so much is lost that kidney failure occurs 3) chronic pyelonephritis - recurrent infection leading to progressive renal scarring, which can lead to end-stage renal disease (ESRD). For example, in reflux nephropathy, intrarenal reflux of infected urine induces renal injury, causing scar formation. Scars may form in utero in pts with renal dysplasia with perfusion defects. Infection without reflux is less likely to produce injury. Dysplasia may be acquired from obstruction 4) obstructive uropathy - obstruction of urinary tract at any level results in inc intraluminal ureteral pressure. With prolonged obstruction, ureteral peristalsis is overcome and increased hydrostatic pressures are transmitted directly to the nephron tubules. As pressures in proximal tubule and Bowman space inc, glomerular filtration rate (GFR) falls. After 12-24 hours of complete obstruction, intratubular pressure dec to preobstruction levels. If complete obstruction is not relieved, a depressed GFR is maintained by decreases in renal blood flow mediated by thromboxane A2 and angiotensin II (AII). With continued obstruction, renal blood flow progressively falls, resulting in ischemia and incremental nephron loss

Discuss the macrovascular complications of diabetes

1) DM = risk factor in development of atherosclerosis 2) when macrovascular complications occur they are more severe & more widespread (esp peripheral vascular disease) 3) stroke is 2x as likely 4) MI is 3-5x as likely 5) amputation of foot for gangrene is 50x as likely

Discuss the clinical use of direct oral anticoagulants and how they differ from warfarin and heparin

1) DOACs - dabigatran, rivaroxaban, apixaban, edoxaban and betrixaban 2) vs warfarin - rapid onset action and relatively short half-lives; hence, they carry out their function more rapidly and effectively, and allow for drugs to quickly reduce their anticoagulation effects, don't need routine monitoring and dose adjustments, equally effective, less influenced by diet and medications compared to warfarin, but presently no countermeasure for most NOACs unlike in warfarin & much more expensive 3) vs heparin - similar to an oral version of LMWH but act by inhibiting factor Xa directly, unlike the heparins and fondaparinux, which work via antithrombin activation 4) use - direct factor Xa inhibitors: eg/ rivaroxaban, apixaban and edoxaban

Describe a differential diagnosis in pts presenting with pain or swelling of the lower limb and ix

1) DVT 2) superficial thrombo-phlebitis 3) chronic venous insufficiency - VVV LAPS: varicose veins, venous ulcers, venous stars, lipodermatosclerosis, atrophy blanche, pitting oedema, scars 4) cellulitis - 4 signs of inflammation: pain, heat, swelling, redness 5) acute arterial ischaemia - identified by 6 Ps: pain, pallor, parasthesia, paralysis, pulseless, cold 6) hypoproteinaemia 7) ruptured Baker's cyst 8) ix - a) compression or Doppler ultrasound - sens and spec over 90% b) contrast venography (best test) c) impedance plethysmography d) B mode venous compression NB/ diagnostic ability of these tests dec quite a lot for lesions in calf (~70%)

Outline normal venous physiology and describe the roles of superficial, deep and perforating veins and venous valves

1) Deep Venous System: a) deep primary veins drain venous blood from lower extremity b) include: common + deep femoral, external iliac, popliteal, tibial, peroneal c) located within muscle fascia - allows a high volume & pressure of blood to pass through veins. Account for ~90-95% of venous blood return to heart. Deep veins can form DVTs 2) Superficial Veins a) superficial veins drain blood from skin. Blood travels from superficial through perforator to deep veins. Superficial veins located near surface of skin, outside muscle fascia. Account for ~5-10% of venous blood return to heart b) 2 primary superficial veins: Small Saphenous Vein (SSV) + Great Saphenous Vein (GSV) c) GSV = longest vein in body, running medially from dorsal vein in foot up to common femoral vein in groin, where it empties. Where GSV empties into common femoral vein = Saphenofemoral Junction (SFJ). Typical GSV has ~7 valves throughout its entire length & is most common superficial vein to develop venous reflux d) small saphenous vein originates at back of ankle near the outer malleous bone & usually runs up back of lower leg to popliteal vein behind the knee 3) Perforator Veins: connect superficial to deep veins. Have 1-way valves to direct blood from superficial to the deep system a) include: Cockett/Boyd's/Dodd's/Hunterian Perforator b) Boyd's perforators - common sites for primary varicose veins. These veins connect GSV to posterior tibial vein. Hunterian perforators connect GSV to superficial femoral vein, common causes of medial thigh varicosities c) main points of drainage = saphenofemoral and saphenopopliteal junctions 4) Reticular Veins: connect branch veins to any of deep, superficial, or perforating veins NB/ competent valves & calf muscles prevent backflow of venous blood. Superficial venous insufficiency caused by failure of valves in superficial + perforator vessels leads to venous pooling & chronic venous hypertension = dilation of vessels & varicosities. Deep venous insufficiency occurs when deep valves fail (usually secondary to Hx of DVT), causing a backflow of blood into superficial system & swelling of veins + legs NB2/ trendelenburg test - pt goes from lying to standing, and if varicose veins fill = communicating vein leaks (or incompetence below saphenofemoral junction). If it takes releasing the fingers from pts leg to cause varicosities to fill = sapheno-femoral incompetence

Define heart failure and classify common causes. Describe the typical history and clinical examination findings, investigations and management together with morphology and histological changes in the lungs and liver

1) Definition - a) complex clinical syndrome resulting from the impaired ability of the heart to cope with the metabolic needs of the body, resulting in breathlessness, fatigue, and fluid retention b) Heart failure with preserved ejection fraction (HFpEF) - have s/s of heart failure with normal or near-normal LV function and no significant valvular abnormalities. Likely to be due to diastolic dysfunction, impaired systolic function on exercise, abnormal ventricular-arterial coupling, inflammation and endothelial dysfunction, chronotropic incompetence, altered myocardial energetics and peripheral skeletal muscle metabolism and perfusion, pulmonary hypertension, and renal insufficiency c) Also have heart failure with REDUCED ejection fraction (HFrEF) - LVEF <40%, and heart failure with mid-range ejection fraction (HFmrEF) - LVEF 41-49% with structural heart disease and/or diastolic dysfunction. 2) Common causes - a) most common causes of HFpEF - HTN, CAD, DM b) less common - cardiomyopathy (hypertrophic or restrictive), infiltrative causes (amyloid, haemochromatosis, storage and mitochondrial disease), radiation, chemotherapy, endomyocardial fibrosis, amyloid c) Common precipitating factors of heart failure exacerbation include: volume overload (e.g., in valvular heart disease), tachycardia, uncontrolled hypertension, ischaemia, arrhythmia (e.g., atrial fibrillation), systemic stressors (e.g., anaemia, fever, infection, thyrotoxicosis), NSAIDs 3) s/s - a) exertional dyspnoea b) orthopnoea +/- paroxysmal nocturnal dyspnoea c) abdominal fullness - swelling or pain in the upper abdomen from fluid retention and hepatic/intestinal congestion d) rales e) jugular venous distension f) hepatojugular reflux, congestive hepatomegaly g) peripheral oedema h) laterally displaced apical impulse 4) ix - a) serum electrolytes - hypervolaemic hyponatraemia is common in severe heart failure due to expansion of extracellular volume, K+ usually normal, but thiazide and loop diuretics may result in hypokalaemia b) renal function tests - renal failure may be the aetiology of fluid overload, or CKD may be a risk factor leading to diastolic dysfunction - so may have inc creatinine & dec eGFR c) liver enzymes - marker of passive liver congestion: inc AST & ALT d) B-natriuretic peptide (BNP)/N-terminal prohormone B-natriuretic peptide (NT-pro-BNP) - BNP >400 nanograms/L (>400 picograms/mL) may be indicative of heart failure. However, BNP does not reliably distinguish between systolic heart failure and HFpEF. Causes of elevated BNP other than heart failure include LV hypertrophy, MI, tachycardia, RV overload, hypoxaemia, renal dysfunction, sepsis, COPD, diabetes, age >70 years, and cirrhosis. BNP <100 nanograms/L (<100 picograms/mL) suggests a non-cardiac aetiology of dyspnoea e) FBC - may show anaemia (precipitating cause) f) 12-lead ECG - may reveal evidence of prior MI, conduction defects, and arrhythmias common to HFpEF, such as atrial fibrillation. Increased QRS voltage may infer the presence of left ventricular hypertrophy; low QRS voltage can suggest infiltrative cardiomyopathy. g) CXR - can show cardiomegaly, pulmonary oedema, pleural effusion h) Doppler echocardiography - normal LV systolic function (ejection fraction >50%); abnormal E/A ratio (normal E/A = 1.0-1.5), can also do tissue Doppler imaging i) others - cardiac magnetic resonance (CMR) imaging, CT angiography, radionuclide ventriculography (MUGA scan), stress testing, cardiac catheterisation and coronary angiography 5) mx of acute decompensated HFpEF - a) 1st line = diuretic eg/ furosemide (40mg IV every 1-2hrs) or bumetanide b) + O2 if sats <90% c) + ACEI or ARB eg/ ramipril: 1.25 to 10 mg or irbesartan or candesartan d) 2nd line = nesiritide + O2 + ACEI/ARB e) with hypertensive emergency - + IV vasodilator eg/ glyceryl trinitrate: 5 micrograms/minute IV or nitroprusside f) with AF - +B-blocker or CCB or digoxin + anticoagulation +/- amiodarone (severe supraventricular tachyarrhythmia) g) with sinus tachycardia - +/- B-blocker or CCB h) With myocardial ischaemia, previous MI, or HTN - +B-blocker + aspirin +/- revascularisation 6) Mx of chronic HFpEF - a) Mx risk factors eg/ BP, weight, dyslipidaemia, DM b) +/- diuretic eg/ furosemide: 20-80 mg orally once or twice daily initially c) HTN - +ACEI or ARB +/- B-blocker

Outline the management of a patient with impaired motor function

1) Duchenne MD - glucocorticoids eg/ prednisolone can delay progression, chest physio, walking aids, later will need respiratory help etc 2) muscular dystrophies - physical therapy, spinal supports, and splints for the limbs; prevention of obesity; infections are promptly treated; identification of carriers of the trait and genetic screening and counseling are important 3) myasthenia gravis - removal of the thymus, high doses of corticosteroids (depress immune response) and anticholinesterase medications (stimulate transmission of nerve impulses), and plasmapheresis (autoimmune antibodies are removed from the blood) 4) bacterial myositis - abx and minor surgery 5) polymyositis - high-dose corticosteroids, often combined with a cytotoxic immunosuppressant drug eg/ cyclophosphamide

List the most common bacteria cultured in acute cholecystitis

1) E. coli 2) Klebsiella 3) Streptococcus 4) Clostridium NB/ these are gas producing organisms and may lead to emphysematous cholecystitis

Valvular heart disease - outline the non-invasive assessment

1) ECG - outlines ischaemia, hypertrophy or arrhythmia 2) CXR - shows cardiomegaly, provides evidence of pulmonary oedema: pulmonary congestion with upper lobe diversion, fluid in fissures, Kerley B lines (horizontal lines from pleural inwards, most visible in costophrenic angles) and pulmonary oedema 3) BNP & pro BNP - inc in heart failure. Values > 100 have strong correlation with disease. Test has a strong -ve predictive value (if BNP is low, heart failure is unlikely) 4) trans-thoracic echocardiography - can visualise major causes & pathologies. Used if first stage investigations are abnormal. Examples include chamber dimensions, systolic and diastolic functions, regional wall motion abnormalities, valvular disease and cardiomyopathies 5) trans-oesophageal echocardiography

Describe the investigation of a pt with suspected angina

1) ECG - usually normal but may have ST depression, flat or inverted T waves, or signs of previous MI 2) exclude precipitating factors eg/ anaemia, diabetes, hyperlipidaemia, thyrotoxicosis, temporal arteritis 3) exercise test 4) scintigraphy 5) echocardiography 6) angiography 7) blood tests - FBC, cholesterol

List the clinical investigations employed to confirm a clinical suspicion of temporal arteritis

1) ESR - ≥50 mm/hour by the Westergren method is one of the American College of Rheumatology diagnostic criteria, but can be elected in many other conditions eg/ anaemia and paraproteinaemias 2) CRP - inc in most pts 3) FBC - typically, pts have a normochromic, normocytic anaemia with a normal WCC & inc platelet count; mild leukocytosis may occur 4) LFTs - transaminases and alkaline phosphatase are often mildly elevated 5) temporal artery biopsy - definitive test for diagnosis of GCA. All patients with suspected GCA should undergo temporal artery biopsy. If the biopsy on one side is normal, a second biopsy on the contralateral side should be performed. Need 3-5cm sample of temporal artery. Histopathology typically shows granulomatous inflammation; may show multinucleated giant cells; inflammatory infiltrate may be focal and segmental 6) temporal artery ultrasound - if biopsy not readily available, may show wall thickening (halo sign), stenosis or occlusion 7) consider - aortic arch angiography, may show stenosis or occlusion of the subclavian, axillary, or proximal brachial arteries 8) emerging tests - FDG-PET scan of head to mid-thigh, US of the upper extremity arteries

Describe the use of FAST (focused assessment with sonography for trauma) in trauma

1) FAST - US can detect fluid eg/ blood or GI contents in the abdominal cavity, and is a noninvasive procedure and relatively safe for the patient 2) indications - blunt abdominal trauma, not stable to go to CT, evaluation of the torso for free fluid suggesting injury to the peritoneal, pericardial, and pleural cavities, particularly in cases of trauma, evaluate the lungs for pneumothorax, stable penetrating trauma 3) benefits - dec time to diagnosis for acute abdominal injury in BAT, can diagnose & assess haemoperitoneum, noninvasive, can be integrated into the primary or secondary survey and can be performed quickly, without removing patients from the clinical arena, safe in pregnant patients and children 4) contraindications - no absolute CI, although if it is clear that the patient requires emergent surgical intervention, then a relative contraindication to performing the examination may exist 5) limitations - limitations in ability to detect mesenteric, diaphragmatic, or hollow viscous injury, and patients with isolated penetrating injury to the peritoneum, limited in identifying retroperitoneal hemorrhage, operator dependant

List the common tests used in the diagnosis of gallstones

1) FBC - inc WBC suggests inflammation from a complication of cholelithiasis: acute cholecystitis, cholangitis, or pancreatitis. 2) serum LFTs - cholelithiasis: normal; choledocholithiasis: elevated alkaline phosphatase, elevated bilirubin 3) serum lipase and amylase - 3x inc in acute pancreatitis 4) abdominal ultrasound - best single test (>95% accurate) for cholelithiasis - sludge in the gallbladder is key to diagnosis, shows - cholelithiasis: stones in the gallbladder; choledocholithiasis: stones in the bile duct +/- bile duct dilatation 5) magnetic resonance cholangiopancreatography (MRCP) - quite specific overall, but limited value for detecting small stones (<5 mm) 6) endoscopic ultrasound scan (EUS) - for suspected choledocholithiasis not confirmed by abdominal US, particularly if cannot undergo an MRCP (claustrophobia, implanted devices) or to confirm choledocholithiasis in high-risk patients prior to a therapeutic ERCP, shows stones in gallbladder or bile duct 7) endoscopic retrograde cholangiopancreatography (ERCP) - preferred intervention if high risk of bile duct stones (+ve imaging, symptoms, +/- blood tests), due to its therapeutic capability to remove an obstructing stone 8) abdominal CT scan - CT abdomen with IV contrast can be used to investigate suspected ascending cholangitis, bile duct dilation with choledocholithiasis; acute pancreatitis: diffuse or segmental enlargement of the pancreas with stones in the gallbladder and possibly the bile or pancreatic ducts

List the common tests used in the diagnosis of calculus biliary tract disease

1) FBC - inc WCC 2) USS - thick walled shrunken gall bladder, pericholecystic fluid, stoned, common bile duct (dilated if >6mm - most useful investigation for diagnosis of acute cholecystitis 3) HIDA cholescintigraphy (if diagnosis uncertain after USS) 4) plain AXR - only shows 10% gallstones, may identify porcelain GB NB/ lab tests unlikely to be altered in biliary colic, acute cholecystitis may be associated with moderate leucocytosis & raised inflammatory markers. LFTs may show an obstructive pattern

Discuss the diagnostic aids available for the diagnosis of large bowel obstruction

1) FBC - inc WCC may indicate an infective or inflammatory cause or complication such as perforation or impending perforation, anaemia may be found with malignancy (microcytic) 2) U&E - may be deranged from dehydration, fluid shifts, or sepsis, may have hypokalaemia 3) serum amylase/lipase - can be inc with any significant intra-abdominal event 4) coagulation studies - coagulopathy may be present in sepsis from perforation 5) erect chest x-ray - may show perforation 6) plain abdominal x-ray - a) colonic dilation b) level of obstruction may be determined by a cut-off beyond which colon or rectum is empty of gas c) colonic volvulus - identified in 75% of x-rays; characteristic kidney- or coffee-bean shape seen with the "apex" locating the origin of volvulus (eg/ sigmoid, caecal) d) sigmoid volvulus - dilated inverted U-shaped loop of colon projected towards RHS of abdomen; opposing colonic walls produce radio-opaque line; proximal large and small bowel dilation may also be evident e) caecal volvulus - dilated right colon rotates to the LHS & dilated small bowel may be present f) malignant obstruction - colon distended to the point of obstruction with a paucity of distal gas g) intramural gas ominously suggests colonic ischaemia 7) contrast enema - gives info on level, degree, and type of obstruction a) contrast ends as a bird's beak in colonic volvulus, which confirms diagnosis where doubt exists on abdominal x-ray 8) CT abdomen and pelvis - obstruction confirmed in >90% 9) flexible/rigid endoscopy - confirms diagnosis & therapeutic, biopsy confirms malignancy 10) biopsy - confirms malignancy, typically adenocarcinoma, although other sub-types exist

Describe and interpret relevant investigations in a patient with suspected COPD

1) FBC - inc: Hb, PCV, CRP 2) CXR - hyperinflation (>6 ribs above diaphragm), flat hemidiaphragms 3) ECG - R atrial & ventricular hypertrophy (cor pulmonale), possibly echo 4) ABG - normal or PaO2 dec +/- hypercapnia 5) spirometry 6) lung function tests - dec in both FEV + FVC, dec in ratio <70% 7) CT - may outline bullae 8) a-1 antritrypsin investigation in younger pts or non-smokers NB/ not normally sputum

Outline a plan of investigation for a patient complaining of easy bruising

1) FBC - low Hb may suggest that the bruising is long standing or associated with bleeding elsewhere eg/ GI bleeding; isolated low platelet count suggests thrombocytopenia as likely cause for bruising; abnormal total and differential WCC + low platelet count suggests a bone marrow disorder 2) blood film - to identify morphological abnormalities in blood cells, including the shape and colour of platelets, which can suggest an inherited platelet disorder 3) clotting screen (prothrombin time, activated partial thromboplastin time, and fibrinogen) - if abnormal in a patient with bleeding symptoms, suggests an inherited or acquired coagulation disorder 4) INR 5) U&Es + LFTs - renal impairment can cause bruising from acquired platelet dysfunction, and liver failure can lead to dec production of coagulation factors and platelet dysfunction

Describe the nature of restrictive lung disease & contrast this with features of obstructive lung disease

1) FEV1 = forced expiratory volume in 1s, FVC = forced vital capacity 2) obstructive defect - involve maintenance of normal total lung capacity, but dec in ability to inspire & expire as easily eg/ asthma, COPD - FEV1 dec more than FVC so FEV1/FVC ratio <75% 3) restrictive - processes that dec pts total lung capacity, without loss of ability to inspire & expire adequately eg/ lung fibrosis - FVC dec, FEV1/FVC ratio is normal (~75-80%), or inc (~90%) NB/ other restrictive conditions - sarcoidosis, pneumoconiosis, interstitial pneumonias, connective tissue diseases, pleural effusion, obesity, kyphoscoliosis, neuromuscular problems

Outline the conditions that may cause ventilatory failure due to neuromuscular disease

1) Guillain-Barre 2) myasthenia gravis 3) polymyositis 4) tetanus 5) botulism 6) organophosphate poisoning 7) MS 8) amyotrophic lateral sclerosis

Describe the relationship between H. pylori, smoking and non-steroidal anti-inflammatory drugs and peptic ulcer disease and the mechanisms by which they cause peptic ulceration

1) H. pylori -spiral-shaped bacteria that cause peptic ulcer disease by damaging mucous coating that protects the lining of the stomach and duodenum. Once its damaged the mucous coating, powerful stomach acid can get through to the sensitive lining. Stomach acid + H. pylori irritate the lining of the stomach or duodenum and cause a peptic ulcer 2) smoking - inc risk of H. pylori infection, slows the healing of peptic ulcers, and inc likelihood that peptic ulcers recur. Smoking hasn't been shown to inc acid production, but does inc production of other substances that may harm stomach lining eg/ pepsin (protein enzyme). Smoking also decreases factors that protect or heal the lining eg/ blood flow to the lining, secretion of mucus, production of sodium bicarbonate by the pancreas 3) NSAIDs - work by blocking or dec amount of COX so dec pain + inflammation. But prostaglandins help protect stomach lining from stomach acid (mucus) and helps control bleeding

List the risk factors which may predispose a patient to TB or fungal meningitis

1) HIV/AIDS 2) excessive alcohol use & malnutrition 3) weakened immune system 4) DM 5) high prevalence (geography) 6) overcrowding 7) corticosteroid use 8) malignancy

Describe the clinical features of lymphoma. Classify lymphomas into Hodgkin's and Non-Hodgkin's disease and to high- and low- grade groups. Outline the principles of treatment

1) Hodgkin's - a) s/s - I) lymphadenopathy - typically painless & mostly cervical and/or supraclavicular nodal chain II) risk factors eg/ EBV infection, fmx, and young adults from higher socio-economic class III) less common - B symptoms (fever, night sweats, weight loss), dyspnoea & cough, chest pain etc b) tx - I) stage I to stage IV: 1st line - chemotherapy ± radiotherapy: doxorubicin + bleomycin + vinblastine + dacarbazine + radiotherapy II) if nodular lymphocyte predominant: 1st line - radiotherapy c) staging - I) involvement of 1 lymph node region or lymphoid structure (e.g., spleen, Waldeyer ring) II) involvement of 2+ lymph node regions on the same side of the diaphragm III) involvement of lymph node regions or structures on both sides of the diaphragm IV) involvement of extra-nodal site(s) beyond that designated 2) non-Hodgkin's - a) s/s - I) B symptoms II) lymphadenopathy - peripheral III) splenomegaly IV) less likely - hepatomegaly, dyspnoea & cough, abdo pain, headache etc b) tx - B-cell lymphomas - I) diffuse large B-cell lymphoma; stages I-II: 1st line - R-CHOP-21: rituximab + cyclophosphamide + doxorubicin + vincristine + prednisolone; +/- radiotherapy; +/- CNS prophylaxis (methotrexate); +/- Mesna prophylaxis; +/- growth factor prophylaxis; +/- antimicrobial prophylaxis T-cell lymphomas I) enteropathy-type T: 1st line - CHOP: cyclophosphamide + doxorubicin + vincristine + prednisolone; +/- autologous transplantation c) staging - I: single lymph node group II: multiple lymph node groups on same side of diaphragm III: multiple lymph node groups on both sides of diaphragm IV: multiple extranodal sites or lymph nodes and extranodal disease

Describe and perform the proper technique for intra-muscular and IV injections; as well as injection of local anaesthetics; discuss potential complications of the procedure

1) IM - a) severe pain at the injection site b) tingling or numbness (nerve injury) c) redness, swelling, or warmth at the injection site (infection) d) drainage at the injection site e) prolonged bleeding f) signs of an allergic reaction, such as difficulty breathing or facial swelling g) muscle atrophy h) injury to bone i) cellulitis j) sterile abscesses 2) IV - a) site infection b) anaphylaxis or other allergic reaction c) infiltration d) hematoma e) air embolism f) phlebitis g) extravascular drug administration h) intraarterial injection

Differentiate symptoms of ischaemic rest pain and neuropathy as a cause of foot pain and contrast gangrene in diabetic and non -diabetic patients

1) IRP - continuous burning pain of the lower leg or feet. Begins, or aggravated, after elevating limb & relieved by sitting or standing. More severe than IC 2) neuropathy - prickling and tingling sensation in affected body part, numbness and less of an ability to feel pain or changes in temperature, particularly in feet. Burning or sharp pain, usually in feet, allodynia, loss of balance or co-ordination caused by less ability to tell position of feet or hands - generally glove and stocking distribution 3) gangrene in diabetics - 4) gangrene in non-diabetics -

Describe the main incisions for cardiac surgery and outline the difference between open and closed heart surgery and outline the principles involved in cardio-pulmonary bypass

1) Incisions - a) Median sternotomy - most common thoracic incision, predominately for open heart surgery, such as valve replacements, CABG, or cardiac transplant, incision from substernal notch to xiphoid process, before cutting directly through the sternum to enter the thoracic cavity b) Pacemaker - 4-5cm incision is made in the left infraclavicular region, either: horizontal, oblique and deltopectoral c) Thoracotomy - used to access the pleural space of the thorax, 3 main subtypes: posterolateral incision, anterolateral incision, and axillary incision d) Posterolateral thoracotomy - gold standard for access to the thorax, gives access all the thoracic viscera, and is mainly used for pulmonary resections (pneumonectomy or lobectomy), chest wall resection, or oesophageal surgery, goes from between the scapula and mid-spinal line, and extends laterally to the anterior axillary line e) Anterolateral thoracotomy - used in a variety of operations for cardiac, pulmonary, and oesophageal pathology, incision from lateral border of the sternum to the mid-axillary line at the 4th or 5th intercostal space f) Axillary thoracotomy - muscle sparing approach, used for pneumonectomy and pneumothorax operations, incision between the posterior border of the pectoralis major and anterior border of latissimus dorsi muscles, through the 4th or 5th intercostal space. 2) Open vs closed heart surgery - 3) Open-heart surgery - any procedure in which the heart is literally opened. Heart is cooled down and the beating stopped, while a heart-lung bypass machine takes over the job of pumping oxygen-rich blood to the rest of the body. Mostly median sternotomy 4) Closed-heart surgery — open the chest, but heart doesn't stop. Mostly for major blood vessels, such as the aorta or pulmonary arteries eg/ aortic coarctation repair, "shunt" operations to increase blood flow to the lungs, vascular ring division, patent ductus arteriosus ligation or division, and pulmonary artery repair 5) Cardio-pulmonary bypass - technique that temporarily takes over the function of the heart and lungs during surgery - to provide a bloodless, stable surgical field, maintaining the circulation of blood and the oxygen content of the patient's body. Cardiopulmonary bypass pumps are operated by perfusionists. CPB is a form of extracorporeal circulation. Extracorporeal membrane oxygenation is generally used for longer-term treatment. a) Actions - oxygenation via increasing FiO2, removal of CO2 via increasing gas flow, non-pulsatile circulation of blood

List the surgically or radiologically curable causes of hypertension

1) Kidney disese - a) polycystic kidney disease b) Renovascular HTN - caused by fibromuscular dysplasia or atheromatous stenosis or diabetes c) some renal tumors can cause hypertension eg/ young pt with HTN - juxtaglomerular cell tumor, Wilms' tumor, and renal cell carcinoma, all of which may produce renin d) HTN secondary to other disease eg/ CKD, renal artery stenosis, renal segmental hypoplasia (Ask-Upmark kidney) 2) Endocrine - a) neurogenic hypertension - excessive secretion of NA + A promotes vasoconstriction resulting from chronic high activity of the sympathoadrenal system, the sympathetic nervous system and the adrenal gland b) Pheochromocytoma -tumor which results in an excessive secretion of A + NA which promotes vasoconstriction c) Hyperaldosteronism (Conn's syndrome) - idiopathic hyperaldosteronism, liddle's syndrome (pseudoaldosteronism), glucocorticoid remediable aldosteronism d) Cushing's syndrome - excess secretion of glucocorticoids causes the hypertension e) Hyperparathyroidism f) Acromegaly g) Hyperthyroidism or Hypothyroidism 3) Adrenal - a) primary aldosteronism b) Congenital adrenal hyperplasia c) 17 alpha-hydroxylase deficiency d) 11β-hydroxylase deficiency, aka apparent mineralocorticoid excess syndrome e) glucocorticoid remediable aldosteronism f) Cushing's syndrome - inc cortisol g) Neuroendocrine tumors eg/ pheochromocytoma 4) Drugs - alcohol, nasal decongestants with adrenergic effects, NSAIDs, MAOIs, adrenoceptor stimulants, combined methods of hormonal contraception (containing ethinylestradiol), steroids, nicotine, certain antidepressants (such as venlafaxine), buspirone, carbamazepine, bromocriptine, clozapine, and cyclosporine, rebound hypertension eg/ clonidine and methyl-dopa, other herbal or "natural products" associated with HTN include ma huang, St John's wort, and licorice 5) Other causes - neurologic disorders, OSA, scleroderma, neurofibromatosis, pregnancy, cancers eg/ kidney tumour, malformed aorta, slow pulse, ischemia: these cause reduced blood flow to the renal arteries, with physiological responses as already outlined, aortic valve disease, coarctation of the aorta, atherosclerosis, anaemia, fever, white coat HTN, perioperative hypertension (just before, during or after surgery), arsenic exposure, hypokalaemia 6) ABCDE mnemonic can be used to help determine a secondary cause of hypertension - A: Accuracy, Apnea, Aldosteronism B: Bruits, Bad Kidney C: Catecholamines, Coarctation of the Aorta, Cushing's Syndrome D: Drugs, Diet E: Erythropoietin, Endocrine Disorders

Outline commonly used drugs to treat Parkinson's disease and their common adverse effects

1) Levo-dopa (co-beneldopa, co-careldopa, sinemet) + peripheral decarboxylase inhibitor (eg/ carbidopa) to prevent peripheral breakdown 2) can use dopamine agonist (rotigotine, ropinorole) - activate post synaptic receptors (but neuropsychiatric S/E eg/ obsession, sexual virilisation) 3) others - MAOB inhibitors (selegiline, rasagaline), amantadine (for dyskinesias), Catechol-O-Methyl transferase (COMT) - Entacapone (inhibit LDOPA breakdown), Apomorphine 4) interventions (younger pts) eg/ DBS 5) non-pharmacological - CGA: social support, care, OT, environment, physio (esp if fall) 6) non-motor symptoms: constipation, sleep (vivid dreams), autonomic dysfunction, mood, dementia (rivastigmine - cholinergic agent) 7) with nausea and/or vomiting on carbidopa/levodopa - + additional carbidopa or domperidone 8) with nausea and/or vomiting on dopamine agonist - + domperidone 9) with refractory tremor - + pharmacotherapy or deep brain stimulation: trihexyphenidyl or amantadine 10) with wearing off (motor fluctuations), taking carbidopa/levodopa - + COMT inhibitor, dopamine agonist, MAO-B inhibitor, or switch to extended-release carbidopa/levodopa: entacapone or opicapone or ropinirole or pramipexole 11) with unpredictable off-times, motor fluctuations, or freezing - + apomorphine or as-needed doses of carbidopa/levodopa

Describe the characteristic, and contrasting, features of chest pain resulting from myocardial ischaemia, aortic dissection, pleural disease, oesophageal disease, musculoskeletal disease

1) MI - a) similar to angina but more severe (not relieved by GTN spray) b) central (retrosternal) chest tightness or heaviness brought on by exertion & relieved by rest c) may radiate to arm(s), neck, jaw, teeth d) precipitants - cold, emotion, heavy meals e) associated symptoms - dyspnoea, nausea, sweatiness, faintness f) pts may experience "angor animi", a feeling of impending doom 2) aortic dissection - a) sudden tearing chest pain which can radiate to the back + shoulder blades b) pts commonly hypertensive or Marfan-oid c) persistent, most severe at onset d) as dissection expands can lead to occlusion so can have: hemiplegia (carotid artery), unequal arm pulses + BP or acute limb ischaemia, paraplegia (anterior spinal artery), anuria (renal arteries), aortic valve incompetence, inferior MI, cardiac arrest 3) pleural disease - a) sharp pain, worse on inspiration and coughing & not relieved by GTN b) not central, may be one sided, no radiation c) often associated with breathlessness or cyanosis 4) oesophageal disease - a) heartburn (burning, retrosternal discomfort after meals, lying, stooping or straining, relieved by antacids + GTN) b) belching c) acid brash (acid or bile regurgitation) d) oesophageal spasm may be mistaken for MI/angina e) often associated with a history of dyspepsia or dysphagia NB/ relieved with GTN after ~20 minutes - 2 minute relief in angina 5) MSK disease - a) localised to one location on the chest b) tender to palpitation c) more sensitive when moving and respiring

Explain the advantages of MRI over CT scan of the head

1) MRI does not use ionizing radiation, and is thus preferred over CT in children and patients requiring multiple imaging examinations 2) MRI has a much greater range of available soft tissue contrast, depicts anatomy in greater detail, and is more sensitive and specific for abnormalities within the brain itself 3) MRI scanning can be performed in any imaging plane without having to physically move the patient 4) MRI contrast agents have a considerably smaller risk of causing potentially lethal allergic reaction 5) MRI allows the evaluation of structures that may be obscured by artifacts from bone in CT images

Describe the epidemiological features of MS, with specific reference to gender, age of onset and geographic distribution

1) MS is most commonly diagnosed between 20 and 40 years old. However, it can occur in the paediatric age group to as young as 2 years, where it may be confused for acute disseminated encephalomyelitis - most common cause of neurological disability in young adults 2) female to male ratio of around 3:1, ~1/800 prevalence 3) geographic gradient, with higher incidence at latitudes closer to the poles, has been linked with MS. Individuals of European descent are most commonly affected; however, affected black Americans may have more aggressive courses due to a combination of socioeconomic and genetic factors, as well as later diagnosis

Define the following: Murphy's sign, Courvoisier's sign, T-tube (including purpose and circumstances of use), gallstone ileus

1) Murphy's sign - a) abdo exam - pt fully expires b) lay 2 fingers over RUQ (under the costal margin in the mid-clavicular line) c) ask pt to breath in deeply d) causes pain & arrest on inspiration as inflamed gall bladder impinges fingers NB/ only +ve if doesn't cause pain in LUQ 2) Courvoisier's sign - a) non-tender gall bladder enlargement with jaundice is unlikely to be caused by gallstones b) sign = presence of non-tender, enlarged gall bladder with jaundice 3) T-tube - a) typically performed 5-10 days after gallbladder removal b) contrast dye injected through t-shaped rubber tube placed in common bile duct during surgery - x-rays taken to detect any residual stones or other abnormalities c) purpose - to detect stones or stone fragments and other abnormalities in the bile ducts eg/ strictures (narrowings), abnormal growths, and fistulae (abnormal openings) NB/ can also be used as a drain 4) gallstone ileus sign - a) stone erodes through gall bladder into duodenum, may then obstruct terminal ileum b) AXR shows air in CBD (pneumobilia), small bowel fluid levels, stone

Outline the common groups of drugs used to treat hypertension including indications, contraindications, side effects and rational combinations of drugs

1) Mx of HTN - a) Step 1 - I) If <55 or diabetic = ACEI - if pt can't tolerate eg/ due to cough, switch to ARB II) If >55 or black of African or Caribbean origin = CCB, if pt can't tolerate switch to diuretic b) Step 2 = A + C c) Step 3 = A + C + D (thiazide like eg/ indapamide) d) Step 4 = A + C + D + spironolactone (potassium sparing diuretic + aldosterone antagonist), consider adding a-blocker, B-blocker, or loop diuretic 2) Medications used in HTN - a) ACEI eg/ enalapril, Lisinopril, perindopril, ramipril - I) Inhibit ACE, so dec angiotensin = dec arterial & venous vasoconstriction, dec aldosterone so dec salt & water retention II) SE/CI - cough (switch to ARB), inc K+, avoid in renovascular disease as can dec renal function b) CCB eg/ diltiazem, verapamil, dihydropyridines (amlodipine) - I) Verapamil - exerts most of its actions on the heart II) Dihydropyridines - most effect on arteriole SM III) Inhibit voltage gated Ca channels on vascular SM = vasodilation & dec BP IV) SE/CI - ankle oedema, headache, face flushing, postural hypotension, can exacerbate heart failure c) Thiazide-like diuretics eg/ indapamide - I) Inhibit Na/Cl in distal convoluted tubule = dec circulating volume + vasodilation II) SE/CI - hypokalaemia, postural hypotension, impaired glucose control, alter lipid profile, ineffective if eGFR <30, exacerbates gout d) Spironolactone - I) Aldosterone antagonist, dec Na + H20 retention II) SE/CI - hyperkalaemia (esp with ACEI/ARB), gynaecomastia III) Only give 25mg dose

Discuss examples of the major screening programmes currently operating in the UK

1) NHS Abdominal Aortic Aneurysm Screening Programme - a) dec premature deaths from ruptured AAAs among men aged >65, by up to 50% b) offers all men abdo US when they turn 65 while men over 65 who have not previously been tested can self-refer for screening 2) NHS Bowel Cancer Screening Programme - a) detects bowel cancer at an early stage when treatment is more likely to be effective, also detects polyps (which can be removed) b) screening kit is offered to men and women aged 60-74 every 2 years, kit is completed at home and posted to a lab c) one-off bowel scope screening test, using flexible sigmoidoscopy, for those aged 55, is also being implemented across England 3) NHS Breast Screening Programme - a) uses mammography to look for abnormalities in breast tissue b) women aged 50-70 invited for breast screening every 3 years, if >70 can continue to have breast screening by making an appointment at local screening unit every 3 years 4) NHS Cervical Screening Programme - a) screening is offered every 3 years to all women aged 25-49 and every 5 years if aged 50-64 5) NHS Diabetic Eye Screening Programme - a) offers screening every 12 months to all people with diabetes aged >12 6) NHS Fetal Anomaly Screening Programme - a) offers US to all pregnant women to assess chance of baby being born with Down's syndrome or abnormalities with the fetus b) 1st scan at 10-14 weeks + blood test c) scan fetal abnormalities at 18-21 weeks 7) NHS Infectious Diseases in Pregnancy Screening Programme = a) recommends screening for all pregnant women for hepatitis B, HIV and syphilis 8) NHS Newborn and Infant Physical Examination Programme - a) detailed physical exam to screen newborn babies for problems with their eyes, heart, hips or testes 9) NHS Newborn Blood Spot Screening Programme - a) screens newborn babies for 9 rare but serious conditions: phenylketonuria (PKU), congenital hypothyroidism (CH), sickle cell disease (SCD), cystic fibrosis (CF), medium-chain acyl-CoA dehydrogenase deficiency (MCADD), maple syrup urine disease (MSUD), isovaleric acidaemia (IVA), glutaric aciduria type 1 (GA1) and homocystinuria (HCU) 10) NHS Newborn Hearing Screening Programme - a) hearing screening test for babies during first few weeks of their lives to find those who are born with hearing loss 11) NHS Sickle Cell and Thalassaemia Screening Programme - a) uses a questionnaire about family origin and, if necessary, offers blood tests to screen pregnant women for sickle cell disease and thalassaemia major, also screens newborn babies for sickle cell disease

Describe treatment options of pneumothorax including chest aspiration or intercostal underwater chest drain

1) Need to remove air from pleural space & dec likelihood of recurrence 2) initial tx - observation with supplemental oxygen therapy, percutaneous aspiration of the air in the pleural space, and chest-tube thoracostomy, depending on the type and size of the pneumothorax, video-assisted thoracoscopy or thoracostomy may be necessary to eliminate the site of the air leak 3) pleurodesis is used to limit the likelihood of recurrence (by mechanical abrasion of the pleura or by introduction of a substance into the pleural space that irritates the pleural surfaces with subsequent adhesion of the parietal and visceral pleurae) 4) clinically stable patients with small primary spontaneous pneumothorax can be observed and treated conservatively with supplemental high-concentration (10 L/min) oxygen and observation without invasive intervention - if stable for 4 to 6 hours can be released with follow-up in several days (seek attention if SOB) 5) if large, then percutaneous needle aspiration should be undertaken (IV cannula into the pleural space at the intersection of the midclavicular line and the second or third intercostal space, large syringe withdraws air from the pleural space), once no further air can be aspirated the cannula should be removed and a chest x-ray obtained, more effective if pt <50yo 6) if aspiration fails, a chest tube or small-bore catheter should be inserted into the pleural space - chest tube thoracostomy 7) if persistent air leak and the drain continues to bubble after 48 hours, patients may be considered for negative pressure suction (high-volume low-pressure systems) to resolve the pneumothorax 8) further tx only necessary if air leak persists or pt has an ipsilateral recurrent pneumothorax - video-assisted thoracoscopy with stapling of the air leak and pleurodesis is the procedure of choice in most circumstances 9) thoracoscopic wedge resection is an alternative procedure to stem the air leak should it persist, often performed together with mechanical pleurodesis to prevent recurrence of the pneumothorax 10) if recurrent pneumothoraxes then consider pleurectomy (procedure to remove the lining between the lungs and the chest wall, helps surface of the lung to stick to the chest wall, in order to prevent further lung collapse)

List the incidence and complications of undernutrition

1) PEM more common in developing countries - Marasmus (generalised starvation) or Kwashiorkor (2nd-child syndrome, low protein but high CHO) 2) incidence in UK - 10.8% population (2015) 3) ~30% hospital pts undernourished. Elderly at risk, esp if have poor culinary skills 4) complications - a) somatic protein depletion - loss of tissue as general wasting of muscles b) visceral protein depletion - protein loss from liver, pancreas & gut, heart & kidneys lose mass in parallel with rest of body, brain retains size & structure c) when protein lost from SkM, muscle fibres shrink + spaces between are filled with extracellular watery gel - in elderly some of excess fluid collects in legs during day = oedema d) fatty marrow disappears & replaced with aqueous material e) easily infected (dec immune system) + poor wound healing f) gut becomes thin & weight lost from muscle + mucosa - in moderate undernutrition gut functions normally, in severe undernutrition diarrhoea can be severe (even without infection) & resulting dehydration can prove fatal g) fatigue, failure of reproduction, depression, loss of libido, anxiety, ultimately death NB/ organ function & body systems normal until lots of weight lost - in starvation both metabolic rate & body temperature fall NB2/ malnutrition of nutrients - vitA (night blindness, xeropthalmia, keratomalacia), thiamin (Beriberi, wernickes encephalopathy), riboflavin (angular stomatitis), niacin (Pellagra), vitB6 (polyneuropathy), B12 + folate (Megaloblastic anaemia), vitC (scurvy), vitD (rickets, osteomalacia), vitE (neurological disorders), vit K(coag defects)

Describe the risk factors, classification, clinical features and investigations of atrial fibrillation

1) Risk factors - inc age, HTN, DM, CHF, valvular heart disease, CAD, other atrial arrhythmias, cardiac or thoracic surgery, hyperthyroidism 2) Classification - fast or slow AF (CHECK) 3) s/s - irregular pulse rate, palpitations, hypotension, inc JVP, added heart sounds (underlying valvular disease eg/ mitral stenosis due to rheumatic heart disease, may be audible, gallop rhythm may be heard in heart failure, pericardial rub may be heard in cases of pericarditis) 4) to decide if need anticoagulation do CHA2DS2-VASc vs HAS BLED a) CHA2DS2-VASc - I) 2 points each: hx stroke, TIA, or thromboembolism, >74 II) 1 point each: 65-74, hx HTN, diabetes, CHF, valvular disease, female b) HAS-BLED - HTN (H), abnormal renal or hepatic function (A), stroke (S), bleeding or its risks (B), labile INRs (L), elderly age group (>65 years) (E), and drugs/alcohol (drugs such as antiplatelets) (D) are given 1 point each. Based on this scoring system, the risk of bleeding is significantly higher for scores of ≥3. 5) Ix - a) ECG - absent P waves; presence of fibrillatory waves that vary in size, shape, and timing; irregularly irregular QRS complexes b) serum electrolytes - may show high or low potassium, or low magnesium c) cardiac biomarkers - inc CK-MB or troponin with myocardial ischaemia d) TFTs - thyrotoxicosis may present with AF: suppressed TSH with elevated free T4 and/or T3 e) CXR - pneumonia, pericarditis, or heart failure may precipitate new-onset AF, may also show cardiomegaly, in particular LA enlargement; signs of heart failure f) transthoracic echocardiogram - may show abnormalities, such as LVH, left atrial enlargement, segmental or global wall motion abnormalities, valvular stenotic or regurgitation abnormalities, cardiomyopathy with low LVEF, or pericardial disease. g) transoesophageal echocardiogram (TOE) - necessary in patients presenting with new-onset AF, where the exact onset of AF is unclear, and cardioversion is necessary to restore sinus rhythm, may show presence of atrial thrombus h) others - electrophysiological study, exercise stress tests

Discuss the methods available for treating abdominal and thoracic aortic aneurysms and discuss the indications, contraindications and risk factors for surgery in patients with an abdominal aortic aneurysm

1) Ruptured AAA - a) 1st line = resuscitation eg/ O2, endotracheal intubation and assisted ventilation if the patient is unconscious, central venous catheter, arterial catheter and urinary catheter, target SBP of 50-70 mmHg and withholding fluids is advocated preoperatively b) + urgent surgical repair - endovascular aneurysm repair (EVAR) is preferred - graft is inserted into a blood vessel in groin then passed up into the aorta, open surgery - graft placed in the aorta through open surgery of abdomen c) + perioperative abx - cover gram-+ve and -ve organisms and prevent graft infection 2) AAA - symptomatic, but not ruptured - a) 1st line - urgent surgical repair b) + preoperative CV risk reduction - low-dose aspirin, control HTN c) + perioperative abx 3) AAA - incidental finding: small asymptomatic - a) 1st line - surveillance, surgical referral of smaller AAA is usually reserved for rapid growth, or once the threshold diameter for aneurysm repair is reached on repeated ultrasonography b) + CV risk mx -stop smoking, inc exercise, low-dose aspirin, control HTN, perioperative statins, perioperative B-blockade may be reasonable in patients at high risk of myocardial ischaemia 4) AAA - incidental finding: large asymptomatic - a) 1st line - elective surgical repair: repair is indicated in patients with large asymptomatic AAA (diameter >5.5 cm in men or 5.0 cm in women) b) + preoperative CV risk reduction - low-dose aspirin, control HTN, perioperative statins, perioperative B-blockade if high risk of myocardial ischaemia c) + perioperative abx 5) Indications for surgery in pts with a AAA - a) Emergency surgery for symptomatic or ruptured aneurysm b) Elective surgery c) Aneurysms 5 cm diameter or growing AAA 5 mm/year 6) CI for surgery in pts with a AAA - a) severe state of the patient b) recent MI c) cardiac lesions with circulatory insufficiency of stage II B or III d) fresh haemorrhage to the brain, hepatic and renal lesions with azotemia e) atherosclerotic occlusion of vessels in lower extremities with a complete block of all major vessels f) symptomatic aneurysms <5 cm in diameter, without growth in patients > 75 years & considerable perioperative risk - sonography done 3-monthly as a continuing control 7) Risks - a) graft leaking or slipping out of position b) wound infection or infection of the graft c) heavy bleeding from groin d) blood clot, heart attack or stroke e) heart attack or stroke f) erectile dysfunction or ejaculation problems in men 8) Thoracic AA - a) thoracic aorta > 4.5 cm is generally defined as aneurysmal b) size > 6 cm is the distinction for treatment, which can be either endovascular or surgical, with the former reserved for pathology at the descending aorta c) aneurysms in ascending aorta may require surgery at smaller size than if descending aorta

Describe the clinical syndrome that would arise from S1 root lesion; C5 root lesion; Median nerve compression at the carpal tunnel; Ulnar nerve palsy; Peripheral neuropathy; Neuromuscular junction disorders; myopathy

1) S1 root lesion - a) weakness of gastrocnemius muscle (back of calf) - difficulty with foot push off b) numbness runs on outside of foot c) loss of ankle jerk tap d) motor loss - plantar flexion e) dramatic onset during twisting, lifting or bending 2) C5 root lesion - a) shoulder pain + deltoid weakness (lateral arm) b) small area of numbness on shoulder c) biceps reflex diminished 3) median nerve compression at carpal tunnel - a) paraesthesia on palmar aspect of first 3.5 digits b) wasting of Pollicus Brevis c) pt wakes at night with paraesthesia/pain radiating to forearm, relieved by hanging hand down d) RFs incl: DM, RA, pregnancy, obese, hypotension, dialysis 4) ulnar nerve palsy - a) medial 1.5 digits b) wasting of hypothenar mm, interossei and medial 2 lumbricals c) occurs as nerve runs through cubital tunnel at elbow, or deep motor branch from palmar trauma 5) radial nerve palsy - a) abnormal sensation on dorsum of hand + thumb b) compression of nerve against humerus = wrist drop c) damage to posterior interosseous nerve in forearm gives wrist drop without brachioradialis weakness 6) peripheral neuropathy 7) neuromuscular junction disorders 8) myopathy

Describe the mechanisms of actions of the main drugs used to treat asthma

1) SABA bronchodilators - reverse bronchospasm (early) & rapid relief, act on B2-adrenoceptors on SM to inc cAMP eg/ salbutamol 2) LABA preventors - may be anti-inflammatory eg/ salmeterol, corticosteroids 3) xanthines - bronchodilators as phosphodiesterase inhibitors eg/ theophylline 4) muscarinic M-receptor antagonists - block parasympathetic bronchoconstriction eg/ ipratropium, tiotropium 5) corticosteroids - anti-inflammatory, preventative (don't reverse an attack) - lead to altered gene transcription eg/ beclometasone (inhaled) or prednisolone (oral) 6) leukotriene receptor antagonists (LTRAs) - antagonise actions of LTs, preventative & bronchodilator eg/ montelukast

Stepped care for asthma

1) SABA eg/ salbutamol PRN 2 puffs of 100 micrograms, for short-lived wheeze (blue inhaler) 2) + low dose ICS daily, 2 puffs (100 or 50 micrograms depending on brand, brown inhaler) 3) + LABA eg/ Fostair a) assess response - if beneficial & control adequate maintain and review in 3/12 b) if benefit but control still inadequate, inc dose of ICS to medium dose c) if poor response, stop LABA and inc dose of ICS to high dose 4) add on therapy - LTRA, tiotropium; try for 1/12, if no benefit stop & inc dose of ICS; if benefit seen continue but not adequate, inc dose of ICS 5) if control still inadequate consider oral steroid

Describe the spectrum of acute coronary syndromes

1) ST segment elevation myocardial infarction (STEMI) - coronary artery completely blocked, inc troponin (released at 4-6 hrs and peaks at 24 hrs), release of CK-MB may be an earlier indicator 2) non-ST-segment elevation syndromes (non-STEMI) - artery is only partly blocked - only part of heart muscle being supplied by artery is affected. Inc troponin. ECG often shows T wave inversion or ST depression 3) unstable angina - blood clot causes dec blood flow but not a total blockage. Heart muscle supplied by affected artery does not die (infarct). Chest discomfort provoked by minimal exertion or at rest. Plasma troponin and creatine kinase are normal. ECG is normal or shows ST depression and/or T wave inversion

Interpret the ECG in suspected acute coronary syndromes and integrate interpretation with other relevant investigations

1) STEMI - ST elevation eg/ V3+4 = anterior a) need ST elevation in at least 2 anatominally contiguous leads (eg/ V1+2, V5+6, V4+5, aVF + III) b) most ST elevations are accompanied by reciprocal ST depressions c) later have T wave inversion I) ST elevation in II + III + aVF = inferior MI (STEMI), usually RCA occlusion II) ST elevation in V1-4 = anterior elevation MI, usually LAD occlusion III) ST elevation in I, aVL, V5, V6 = lateral MI, either Cx occlusion or LAD/diagonal d) pathological Q waves may appear if infarct is large - Q waves are large & deep 2) STEMI ECG changes seen + elevated troponin = STEMI 3) NSTEMI - a) ST segment depressions b) T wave changes - inversions or flat c) unlike STEMI the leads that depress don't necessarily reflect the ischemic area d) don't normally get pathological Q waves 4) NSTEMI ECG changes seen + elevated troponin = NSTEMI 5) NSTEMI ECG changes seen + normal tropnonin = unstable angina

Discuss the evidence and indications for using lipid-lowering drugs in the prevention of cardiovascular disease, together with their side effects

1) Statins - a) Secondary prevention = atorvastatin 80mg b) Dec CV disease events & total mortality irrespective of initial cholesterol concn c) Advise pts to report promptly if unexplained muscle pain/tiredness/weakness, rarely causes inc in level of enzymes that signal liver inflammation - s/s include: fatigue or weakness, loss of appetite, pain in your upper abdomen, dark-colored urine, or yellowing of skin or eyes, can cause inc blood sugar or type 2 diabetes d) Take care with interactions - amiodarone, clarithromycin, itraconazole, amlodipine, grapefruit 2) Other lipid lowering drugs - a) Ezetimibe - inhibits intestinal absorption of cholesterol b) Fibrates eg/ bezafibrate - dec serum TGs c) Bile acid sequestrants eg/ cholestyramine - interfere with absorption of fat-soluble vitamins, may cause constipation, other drugs need to be taken an hr before or 4hrs after

Describe the clinical features and management principles of other overdoses that present commonly to the Emergency Department, including tricyclic antidepressants, benzodiazepines, opiates

1) TCA - a) s/s - I) change in mental status - dec LOC from slight drowsiness to coma II) tachycardia - normally sinus III) hypotension IV) mydriasis V) warm, dry, flushed skin VI) dec or absent bowel sounds VII) urinary retention VIII) ophthalmic signs - divergent squint, internuclear ophthalmoplegic, and gaze paralysis; nystagmus IX) neurological signs - ataxia, myoclonic and choreoathetoid mvmt, inc muscle tone, hyperreflexia, and extensor plantar responses b) mx - I) ABCDE including obtaining IV access, cardiac monitor, ECG II) supportive care and monitoring - acidosis, hypoxia, and electrolyte abnormalities should be corrected initially III) with ingestion within 2 hours: + GI decontamination - activated charcoal: 1 g/kg orally as a single dose, repeat every 2-4 hours if required IV) with QRS >100 ms or pH <7.45, or arrythmias: + sodium bicarbonate bolus then infusion - sodium bicarbonate: 1-2 mEq/kg IV V) with arrhythmias: + sodium bicarbonate & anti-arrhythmic therapy - may need magnesium sulphate VI) with hypotension: + IV fluids plus sodium bicarbonate +/- vasopressor or glucagon or extracorporeal membrane oxygenation - noradrenaline (norepinephrine): 2-20 micrograms/min IV infusion VII) with seizures: + sodium bicarbonate & benzodiazepine or barbiturate or propofol - diazepam: 10-20 mg IV or lorazepam: 4 mg IV 2) benzodiazepines - a) s/s - I) impaired mental status - impaired attention or memory, esp loss of anterograde memory. More subtle signs of impaired mental status include inappropriate behaviour or judgment, and labile mood II) drowsiness, slurred speech, ataxia III) dec deep tendon reflexes b) mx - I) supportive - airway maintenance, cardiorespiratory monitoring and support, and IV fluids II) +/- flumazenil - 0.2 mg IV over 30 seconds; may give a further 0.3 mg after 30 seconds if necessary; further doses of 0.5 mg may be given at 1 minute intervals as necessary, maximum 2 mg total dose 3) opiates - a) s/s - I) miosis - constricted pupils usually seen II) bradypnoea - RR <12 breaths/minute in an unconscious patient presenting via emergency medical services best predicts a response to naloxone and overdose III) altered mental status - including drowsiness and sleepiness IV) dramatic response to naloxone V) may see track marks, fresh needle marks VI) dec GI motility eg/ N&V, constipation, and abdominal pain b) mx - I) ventilation prior to naloxone administration - maintain O2 sats >94%. Support the airway and breathing, esp if stupor & RR <12 breaths/minute. Airway maintained through chin-lift, head-tilt, or jaw-thrust manoeuvres II) + naloxone - 0.4 to 2 mg IV.IM/subcut, repeat dose every 2-3 minutes, titrate dose by 0.2 to 0.4 mg increments according to response, maximum 10 mg/total dose

Select appropriate investigations to aid diagnosis and interpret these for the acute abdomen

1) U&Es + urinalysis 2) FBC 3) amylase 4) LFT 5) ABG 6) erect CXR, AXR (may show Rigler's sign) 7) may have laparoscopy 8) CT + USS can help diagnosis 9) pregnancy test 10) CRP & ESR 11) lactate 12) E/MRCP

Describe the clinical difference between upper and lower motor neuron limb weakness, with specific reference to findings on inspection, tone, deep tendon reflexes and pattern of weakness

1) UMN - a) muscle weakness: extensors (upper limbs) + flexors (lowers) = pyramidal weakness b) dec control of active mvmt, particularly slowness c) antigravity muscles stronger d) no wasting (of disuse) e) extensor plantar response f) spasticity g) clasp-knife response h) babinkski sign i) inc deep tendon reflex (hypereflexia) j) protator drift 2) LMN - a) muscle paresis or paralysis b) fibrillations c) fasciculations d) hypotonia or atonia e) hyporeflexia (no Babinkski) f) weakness limited to segmental or focal pattern - root innervated pattern g) wasting

Describe the cause of uvular deviation and outline the underlying cranial nerve innervation

1) Uvula innervated by CN 10 (vagus) + 9 2) deviation of the uvula to one side may imply two things - a) if LMN lesion of the Vagus nerve on one side, the uvula deviates to the opposite side b) if UMN lesion of the Vagus nerve on one side, the uvula to deviates to the same side 3) weakness of CN 9 or 10 may be due to - infection, compression (tumour), stroke (tongue, palate and pharynx are deviated along with the uvula), physical injury eg/ swallowing a big object or something that could cut the uvula, or when there is an injury to the throat or jaw caused by an accident or when a person is playing a sport, peritonsillar abscesses (deviation to the opposite side of the affected tonsil), uvula may also get elongated and deviate to one side due to postnasal drips or allergies, post-tonsillectomy, recurring, severe tonsillitis, congenital

Describe the special considerations necessary for dressing and bandaging wounds citing circumstances where it is necessary to protect the wound from the patient.

1) a wound will require different management and treatment at various stages of healing. No dressing is suitable for all wounds; therefore frequent assessment of the wound is required 2) dry wound - hydrocolloid, films, non-adherent dressing 3) minimal exudate - hydrogel, hydrocolloid, silicone 4) moderate exudate - calcium alginate, hydrofibre, foams 5) heavy exudate - hydrofibre, foam, absorbent dressing 6) burn patients need special dressings to retain moisture and maintain temperature may need to protect wound from pt eg/ moderate-severe dementia, delirium, young children

Discuss the potential causes of splenic rupture

1) abdominal blow or blunt trauma eg/ motor vehicle accidents, injury during contact sports, such as football and hockey, domestic violence 2) splenomegaly - a) infectious mononucleosis b) haemolytic diseases eg/ hemolytic anemia, certain lymphomas c) malaria

Describe the steps in the drainage and culture of a superficial subcutaneous abscess

1) abscesses always need drainage as infection will not resolve unless the pus is drained. The abscess cavity is avascular and therefore antibiotic treatment alone will not resolve the infection. [1, 3] If left untreated, an abscess can progress in involve deeper tissue and has the potential to develop into a life-threatening, systemic infection 2) many abscesses can be drained at the bedside under local anesthesia eg/ ¼% Marcaine or 1% lidocaine with or without epinephrine (dec bleeding). Inject the skin surrounding the abscess cavity instead of the cavity itself 3) when extent of the abscess cavity cannot be elucidated at bedside, or pain is too great to perform drainage under local anesthesia, exam and drainage in theatre under sedation or general anesthesia is necessary 4) position patient to provide maximal exposure 5) goal of treatment is to remove all necrotic debris and pus from the abscess cavity. Opening the abscess widely to allow all contents to drain is important. The wound should remain open after the procedure and be allowed to heal be secondary intent by packing the wound and changing the packing frequently 6) technique - a) prep and drape in a sterile fashion maintaining adequate exposure to site b) draw local anesthetic into the syringe using the 18-gauge needle and inject skin surrounding the abscess using the 27-gauge needle c) make an incision directly over, extending the entire length of the area of greatest fluctuance d) use forceps to stretch open the incision, allowing the contents of the cavity to drain. Insert finger or forceps into abscess cavity to break up any loculations e) flush cavity with irrigant eg/ vancomycin, gentamicin, hydrogen peroxide, iodine, Hibiclens, sterile water, or saline f) pack abscess cavity with sterile gauze. Alternatively, a Penrose drain may be left in place and the skin closed primarily 7) send for culture, results will determine if pt needs post-op abx

Discuss the symptomatic management of peptic ulcer disease. List the complications of peptic ulcer disease and describe subsequent treatment

1) active bleeding ulcer - a) 1st line: endoscopy +/- blood transfusion b) + PPI c) 2nd line: surgery (perforated ulcer) 2) no active bleeding, H. pylori -ve - a) 1st line: tx underlying cause (eg/ NSAIDs) b) + PPI for 8/52 c) 2nd line: H2 antagonist or sucralfate or misoprostol 3) no active bleeding, H. pylori +ve - a) 1st line: H pylori eradication therapy b) 2nd line: alternative regimen c) 3rd line: acid suppression therapy (PPI) 4) frequent recurrences, large or refractory ulcers = acid suppression therapy (PPI) 5) complications - a) penetration (of stomach or duodenal wall) - same tx as above, may need surgery b) gastric outlet obstruction - high dose PPI, may need endoscopic dilation c) upper GI bleed - A&E d) perforation - A&E

Outline the treatment options and monitoring of treatment response of EAA

1) acute - a) 1st line - avoidance of antigen b) with acute and sub-acute symptoms - + corticosteroid taper: prednisolone: 0.5-1 mg/kg/day PO, taper dose by 5-10 mg/day every other day for 6 weeks c) with chronic symptoms - + long-term low-dose corticosteroid therapy: prednisolone: 10 mg PO OD on alternate days d) secondary prevention - avoidance of the causative agent eg/ workplace reassignment or environmental interventions such as dehumidifiers and adequate/well-maintained ventilation systems. Environmental hygiene is also important: for example, cleaning out the crawl space

Describe a management plan for patients with stable COPD and for those presenting with an acute exacerbation of COPD

1) acute - a) SABA & SAMA neb + prednisolone (5 days 30mg) b) +/- airway clearance c) +/- O2 d) +/- abx (doxycycline) e) +/- invasive or non-invasive positive-pressure ventilation f) +/- IV aminophylline if pt not responding to tx (with senior review) 2) stable - a) medication - I) occasional breathlessness = SABA or SAMA II) persistent breathlessness & exercise limitation = LAMA (stop SAMA) or LABA or combine if not controlling alone III) frequent exacerbations = consider LABA + ICS combo eg/ Symbicort (>2 exacerbations in last yr + FEV1 <50%) IV) if persistent problems = combo inhaler containing LAMA + LABA + ICS V) chronic productive cough = carbocisteine b) + vaccines (influenza & pneumococcal) c) + smoking cessation d) + pulmonary rehab e) +/- O2 (LTOT tells you if severe) f) +/- dietician input g) +/- surgery NB/ if drug has no benefit then stop, if benefit but not controlled then step up and keep using that tx as well

Classify asthma into acute, severe, and life-threatening, and discuss the characteristics of a typical patient who is likely to present with each type

1) acute attack - a) tachypnoea b) hyper-inflated chest c) hyper-resonant percussion d) dec air entry - marked SOB e) pulsus paradoxus (massive drop in BP on inspiration causes impalpable pulses) f) bilateral widespread expiratory wheezing 2) severe attack - a) can't complete sentences b) pulse >110bpm c) RR >25/min d) PEF = 33-50% 3) life-threatening attack - a) silent chest b) confusion c) exhaustion d) cyanosis (PaO2 <8kPa, PaCO2 = 4.6-6) e) bradycardia f) PEF <33%

Describe the symptoms and signs and management of bile duct stones

1) acute cholecystitis - light-colored stools & dark urine, AC may cause gall bladder to adhere to jejunum or duodenum = fistula formation + transfer of stones into SI = gallstone ileus (can occlude bowel) 2) yellowish skin or eyes (jaundice) 3) itching 4) abdominal pain in RUQ 5) nausea & vomiting 6) weight loss & loss of appetite 7) fever 8) obstruction of CBD = infection, generally from bacteria in duodenum (cholangitis) 9) Charcot's triad in severe disease: swinging fever with rigors and chills, RUQ pain, jaundice 10) acute pancreatitis in 5% of pts with gallstones, more commonly in pts with multiple small stones - may temporarily obstruct pancreatic duct or allow duodenal contents to enter pancreas, leading to pancreatitis 11) mx - a) symptomatic cholelithiasis = cholecystectomy (mostly laparoscopic) b) choledocholithiasis +/- symptoms - I) 1st line - endoscopic retrograde cholangiopancreatography (ERCP) - combination of biliary pain, gallbladder containing stones, dilated common bile duct (>6 mm) on US, and abnormal liver biochemistry (esp bilirubin >68 micromol/L or >4 g/dL) or pancreatic enzyme elevation suggests stone may have migrated into common bile duct, causing an obstructive complication such as cholangitis or pancreatitis choledocholithiasis is best detected through endoscopic ultrasound or MRCP, ERCP with biliary sphincterotomy and stone extraction is tx of choice II) +/- lithotripsy, papillary balloon dilation, or long-term biliary stenting III) 2nd line - laparoscopic common bile duct exploration, bile duct stones need tx as associated with potential development of complications eg/ cholangitis, pancreatitis, and hepatic abscesses c) asymptomatic cholelithiasis - observation, doesn't need tx

Describe the clinical presentation of; Acute compression of the cauda equina; Acute lesion of the thoracic cord; L5/S1 root impingement due to disc prolapsed

1) acute compression of the cauda equina - a) severe back pain b) saddle anesthesia i.e., anesthesia or paraesthesia involving s3 to s5 dermatomes, including the perineum, external genitalia and anus; descriptively, numbness or "pins-and-needles" sensations of the groin and inner thighs which would contact a saddle when riding a horse c) bladder and bowel dysfunction, caused by dec tone of the urinary and anal sphincters, detrusor weaknesses causing urinary retention and post-void residual incontinence as assessed by bladder scanning the patient after the patient has urinated d) sciatica-type pain on one side or both sides, although pain may be wholly absent e) weakness of the muscles of the lower legs (often paraplegia) f) achilles (ankle) reflex absent on both sides g) sexual dysfunction h) absent anal reflex and bulbocavernosus reflex i) gait disturbance j) severe back pain, saddle anesthesia, incontinence and sexual dysfunction are considered "red flags", i.e. features which require urgent investigation 2) acute lesion of the thoracic cord - a) paraesthesia, a tingling or burning sensation in affected dermatome b) muscle spasticity, weakness, or complete paralysis c) lumbosacral - dec control of the legs and hips, genitourinary system, and anus. Bowel and bladder function are regulated by the sacral region. It is common to experience sexual dysfunction after injury, as well as dysfunction of the bowel and bladder, including fecal and urinary incontinence. May have priapism d) thoracic - affect the muscles in the trunk, T1-T8 injuries result in inability to control the abdominal muscles. Trunk stability may be affected; even more so in higher level injuries. The lower the level of injury, the less extensive its effects. Injuries from T9 to T12 result in partial loss of trunk and abdominal muscle control. Thoracic spinal injuries result in paraplegia, but function of the hands, arms, and neck are not affected e) lesions above the T6 level causes autonomic dysreflexia (AD), in which the blood pressure increases to dangerous levels, high enough to cause potentially deadly stroke f) cervical - result in full or partial tetraplegia (quadriplegia) g) C1-4 = cannot breathe without mechanical ventilation h) additional signs and symptoms of cervical injuries include low HR & BP, problems regulating body temperature, and breathing dysfunction 3) L5/S1 root impingement due to disc prolapse - a) leg pain (aka sciatica) b) herniated disc at L4-L5 usually causes L5 nerve impingement. In addition to sciatica pain, this type of herniated disc can lead to weakness when raising the big toe and possibly in the ankle, also known as foot drop. Numbness and pain can also be felt on top of the foot c) herniated disc at L5-S1 usually causes S1 nerve impingement. In addition to sciatica, this type of herniated disc can lead to weakness when standing on the toes. Numbness and pain can radiate down into the sole of the foot and the outside of the foot

Describe the management of a patient once a subdural haemorrhage is detected, with specific reference to who's advice should be sought

1) acute haematoma - a) <10 mm size: midline shift <5 mm non-expansile without significant neurological dysfunction - I) 1st line - observation, monitoring, and follow up imaging II) if GCS <9 need intracranial pressure monitoring & consider monitoring of cerebral oxygenation, together with continuous EEG monitoring for seizures III) follow-up CT scan 2 to 3 weeks after discharge. Imaging should be obtained immediately if new neurological symptoms, headache, nausea, vomiting, or dizziness develop IV) + prophylactic antiepileptics eg/ phenytoin: 10-20 mg/kg IV loading dose then 4-6 mg/kg/day IV, or levetiracetam; for 1/52 V) +/- correction of coagulopathy - if on anticoagulation stop or reverse antiplatelet or anticoagulant agent VI) +/- intracranial pressure-lowering regimen eg/ raising the head of the bed to 30°, using the reverse Trendelenberg position if spinal instability or injury is present, analgesics & sedation, paralytics in intubated patients, hyperventilation to pCO₂ of 30 to 35 mmHg should be used only for short periods when urgent reduction of ICP is needed b) ≥10 mm size or midline shift >5 mm or expansile or significant neurological dysfunction - I) 1st line - surgery: decision of what type of surgery to perform depends on the radiographic appearance of the haematoma and the surgeon's preference eg/ burr hole craniotomy, trauma craniotomy, hemicraniectomy II) + monitoring - all patients with GCS <9 need ICP monitoring (by ventriculostomy, subarachnoid bolt, or intraparenchymal ICP monitor), pressure of brain tissue oxygen monitor for partial pressures of oxygen in brain tissue areas of interest; jugular bulb monitoring for global cerebral oxygenation; and continuous electroencephalographic monitoring for seizures, as well as to guide barbiturate coma when necessary III) + prophylactic antiepileptics eg/ phenytoin or levetiracetam IV) +/- correction of coagulopathy V) +/- intracranial pressure-lowering regimen d) chronic haematoma - I) 1st line - antiepileptics eg/ phenytoin or levetiracetam II) +/- elective surgery eg/ twist-drill craniotomy with continuous catheter drainage, burr hole irrigation and drainage, standard craniotomy (2nd line), subdural-peritoneal shunting (3rd line) II) +/- correction of coagulopathy III) +/- intracranial pressure-lowering regimen NB/ may need help from epileptic specialist for regimens involving AEDs, also need neurosurgeon, may need haematologist

Outline the management of a transfusion reaction

1) acute haemolytic transfusion reaction - a) 1st line: I) fluid resuscitation - 0.9% saline II) + supportive therapy - airway support, vasopressors etc III) + discontinuation of blood transfusion NB/ want urine output >100 mL/hour, to prevent oliguric renal failure b) +/- forced diuresis - mannitol: (20%) 0.5 g/kg IV bolus, then 0.1 g/kg/hour IV infusion, titrated according to response, used if urine output is not adequate despite fluid resuscitation c) +/- dialysis - if progressive acute renal failure 2) anaphylactic reaction or severe allergic reaction - a) 1st line: I) adrenaline (epinephrine) - IM 1:1000 0.5mg every 10-15 mins II) + supportive care - airway, cystalloids III) + discontinuation of blood transfusion b) +/- inhaled bronchodilator - salbutamol 2.5 to 5 mg every 4-6 hours when required c) +/- glucagon - 1-2 mg IV bolus, then 1-5 mg/hour IV infusion if required d) +/- antihistamines - diphenhydramine: 50-100 mg IV + ranitidine 50 mg IV e) +/- corticosteroid - methylprednisolone 10-40 mg IV, repeat as needed 3) transfusion-related acute lung injury (TRALI) - a) 1st line: supportive care - O2 by mask for a brief period to mechanical ventilation for a period of days, resolution generally occurs rapidly, typically within <7 days following transfusion

Outline the general principles of treatment of acute leukaemias

1) acute myeloid - a) AML is aggressive & develops rapidly, so tx will begin a few days after diagnosis b) supportive care for all - I) hydroxycarbamide + allopurinol or rasburicase II) transfusions: RBC and platelet transfusions may be required. Platelet counts need to be maintained >50,000/microlitre, and activated PTT and fibrinogen levels normalised by infusions of fresh frozen plasma and cryoprecipitate III) Infections: pts with a febrile illness need to be investigated and treated appropriately with anti-infective agents. Central venous access is advisable c) induction - aims to kill as many leukaemia cells in blood & bone marrow as possible I) intensive chemotherapy - mixed regime using PICC or central line: cytarabine + idarubicin or daunorubicin II) non-intensive chemotherapy if not fit enough to withstand the effects of intensive chemotherapy - involves using an alternative type of chemotherapy to the standard intensive therapy, which is designed more to control leukaemia rather than cure it III) All Trans-Retinoic Acid (ATRA) - if have subtype acute promyelocytic leukaemia - works by changing the immature white blood cells (blast cells) into mature healthy cells, and can reduce symptoms very quickly d) consolidation - aims to prevent cancer relapsing, by killing any remaining leukaemia cells that may be present in the body: cytarabine or cytarabine + mitoxantrone I) often involves regular injections of chemotherapy on an outpatient basis e) other treatments - I) radiotherapy - to prepare the body for a bone marrow or stem cell transplant, or to treat advanced cases that have spread to the nervous system and/or brain II) bone marrow and stem cell transplants - if chemotherapy doesn't work III) azacytidine - possible alternative if can't have a stem cell transplant 2) acute lymphoid - a) induction chemotherapy: prednisolone or dexamethasone or vincristine or doxorubicin (+ dexrazoxane if using latter) b) +/- rituximab; +/- tyrosine kinase inhibitor eg/ imatinib or dasatinib c) + fluid therapy + allopurinol or rasburicase d) + prophylactic antimicrobials: ofloxacin or levofloxacin or ciprofloxacin + trimethoprim/sulfamethoxazole or pentamidine + fluconazole or itraconazole + aciclovir e) + supportive care: norethisterone, semen cryopreservation (males post-puberty) f) +/- haematopoietic growth factor: filgrastim or pegfilgrastim etc g) +/- platelet and red cell transfusions

Describe the clinical features of acute leukaemia and discuss the laboratory diagnosis

1) acute myeloid - a) s/s - I) pallor II) ecchymoses or petechiae III) fatigue IV) dizziness V) palpitations VI) dyspnoea VII) infections or fever VIII) lymphadenopathy IX) hepatosplenomegaly X) mucosal bleeding eg/ gums, nose, menorrhagia b) ix - I) FBC with differential - anaemia, macrocytosis, leukocytosis, neutropenia, and thrombocytopenia II) peripheral blood smear - immature blast cells, presence of Auer rods III) coagulation panel - PT and activated PTT may be mildly prolonged with normal fibrinogen and D-dimer IV) U&E + LFTS + serum lactic dehydrogenase - baseline V) serum electrolytes - may be normal; calcium, potassium, phosphorus, uric acid, and lactic acid may be inc; calcium may be dec VI) bone marrow biopsy or aspiration - needed for diagnosis: bone marrow hypercellularity and infiltration by blasts; blasts ≥20%, Auer rods, morphological description of type of blast VII) consider - immunophenotyping and molecular studies (blasts express surface antigens and molecular markers that help to identify their specific lineage), LP (if inc WBC count, monocytic lineage, or neurological s/s at presentation), HLA antigen typing (for stem cell donor), CXR, echo, multi-gated acquisition scan 2) acute lymphocytic - a) s/s - I) risk factors eg/ <6 yo, late 30s or mid 80s, hx of malignancy; previous chemo or radiotherapy, smoking; genetic disorders (eg/ Klinefelter's or trisomy 21); fmx of acute lymphocytic leukaemia II) lymphadenopathy III) hepatosplenomegaly IV) pallor, ecchymoses, or petechiae V) fever VI) fatigue, dizziness, palpitations, and dyspnoea VII) epistaxis, menorrhagia VIII) papilloedema, nuchal rigidity, meningismus IX) focal neurological signs eg/ 3rd, 4th, 6th, 7th CNs X) painless unilateral testicular enlargement XI) renal enlargement XII) bony pain b) ix - I) FBC with differential - normocytic normochromic anaemia with low reticulocyte count, can have leucocytosis, many have severe neutropenia & thrombocytopenia II) peripheral blood smear - shows leukaemic lymphoblasts III) serum electrolytes - inc calcium, potassium, phosphorus, uric acid, and lactic acid may be seen IV) U&E + LFTs + lactic dehydrogenase for baseline V) coagulation profile - prothrombin time, partial thromboplastin time, and levels of fibrinogen and D-dimers should be measured in any patient with bleeding or petechiae - results are variable VI) bone marrow aspiration and trephine biopsy - needed for diagnosis: bone marrow hypercellularity and infiltration by lymphoblasts VII) consider - immunophenotyping (on bone marrow, or peripheral blood if cell count is raised, to identify blast specific lineage), thiopurine methyltransferase (TPMT) phenotype, cytogenetics & molecular studies - for prognosis, HLA-typing (for donor stem cells), LP (for cytology if evidence of focal neurology or meningism), pleural tap if appropriate, CT/MRI brain (if dec GCS, meningism, or focal neurology)

Describe the pathological features and complications of acute and chronic pyelonephritis

1) acute pyelonephritis = exudative purulent localised inflammation of renal pelvis (collecting system) & kidney. Kidney parenchyma presents in interstitium abscesses (suppurative necrosis), consisting in purulent exudate (pus): neutrophils, fibrin, cell debris and central germ colonies (hematoxylinophils). Tubules are damaged by exudate and may contain neutrophil casts. In the early stages, the glomerulus and vessels are normal. Gross pathology often reveals radiations of bleeding & suppuration through renal pelvis to renal cortex. Generally ascending, but may be haematogenous. Symptoms - loin pain and tenderness, fever, rigors, frequency, urgency and dysuria. Complications - sepsis, abscess, papillary necrosis 2) chronic pyelonephritis - implies recurrent kidney infections and can result in scarring of the renal parenchyma and impaired function, especially in the setting of obstruction. A perinephric abscess (infection around the kidney) and/or pyonephrosis may develop in severe cases. Commonly occurs in presence of vesicoureteric reflux. Macroscopically, deep scars are present, capsule is thickened, and internal plumbing is distorted and distended. Main complication = ESRD NB/ permanent kidney scars can lead to: CKD, HTN, kidney failure. Usually occur in people with a structural problem in urinary tract, kidney disease from other causes, or repeated pyelonephritis. Infection in kidneys may spread to blood = sepsis

Describe the investigation and management of an acute MS relapse

1) acute relapse - episode of neurological disturbance, of the kind seen in multiple sclerosis, that lasts for > 24 hours, and for which there is no other cause such as fever. Typically evolves over a few days, reaches a plateau, and then remits to a variable degree over a few weeks or months 2) ix as above 3) mx - a) acute relapse affecting function - I) 1st line - methylprednisolone 1000mg IV for 3/7 II) with severe worsening - + plasma exchange III) education regarding relapses IV) rehabilitation

Describe how renal stones are treated including use of non-operative methods of treatment

1) acute renal colic non-pregnant - a) 1st line - conservative management: hydration, pain control, and anti-emetics eg/ crystalloids + ketorolac +/- morphine sulfate with ondansetron 2) confirmed stone: no evidence of obstruction & non-pregnant - a) 1st line - hydration, pain control, and anti-emetics as above, if ureteric stones <10 mm without complicating factors (urosepsis, intractable pain and/or vomiting, impending acute renal failure, obstruction of a solitary or transplanted kidney, or bilateral obstruction) can be managed expectantly b) If bacteriuria - I) + abx eg/ trimethoprim/sulfamethoxazole for 1-2 weeks or nitrofurantoin II) + surgical decompression - drainage: a urologist can place a ureteric stent past the obstructing stone and achieve renal drainage, or percutaneous nephrostomy by an interventional radiologist may be performed c) stones <10 mm - + medical expulsive therapy (MET) eg/ a blockers: tamsulosin: 0.4 mg, or alfuzosin or silodosin, all for 4-6 weeks or until stone passed, then surgery d) stones ≥10 mm or failed medical therapy - + surgical removal: I) extracorporeal shock wave lithotripsy (ESWL) & ureteroscopy 1st-line II) percutaneous antegrade ureteroscopy considered in select cases with very large (>15 mm) stones impacted in the upper ureter or if retrograde access not possible III) percutaneous nephrostolithotomy (PCNL) reserved for renal and proximal ureteric stones (in the lower pole) or large (>20 mm), have failed therapy with ESWL and ureteroscopy, or are associated with complex renal anatomy IV) laparoscopic or open surgical stone removal in rare cases where ESWL, ureteroscopy, and percutaneous ureteroscopy fail, or are unlikely to be successful 3) confirmed stone: with evidence of obstruction & non-pregnant - a) 1st line - hydration, pain control, and anti-emetics, as above b) + surgical decompression c) + surgical removal - as above d) with infection - + abx eg/ gentamicin or amoxicillin + gentamicin or cefuroxime etc; pts with urinary calculi + fever and other signs or symptoms of infection need emergency urological consultation for drainage and intravenous antibiotics for 2/52 4) pregnant - specialist referral 5) following an acute episode & non-pregnant - a) long-term dietary modification is essential for preventing future calculi b) inc fluids (>2L/day) c) dec sodium, protein, and oxalate recommended for stone prevention, inc citrus fruit

Outline the (metabolic) complications of pancreatitis

1) acute renal failure - caused by circulating toxins or rhabdomyolysis, complication with poor outcome 2) pancreatic abscess 3) necrotising pancreatitis 4) pancreatic insufficiency 5) chronic pancreatitis 6) portal vein/splenic thrombosis 7) enteric fistulas 8) intestinal obstruction 9) sepsis 10) retroperitoneal bleeding 11) infected pancreatic necrosis 12) acute lung injury/ARDS 13) disseminated intravascular coagulation 14) multiorgan failure 15) pseudocyst 16) gastrointestinal bleeding 17) intraperitoneal bleeding 18) pancreatic ascites 19) pancreatic effusion

Describe the commonly observed serum and urinary electrolytes and osmolality in patients with the following conditions: acute renal tubular necrosis; dehydration; inappropriate ADH secretion; diabetes insipidus; congestive cardiac failure

1) acute renal tubular necrosis - a) electrolytes - urine: Na+ >40 b) osmolality - <400 (injured tubules can't reabsorb all filtered water) 2) dehydration - a) electrolytes - urine: Na+ <30 b) osmolality - >600 3) inappropriate ADH secretion - a) electrolytes - hyponatraemia in serum, urine: >40 if normal diet b) osmolality - >200 (excess water reabsorption by distal nephron) 4) diabetes insipidus - a) electrolytes - urine: Na+ >40 b) osmolality - <100 (lack of ADH) 5) congestive cardiac failure - a) electrolytes - urine: Na+ <30 b) osmolality - >100

Distinguish between acute and chronic type II respiratory failure and respiratory and metabolic causes of acidosis

1) acute vs chronic: a) acute - life-threatening derangements in ABGs + acid-base status over mins to hours so pH is <7.3 b) chronic - less dramatic and may not be as readily apparent - develops over several days or longer = time for renal compensation & inc bicarbonate concn, so pH only slightly dec 2) respiratory causes of acidosis - type 2 respiratory failure due to any lung, neuromuscular, or physical cause - most common cause = COPD (also most common cause of T2 resp failure) a) acute = morphine overdose, airway collapse, foreign body, etc. b) chronic acidosis will exist alongside metabolic compensation, whereas acute respiratory acidosis will not - metabolic compensation is not a fast process. 3) metabolic causes of acidosis - a) inc anion gap - lactic acid (shock, infection, tissue ischameia), urate (renal failure), ketones (DM, alcohol), drugs/toxins (salicylate, biguanides, ethylene glycol, methanol) b) normal anion gap - renal tubular acidosis, diarrhoea, drugs (acetazolamide), Addison's, pancreatic fistula, ammonium chloride ingestion NB/ respiratory acidosis caused by a high pCO2, metabolic acidosis caused by negative base excess

Describe the pathway of care developed within the hospital for non-STEMI/UA

1) admit to CCU & monitor closely 2) if SaO2<90% or breathless = low flow O2 3) morphine 5-10mg IV + metoclopramide 10mg IV 4) nitrates - GTN spray or sublingual tablets PRN 5) aspirin 300mg PO + 2nd antiplatelet eg/ clopidogrel, ticagrelor, prasugrel 6) oral B-blocker eg/ metoprolol 50mg/12hr if hypertensive/tachycardia/LV function <40% a) if B-blocker contraindicated (asthma, COPD, LVF, bradycardia, coronary spasm) give rate-limiting calcium antagonist eg/ verapamil or dilitazem 7) fondaparinux 2.5mg OD SC or LMWH 1mg/kg/12hr SC (anti-coagulants) 8) IV nitrate if pain continues eg/ GTN 50mg in 50mL 0.9% saline at 2-10mL/hr, titrate to pain, maintain systolic BP >100mgHg 9) record ECG whilst in pain - a) invasive strategy (high-risk pt): rise in troponin, dynamic ST or T wave changes, secondary criteria (diabetes, CKD, LVEF <40%, early angina post MI) = infusion of GPIIb/IIIa antagonist (tirofiban) & refer for angiography as inpt b) conservative strategy (low risk pt): no recurrence of chest pain, signs of heart failure, normal ECG, normal troponin, no inducible ischaemia = discharge if repeat troponin negative NB2/ "BROMANCE" = Beta-blocker, Reassurance, Oxygen, Morphine, Aspirin, Nitrates (GTN), Clopidogrel & Enoxoparin

Describe the clinical presentation of an acute hydrocephalus

1) adults - a) headaches: these are more prominent in the morning because CSF is resorbed less efficiently in the recumbent position. this can be relieved by sitting up. as the condition progresses, headaches become severe and continuous b) neck pain: if present, neck pain may indicate protrusion of cerebellar tonsils into the foramen magnum c) nausea that is not exacerbated by head movements d) vomiting: sometimes explosive, vomiting is more significant in the morning. e) blurred vision (and episodes of "graying out"): these may suggest serious optic nerve compromise, which should be treated as an emergency - papilloedema f) drowsiness g) can cause uncal or tonsillar herniation, with resulting life-threatening brain stem compression 2) infants - a) fontanelle (soft spot) bulging & head is larger than expected b) irritability c) vomiting d) sleepiness e) seizures

Discuss the aetiology, diagnosis and management of herpes simplex encephalitis

1) aetiology - a) caused by HSV, an enveloped, double-stranded DNA virus. HSV-1 and HSV-2 are both members of the larger human herpesvirus (HHV) family, which also includes varicella-zoster virus (VZV) and cytomegalovirus (CMV) b) HSV-1, or HHV-1, is the more common cause of adult encephalitis 2) diagnosis - a) s/s - I) may have a prodrome of malaise, fever, headache, and nausea II) followed by acute or subacute onset of an encephalopathy - lethargy, confusion, and delirium III) most common symptoms of HSE: fever, headache, psych symptoms, seizures, vomiting, focal weakness, memory loss b) ix - I) MRI brain - preferred imaging study, temporal lobe involvement, sometimes haemorrhagic, and early involvement of white matter are typical II) CT brain - may show changes in the temporal +/- frontal lobe, but CT is less sensitive than MRI III) EEG - focal abnormalities eg/ spike and slow- or periodic sharp-wave patterns over the involved temporal lobes, or diffuse slowing may be observed IV) analysis of CSF - typical "viral profile" is identified. RBC & xanthochromia may be seen V) PCR - CSF should be sent for HSV-1 and HSV-2 PCR study VI) brain biopsy - with the advent of PCR technology, the role of brain biopsy is diminishing 3) mx - a) monitor for increased intracranial pressure (ICP) and seizures - if present need ICU I) tx of brain oedema ranges from simple measures (eg/ elevating head of bed, gentle diuresis with furosemide) to more complex measures (eg/ mannitol and steroids, intubation with hyperventilation) II) mx of seizures - AEDs, benzos to abort status epilepticus b) antiviral Therapy - acyclovir (30 mg/kg/d IV) c) steroids - may be used

Describe the aetiology and pathology of primary and secondary liver neoplasms

1) aetiology - a) aetiology of HCC is multifactorial b) major risk factors - chronic hep B&C, heavy alcohol consumption, DM; also fmx of liver cancer in a first-degree relative appears to be a significant risk factor c) ¼ patients have no known risk factor 2) pathophysiology - a) chronic inflammation and cirrhosis play important roles in hepatocellular carcinogenesis b) acquisition of hep B virus early in life results in longer duration of disease that may lead to chronic inflammation and cirrhosis. HBV is a direct carcinogen and may cause methylation of the P16 gene, known to cause hepatocellular carcinogenesis leading to HCC. In addition, mutation of the X gene of HBV has also been found to play a role c) hypothesised that high levels of oestrogen and lipid peroxidation in patients with chronic hep C virus (HCV) may result in the development of the carcinoma. [18] The HCC related to HCV is almost always due to cirrhosis d) chronic HBV and HCV infection with metabolic disorders may lead to generation of reactive oxygen species from oxidative stress that can cause damage to DNA, resulting in mutation of cancer-related genes such as p53 e) aflatoxin is a mycotoxin naturally produced by Aspergillus , and commonly contaminates soya beans. In those patients who have had long-term exposure to aflatoxin, it has been found to cause mutations in the p53 tumour-suppressor gene f) pathogenesis of HCC usually begins with the development of dysplastic nodules, which can be classified as high-grade or low-grade dysplasia. High-grade dysplasia is a precancerous condition leading to a high rate of conversion to HCC

Describe the aetiology, morphology and pathological consequences of carcinoma of the breast

1) aetiology - cause is not entirely understood. Random changes or mutations in body's DNA leads to breast cancer. These changes in DNA can be inherited or acquired. Carcinomas are cancers of epithelial component of the breast - consists of cells lining lobules and terminal ducts; under normal conditions, these epithelial cells are responsible for making milk 2) morphology - may be in situ or invasive. 2 main cell types: ductal, and lobular 3) in situ cancer is usually impalpable and diagnosed on mammography or biopsy. 4) consequences - a) treatment eg/ pain and stiffness in arms and shoulders after surgery, and the skin in these areas may be tight. Build-up of excess lymph fluid = lymphoedema b) metastasis NB/ most common form of cancer in women. Rare < 35. NB2/ s/s: palpable mass, nipple discharge/retraction/rash, dimpling breast tissue, local oedema. Lump may feel hard and spiky, not well circumscribed & fixed to chest wall. LNs may be palpable depending on stage of tumour. Spread via direct extension, lymphatics, blood (lung liver bone), and through pleura/peritoneum

Describe the pathological process involved in temporal arteritis

1) aetiology - exact cause is unknown; however, genetic and environmental factors are thought to contribute to development of GCA. The condition is probably triggered by an environmental cause in a genetically predisposed person. a) genetic polymorphisms of HLA antigen class II region, specifically HLA-DRB1-04 and DRB1-01 alleles, are associated with susceptibility to GCA b) several infectious agents have been implicated, but conclusive evidence is lacking eg/ Mycoplasma and Chlamydia pneumoniae , parvovirus B19, parainfluenza virus 2) pathophysiology - a) GCA is an immune-mediated vasculitis characterised by granulomatous inflammation in the wall of medium-sized and large arteries - extracranial branches of the carotid artery are preferentially involved, although the aorta and its major branches are often targeted as well. Affected arteries contain inflammatory lesions arranged in granulomas composed of T cells and macrophages. Multinucleated giant cells are present in about 50% of cases b) initial immune insult is thought to occur in the outer or adventitial layer of the arterial wall - highly activated resident dendritic cells attract & present antigen to T cells. CD4 T cells enter the artery through the vasa vasorum and, on activation, proliferate and undergo clonal expansion in the vessel wall. T cells release cytokine interferon gamma, which stimulates tissue-infiltrating macrophages and induces formation of multinucleated giant cells c) macrophages in the adventitia produce inflammatory cytokines interleukin 1 & 6, which are responsible for systemic inflammation and the acute-phase response, resulting in elevation of inflammatory markers d) in the muscular or medial layer of the artery, macrophages cause tissue damage by releasing matrix metalloproteinases and ROS, resulting in oxidative stress. Thus, the internal elastic lamina, (separates intimal and medial layers), becomes fragmented. Multinucleated giant cells tend to lie at the media-intima border, often close to fragments of the internal elastic lamina e) in response to immunological injury, the artery releases growth and angiogenic factors (platelet-derived growth factor and vascular endothelial growth factor) that result in proliferation of myofibroblasts, new vessel formation, and marked thickening of the intimal or inner layer f) process of intimal expansion and hyperplasia leads to narrowing and occlusion of the vessel lumen, ultimately causing tissue ischaemia. Clinical symptoms derive from ischaemia in organs such as the eye and brain, leading to blindness and stroke. Low-grade inflammation has also been demonstrated in temporal artery biopsy specimens from patients with polymyalgia rheumatica but without overt clinical arteritis

Describe the aetiology, morphology and pathological consequences of cholelithiasis

1) aetiology - gallstone formation = certain substances in bile are present in concns that approach the limits of their solubility. When bile is concentrated in the gallbladder, it can become supersaturated with these substances, which then precipitate from the solution as microscopic crystals. Crystals are trapped in gallbladder mucus, producing gallbladder sludge. Over time, the crystals grow, aggregate, and fuse to form macroscopic stones. Occlusion of the ducts by sludge and/or stones produces the complications of gallstone disease. 2 main substances involved in gallstone formation are cholesterol and calcium bilirubinate 2) morphology -varying from 1mm-5cm. Single or only a few gallstones they tend to be round. Larger numbers tend to be faceted due to the rubbing of one gallstone against another. Brown pigment gallstones may be crumbly and irregular 3) pathological consequences - symptoms & complications result from effects occurring within the gallbladder or from stones that escape the gallbladder to lodge in the CBD. a) biliary colic - sporadic and unpredictable episodes, pain localized to the epigastrium or RUQ, sometimes radiating to the right scapular tip, pain begins postprandially, described as intense and dull, typically lasts 1-5 hours, increases steadily over 10-20 minutes, and then gradually wanes, pain that is constant; not relieved by emesis, antacids, defecation, flatus, or positional changes; sometimes accompanied by diaphoresis, nausea b) acute cholecystitis - well-localized pain in RUQ, usually with rebound and guarding; positive Murphy sign (nonspecific); frequent presence of fever; absence of peritoneal signs; frequent presence of tachycardia and diaphoresis; in severe cases, absent or hypoactive bowel sounds c) other complications - cholecystitis, cholangitis, pancreatitis

Describe the aetiology, morphology and pathology of carcinoma of the large bowel

1) aetiology - multifactorial disease process - genetic factors (adenomatosis coli gene (APC/FAP), hereditary non-polyposis CRC syndrome (HNPCC)), environmental exposures (diet - low fibre, high in protein + animal fats = risk factor), and inflammatory conditions of the digestive tract 2) morphology - usually single, polypoid or ulcerated mass, may cause serosal puckering if muscularis propria is involved. R colon tumors tend to be polypoid and exophytic, L colon tumors tend to be annular, encircling lesions. Usually well or moderately differentiated gland forming carcinoma with marked desmoplasia, particularly at edge of tumor. Glands often filled with necrotic debris ("dirty necrosis"), in both primary and metastatic sites. Inflammatory cells and scattered neuroendocrine cells are common. Intramural venous invasion may be easier to identify using an elastin stain 3) tubular adenomas - rectum and sigmoid colon. Start off as smaller swellings but develop into pedunculated structures, with hyperchromatic dysplastic glands 4) villous adenomas - rectum. Form a smaller mass which may be quite large and have a delicate frond-like structure, with a broad base and no pedicle. Fronds are formed of dysplastic epithelium. Present with rectal bleeding, but may present with consequences of inc mucus secretion. Risk of cancerous development increases rapidly >4cm. Many polyps are mixed type, tubulovillous (9%) which commonly develop into carcinoma - which forms in villous component (which forms ~25% of the lesion) 5) 20% cancers in caecal region. Tend to be polypoid, ulcerating carcinomas which have an everted edge and ulcerating central growth. Lumen is wide and faeces are liquid, which means obstruction is not as common - and tumours present late, with anaemia, abdominal mass and liver mets 6) 50% cancers in recto-sigmoid region, these carcinomas are usually stenosing, (annular and constricting) carcinomas. Lumen becomes narrow, and at this point faeces are solid, and so obstruction is common. Hypertrophy and dilation of the proximal bowel is one consequence. These tumours may present with altered bowel habit, obstruction or rectal bleeding 7) consequences - blockage of colon, causing bowel obstruction, cancer returning in the colon or spreading to other organs or tissues (metastasis), development of a second primary colorectal cancer

Describe the aetiology, clinical presenting features and management of a urethral stricture

1) aetiology - narrowing of the urethra caused by injury, instrumentation, infection and certain non-infectious forms of urethritis. Inflammation or scar tissue causes narrowing 2) presentation - hallmark sign = weak urinary stream. Other symptoms include: a) splaying of the urinary stream b) urinary frequency & urgency c) straining to urinate d) pain during urination e) UTI f) prostatitis g) inability to completely empty bladder h) some ps with severe urethral strictures = acute urinary retention (emergency) i) hydronephrosis and renal failure may also occur j) may result in overflow incontinence k) flow may be slow to start, slow at best and slow to finish 3) corrective tx necessary only if stricture causes problems - need annual follow-up to check for recurrence. Tx options: a) catheterization - usual first step for urine blockage, may also have abx if infection b) dilation - small wire through urethra and into the bladder, progressively larger dilators pass over the wire to gradually increase the size of the urethral opening, outpatient procedure may be an option for recurrent urethral strictures c) urethroplasty - surgically removing the narrowed section of urethra or enlarging it, may also involve reconstruction of the surrounding tissues with graft during reconstruction, recurrence of urethral stricture after a urethroplasty is low, first line when possible d) endoscopic urethrotomy - cystoscope into urethra, then insert instruments through the cystoscope to remove the stricture or vaporize it with a laser, gives faster recovery, minimal scarring and less risk of infection, although recurrence is possible e) implanted stent or permanent catheter - severe stricture and choose not to have surgery, several disadvantages, including a risk of bladder irritation, discomfort and urinary tract infections. They also require close monitoring. Urethral stents are often a measure of last resort and are rarely used NB/ most strictures iatrogenic & caused by urethral catheterisation. Other trauma may lead to stricture, as can infections (such as gonorrhoea), and invasive tumours. Meatal and bulbar stricture may occur congenitally

Outline the pathology and natural history of malignant breast neoplasms

1) aetiology - unknown, however, 2 groups of aetiological factors suggested: a) genetic factors: estimated cumulative lifetime incidence of breast cancer and ovarian cancer in BRCA mutation carriers is 87% and 44%, respectively b) hormonal factors: role of oestrogen in the stimulation of mammary tumours 2) pathophysiology - a) malignant cells of the breast result from a cascade of genetic events involving the uncontrolled expression of endogenous growth factors and signalling pathways eg/ steroid hormones, especially oestrogens, ErbB family, myc family, PI3K/AKT signalling pathway, NFkappaB, PDGFR, Src, and IGF

Describe the pertinent epidemiological features of temporal arteritis, including age at presentation and association with other inflammatory conditions

1) age - typically occurs in people >50 yo, incidence rises steadily after age 50 and is highest between 70-80 years 2) W:M = 3:1 3) incidence of giant cell arteritis (GCA) varies by race and geographical location - most common among white people of Scandinavian descent and is rare among black people 4) associated conditions - a) varicella-zoster virus antigen was found in 74% of temporal artery biopsies, suggesting that the VZV infection may trigger the inflammatory cascade b) may coexist with polymyalgia rheumatica (PMR) - sudden onset of pain and stiffness in muscles (pelvis, shoulder) of the body and is seen in the elderly c) other diseases associated with temporal arteritis are SLE, RA, and severe infections d) GCA can involve branches of the aorta as well, leading to an aortic aneurysm or dissection. For this reason, patients should be followed with serial chest X-rays

Describe the concept and causes of Acute Kidney Injury (AKI) leading to ARF

1) age >75 2) chronic kidney disease 3) cardiac failure/MI 4) peripheral vascular disease 5) chronic liver disease 6) diabetes 7) drugs (esp newly started) including diuretics 8) sepsis 9) poor fluid intake/inc losses 10) history of urinary symptoms 11) renal artery thrombosis 12) massive hypotension/haemorrhage 13) burns 14) pancreatitis 15) globinaemias 16) pregnancy (pre-eclampsia, eclampsia, abruption placentae) NB/ essentially, pre-renal causes lead to ATN - may develop into "renal failure"

Describe the pathology underlying COPD and emphysema

1) airway inflammation - chronic inflammation of airways due to inhaling irritants (eg/ tobacco smoke) a) causes neutrophils, CD8+ T-lymphocytes, B cells & macrophages to accumulate - initiate inflammatory cascade which triggers release of inflammatory mediators eg/ TNF-a, IFN-y, MMP-6 + 9, CRP & IL1,6,8. - leads to tissue damage & inflammatory effects 2) structural changes - airway modelling leads to narrowing of airways a) caused by: peribronchial fibrosis, build up of scar tissue from damage to airways, over-multiplication of epithelial cells lining airways b) parenchymal destruction due to destruction of supporting structures of alveoli = loss of elasticity (emphysema) - small airways collapse on exhalation, impeding air flow 3) mucocillary dysfunction - smoking + inflammation enlarge mucus glands lining airway, causing goblet cell metaplasia & healthy cells replaced by mucus-secreting cells a) inflammation causes damage to mucocillary transport system responsible for clearing mucus in airways - both = excess mucus which blocks airways 4) major pathological finding in chronic bronchitis = inc in no of mucus secreting goblet cells in bronchial mucosa, particularly larger bronchi NB/ small airways affected earlier in disease 5) emphysema = dilation & destruction of lung tissue distal to the terminal bronchiole, classified according to site of damage into centri-acinar (concd around respiratory bronchioles, more distal branches preserved), pan-acinar (less common destruction of whole acinus & bullae formation) and irregular (parenchymal, no regard for acinar structure) 6) normally - inflammatory proteases inactivated by eg/ alpha-1-antitrypsin a) if not inactivated, lung tissue is destroyed b) inc inflammation (smoking) or dec anti-protease activity (a1a deficiency) will result in net destruction of lung tissue c) emphysema leads to expiratory airflow limitation & air trapping d) loss of elasticity inc TLC, loss of alveoli leads to dec gas exchange & v/q mismatch - leads to fall in PaO2 & inc respiratory work demand e) release of elastase by neutrophils may help produce emphysema f) alpha-1 antritrypsin deactivated by cigarette smoke thus proteinase goes wild - smoke interferes with surfactant, favouring overdistention

Outline the treatment for acute ventilatory failure

1) airway obstruction - a) 1st line - clear airway eg/ manually, direct laryngoscopy or bronchoscopy, emergency cricothyroidotomy or tracheostomy to manage structural obstructions at the glottic or infra-glottic level b) + O2 - by nasal cannula, mask, or non-invasive positive pressure ventilation (NIPPV) to all hypoxic patients as part of, or immediately after, initial ABCDE assessment c) +/- tx underlying cause eg/ abx for infection, adrenaline for anaphylaxis, opiate reversal with naloxone, bronchodilation/corticosteroid therapy for chronic lung disease, decompression/chest tube insertion for pneumothorax, fluid resuscitation for hypovolaemia, thrombolysis/embolectomy for PE, radiotherapy for malignancy, and surgery for trauma or malignancy 2) no airway obstruction - a) conscious & stable - I) 1st line - O2: by nasal cannula, mask, or non-invasive positive pressure ventilation (NIPPV - only if awake) as part of, or immediately after, the initial ABCDE assessment II) +/- tx of cause - see above III) 2nd line - NIPPV - NIPPV can be used for conscious patients with spontaneous respiration and intact gag reflexes, and without rapid deterioration or vital organ compromise. BiPAP or CPAP favoured for respiratory failure secondary to acute congestive heart failure as they are not associated with a significant decrease in cardiac output b) unconscious & stable - I) 1st line - O2: by nasal cannula or mask; tx underlying cause II) 2nd line - endotracheal intubation and mechanical ventilation III) +/- rapid sequence induction (RSI) - intubation can be facilitated using RSI of anaesthesia (with sedatives and paralytics) prior to intubation c) unstable - I) 1st line - O2 II) + endotracheal intubation and mechanical ventilation III) +/- rapid sequence induction (RSI) IV) +/- tx of underlying causes

Discuss the different types of skin disinfectants used in theatres and surgical wards

1) alcohols eg/ ethanol, isopropanol - hand disinfection in outpatient clinic a) wide microbicidal activity, noncorrosive; but not universally sporicidal, limited residual activity, fire hazard 2) aldehydes eg/ glutaraldehyde, ortho- phthaladehyde - disinfecting equipment a) wide microbiocidal activity, sporicidal and fungicidal; but potentially toxic 3) oxidising agents - a) NaOH - disinfecting surfaces & water & equipment I) wide microbiocidal activity, sporicidal and fungicidal; but corrosive b) hydrogen peroxide - used for fogging (high level disinfectant in theatre) I) wide microbiocidal activity, sporicidal and fungicidal; but unstable compound 4) phenolics - surface disinfectant a) easily available & cheap; but corrosive to skin & resistance common 5) quaternary ammonium compounds - low level disinfectant a) active against enveloped viruses; but low sporicidal activity

For each type of shock outline the general principles of fluid, pharmacological and surgical management as appropriate

1) all patients - a) 1st line - assessment of airway, breathing, and circulation: ensure a patent airway (intubation if required), supplemental O2 if needed, continuous monitoring to monitor response to therapy and to guide treatment b) + tx of underlying cause: I) tension pneumothorax - urgent decompression by needle thoracocentesis followed by a chest drain II) cardiogenic shock secondary to a large myocardial infarction - urgent re-vascularisation of coronary arteries, either by angioplasty or surgery III) cardiac tamponade - drainage; pericardiocentesis under ECG monitoring, a pericardial drain or surgical pericardial window may be required IV) PE - thrombolysis or emergency embolectomy V) anaphylactic shock - IM adrenaline, supplemented by high-dose corticosteroids and antihistamines VI) septic shock - generous IVI & early broad-spectrum abx b) +/- vasopressors: noradrenaline - 0.5-1 microgram/minute IV, or dopamine - 5-10 micrograms/kg/minute IV c) cardiogenic shock not suspected: + IVI; +/- blood products; +/- tranexamic acid cardiogenic shock suspected: +/- IVI; +/- glyceryl trinitrate (5-20 micrograms/minute IV); +/- inotropic support (dobutamine - 2.5 micrograms/kg/minute IV); +/- intra-aortic balloon pump

Outline medical and surgical management for TIA

1) all types - a) + lipid-lowering agent eg/ atorvastatin: 10-80 mg, or simvastatin or rosuvastatin etc b) with HTN - + lifestyle modifications +/- antihypertensive therapy eg/ salt restriction, weight loss, healthy diet, exercise, and limited alcohol consumption, antihypertensive therapy should be started in patients with a BP of ≥130/80 mmHg; diuretics, ARBs, and ACEI have all been studied and found effective in reducing stroke risk c) with suboptimal lifestyle - + lifestyle modifications eg/ stop smoking, dec alcohol, dec weight, inc exercise control diabetes 2) atherosclerotic or small-vessel TIA - a) 1st line - antiplatelet therapy eg/ aspirin: 75-300 mg PO OD, or aspirin/dipyridamole or clopidogrel (75 mg); antiplatelet should be started in the first 24 hours, after intracranial hemorrhage is ruled out b) with ≥50% carotid stenosis - + carotid endarterectomy or stent 2) cardioembolic TIA without ischaemic heart disease - a) 1st line - anticoagulation eg/ warfarin: 2-10 mg, or dabigatran or rivaroxaban etc b) 2nd line - antiplatelet therapy eg/ aspirin: 300 mg; only if anticoagulation is CI 3) cardioembolic TIA and ischaemic heart disease - a) 1st line - anticoagulation eg/ warfarin: 2-10 mg, or dabigatran etc 4) mechanical heart valves or rheumatic heart disease already on therapeutic anticoagulation - a) 1st line - aspirin + continued warfarin eg/ warfarin: 2-10 mg orally once daily initially, adjust according to INR; + aspirin: 75 mg orally once daily

List the anatomical and physiological factors predisposing to gastro-oesophageal reflux disease

1) anatomical - 1+ feature of: a) malformation in oesophageal hiatus in diaphragm b) lower oesophageal sphincter (high-pressure zone (HPZ) dysfunction - transient relaxation of LES (most common), permanent LES relaxation, and transient inc of intra-abdominal pressure that overcomes LES pressure eg/ delayed gastric emptying 2) physiological features: a) failure of oesophageal defense mechanisms - oesophageal clearance and mucosal resistance b) possibly - hiatus hernia c) obesity - inc oesophageal acid exposure d) pregnancy e) trauma f) poor posture g) spicy food h) Zollinger-Ellison syndrome i) H. Pylori j) NSAIDs k) smoking/alcohol

Discuss a diagnosis of angina with a patient in lay terms

1) angina is a pain that comes from the heart, caused by narrowing of the heart (coronary) arteries due to an atheroma = dec blood supply to a part, or parts, of heart muscle a) blood supply may be good enough when you are resting, when heart works harder (walk fast or climb stairs & HR inc) heart muscle needs more blood and oxygen. If the extra blood that your heart needs cannot get past the narrowed coronary arteries, the heart responds with pain. 2) tx - statin medicine to dec cholesterol, low-dose aspirin to help prevent a heart attack, B-blocker to help protect heart & prevent angina pains, ACEI in some cases, sometimes angioplasty or surgery to widen, or to bypass, narrowed arteries

Outline treatment options of angioplasty or coronary artery bypass grafting

1) angioplasty (aka balloon angioplasty or percutaneous transluminal angioplasty) - minimally invasive, endovascular procedure to widen narrowed or obstructed arteries or veins, typically to treat arterial atherosclerosis a) deflated balloon attached to a catheter is passed over a guide-wire into the narrowed vessel and then inflated to a fixed size b) balloon forces expansion of the blood vessel & surrounding muscular wall, allowing an improved blood flow c) stent may be inserted at the time of ballooning to ensure the vessel remains open, and the balloon is then deflated and withdrawn 2) coronary artery bypass - surgical procedure to restore normal blood flow to an obstructed coronary artery a) 2 main approaches - L internal thoracic artery (internal mammary artery) is diverted to the left anterior descending branch of the left coronary artery, or great saphenous vein is removed from a leg; one end is attached to the aorta or one of its major branches, and the other end is attached to the obstructed artery immediately after the obstruction to restore blood flow b) surgery usually performed with the heart stopped, necessitating the usage of cardiopulmonary bypass, however can be done on a beating heart either without using the cardiopulmonary bypass

Discuss the indications and contraindications for primary percutaneous coronary intervention and thrombolysis

1) anti-platelets: 300mg PO aspirin + clopidogrel 2) thrombolysis: a) ECG criteria - ST elevation >1mm in 2+ limb leads, or >2mm in 2+ chest leads or new LBBB or posterior changes (deep ST depression & tall R waves in leads V1-3) b) contraindications - previous intracranial haemorrhage, GI bleeding (<1 month), ischaemic stroke <6 months, cerebral malignancy/AVM, recent surgery/trauma/head injury (< 3 weeks), known bleeding disorder, aortic dissection, non-compressible punctures <24hrs eg/ liver biopsy, lumbar puncture c) relative CI - TIA <6months, oral anticoagulant therapy, pregnancy <1 week post-partum, refractory hypertension (>180/110), advanced liver disease, IE, active peptic ulcer, prolonged/traumatic resuscitation d) use - tissue plasminogen activators eg/ Alteplase, reteplase, tenecteplase 3) primary PCI: offered to all with acute STEMI & can be at a PCI centre within 2hrs of medical contact. Non-surgical procedure to treat stenosis of coronary arteries - coronary angioplasty, using a balloon catheter in which a deflated balloon is advanced into the obstructed artery and inflated to relieve the narrowing 4) rescue PCI: if PCI can't be performed (>2hrs to centre) fibrinolysis is performed. If this fails have rescue PCI

Outline measures undertaken in secondary stroke prevention

1) anticoagulation with warfarin for atrial fibrillation - INR should be 2.0-3.0 2) antiplatelet drug therapy for non-atrial fibrillation patients eg/ clopidogrel, aspirin 3) carotid endarterectomy for carotid artery stenosis 4) statin therapy with intensive lipid-lowering effects is recommended for patients with ischaemic stroke or TIA, to lower the risk of stroke and cardiovascular events 5) in stroke patients or those with TIA and obesity, the treating physician should consider sleep studies because sleep apnoea is common among this subgroup of patients and treatment of apnoea with continuous positive airway pressure may improve outcomes 6) in patients with embolic stroke of undetermined source and a patent foramen ovale (PFO) who have a high ROPE score, closure of the PFO may be of benefit for secondary stroke prevention

Describe the location of Broca's and Wernicke's areas and explain their function in language

1) aphasia: partial or complete loss of language skills resulting from an organic cause 2) Broca's aphasia (expressive aphasia, non-fluent aphasia) - L inferior frontal gyrus = Broca's area - required for fluent speech production skills a) Broca's aphasia: spontaneous speech confined to single words or syllables, speech described as 'telegraphic' - highly reduced form of speech in which unessential words left out, lack of function words: 'for' 'with' 'to' 'about' etc, anomia, agrammatism 3) Wernicke's aphasia (receptive aphasia, fluent aphasia) - L sup temporal gyrus = Wernicke's area - required for reception & processing of language a) Wernicke's aphasia: fluent speech production but meaningless unintelligible strings of words, sounds, jargon - 'word salad', sometimes anomia, anosognosia, can't monitor language production, semantic paraphrasias (confuse meanings) eg/ 'horse' for 'cow', deep dyslexia

Describe the CT scan appearance of a subdural haemorrhage, and how it would change with time

1) appearance of SDHs on CT varies with clot age and organisation - 2) hyperacute - 1st hour, relatively isodense to the adjacent cortex, with a swirled appearance due to a mixture of clot, serum and ongoing unclotted blood. Often a degree of underlying cerebral swelling (especially if young) which accentuates the mass-effect created by the collection 3) acute - crescent-shaped homogeneously hyperdense extra-axial collection that spreads diffusely over the affected hemisphere. As the clot starts to retract the density increases & is thus hyperdense relative to the cortex 4) subacute - as the clot ages and protein degradation occurs, the density starts to drop. At some point between 3-21 days, the density will drop & become isodense to the adjacent cortex, making identification potentially tricky, Key to identification is visualising a number of indirect signs, including: CSF-filled sulci do not reach the skull but rather fade out into the subdural; mass-effect including sulcal effacement (distortion) and midline shift ; apparent thickening of the cortex 5) chronic - >3/52 old, subdural collection becomes hypodense & be isodense to CSF, and mimic a subdural hygroma, crescentic shape may change to a biconvex one 6) acute on chronic - 2nd episode of acute haemorrhage into a pre-existing chronic subdural haematoma. It typically appears as a hypodense collection with a haematocrit level (located posteriorly)

Demonstrate through discussion understanding of the importance of the 'aqueous environment' in body composition, the distribution of fluids and electrolytes in the body compartments, and the role of the kidney in regulating fluid and electrolyte balance

1) aqueous fluid - a) chemical reactions of life take place in aqueous solutions, an appropriate balance of solutes inside and outside of cells must be maintained to ensure normal function b) humans are 50-60% water, brain and kidneys have the highest proportions of water, which composes 80-85% of their masses 2) distribution of fluids & electrolytes - a) intracellular fluid (ICF) compartment includes all fluid enclosed in cells by their plasma membranes. ICF is the principal component of the cytosol/cytoplasm & makes up 60% of the total water in the human body. Fluid volume tends to be very stable, because the amount of water in living cells is closely regulated. If the amount of water inside a cell falls to a value that is too low, the cytosol becomes too concentrated with solutes to carry on normal cellular activities; if too much water enters a cell, the cell may burst and be destroyed b) extracellular fluid (ECF) surrounds all cells in the body & has 2 primary constituents: fluid component of the blood (plasma) and the interstitial fluid that surrounds all cells not in the blood. Makes up 33% body's water content with 20% ECF found in plasma. Plasma travels through the body in blood vessels and transports a range of materials, including blood cells, proteins (including clotting factors and antibodies), electrolytes, nutrients, gases, and wastes. Gases, nutrients, and waste materials travel between capillaries and cells through the IF. Cells are separated from the IF by a selectively permeable cell membrane that helps regulate the passage of materials between the IF and the interior of the cell c) blood plasma has high concentrations of sodium, chloride, bicarbonate, and protein. The IF has high concentrations of sodium, chloride, and bicarbonate, but lower concentration of protein. In contrast, the ICF has elevated amounts of potassium, phosphate, magnesium, and protein. Overall, the ICF contains high concentrations of potassium and phosphate (HPO42−HPO42−), whereas both plasma and the ECF contain high concentrations of sodium and chloride 3) role of the kidney in regulating fluid and electrolyte balance - a) kidneys match renal excretion to intake of water and electrolytes to regulate the osmolality and volume of body fluids. Deficits of water or electrolytes are compensated for by inc intake and retention, whereas excesses are compensated for by inc urinary excretion. Works through secretion of ADH which facilitates uptake of water in distal collecting tubule b) kidneys are the main route for the excretion of salt and water and have an important role in the control of body fluid osmolality and ECF volume

Differentiate symptoms and signs of acute arterial from acute venous occlusion

1) arterial = 6Ps: a) pain - earliest & major sign, rapid peak, sharp, distal to or below obstruction b) paresthesia - sensory; touch, pressure, numbness; motor- can't move- not recover c) pallor - mottled, no edema d) pulse changes - diminished to absent e) poikilothermia - adapt to air temperature f) paralysis - muscle rigidity 2) arterial - acute, dramatic changes & sudden, 1 extremity, pain unrelenting - distal to or below obstruction, absent or diminishing pulse 3) venous - a) dull ache/cramping not reproducible consistently with activity b) unilateral or bilateral c) no neurological changes or deficits d) pain relieved by elevation- worse later in the day, less at night e) thick, tough, woody, brawny, brown pigmented skin f) veins full when leg slightly dependent g) scarring from recurrence of ulcers h) pulses intact - may be difficult to locate due to oedema i) swelling, redness, warmth and pain j) engorgement of superficial vessels k) ankle oedema l) Homan's sign (pain on dorsiflexion of foot) may be present

Describe the vascular abnormalities which may predispose a patient to developing a subarachnoid haemorrhage

1) arteriovenous malformation - congenital defect, tangle of abnormal and poorly formed blood vessels (abnormal connections between arteries and veins without capillaries). Higher rate of bleeding than normal vessels due to abnormal BP distribution 2) vasculitis - inflammation of blood vessel wall involving the brain and occasionally spinal cord. Affects all vessels. If blood flow in a vessel with vasculitis is reduced or stopped, the parts of the body that receive blood from that vessel begins to die. Can produce a wide range of neurological symptoms eg/ headache, skin rashes, feeling very tired. 10% have associated bleeding in brain 3) abnormal blood vessels associated with tumour 4) berry aneurysm - congenital weakness in elastic tissues lining arteries. Thin walled aneurysms develop at sites of deficiency in arteries eg/ branch points. 40% occur at ICA, 30% at anterior communicating artery, and 20% at MCA. Often multiple, and may range in size from 1-2mm to 1-2cm

Describe the clinical presentation of a patient with cystic fibrosis with respect to disease of the lung and pancreas

1) as normal digestive function is possible with <5% pancreatic function, CF can present at any age - may have no s/s 2) most common presentation = recurrent LRTI with chronic sputum production 3) antenatal - a) amniocentesis/chorionic villus sampling (CVS) b) US shows bowel perforation/hyperechogenic bowel 4) perinatal - a) screening - immunoreactive trypsinogen (IRT) measured on Guthrie card at day six of life b) bowel obstruction with meconium ileus (bowel atresia). c) haemorrhagic disease of the newborn d) prolonged jaundice 5) infancy and childhood - a) recurrent respiratory infections b) diarrhoea, rectal prolapse c) failure to thrive d) nasal polyps e) acute pancreatitis f) portal HTN & variceal haemorrhage g) pseudo-Bartter's syndrome, electrolyte abnormality h) hypoproteinaemia and oedema 6) adolescence/adulthood - a) screening b) recurrent respiratory infections c) atypical asthma or bronchiectasis d) male infertility & congenital bilateral absence of vas deferens e) heat exhaustion/electrolyte disturbance f) portal HTN & variceal haemorrhage 7) signs - a) finger clubbing b) cough with purulent sputum c) crackles & wheezes (mainly in upper lobes) d) FEV1 showing obstruction

Describe the clinical features of the main conditions associated with asbestos inhalation

1) asbestososis - inhalation of asbestos fibres - mix of silicates of Fe, Mg, nickel, cadmium + Al resistant to heat, acid and alkali causes restrictive ventilatory defects a) chrysotile (90%) = least fibrogenic, crocidolite (6%) = most fibrogenic b) degree of asbestos exposure related to degree of pulmonary fibrosis (may involve fibrosis of pleura) c) progressive dyspnoea, clubbing, bilateral basal fine end inspiratory crackles d) inc risk of bronchial adenocarcinoma + mesothelioma e) management - symptomatic, pts eligible for compensation, can give corticosteroids 2) malignant mesothelioma - tumour of mesothelial cells due to asbestos irritation, usually in pleura, can be in peritoneum or other organs - 90%report exposure to asbestos but only 20% have asbestososis, latent period can be up to 45yrs a) chest pain, dyspnoea, weight loss, finger clubbing, recurrent pleural effusions b) signs of metastases - lymphadenopathy, hepatomegaly, bone pain, abdominal pain/obstruction c) tests - CXR/CT, pleural thickening/effusion, bloody pleural fluid d) diagnosis - made on history after thorascopy, often only made at post-mortem e) management - pemetrexed + cisplatin chemotherapy, surgery & radiotherapy controversial - prognosis is poor (esp without pemetrexed <2yrs) NB/ fibrosis may be identified on CT scan (honeycomb lung) but not on CXR NB2/ no of cases in UK has inc since 1980 & expected to peak 2010-2020

Outline the general principles in the assessment and treatment of a patient who has taken an overdose

1) assessment - a) ABCDE - evaluate if any time critical features: I) impaired ABCD's II) dec LOC or RR III) extreme hypotension (BP <70 mmHg) common in sedative and anti-depressant overdose IV) arrhythmias V) convulsions VI) hypothermia or hyperthermia b) hx - I) what (drug), where, when (how long ago), how (method of poisoning eg/ ingestion, inhalation) II) quantity of substance III) have they had any tx yet eg/ from carers, or health professionals IV) intentional overdose/self-harm: urgently establish the likely physical risk, the person's emotional and physical state, and any requirement for further support services eg/ police. A rapid mental health assessment should be undertaken including assessing suicide risk 2) tx - in all cases of overdose, management is based upon: a) identification of poisons b) specific treatment for specific poisons - consult toxbase etc c) rapid access to hospital d) tx concurrent problems eg/ convulsion, hypothermia e) tx ABCDE: I) A/B - high concn O2 via non-re-breathe mask so O2 sats >95% (except COPD where 88-92%) II) C - IV access III) D - always check blood glucose level and correct if <4.0mmol/l with glucose 10% IV) E - remove pt from source of exposure eg/ if liquid irrigate V) if patient vomits, retain a sample, for inspection at hospital, but don't induce vomiting. If swallowed caustics and petroleum products dilute by giving a glass of milk at the scene

Name the main anatomical landmark(s) used in guiding a lumbar puncture, and the coincident level in the spine

1) at birth, the inferior end of the spinal cord is opposite the body of L3, distal to this point is the cauda equina and its nerve roots a) with ageing the vertebral column grows much faster than the spinal cord so, by adulthood, the spinal cord only reaches the inferior border of L1, or the superior aspect of L2, distal to this is the cauda equina 2) in order to avoid transfixing the spinal cord during LP, the needle is placed distal to L2 - means needle enters SA space at level of cauda equina 3) landmarking - imaginary line that crosses the lumbar region of the back joining the posterior superior iliac crests, as crosses L3-L4 interspace, then find L2-L3 space and L4-L5 space and use the one with the biggest space 4) CSF resides in SA space between pia mater & arachnoid mater, so needle passes between two vertebral processes and continues through the interspinal tissues and into the subarachnoid space 5) tissues pierced are (in order): skin, subcutaneous tissue, supraspinal ligament, interspinal ligament, ligamentum flavum, dura mater, the arachnoid mater and into the subarachnoid space

Describe the common sites and relative incidence of atherosclerotic arterial aneurysms; list the symptoms, signs and differential diagnosis of a ruptured abdominal aortic aneurysm and outline an emergency management plan

1) atherosclerotic arterial aneurysm - a) more common sites are in descending, strong-flow vessels (in descending order of incidence): abdominal aorta, iliac, popliteal, femoral arteries and thoracic aorta b) berry aneurysms occur more in circle of Willis but are not atherosclerotic. c) AAA present in 5% population >60 years, 5x more common in males and develops in 1 in 4 children of affected individuals (some familial component) d) s/s of AAA - intermittent or continuous abdominal pain that radiates to the back, iliac fossa, or groins (don't dismiss as renal colic) I) collapse (hypotension) II) expansile abdominal mass III) shock IV) tachycardia V) profound anaemia VI) sudden death VII) if aneurysm is throwing off emboli, pts may have "trash feet" - discoloured digits secondary to arterial occlusion e) DD of AAA - I) acute gastritis II) appendicitis III) pancreatitis IV) gallstones (cholelithiasis) V) large or small-bowel obstruction VI) myocardial infarction NB/ asymptomatic until rupture f) mx - g) screening - ) AAA can be discovered by a simple abdominal aortic aneurysm screening or USS. Low cost outpatient scan is a highly accurate, painless and completely safe 2) screening has been proven to lower the death rate in the at-risk population. British Multicenter Aneurysm Screening Trial - pts screened had dec death rate from AAA by 50% over four years 3) people who undergo screening: men & women over 60 who have these risk factors - a) smokers - over 100 cigarettes in their life (especially heavy smokers) b) diabetics c) known CV disease (high cholesterol, heart problems, stroke history, high BP) d) obese individuals e) family history of AAA NB/ aneurysm = permanent dilation of artery to 150% normal diameter. Operation if aneurysm >5.5cm, expanding >1cm/year, symptomatic. Pts with AAA <5.5cm followed up with regular US surveillance. Prophylactic surgery preferred to emergency surgery with rupture

Describe ECG features of left and right bundle branch block and atrial and ventricular arrhythmias

1) atrial - a) AF (see below) - no P waves, irregularly irregular b) atrial flutter - rate of 150 bpm - complex tachycardia with 2:1 conduction, reveal flutter waves by slowing ventricular response, showing underlying atrial rhythm, saw-toothed pattern c) heart block d) Wolff-Parkinson-White syndrome - abnormalities caused by extra muscle pathways between the atria and the ventricles so electrical signals arrive at the ventricles too soon, and the signals are sent back to the atria, ECG - PR interval <120ms, Delta wave - slurring slow rise of initial portion of the QRS, QRS prolongation >110ms, ST Segment and T wave discordant changes - i.e. in the opposite direction to the major component of the QRS complex, pseudo-infarction pattern e) premature supraventricular contractions - atria contract too soon, causing the heart to beat out of sequence, ECG - abnormal (non-sinus) P wave is followed by a QRS complex, P wave typically has a different morphology and axis to the sinus P waves, abnormal P wave may be hidden in the preceding T wave = "peaked" or "camel hump" appearance 3) ventricular - a) ventricular tachycardia - SA node no longer controls the beating of the ventricles, so other areas along the lower electrical pathway take over the pacemaking role. Heart muscle does not beat normally, inc pulse, feel as if your heart is "skipping beats", ECG - broad complex tachycardia b) ventricular fibrillation - instead of one misplaced beat from the ventricles, have several impulses that begin at the same time from different locations—all telling the heart to beat, faster, chaotic heartbeat that sometimes reaches 300 beats a minute, ECG - chaotic irregular deflections of varying amplitude, no identifiable P waves, QRS complexes, or T waves, rate 150 to 500 per minute, amplitude decreases with duration c) premature ventricular contractions - ventricles contract too soon, out of sequence with the normal heartbeat, ECG - broad QRS complex (≥ 120 ms) with abnormal morphology, premature — i.e. occurs earlier than would be expected for the next sinus impulse, discordant ST segment and T wave changes, usually followed by a full compensatory pause, retrograde capture of the atria may or may not occur d) supraventricular arrhythmias - begin in atria or atrial conduction pathways e) supraventricular tachycardia (SVT) or paroxysmal supraventricular tachycardia (PSVT) - electrical signals in atria fire abnormally, which interferes with electrical signals coming from the SA node, causes abnormally fast heart rhythm arising from improper electrical activity in the upper part of the heart. 4 main types: atrial fibrillation, paroxysmal supraventricular tachycardia (PSVT), atrial flutter, and Wolff-Parkinson-White syndrome, ECG - rapid, regular heart rate of 150-250 bpm in atria

Describe the pathway of care developed within the hospital for STEMI

1) attach ECG monitor & record 12 lead ECG 2) IV access - bloods for FBC, U&E, glucose, lipids, cardiac enzymes 3) brief assessment - a) history of CV disease, risk factors for IHD b) exam - pulse, BP (both arms), JVP, murmurs, signs of CCF, upper limb pulses, scars from previous cardiac surgery, CXR if needed c) contraindications to PCI or fibrinolysis? 4) aspirin 300mg (unless given by GP/paramedics) 5) morphine 5-10mg IV + metoclopramide 10mg IV 6) STEMI on ECG - PCI available within 120 min? a) yes - primary PCI b) no - fibrinolysis, transfer to primary PCI centre for rescue PCI if fibrinolysis unsuccessful, or angiography NB/ "MONA" = Morphine, Oxygen, Nitrates, Aspirin

List the common situations in which appendicitis is difficult to diagnose or manage

1) atypical presentation 2) pregnant women 3) appendix in different part of the abdomen

Outline the treatment options for hepatitis due to autoimmunity, hepatitis B or C and paracetamol overdose

1) autoimmune - a) non-severe disease - I) 1st line - observation and monitoring only II) expected tx <6 months or immunosuppressant contraindication: +/- corticosteroid monotherapy: prednisolone III) expected tx >6 months with no immunosuppressant contraindication: +/- corticosteroid + immunosuppressant therapy: prednisolone + azathioprine or mercaptopurine b) severe disease - I) as above +/- liver transplant 2) hep B - a) acute HBV infection - I) 1st line - supportive care II) with fulminant hepatitis or hepatic failure: +/- antiviral therapy ± liver transplant: lamivudine or entecavir or tenofovir disoproxil b) chronic HBV infection - I) 1st line - antiviral therapy or peginterferon: entecavir or tenofovir disoproxil or peginterferon alfa 2a c) chronic HBV infection with cirrhosis - I) with compensated cirrhosis: antiviral therapy or peginterferon - entecavir or tenofovir disoproxil or peginterferon alfa 2a II) with decompensated cirrhosis: antiviral therapy - entecavir or tenofovir disoproxil; +/- evaluation for liver transplant 3) hep C - a) recent infection - I) 1st line - monitor patient or start treatment b) chronic infection - I) antiviral therapy: elbasvir/grazoprevir or glecaprevir/pibrentasvir or ledipasvir/sofosbuvir or sofosbuvir/velpatasvir 4) paracetamol overdose - a) <8 hours since ingestion: 1st line - acetylcysteine: 150 mg/kg IV infusion over 1 hour, then 50 mg/kg infusion over 4 hours, then 100 mg/kg infusion over 16 hours I) once the 4-hour paracetamol level is obtained, it should be plotted against time since ingestion on the relevant nomogram. If the plot falls on or above the appropriate treatment line, the patient should be treated with acetylcysteine. If the plotted level falls below the treatment line, most patients will not require acetylcysteine II) blood tests are repeated after completion of a typical course of IV acetylcysteine therapy (21 hours) III) + supportive care: may need intubation and ventilation or vasoactive infusions for blood pressure support, psychological support for mental illness, and education for those with accidental overdoses IV) +/- anti-emetic: ondansetron; +/- activated charcoal (may be considered in a cooperative patient presenting within 2 hours ingestion) V) >8 hours since ingestion: acetylcysteine + supportive care +/- anti-emetic VI) +/- evaluation for liver transplant

Classify the causes of hyperthyroidism and outline the pathological features of Graves' disease, toxic adenoma and toxic multinodular goiter

1) autoimmune thyroid disease aka Graves (80%) - mainly women 40-60, circulating IgG autoantibiodies bind to & inactivate GPC-thyrotropin-R = smooth thyroid enlargement & inc hormone production (esp T3), also react with orbital autoantigens 2) toxic adenoma - solitary nodule producing T3 + T4 (5%) - on isotope scan the nodule is hot and the rest of the gland is suppressed 3) toxic multinodular goiter (15%) - seen in elderly & iodine deficient. Nodules secrete thyroid hormones - surgery for compressive symptoms from enlarged thyroid NB/ may affect 2-5% of women (sex ratio 5:1) aged 20-40 NB2/ also - de Quervain's thyroiditis (5%), postpartum thyroiditis, amiodarone-induced thyrotoxicosis NB/ features of Grave's - 1) eye disease - exopthalmos, opthalmoplegia 2) pretibial myoxedema 3) thyroid acropachy

Describe the common bacterial and viral organisms causing meningitis in adult life

1) bacterial - a) Hib in many developing countries b) S pneumoniae is the most common cause of bacterial meningitis in the US and many countries worldwide c) N meningitidis d) Listeria monocytogenes - in pts using immunosuppressive drugs, people who abuse alcohol, and patients with diabetes mellitus 2) viral - a) human enteroviruses are the most common cause of viral meningitis - include coxsackieviruses, echoviruses, and polioviruses b) herpes viruses eg/ herpes simplex virus (HSV)-2 c) arboviruses cause neurological infection in many parts of the world eg/ West Nile virus, Eastern and Western equine encephalitis viruses, La Crosse virus, and St. Louis encephalitis virus

Describe the diagnostic features and treatment of common skin infections

1) bacterial - a) cellulitis - infection of loose connective tissue I) most commonly group A streptococcus pyogenes II) s/s - fever, malaise and systemic upset. Affected limb (mainly lower) normally erythematous, hot, painful and tender; if very severe blistering may occur and skin necrosis of dermis III) tx - oral or IV Benzylpenicillin or Flucloxacillin for 10-14 days (4-6 weeks if severe). Recurrent - prophylaxis with Penicillin b) impetigo - contagious superficial pyogenic infection of skin I) most common in young children, due to staph aureus II) s/s - skin lesions with yellow-crusted surface often around mouth, nose and limbs. Regional lymphadenopathy, rarely bullae III) tx - topical Mupirocin partially effective, oral Flucloxacillin (or erythromycin) for 7-10 days. Should bathe separately and use their own towel b) staph aureus - directly invades epidermis, hair follicle, or toxin made eg/ ecthyma (chronic infection of skin - ulcers with adherent crust, may have scarring), superficial folliculitis (infection of superficial hair follicle producing small pustules which heal without scarring), impetigo, furunculosis - boil (acute necrotic infection of a single hair follicle), carbuncle (collection of boils), infected eczema etc tx - Flucloxacillin 2) viral - a) shingles -dermatomal varicella-zoster infection due to reactivation of virus in nervous tissue in dorsal root ganglion cells and schwann cells I) s/s - pain & tingling for 24-48 hrs then dermatomal eruption of vesicles, can have post-herpetic neuralgia II) tx - if within 72 hours of pain/tingling: oral aciclovir for 5 days b) warts - many different types I) s/s of common wart - elevated multiple papules with a rough, papillomatous and hyperkeratotic surface, vary in size. Most often on backs of fingers or toes, around nails & knees. Can resemble a cauliflower II) tx - topical paints (from pharmacists), cryotherapy with liquid nitrogen, curettage & cautery - curettage followed by haemostasis using silver nitrate or electrocautery, formalin soaks and podophyllin - resistant warts c) herpes simplex - HSV Type I (cold sores) or HSV Type II (anogenital): both persist in sensory root ganglia after primary infection & can be shed asymptomatically (especially in 1st year) I) s/s of cold sores - commonly on vermillion border of lip, pain & tingling then acute vesicular eruption II) s/s of genital herpes - significant pain & tingling, discomfort then vesiculation III) tx - cold sores - topical acyclovir; genital herpes and complicated infections - oral aciclovir for 5+ days 3) fungal - a) Athlete's foot/tinea - many different types based on body part eg/ capitis, cruris, corporis, manuum, pedis, unguium I) s/s - unilateral with clear border, scaly plaques, outward spread with red scaly border and central clearing, pruritis II) tx - localised: topical anti-fungal agents eg/ Azoles (Miconazole) or topical Terbinafine; for nail, scalp and hair infection or widespread skin infection = systemic agent eg/ Terbinafine or Itraconazole b) yeast infections - I) s/s - small satellite lesions and sometimes pustules at the edges of the eruption, unlike the clear, well defined edge of tinea II) tx - polyenes eg/ Nystatin or Amphotericin B if systemic infection, other common tx - topical Azoles eg/ Clotrimazole or oral Fluconazole 4) parasitic - a) head lice - I) s/s - persistent itching in scalp (pruritis), erythema, excoriations, egg casings or nits - occasionally can see live lice on hair II) tx - fine combing to remove nit casings & using a suitable hair conditioner, pediculocides eg/ Malathion, Permethrin and Carbaryl not very effective & resistance reported - leave on scalp for 12h, repeat in 10 days b) scabies - I) s/s - itchy erythematous papules which become excoriated with time, can see burrows eg/ finger webs, flexures, axillae, breasts, ankles and insteps tx - Permethrin (leave on for 12hrs) or Malathion (leave on for 24hrs). Repeat 1/52 after initial therapy (days 1 & 8), tx any close contacts ie members of family, all members of a ward, wash clothes and bed linen

List the daily requirements of fat, protein and carbohydrate utilised by the body. Knowing a patient's weight and level of stress, calculate the daily requirements for calories, protein and carbohydrates

1) basic energy requirements (BER) = basal metabolic rate (BMR) + general daily activities 2) for every kg of body weight approximately 1.334 calories is required every hour eg/ person weighing 60kg needs 1.334 × 24hrs × 60kg = 1921 calories/day 3) dietary guidelines for americans recommends following blend: a) 55% carbohydrates (sugar, sweets, bread, cakes) b) 25% fats (dairy products, oil) c) 20% protein (eggs, milk, meat, poultry, fish) - 0.75g/kg NB/ UK - estimated average daily requirement is 2000 calories for women and 2500 for men 4) for every kg of body weight approximately 1.334 calories is required every hour eg/ person weighing 60kg needs 1.334 × 24hrs × 60kg = 1921 calories/day 5) 55% carbs, 20% protein 6) daily expenditure = BMR x Stress factor x Activity factor (PAL) 7) stress factors: 1.2 for elective surgery, 2.0 for severe burn, 1.5 for wound, infection or trauma 8) PAL: 1.2 for patient confined to bed, 1.3 for patients out of bed NB/ protein + CHO = (4kcal/g), fat = (9kcal/g), 4.2kJ = 1 kcal

Identify & define the physical findings associated with the acute abdomen & relate these to the basic underlying pathology

1) bending forward - chronic pancreatitis 2) jaundiced - CBD obstruction 3) dehydrated - peritonitis, small bowel obstruction 4) abdomen - flat, distended (ascites) etc 5) palpation - check hernia sites, rigidity on abdomen suggests peritonitis 6) PR and PV exams for abnormalities 7) important signs: a) Cullen's + Grey-Turner's - retroperitoneal haemorrhage b) Kehr's - splenic rupture or ectopic pregnancy rupture c) McBurney's - appendicitis d) Murphy's - acute cholecystitis e) Iliopsoas + obturator + Rovsing's- appendicitis f) Chandelier - pelvic inflammatory disease NB/ if a child or pregnant woman - most likely is appendicitis NB2/ pt may look ill, shocked, pale, sweaty, weak rapid pulse. Guarding and Rigidity suggest peritonitis. High pitched, tinkling bowel sounds suggest fluid obstruction. Distention suggests obstruction. Fever may be present

Describe the symptoms and signs of biliary colic and contrast with acute cholecystitis

1) biliary colic - a) gallstones (cystic duct obstruction or CBD obstruction) b) constant, crescendoing epigastrum/RUQ pain that radiates to back, R shoulder & subscapular - can be related to overindulgence of high fat food (normally mid-evening, lasting until early hours of morning) c) nausea & vomiting with severe attacks d) +/- jaundice 2) compared to acute cholecystitis - initial presentation may be similar to biliary colic - however, over a number of hours there is progression, with local, severe RUQ pain associated with tenderness & guarding, another differentiating factor is pyrexia & no inc WCC, occasionally gall bladder may become distended with pus (empyema) NB/ acute cholecystitis = initial stage is gall bladder obstruction. In 95% of cases a gall stone can be identified as the cause. Obstruction = inc glandular secretion, leading to progressive distension which may compromise vasculature. Inflammatory reaction secondary to obstruction. Infection is another secondary phenomenon

Outline the medical management of a patient with biliary colic and acute cholecystitis including appropriate antibiotic regimen

1) biliary colic - a) analgesia - NSAIDs and PRN opioids + antiemetic b) lifestyle - low fat diet, weight loss, inc exercise c) high chance of symptom recurrence or complications of gallstones eg/ cholecystitis, or acute pancreatitis, so elective cholecystectomy is warranted (or review for consideration for a cholecystectomy) within 6 weeks of first presentation 2) acute cholecystitis - a) IV abx eg/ co-amoxiclav +/- metronidazole b) fluid resuscitation c) if s/s of sepsis, management should be adapted accordingly d) NG tube should be placed if the patient is vomiting and the patient NBM for US e) analgesia - simple analgesics with PRN opioids + antiemetics f) laparoscopic cholecystectomy indicated within 1 week, however ideally done within 72hr of presentation; if not fit for surgery and not responding to abx, a percutaneous cholecystostomy can be performed to drain the infection

Explain how an extradural haemorrhage arises, including the artery involved

1) bleeding is typically from a torn meningeal artery, usually middle meningeal artery (75%) 2) pain (often severe headache) is caused by the stripping of dura from the bone by the expanding haemorrhage 3) occasionally, an EDH can form due to venous blood, typically a torn sinus with an associated fracture 4) EDHs are generally unilateral in > 95% of cases, however, bilateral or multiple EDHs are reported 5) >95% are supratentorial, temporoparietal: 60%, frontal: 20%, parieto-occipital: 20%

List the risks and complications of angiographic studies and describe their management

1) bleeding or bruising at the insertion site - pressure, pain killers if needed 2) tearing or cracking of the artery lining - surgery 3) blood clots - DVT, PE - VTE prophylaxis 4) infection - aseptic technique, abx 5) stroke - thromboprophylaxis 6) allergic reaction or kidney problems from the x-ray dye - adrenaline 7) heart attack, stroke, or death (very rare) 8) need for emergency cardiac or leg surgery (very rare) 9) abnormal heartbeat (arrhythmia) - cardioversion 10) impaired kidney function (very rare) 11) pseudoaneurysm (bubble) in the artery

Describe the investigation of a patient with suspected cirrhosis

1) blood - a) LFT- normal or inc bilirubin, inc ALT + AST + AP + GGT, later stages with loss of synthesis look for dec albumin + inc INR b) dec WCC + platelets = hypersplenism c) find cause - ferritin, iron/total iron binding capacity, hepatitis serology , immunoglobulins, autoantibodies, a-fetoprotein, caeruloplasmin in pts <40, a1-antitrypsin 2) liver ultrasound + duplex - may show: a) small or enlarged liver b) splenomegaly c) focal liver lesions d) hepatic vein thrombosis e) reversed flow in portal vein f) ascites 3) MRI - a) inc caudate lobe size b) smaller islands of regenerating nodules + presence of R posterior hepatic notch more common alcoholic than virus-induce cirrhosis c) MRI scoring system based in spleen + liver volume, presence of ascites or varices/collaterals can quantify severity of cirrhosis in a way that correlates with child grades d) ascitic tap - urgent MC&S (neutrophils >250/mm^3 indicates spontaneous bacterial peritonitis) e) liver biopsy NB/ always do history & exam first

Outline important investigations in infective endocarditis

1) blood cultures - 3 sets at different times from different sites - +ve blood cultures (typical organism in 2 separate cultures, or persistently +ve blood cultures) 2) echocardiography - vegetation, abscess, dehiscence of prosthetic valve, or new valvular regurgitation 3) ECG - shows evidence of emboli manifesting as MI or conduction defects 4) FBC - normochromic normocytic anaemia, neutrophilia (polymorphonuclear leucocytes) 5) ESR + CRP raised - can show response to therapy 6) renal function tests (U+Es) - look for microscopic haematuria + proteinuria NB/ -ve EchoCG does not disclude infective endocarditis. TOE has a high negative predictive value

Discuss the common causes of iron deficiency anaemia

1) blood loss eg/ menorrhagia, GI bleeding 2) poor diet (lack of Fe intake) 3) malabsorption eg/ coeliac disease 4) hookworm in tropics (most common cause) 5) supply less than demand (growth, pregnancy)

Outline appropriate investigations for acute appendicitis

1) blood tests - may reveal neutrophil leucocytosis & elevated CRP 2) USS may help to exclude some pathology, but appendix not always visualised 3) CT has high diagnostic accuracy & useful if diagnosis unclear - dec -ve appendicectomy rate but may cause fatal delay 4) 80-85% adults with appendicitis have WBC count >10,500 cells/µL 5) inc CRP 6) inc U-5-HIAA levels in acute appendicitis

Describe the investigation of a semi/unconscious patient

1) bloods - FBC, U&E, LFTs, blood glucose, clotting, calcium & bone profile, toxicology screen (paracetamol, alcohol), artierial blood gas, blood culture: give lots of useful information which may make the diagnosis 2) may need blood paracetamol & aspirin levels 3) CT head - shows space occupying lesions, bleeds and swelling of the brain, less commonly MRI 4) lumbar puncture if no CI - gives information about infection or bleeding (CSF xanthochromia) 5) EEG - may give useful information, especially if epilepsy is suspected 6) may have - ECG, C-xray, etc 7) bedside - urine dip, capillary blood glucose, pupils

Discuss the effect of chronic renal failure on blood and bone based on disturbance of normal renal function. Outline the treatment options and discuss the indications for dialysis. Outline the options for anaemia management and the principles of managing renal bone disease

1) bone - a) mineral and bone disorder in CKD occurs when damaged kidneys and abnormal hormone levels cause calcium and phosphorus levels in a person's blood to be out of balance b) kidneys stop activating calcitriol - low levels of calcitriol in the body create an imbalance of calcium in the blood c) kidneys don't remove the phosphorus in the blood properly, so phosphorus levels rise in the blood. This pulls calcium out of the bones, causing them to weaken d) when kidneys are damaged, the parathyroid gland releases parathyroid hormone into the blood to pull calcium from the bones and raise blood calcium levels. This response restores the balance of phosphorus and calcium; however, it also starves the bones of much-needed calcium e) tx - I) eating, diet, and nutrition - dec intake of phosphorus eg/ processed and packaged foods, beer, cheese, cocoa etc II) meds - if the kidneys do not make adequate amounts of calcitriol may prescribe synthetic calcitriol or, for dialysis patients, in an injectable form as calcitriol helps dec parathyroid hormone levels. Cinacalcet hydrochloride, a calcimimetics, dec parathyroid hormone levels by imitating calcium's effects on the parathyroid gland. May prescribe phosphate binders—such as calcium carbonate, calcium acetate, sevelamer carbonate, or lanthanum carbonate III) dialysis - inc dialysis dose can help control blood phosphorus level. In haemodialysis, the health care provider can adjust the dose by increasing how quickly the blood flows to and from the dialyzer. Another way to adjust the dose involves increasing the time of a person's dialysis session or the number of sessions. In peritoneal dialysis, using more dialysis solution in each fill or increasing the number of fills each day increases the dose IV) parathyroidectomy - if diet, meds, and dialysis can't control parathyroid hormone levels, a surgeon can remove one or more of the parathyroid glands 2) blood - a) anaemia - insufficient RBC due to the loss of kidney cells that produce erythropoietin hormone which stimulates bone marrow to make RBC b) uraemia - when kidneys fail wastes build up in your blood. When dietary protein is broken down a product called urea is formed. High levels of urea, called uremia, is toxic to the body and could lead to a dangerous inflammation of the outer layers of the heart, the pericardium. It is treated with dialysis 3) dialysis - a) process of filtering wastes and extra fluid from the body by means other than the kidneys. The two forms of dialysis are haemodialysis and peritoneal dialysis: I) haemodialysis uses a machine to circulate a person's blood through a filter outside the body. The blood passes from a patient's body through a needle, at nearly 1 pint per minute. The blood then travels through a tube that takes it to the filter, called a dialyzer. Inside the dialyzer, the blood flows through thin fibers that filter out wastes and extra fluid. After the machine filters the blood, it passes back to the body through another tube II) peritoneal dialysis uses the lining of the abdomen to filter a person's blood inside the body. A soft tube called a catheter is placed in the patient's abdomen a few weeks before peritoneal dialysis begins. A person uses the catheter to fill the empty space inside the abdomen with dialysis solution—a kind of salty water—from a plastic bag. While inside the body, the dialysis solution absorbs wastes and extra fluid. After a few hours, the person drains the used dialysis solution into another bag for disposal. The person then restarts the process with a fresh bag of dialysis solution III) CAVH or CAVHD - requires vascular access, systemic heparin, and continuous monitoring. Consider for postop patient with unstable haemo-dynamics, fluid overload, or catabolic patient on TPN b) indications - AEIOU"-"A"- intractable acidosis; "E"- electrolyte disarray (K+, Na+, Ca++); "I" - intoxicants (methanol ethylene glycol, Li, ASA); "O"- intractable fluid overload; "U"- uremic symptoms (nausea, seizure, pericarditis, bleeding). 4) options for anaemia management - a) initiation of ESA therapy in iron deficient patients (erythropoietic stimulating agent) therapy b) tx clinically relevant hyperparathyroidism c) pt education programmes

Distinguish between simple and tension pneumothorax including features that aid in recognition of critically ill patients presenting with a tension pneumothorax

1) both involve air in pleural space - in tension air cannot escape 2) clinical features - respiratory distress, tachycardia, hypotension, distended neck veins, trachea deviated from side of pneumothorax, inc percussion note, dec air entry/breath sounds on affected side 3) tension pneumothorax = valvular mechanism or pressure differences - air sucked into pleural space during inspiration, but not expelled during expiration a) lung deflates further, mediastinum shifts & venous return to heart dec b) tension rare unless pts on positive ventilation

Define femoral hernia

1) bowel enters femoral canal (normally occupied by fat and Cloquet's LN) presenting as mass in upper medial thigh or below inguinal ligament where it points down the leg, unlike inguinal canal which points to the groin 2) more often in women esp middle aged and elderly 3) likely to be irreducible & strangulate due to rigidity of canal's borders NB/ femoral canal - anterior border = inguinal ligament, medial border = lacunar ligament, lateral border = femoral vein, posterior border = pectineal ligament NB2/ Richter's hernias may occur in femoral canal - anatomically defined by only part of the intestinal wall folding through the femoral ring (like pinching one side of a hose)

Describe the clinical presentation of an epidural spinal abscess

1) bowel or bladder incontinence 2) difficulty urinating (urinary retention) 3) fever and back pain 4) clinical triad of fever, back pain, and neurologic deficit is not present in most patients 5) 4-phase sequential evolution has been described - (1) localized spinal pain, (2) radicular pain and paresthesias, (3) muscular weakness, sensory loss, and sphincter dysfunction, (4) paralysis 6) an evident source of infection in skin or soft tissue may be found 7) spinal cord syndromes, typically involving paraparesis with prospective progression to paraplegia (Epidural abscesses at the level of the cauda equina cause symptoms consistent with cauda equina syndrome rather than a spinal cord syndrome) 8) headache and neck pain may be present, especially with cervical epidural abscesses 9) reflexes may vary from absent to hyperreflexia with clonus and extensor plantar (Babinski) responses. Areflexia may indicate spinal shock with transient inhibition of spinal reflexes 10) nuchal stiffness or rigidity may be present, notably with cervical epidural abscesses

List the causes of dysarthria

1) brain tumour 2) cerebral palsy 3) Guillain-Barre syndrome 4) head injury 5) hypothermia 6) lyme disease 7) MS 8) Parkinson's disease 9) stroke 10) Wilson's disease 11) Intercranial Hypertension (formerly Pseudotumor Cerebri) 12) Tay-Sachs, and Late Onset Tay-Sachs (LOTS), disease

Common abnormalities of breath sounds

1) bronchial breathing - consolidation (eg/ pneumonia) or compression of the lung 2) crackles ('rales') - caused by fluid/mucous/pus in the small airways or atelectasis, may be heard on inspiration or expiration, can be course (eg/ ILD, fluid in lungs), medium, or fine (chronic bronchitis, pneumonia, CHF, or atelectasis) 3) wheeze ('rhonchi') - inspiratory or expiratory, polyphonic = asthma or COPD, monophonic - worry about malignancy, causes - pulmonary oedema, interstitial lung disease and chronic bronchitis 4) pleural rub - low-pitched, grating, or creaking sounds that occur when inflamed pleural surfaces rub together during respiration, mostly inspiration. To determine whether the sound is a pleural friction rub or a pericardial friction rub, ask the patient to hold his breath briefly, if the rubbing sound continues, its a pericardial friction rub - a pleural rub stops when breathing stops. Suggests pleurisy, pneumothorax or pleural effusion 5) absence of sound and/or "normal" sounds in areas where they are normally not heard 6) stridor - high-pitched harsh sound during inspiration. Stridor is caused by obstruction of the upper airway, is a sign of respiratory distress and thus requires immediate attention

List questions and physical findings that would identify potential bleeding disorders

1) bruise easily/spontaneously 2) family, past medical, and drug history 3) bleed for a long time/excessively 4) nose/gum bleeds 5) haematomas 6) muscle pain 7) joint pain 8) menorrhagia 9) blood clot (phlebitis), usually in a leg (causing most often swelling, redness, and/or warmth of the leg or shortness of breath) 10) purpura/bruises 11) blood blisters in the mouth 12) pallor 13) pica 14) bleeding 15) muscle haematomas 16) haemarthroses 17) liver disease

Explain the difference between a bulbar and pseudobulbar palsy

1) bulbar palsy and pseudobulbar palsy are motor neuron diseases characterized by the progressive degeneration of cranial motor neurons 2) bulbar vs pseudobulbar - a) aetiology - I) BP: brainstem stroke or tumour; neurodegeneration eg/ ALS, syringomyelia, MS; autoimmune neuropathies eg/ GBS, myasthenia gravis; infectious neuropathies eg/ poliomyelitis, diphtheria, neurosyphilis II) PBP: stroke, neurodegeneration eg/ ALS, progressive supranuclear palsy, MS; injury or malignancy of high brainstem b) pathophysiology - I) BP: bilateral degeneration of CN nuclei 9-12, LMN palsy in respective muscles II) PBP: bilateral degeneration of corticobulbar tracts to nuclei of CV 5, 7, 10-12, UMN palsy of respective nerves c) s/s - I) both - dysphagia, drooling, nasal regurgitation; tongue - can't protrude; palatal mvmt absent; II) BP: normal facial expression; nasal speech; tongue - wasting, fasciculations; gag reflex absent; jaw jerk normal; emotions not affected III) PBP: absent facial expressions; speech spastic & dysarthric; difficulty chewing; dysphagia, drooling, nasal regurgitation; tongue - spastic, no fasciculations/wasting, pointed; gag reflex brisk; jaw jerk exaggerated & clonic; emotional incontinence

Describe the presentation, complications and treatment of popliteal aneurysm

1) bulging and weakness in the wall of the popliteal artery, which supplies blood to the knee joint, thigh and calf 2) Many popliteal aneurysms have no symptoms, otherwise - pain behind the knee, lower leg oedema, foot pain, ulcers on the skin of the feet that don't heal 3) complications - a) main risk = embolisation and occlusion (thrombosis) both of which can cause acute leg ischaemia (sudden loss of blood supply to the lower leg) - as popliteal aneurysms inc in size they become lined by thombi, which can break away & lodge in smaller arteries in calf & foot b) aneurysm rupture - much less common than embolization (opposite of aortic aneurysms where the main risk is of the aneurysm bursting and embolisation is much less common) c) popliteal aneurysms can also indicate that pt is at risk of other aneurysms eg/ aneurysm in other leg, aortic aneurysm or another less common peripheral aneurysm d) if particularly large can also cause compression of neighbouring structures eg/ nerves and veins - in some can lead to DVT e) ultimately all these problems can lead to amputation of the leg 4) tx - a) surgical repair is usually quite successful, durable, and low risk b) best procedure - incision on the leg, removal of the popliteal aneurysm and reconstruction of the blood flow to the foot using either a vein or an artificial artery

Outline the moral and ethical issues associated with renal transplantation

1) cadaveric transplant is problematic because the availability of the organs means that someone has died. Usually, the donor is a young, healthy person who was injured or killed violently - problems include the acquisition of the appropriate consent and the various interpretations of brain-death in different cultures eg/ frowned upon in Islam, in Japan body must remain whole for several days after death 2) globally, more kidneys are being transplanted from living related donors, but this infringes the Hippocratic Oath to "First, do no harm". However, the informed consent, the absence of coercion and the careful planning to minimize the poten¬tial risks, justify this divergence from the oath 3) to extend the donor pool, new categories of donors have recently emerged eg/ emotionally related donors, children, altruistic unrelated donors and executed prisoners. In addition, two questionable means of providing incentive for donation are "re¬warded gifting" and rampant commercialism 4) attempts at increasing the donors to include genetically modified animal organs have led to a new set of ethical and medical concerns - should we breed animals as organ donors 5) organ allocation and the equity of the process - basic dilemma is which patient has the highest priority of the donated organ. Organs are usually allocated based on blood type, tissue match, immune status, medical urgency and the length of time on the waiting list. Two main competing principles are considered in all organ allocation decisions: justice and utility 6) biologically related donors - need to assure the voluntary nature following full knowledge of the potential health risks. 7) a problematic category is practiced in some countries = 'Rampant commercialism': actual buying and selling of organs. Most countries have enacted laws to prohibit the practice of selling organs

Outline the principles of cardiac rehabilitation including advice given regarding driving and employment

1) cardiac rehabilitation is a term which covers all the help you need in getting back to as full a life as possible after a heart event, such as a heart attack or bypass surgery. It's also about supporting you to live with your heart condition, to stay as healthy as possible, and to reduce the chance of another heart event 2) covers advice on protecting the heart, dec risk factors etc, support, and long-term physical activity 3) driving - a) angina - stop driving if angina symptoms whilst driving, or if your symptoms occur whilst resting or with emotion, restart driving when s/s controlled, don't need to inform the DVLA b) MI, ACS, or UA - stop driving for 4 weeks c) elective PCI - stop driving for 1 week, don't need to inform DVLA d) heart surgery - stop driving for at least 4 weeks 4) employment - can normally report back to work within 1-3 months

Identify the cardinal symptoms and signs of common causes of the acute abdomen and describe pain patterns associated with the most common causative conditions

1) cardinal symptoms - pain, rigidity, deep tenderness, and hyperaesthesia 2) epigastrum - pancreatitis, MI, peptic ulcer, acute cholecystitis, perforated oesophagus 3) LUQ - ruptured spleen, gastric ulcer, aortic aneurysm, perforated colon, pyelonephritis 4) umbilicus - intestinal obstruction, acute pancreatitis, early appendicitis, mesenteric thrombosis, aortic aneurysm, diverticulitis 5) LLQ - sigmoid diverticulitis, perforated colon, UC 6) RUQ - acute cholecystitis, duodenal ulcer, hepatitis, hepatomegaly, pyelonephritis, appendicitis 7) RLQ - appendicitis, mesenteric adenitis, Meckel's diverticulitis, perforated caecum, psaos abscess 8) R+LLQs -salpingitis, tubo-ovarian abscess, ectopic pregnancy, strangulated hernia, Crohn's, renal/ureteric stones 9) colicky pain - IBS, bowel obstruction 10) nagging & grumbling pain - biliary colic, cholecystitis, PID, UTI 11) stabbing -AAA 12) burning - oesophagitis, PUD 13) gnawing - pancreatitis, pancreatic cancer

List the causes of Horner Syndrome

1) carotid artery dissection 2) tumor in neck or chest cavity, particularly a neuroblastoma and a tumor of the upper part of the lung (Pancoast tumor) 3) lesion in midbrain, brain stem, upper spinal cord, neck, or eye orbit 4) inflammation or growths affecting the lymph nodes of the neck 5) surgery or other forms of trauma to the neck or upper spinal cord 6) idiopathic 7) ?genetic cause

List and recognise the complications associated with urethral catheterisation; indicate the steps necessary to minimise these risks

1) catheter-related urinary tract infection (uti) - occurs because urethral catheters inoculate organisms into the bladder and promote colonization by providing a surface for bacterial adhesion and causing mucosal irritation. the presence of a urinary catheter is the most important risk factor for bacteriuria a) prevention - closed system used, catheter removed asap 2) catheter-associated urinary tract infections - can lead to urosepsis and septicemia 3) urethral damage - a) occurs primarily in men because the catheter may interfere with drainage of seminal secretions eg/ epididymitis, orchitis, scrotal abscess, prostatitis, and prostatic abscess - can start at the time of insertion of the catheter but increases with long-term catheter use. b) difficulty passing the catheter may mean that the catheter has encountered a urethral stricture, has entered or created a false passage in the urethra, or that its passage is blocked by an obstructing prostate, bladder neck, or sphincter. the catheter may turn on itself and curl in the urethra c) urethritis - inflammation of the urethral meatus, causes discomfort and contributes to a breakdown in tissue integrity. it may be due to the frequent insertion of catheters or forceful catheterization against an obstruction. urethritis may occur more frequently in patients who have latex catheters d) erosion (tearing) of the urethra, primarily the urinary meatus, occurs in individuals who have had indwelling catheters for a long period of time. this erosion is usually secondary to catheter tension on the distal urethra at the meatus. the manner in which a catheter is secured should be alternated to prevent prolonged tension or pressure at an individual site e) creation of a false passage can occur primarily in men with persisting urethral strictures. men with enlargement of the prostate gland are most at risk f) urethral fistulas can develop in patients being managed long term with a urethral catheter. such fistula formation most commonly develops in women between the bladder and the anterior vaginal wall. many times the woman who has developed a fistula will complain of leakage and drainage from the vagina

Discuss the aetiology and presentation of calculi in the kidney and ureter

1) cause - consist of crystal aggregates of uric acid. Stones form in collecting ducts & may be deposited anywhere from renal pelvis to urethra - classically at pelviureteric junction, pelvic brim, vesicoureteric junction a) most common cause is of stones is bladder outlet obstruction (BPH, bladder neck elevation, high residual volume, infection, neurogenic bladder, urethral stricture) b) diverticuli & inflammation are risk factors, foreign bodies act as a nidus for stone formation. Most are mobile within the bladder, but some may form on suture, stent or tumour 2) effects - a) renal colic - excruciating ureteric spasms 'loin to groin' with nausea/vomiting b) renal obstruction - felt in loin between rib 12 & lateral edge of lumbar muscles c) obstruction of mid-ureter - may mimic appendicitis/diverticulitis d) obstruction of lower ureter - symptoms of bladder irritability, pain in scrotum, penile tip, or labia majora e) obstruction in bladder or urethra - pelvic pain, dysuria, strangury, interrupted flow f) pyelonephritis g) pyonephrosis h) can also have - UTI, haematuria, proteinuria, sterile pyuria, anuria

Describe the causes, typical history, examination features, differential diagnosis, management and complications of the acute coronary syndromes unstable angina (UA)

1) cause - coronary artery narrowing caused by a thrombus that develops on a disrupted atherosclerotic plaque and is usually non-occlusive 2) s/s - a) chest pain - inc frequency, inc severity (may occur at rest), retrosternal chest pain radiating to jaw, arm, or neck b) dyspnoea c) 4th heart sound (S4) d) atypical chest discomfort - may present with epigastric pain, recent-onset indigestion, stabbing chest pain, or pleuritic chest pain e) diaphoresis f) nausea g) tachycardia (most commonly sinus), AF & other supraventricular tachycardias are possible h) carotid bruit i) poor peripheral pulses 3) DD - a) cardiac - stable angina, prinzmetal (variant or vasospastic) angina, NSTEMI, STEMI, CHF, pericarditis, myocarditis, aortic dissection b) resp - PE, pleuritis, pneumothorax 4) mx - BROMANCE 5) complications - a) of treatment - bleeding, thrombocytopenia b) CHF, ventricular arrhythmias

Outline the cause of non-descent and mal-descent of the testis, the risks of this condition and its management

1) cause - unknown at present. However, several aetiological factors have been suggested: a) hormonal: may have abnormalities in the pathways/signalling of testosterone, mullerian inhibiting substance, insulin-like 3 hormone or its receptor LGR8, epidermal growth factor, and/or oestrogens b) environmental or maternal toxins: organochlorines, environmental oestrogens, phthalate esters, and pesticides c) maternal alcohol consumption, analgesic consumption, and smoking d) gestational diabetes e) genetics eg/ mutations in insulin-like factor 3 and its receptor, LGR8 f) mechanical: problems with development of the gubernaculum, or cremasteric muscle fibers, a patent processus vaginalis, or impaired intra-abdominal pressure have also been hypothesised to contribute to cryptorchidism g) neuromuscular: abnormalities of the calcitonin gene-related peptide of the genitofemoral nerve or the cremasteric nucleus have been postulated to cause cryptorchidism 2) risks - infertility & malignancy 3) mx - a) retractile testicle(s) - annual follow-up exam: any development of cryptorchidism or testicular asymmetry should be treated with orchiopexy b) undescended testicle(s): prepubertal, no hypospadias - I) palpable testicle(s): 1st line - orchiopexy II) non-palpable testicle: 1st line - examination under anaesthesia + orchiopexy III) non-palpable testicle: 2nd line - surgical exploration - if testis is not palpable on examination under anaesthesia IV) bilateral non-palpable testicles: 1st line - endocrinology +/- genetic evaluation - occasionally, a severely androgenised female with congenital adrenal hyperplasia may present with a phallic structure and presumed bilateral undescended testes, and may have life-threatening metabolic disturbances V) bilateral non-palpable testicles: 2nd line - surgical exploration VI) if hypospadias - need urology referral c) undescended testicle: postpubertal - I) unilateral: 1st line - orchiopexy ± biopsy II) +/- orchiectomy III) bilateral: 1st line - orchiopexy ± biopsy IV) +/- endocrine evaluation - testosterone, LH, FSH, and semen analysis may be considered to determine degree of testicular function +/- orchiectomy

Describe the causes & pathological consequences of acute coronary syndrome

1) causes - a) majority are due to narrowing in blood vessels supplying the heart, usually due to an atheroma within the lining of the artery b) inflammation of coronary arteries (rare) c) stab wound to the heart d) blood clot forming elsewhere in body (eg/ in heart chamber) and travelling to a coronary artery where it gets stuck e) cocaine can cause a coronary artery to go into spasm f) complications from heart surgery 2) consequences - atheromatous plaques in coronary arteries may fissure, predisposing to thrombus formation, or bleeding into plaque can occlude vessel lumen. Regional MI will occur when a coronary artery is blocked. Local ischaemia due to complete occlusion of a vessel will cause full thickness ischaemia, unless thrombus undergoes lysis or collateral circulation develops, in which case infarction will be limited to subendocardial zone. Circumferential subendocardial infarction may occur in general hypoperfusion of the cardiac tissue, often in already stenosed arteries

List the common causes and discuss investigations of dysphagia

1) causes - a) mechanical block eg/ malignant stricture (oesophageal, gastric, pharyngeal cancers), benign strictures (oesophageal web or ring, peptic stricture), extrinsic pressure (lung cancer, mediastinal ln, retrosternal goitre, aortic aneurysm, L atrial enlargement), pharyngeal pouch, hiatus hernia, foreign body, inflammation, infection b) motility disorders eg/ achalasia, diffuse oesophageal spasm, systemic sclerosis, neurological bulbar palsy (pseudobulbar palsy, parkinson's, syringobulbia, chaga's, myasthenia gravis, MS) c) oesophagitis 2) investigations - a) thyroid function test (thyromegaly) b) CSF analysis (MS) c) liver enzymes d) ceruloplasmin levels + 24hr urinary copper (Wilson's disease) e) standardised bedside swallowing assessment f) nasopharyngoscopy/laryngoscopy/transnasal oesophagoscopy/oesophagogastroduodenoscopy g) barium swallow (or oesophagram) h) oesophageal manometry i) x-rays: eg/ foreign body identification, cervical osteoarthritis

List the common causes of peripheral nerve damage and classify peripheral neuropathy into demyelinating and axonal types

1) causes - a) alcoholism - poor dietary choices can lead to vitamin deficiencies b) vitamin deficiencies - B vitamins eg/ B1, 6, 12; vit E; niacin c) autoimmune diseases eg/ Sjogren's syndrome, lupus, RA, Guillain-Barre syndrome, chronic inflammatory demyelinating polyneuropathy and necrotizing vasculitis d) diabetes - >50% people with diabetes develop some type of neuropathy e) exposure to poison - toxic substances include heavy metals or chemicals f) medications - certain meds eg/ chemo g) infections - certain viral or bacterial infections eg/ Lyme disease, shingles, Epstein-Barr virus, hepatitis C, leprosy, diphtheria and HIV h) inherited disorders eg/ Charcot-Marie-Tooth disease i) trauma or pressure on the nerve - traumas eg/ RTA, falls or sports injuries, can sever or damage peripheral nerves; nerve pressure eg/ having a cast or using crutches or repeating a motion such as typing many times j) tumor - malignant or benign growths, can develop on the nerves or press nerves. Also, polyneuropathy can arise as a result of some cancers related to the body's immune response. These are a form of paraneoplastic syndrome k) bone marrow disorders eg/ abnormal protein in the blood (monoclonal gammopathies), a form of bone cancer (osteosclerotic myeloma), lymphoma and amyloidosis i) other diseases eg/ kidney disease, liver disease, connective tissue disorders and hypothyroidism 2) classifying peripheral neuropathy - use nerve conduction studies to divide into types that primarily affect axons and those that primarily affect the myelin sheath a) axonal - axon loss may be seen after trauma to the nerve or as a result of toxic, ischemic, metabolic or genetic conditions. Amplitude of compound muscle action potential (CMAP) correlates with the number of motor nerve axons, and similarly, the amplitude of the sensory nerve action potential (SNAP) reflects the number of sensory nerve axons. Secondary axonal loss often occurs in severe or chronic demyelinating lesions b) demyelination - may be seen in compressive neuropathies, hereditary neuropathies, and acquired immune-mediated neuropathies like Guillain-Barré syndrome (GBS) and chronic inflammatory demyelinating polyneuropathy (CIDP). Loss of myelin is associated with slowing of conduction velocity (< 75% lower limit of normal), marked prolongation of distal latency (> 130% of the upper limit of normal), or both. Amplitude changes can also occur with demyelination due to secondary axonal loss. Any drop in CMAP amplitude or area > 20% implies conduction block and any increase in the CMAP duration > 15% signifies temporal dispersion; both are hallmarks of demyelination

Describe the clinical presentation of malabsorption and outline appropriate investigations. List causes of malabsorption, in addition to coeliac disease

1) causes - a) infection eg/ HIV, intestinal TB, parasites, traveller's diarrhoea b) structural defects eg/ blind loops, fistulae, diverticula, strictures, amyloidosis, lymphoma, IBD, short bowel syndrome c) surgery - bariatric surgery, gastrectomy d) mucosal abnormality - coeliac, cow's milk intolerance, fructose malabsorption e) enzyme deficiency - lactose deficiency, intestinal disaccharidase and enteropeptidase deficiencies, sucrose intolerance f) digestive failure - bile acid/salt malabsorption, bacterial overgrowth, obstructive jaundice, pancreatic insufficiency eg/ carcinoma pancreas or chronic pancreatitis, CF, Zollinger-Ellison syndrome g) systemic disease affecting GI tract eg/ Addison's, carcinoid syndrome, common variable immunodeficiency, thyroid problems. DM, parathyroid problems, malnutrition 2) s/s - a) GI manifestations - asymptomatic or: I) diarrhoea (often steatorrhoea) & flatulence - watery, diurnal and nocturnal, bulky, frequent stools are the clinical hallmark of overt malabsorption II) weight loss III) abdominal bloating & cramps & pain b) extraintestinal manifestations - I) anaemia - microcytic, macrocytic or dimorphic anaemia II) osteopenia III) purpura, subconjunctival hemorrhage, or even frank bleeding (may reflect hypoprothrombinemia secondary to vitamin K malabsorption) IV) prolonged malnutrition may induce amenorrhea, infertility, and impotence V) oedema and ascites may reflect hypoproteinemia associated with protein losing enteropathy caused by lymphatic obstruction or extensive mucosal inflammation VI) dermatitis and peripheral neuropathy may be caused by malabsorption of specific vitamins or micronutrients and essential fatty acids 3) ix - no single, specific test for malabsorption, so ix is guided by s/s - a) blood tests - FBC (anaemia), CRP (inc), LFTs (albumin low), cholesterol (dec), calcium & phosphate (dec) b) specific vitamins like vitamin D or micro nutrient like zinc levels can be checked. Fat soluble vitamins (A, D, E and K) are affected in fat malabsorption. Prolonged prothrombin time can be caused by vitamin K deficiency c) serological studies- determine the underlying cause: IgA Anti-transglutaminase antibodies or IgA Anti-endomysial antibodies for Coeliac disease d) stool studies - microscopy useful in diarrhoea, may show protozoa like Giardia, ova, cyst and other infective agents; low fecal pancreatic elastase is indicative of pancreatic insufficiency e) radiological studies - barium follow through (see SI anatomy), CT abdomen (for structural abnormality), Magnetic resonance cholangiopancreatography (MRCP) to complement or as an alternative to ERCP f) interventional studies - OGD (for detect duodenal pathology & biopsy), enteroscopy, capsule endoscopy, colonoscopy, ERCP g) others - 75SeHCAT (diagnose bile acid malabsorption), glucose hydrogen breath test (bacterial overgrowth), lactose hydrogen breath test (lactose intolerance), sugar probes or 51Cr-EDTA (intestinal permeability)

List the causes of post-operative fever and discuss the diagnostic steps for evaluation

1) causes - a) most common is infection: I) day 1-2 - consider a respiratory source II) day 3-5 - consider a urinary tract source III) day 5-7 - consider a surgical site infection or abscess/collection formation IV) any day post-op - consider infected IV lines or central lines as a source b) iatrogenic eg/ drug-induced reaction - abx, anaesthetic agents etc, or transfusion reaction c) venous thromboembolism - although rare, a PE or DVT can cause a low grade fever without any other overt clinical features d) secondary to prosthetic implantation - with any foreign body eg/ after an AAA repair, a low-grade fever may be evident e) pyrexia of unknown origin 2) ix - a) tailored to the patient eg/ if undergone a bowel resection, post-anastomotic leak is an important differential b) septic screen is essential - normally the source is obvious and your screen can be tailored accordingly: I) blood tests - FBC, CRP, U&Es II) urine dipstick III) cultures - blood, urine, sputum, and wound swab IV) imaging - CXR c) if source cannot be identified through the septic screen, more detailed investigations may be required eg/ CT for anastomotic leak, Doppler US for suspected DVT

Outline a diagnostic plan to identify aetiological factors of faecal impaction

1) causes - a) unhealthy lifestyle eg/ dec physical inactivity, poor intake of water & fluids, low fibre foods b) chronic constipation, or relying heavily on laxatives to relieve bowels - causes long-term suppression of rectal sphincter muscle 'contraction and relaxation', needed to push the solid faeces out. When laxatives are discontinued suddenly, it leads to accumulation of stool in the bowels that slowly develops into an impaction c) certain health problems (affecting the brain) may dec or stop the function of the nerves, which control bowel/intestinal muscle movement d) certain meds prevent or dec normal muscle activity of intestines and rectum. Sometimes, cause the stools to harden, causing a difficulty in expelling them eg/ opioids 2) ix - a) rectal examination b) medical history evaluation c) blood tests to exclude causes d) x-ray of abdo & pelvis, may also need CT or MRI e) barium enema f) sigmoidoscopy g) if suspicion of cancer may have colonoscopy

Outline the causes of male erectile dysfunction, list screening investigations, and list the available treatments

1) causes - a) vascular/arteriogenic is the most common cause - 40% b) diabetes - 30% c) medicine - 15% d) pelvic surgery/radiation/trauma - 6% e) neurogenic - 5% f) endocrine - 3% 2) ix - a) International Index of Erectile Dysfunction (IIEF)& Sexual Health Inventory for Men (SHIM) b) fasting blood glucose: >125 mg/dL in diabetes mellitus c) fasting lipid profile - detects hypercholesterolaemia d) consider - HbA1c if diabetic, testosterone (morning - in cases of suspected hypogonadism), FSH and LH (low = hypogonadotrophic hypogonadism; normal = eugonadotrophic hypogonadism; elevated = hypergonadotrophic hypogonadism), PSA (may have BPH), TSH (look for hypothyroidism), prolactin (rule out prolactinoma), Doppler ultrasonography, pelvic/penile angiography 3) mx - a) in all - I) psychosexual therapy (individual and/or couples) II) weight loss, inc exercise b) 1st line - tx underlying condition: eg/ underlying medical conditions, phosphodiesterase-5 (PDE5) inhibitors in HTN, treatment with testosterone replacement for hypogonadism, diabetic control c) 2nd line - phosphodiesterase-5 (PDE5) inhibitors: sildenafil or tadalafil or vardenafil or avanafil d) 3rd line - intracavernous injection: alprostadil intracavernous or papaverine e) 4th line - intraurethral suppository: alprostadil urethral f) 4th line - vacuum erection device g) 4th line - topical alprostadil h) 5th line - penile prosthesis

Describe the causes, symptoms, signs and initial management of acute arterial occlusion

1) causes - atherosclerosis 2) thrombotic occlusion - 60% = TO of previously stenotic segment 3) embolic occlusion - 30% 4) trauma/iatrogenic/injury 5) popliteal aneurysm 6) aortic dissection 7) acute compartment syndrome NB/ > 80% emboli from LA due to AF, others may come from AAAs. Thrombosis in situ may occur due to plaque rupture, hypovolaemia or pump failure NB2/ predisposing factors include dehydration, hypotension, unusual posture, malignancy, hyperviscosity and thrombophilia 2) s/s - 1) sudden onset in an extremity of the 6 P's: pain, perishingly cold, paresthesias (or anesthesias), pallor, pulselessness, paralysis 2) severe cases may cause loss of motor function. After 6 to 8 h, muscles may be tender when palpated 3) mx -

Describe the causes, morphology, pathological consequences, typical history, examination features, differential diagnosis, management and complications of the acute coronary syndromes ST-segment elevation myocardial infarction (STEMI)

1) causes - atherosclerosis with plaque fissuring or rupture and thrombus formation, rarely from coranry spasm, after chest trauma, or spontaneous coronary or aortic dissection 2) morphology & pathology - a) STEMI typically occurs after abrupt and catastrophic disruption of a cholesterol-laden plaque, resulting in exposure of substances that promote platelet activation and aggregation, thrombin generation, and thrombus formation, causing interruption of blood flow. If the occlusion is severe and persistent, myocardial cell necrosis follows b) on interruption of blood flow in the coronary artery, the zone of myocardium supplied by that vessel immediately loses its ability to shorten and perform contractile work. 3) s/s - a) chest pain - central, severe can occur at rest or with exertion, heavy in nature, radiation to L arm or jaw, lasts >20 mins b) dyspnoea, pallor c) diaphoresis d) N+V e) dizziness or light-headedness f) weakness, distressed g) tachycardia h) additional heart sounds - audible S3 or S4 i) ECG - ST elevation of >1 mm in ≥2 anatomically contiguous leads, repolarisation abnormalities often evolve over time from hyperacute T waves to ST elevation to T-wave inversion to the development of Q waves 4) DD - a) cardiac - UA, NSTEMI, aortic dissection, pericarditis, myocarditis b) resp - PE, pneumothorax, pneumonia c) GI - GORD, oesophageal spasm d) anxiety/panic attack 5) mx - BROMANCES - B-blocker - atenolol 100mg OD Revascularisation - PCI with stent within 90 mins of presentation Oxygen - sats >94% Morphine - 3mg IV every 5-15 mins until pain relief Aspirin - 300mg PO stat, 75mg OD Nitrate - GTN spray or sublingual Clopidogrel - 600mg stat, 75mg OD Enoxaparin - 30mg IV bolus, then 1mg/kg subcut 12-hrly Statin - 80mg atorvostatin OD 6) complications - a) highly likely: CHF, ventricular arrhythmias, recurrent ischaemia and infarction, depression b) medium likely: sinus bradycardia, 1st or 2nd degree or complete heart block, recurrent chest pain b) less likely: acute mitral regurgitation, VSD, acute pericardial tamponade, Dressler's syndrome, in-stent thrombosis, LV thrombus or aneurysm

Describe the causes, typical history, examination features, differential diagnosis, management and complications of the acute coronary syndromes non-ST-segment elevation myocardial infarction (NSTEMI)

1) causes - plaque rupture (transient o near-complete occlusion of coronary artery), dynamic obstruction (vasospasm), progressive luminal narrowing (chronic arterial narrowing from restenosis), inflammatory mechanisms (vasculitis), extrinsic factors leading to poor coronary perfusion (eg/ hypotension, hypovolaemia, or hypoxia), restenosis following PCI, recreational drug use (eg/ cocaine or other stimulants) 2) s/s - a) chest pain - sensation of tightness, heaviness, aching, burning, pressure, or squeezing, often retrosternal and can often radiate to the left arm but may also radiate to the lower jaw, neck, both arms, back, and epigastrium, where it may mimic heartburn b) diaphoresis c) physical exertion - may trigger unstable plaque rupture d) SOB e) weakness f) anxiety g) N+V h) atypical symptoms - more common in women, older people, and those with diabetes: abdo pain, HTN i) ECG - no persistent ST elevation, but may show ischaemic changes such as ST depression or T-wave inversion, ECG may also be normal 4) DD - a) cardiac - aortic dissection, pericarditis, myocarditis, stable ischaemic heart disease, Brugada's syndrome, acute stress cardiomyopathy b) resp - PE c) gastro - peptic ulcer disease, oesophageal spasm, acute cholecystitis, Boerhaave's syndrome d) other - panic attack 5) mx - MONA: a) Morphine: morphine sulfate: 2-5 mg IV every 5-30 minutes when required b) Oxygen: if O2 sats <90%, respiratory distress, or other high-risk features for hypoxaemia c) Nitrate: GTN translingual: 400-800 micrograms (1-2 sprays) every 5 minutes, maximum 3 doses, or GTN: 0.3 to 0.6 mg sublingually every 5 minutes, maximum 3 doses d) Antiplatelet: aspirin 300mg (then 75mg OD thereafter) + clopidogrel 300mg (then 75 mg OD) e) + B-blocker: metoprolol: 50-100 mg orally (regular-release) twice daily f) +/- CCB: diltiazem: 30-90 mg orally (immediate-release) four times daily g) + assess need for invasive or conservative approach I) invasive approach planned - PCI + heparin: 60 units/kg IV bolus, then 12 units/kg/hour infusion or enoxaparin: 30 mg IV bolus, then 1 mg/kg subcutaneously every 12 hours +/- glycoprotein (GP) IIb/IIIa inhibitor: abciximab: 0.25 mg/kg IV bolus, then 0.125 micrograms/kg/min infusion for 12 hours, maximum 10 micrograms/min 6) complications - a) medium likelihood - cardiac arrhythmias, depression b) low likelihood - CHF, cardiogenic shock, ventricular rupture or aneurysm, acute mitral regurgitation, post-MI pericarditis (Dressler's syndrome), VTE, in-stent thrombosis

Describe the ABO blood group and outline its significance in blood transfusion

1) cells that only have A antigen on RBC + anti-B in serum = group A (42%) 2) cells with only B antigen on RBC + anti-A in serum = group B (8%) 3) cells with both A + B antigens on RBC + no anti-A or anti-B = group AB (3%) 4) cells lacking both antigens on RBC + have anti-A & anti-B = group O (47%) 5) Rh group - some pts have Rh factor on RBC - also an antigen and those who have it are called Rh+, otherwise Rh-. A person with Rh- blood does not have Rh antibodies naturally in the blood plasma, but they can develop Rh antibodies if given Rh+ blood (Rh antigens trigger production of Rh antibodies). A person with Rh+ blood can receive blood from a person with Rh- blood without any problems 6) blood transfusions - give same blood type eg/ A to A, If using a different type: a) transfusion will work if person receiving blood has a blood group that doesn't have any antibodies against donor blood's antigens b) blood group O Rh -ve = "universal donors", but can only receive blood from O Rh -ve c) AB Rh +ve = "universal receivers", but can only donate to AB Rh +ve d) Rh+ blood never given to someone with Rh - blood, but other way is ok NB/ anti-X (IgM) = antibodies that cause agglutination of antigens (eg/ X antigens) = clogs BV + leakage of cell contents (Hb = toxic outside cells)

Signs of respiratory failure

1) central cyanosis 2) large volume pulse 3) confusion 4) flapping tremor 5) difficulty breathing or shortness of breath, especially when active 6) coughing up mucous 7) wheezing 8) bluish tint to the skin, lips, or fingernails 9) rapid breathing 10) fatigue 11) anxiety 12) confusion 13) daily headache

Describe the causes of ventilatory failure and outline the effect of chest wall and spinal deformity on respiratory function

1) central depression of respiratory centre - drugs eg/ μ-opioids, sedatives, sudden cessation of blood circulation in brain, severe brain trauma, brain tumors, damage of brainstem 2) neuromuscular - acute eg/ Guillain-Barré syndrome, chronic & relapsing eg/ MS + myasthenia gravis, relentlessly progressive eg/ amyotrophic lateral sclerosis [ALS] 3) chest wall problems - kyphoscoliosis, morbid obesity, ankylosing spondylitis, pectus excavatum 4) COPD - to achieve adequate alveolar ventilation pts with COPD must generate more -ve intrathoracic pressures than normal, so pressure output of inspiratory muscles during resting breathing can be 3x+ higher than healthy subjects 5) kyphoscoliosis scoliosis (lateral curvature of spine) + kyphosis (anteroposterior) a) dec TLC, vital capacity (VC), and FRC without changes in residual volume (RV) b) if severe = risk of respiratory failure, alveolar hypoventilation, cor pulmonale c) advanced disease - inspiratory muscle strength significantly dec & max transdiaphragmatic & inspiratory pressures are dec - active recruitment of intercostal muscles + inspiratory accessory muscles 6) ankylosing spondylitis - inflammatory disease of axial skeleton that mainly affects males, leads to fibrosis & ossification = inc rig cage stiffness due to ankylosis + fusion of costovertebral & sternoclavicular articulations a) despite rib cage stiffness, there is no marked dec in lung volumes (TLC or FRC), or correlation between severity of spinal angulation & functional parameters indicating restriction b) slight dec in lung volumes (possibly due to fusing of ribs) correlates positively with disease duration and activity c) dec in PImax + maximum expiratory pressure indicate weakness caused by intercostal muscle atrophy plus dysfunction of diaphragm, whose activity is generally compensatory and preserved 7) pectus excavatum - most common deformity of anterior thorax = notable concavity of lower third of sternum a) lung volumes at lower limits of normal or slightly dec - compliance normal unless have severe kyphoscoliosis

Outline the indications for the insertion of central venous catheters, Swan-Ganz catheters and arterial catheters

1) central venous catheters - a) access for giving drugs b) access for extracorporeal blood circuits c) haemodynamic monitoring and interventions - insertion of a catheter solely to measure central venous pressure is becoming less common 2) Swan-Ganz (pulmonary arterial) catheter - no hard indications, relative - a) cardiac output measurement esp if arrhythmia or aortic balloon pump, where PiCCO cant be used b) unequal right and left ventricular failure c) complex hemodynamic instability, some combination of obstructive, distributive, cardiogenic and hypovolemic shock d) to differentiate cardiogenic pulmonary oedema from non-cardiogenic e) to guide use of vasopressors, inotropes, fluids and diuretics - when the patient has a hemodynamic problem combined with pulmonary oedema and ventricular dysfunction f) to titrate pulmonary antihypertensives in ARDS (like nitric oxide and prostacyclin) 3) arterial catheter - a) continuous direct BP monitoring - arterial catheter MAP measurements are more accurate than sphygmomanometric BP readings in patients who are morbidly obese, very thin, have severe extremity burns, or have very low blood pressures b) inability to use indirect BP monitoring (eg, in patients with severe burns or morbid obesity) c) frequent blood sampling d) frequent arterial blood gas sampling NB/ for arterial catheterisation need to evaluate patency of ulnar artery using Allen's test, as will be inserting into radial artery

Outline the investigation and management of a patient presenting with a thyroid nodule/swelling

1) check TSH/free thyroxine (fT4) 2) low TSH/high fT4 - a) radioiodine uptake scan b) evaluate for cause of hyperthyroidism based on scan results 3) normal or high TSH - a) neck US 4) intrathyroidal mass - a) if no suspicious features on US follow up only needed b) if suspicious mass on US - need fine needle aspiration & cytology - will be: I) benign - follow up only II) or undetermined atypia - repeat US guided FNA - will be benign or malignant (follow relevant stage) III) or neoplasm/malignant - refer to endocrine or H&N surgeon for thyroidectomy (total or partial) +/- neck dissection 5) extrathyroidal mass - a) evaluate for parathyroid enlargement with calcium/intact PTH b) evaluate for lymphadenopathy with US and US guided FNA biopsy as indicated

Outline carcinoma of the gallbladder, bile duct and ampulla of Vater with regard to presenting symptoms and survival

1) cholangiocarcinoma (tumour of bile duct) - a) s/s - I) risk factors eg/ age >50 years, cholangitis, choledocholithiasis, cholecystolithiasis, other structural disorders of the biliary tract, UC, primary sclerosing cholangitis, liver fluke infection, liver disease, hepatitis C virus, HIV infection, hepatitis B virus, and exposure to thorium dioxide II) painless jaundice III) pruritus IV) can also have weight loss, abdo pain b) survival - 5-year survival for surgical resection alone ranges from 20-43%. For surgical resection with chemotherapy, the 5-year survival rate is 26%. The response rate to chemotherapy alone is <15%. For liver transplant, there is a recurrence rate of 51% within 2 years of the procedure 2) gallbladder carcinoma - relatively uncommon: a) s/s - I) pain in RUQ II) indigestion & dyspepsia (gas) III) bilious vomit & jaundice - both due to obstruction IV) weakness V) dec appetite & weight loss VI) Courvoisier's law states that a palpably enlarged gallbladder which is nontender and accompanied with mild painless jaundice, is unlikely to be gallstone - think cancer somewhere b) survival - 5-yr survival rate of 19% 3) ampulla carcinoma - a) s/s - I) painless jaundice II) pruritus III) may also have scleral icterus, dyspepsia, anorexia, malaise, and weight loss IV) can have Courvoisier's sign (ie, a distended, palpable gallbladder in a patient with jaundice) b) survival - varies in literature, ~50% 5-yr survival

Discuss the management of gallbladder disease

1) cholelithiasis - a) symptomatic cholelithiasis - I) 1st line - cholecystectomy (mainly laparoscopic) b) choledocholithiasis with or without symptoms - I) 1st line - endoscopic retrograde cholangiopancreatography (ERCP) with stone extraction: biliary pain + gallbladder stones + dilated common bile duct (>6 mm) on US + abnormal LFTs or pancreatic enzyme elevation suggests stone may have migrated into the common bile duct, causing an obstructive complication such as cholangitis II) + laparoscopic cholecystectomy: if simultaneous gallbladder and bile duct stones, early laparoscopic cholecystectomy should follow within 72 hours after ERCP and stone extraction III) +/- lithotripsy, papillary balloon dilation, or long-term biliary stenting IV) 2nd line - laparoscopic common bile duct exploration c) asymptomatic cholelithiasis - I) 1st line - observation 2) cholangitis - a) acutely ill - I) 1st line - IV abx + intensive medical management: IV piperacillin/tazobactam or imipenem/cilastatin or cefepime + metronidazole; pt on ICU, IVI II) + biliary decompression: non-operative: ERCP +/- sphincterotomy and placement of drainage stent or percutaneous trans-hepatic cholangiography (PTC) III) +/- lithotripsy; +/- opioid analgesics IV) 2nd line - biliary decompression: surgical + IV abx +/- opioid analgesics 3) cholecystitis - a) mild (grade I): stable without signs of perforation/gangrene - I) 1st line - supportive care: NPO, IVI, abx, and analgesia, together with close monitoring of blood pressure, pulse, and urinary output, should be initiated II) +/- oral abx: cefuroxime or ciprofloxacin or levofloxacin + metronidazole III) +/- NSAID: diclofenac potassium or indomethacin IV) +/- early laparoscopic cholecystectomy or +/- percutaneous cholecystostomy tube or +/- endoscopic transpapillary gallbladder drainage or endoscopic ultrasound-guided gallbladder drainage b) moderate (grade II): stable without signs of perforation/gangrene - I) 1st line - supportive care (as above) + IV abx (as above) II) +/- NSAID III) + early cholecystectomy or cholecystostomy with delayed cholecystectomy c) severe (grade III): suspected gangrene/perforation or evidence of organ dysfunction - I) 1st line - ICU + supportive care II) + IV abx: cefuroxime or ciprofloxacin or levofloxacin + metronidazole III) +/- urgent cholecystostomy followed by delayed elective cholecystectomy

List the common types of gallstones and describe their pathophysiology

1) cholesterol gallstones - most common type (80%), often yellow a) undissolved cholesterol, but may contain other components (crystallised) b) crystallisation dependant on: supersaturation, crystallization-promoting factors and gall bladder motility c) more common in women, with inc age HMG-CoA activity, obesity, dec lecithin + bile salts & inc cholesterol, family history, multiparity, rapid weight loss, OCP, ileal disease, DM, cirrhosis 2) pigment gallstones (<10%) - dark brown or black stones a) form when bile contains too much bilirubin (haemolysis) b) black stones - composed of calcium bilirubinate & network of mucin glycoproteins., have a glass-like cross sectional surface on fracturing c) brown stones - composed of fatty acid calcium salts + calcium bilirubinate, almost always found in biliary stasis or infection, particularly with E.Coli or Klebsiella infections, common cause of recurrent bile duct stones 3) mixed stones - calcium salts, pigment + cholesterol NB/ gallstones uncommon before 30, less common in Asian & African groups NB2/ more common in pts with concurrent GI disease eg/ UC and Crohn's (less bile salts reabsorbed)

Define the classifications of operative procedures with reference to their potential for infectious complications (clean, potentially contaminated, contaminated and dirty); discuss the importance of this classification system

1) clean - incision in which no inflammation is encountered in a surgical procedure, without a break in sterile technique, and during which the respiratory, alimentary and genitourinary tracts are not entered 2) clean-contaminated - incision through which the respiratory, alimentary or genitourinary tract is entered under controlled conditions but with no contamination encountered 3) contaminated - incision undertaken during an operation in which there is a major break in sterile technique or gross spillage from the gastrointestinal tract, or an incision in which acute, non-purulent inflammation is encountered. Open traumatic wounds that are more than 12-24 hours old also fall into this category 4) dirty or infected - incision undertaken during an operation in which the viscera are perforated or when acute inflammation with pus is encountered during the operation (for example, emergency surgery for faecal peritonitis), and for traumatic wounds where treatment is delayed, and there is faecal contamination or devitalised tissue present

Describe the features of 'accelerated phase' or 'malignant' hypertension

1) clinical - a) BP >180/110 mmHg with signs of papilloedema and/or retinal haemorrhage b) malignant hypertension may complicate the course of both essential and secondary hypertension c) change in BP occurs rapidly (months-1/2years) d) more commonly affects people in their 3rd and 4th decade 2) pathological - a) the reason some pts develop malignant hypertension whereas others do not is unknown b) characteristic vascular lesion is fibrinoid necrosis of arterioles and small arteries (especially kidney + abdominal viscera), which causes clinical manifestations of end-organ damage. RBCs damaged as they flow through vessels obstructed by fibrin deposition = microangiopathic hemolytic anaemia c) due to major organ involvement it causes: LVH, acute heart failure, encephalopathy, severe renal damage (death in uraemia!) and papilloedema d) dilatation of cerebral arteries following a breakthrough of the normal autoregulation of cerebral blood flow. Normally cerebral blood flow is kept constant by cerebral vasoconstriction in response to increases in BP. In patients without hypertension, flow is kept constant over a mean pressure of 60-120 mm Hg. In patients with hypertension, flow is constant over a mean pressure of 110-180 mm Hg because of arteriolar thickening. When BP is raised above the upper limit of autoregulation, arterioles dilate = hyperperfusion and cerebral oedema, which cause the clinical manifestations of hypertensive encephalopathy

Describe the presenting clinical features, natural history, .and mx of renal cell carcinoma

1) clinical - 50% found incidentally: a) mean age 55yrs b) 2:1 male:female ratio c) painless haematuria d) loin pain e) abdominal or flank mass f) anorexia g) malaise h) weight loss i) PUO - often in isolation j) can have invasion of left renal vein compressing L testicular vein + varicocele k) spread may be direct (renal veins), via LN, or haematogenous (bone, liver, lung) l) local invasion through capsule, into perinephric fat and lymph nodes m) risk factors - smoking, obesity, NSAIDs, renal cyst disease (haemodialysis) and hypertension n) hypercalcaemia may be present, due to production of PTHrP 2) pathological features - tissue of origin for renal cell carcinoma (RCC) = proximal renal tubular epithelium. Renal cancer occurs in a sporadic (nonhereditary) and a hereditary form, and both forms are associated with structural alterations of the short arm of chromosome 3 (3p) a) at least 4 hereditary syndromes associated with renal cell carcinoma are recognised: von Hippel-Lindau (VHL) syndrome (chromosome 3), hereditary papillary renal carcinoma (HPRC), familial renal oncocytoma (FRO) associated with Birt-Hogg-Dube syndrome (BHDS) 3) mx a) small renal mass (<4cm) or RCC stage 1 or 2 - I) surgical candidate 1st line - surgery: surgical resection II) +/- consideration for clinical trials (post-nephrectomy) III) + local ablation therapy eg/ radiofrequency ablation (RFA) and cryoablation IV) non-surgical candidate - surveillance +/- local ablation therapy +/- consideration for clinical trials b) RCC stage 3 - I) surgical candidate 1st line - surgery: radical nephrectomy II) +/- consideration for targeted molecular therapy eg/ sunitinib III) non-surgical candidate 1st line: consideration for clinical trials c) RCC stage 4 (metastatic disease) - I) 1st line - targeted molecular therapy eg/ pazopanib or sunitinib or aldesleukin or bevacizumab + interferon alfa 2b or temsirolimus etc II) +/- consideration for surgery - cytoreductive (or debulking) nephrectomy, metastasectomy can be done at the same time as renal surgery, or on another occasion III) +/- consideration for clinical trials IV) +/- consideration for chemotherapy V) +/- consideration for palliative local radiation VI) +/- biophosphonate therapy for bone metastases eg/ zoledronic acid

Describe the clinical features of acute renal failure ARF

1) clinical features - a) electrolyte imbalance: hyperkalaemia, acidosis, hyponatraemia (overdrinking), hypocalcaemia (less vit D), hyperphosphataemia b) pulmonary oedema c) uraemia (+ anorexia, N+V, pruritis) and oliguria d) signs related to cause - signs of infection, affected lung basement membranes in Goodpasture's, hepatorenal syndrome in chronic liver disease, metabolic & vascular changes (eyes) in diabetes & hypertension, muscle pain with rhabdomyolysis e) need CV exam to elicit whether there is heart failure or endocarditis f) examination of prostate + bladder may reveal a post-renal cause 2) biochemical features - a) rise in creatinine >26umol/L in 48hrs or >1.5 x baseline b) urine output <0.5mL/kg/hr for >6 consecutive weeks NB/ abrupt deterioration of parenchymal renal function, usually reversible, over a days or weeks

Describe the clinical features of a DVT and the use of the d-dimer test to triage pts with a suspected DVT

1) clinical features in affected leg: a) swelling, pain, warmth, redness, engorged superficial veins, oedema b) Homan's sign (pain on dorsiflexion of foot) often present, but is not diagnostic c) thrombosis in iliofemoral region can present with severe pain d) complete occlusion = cyanotic discolouration of limb & severe oedema e) can occur asymptomatically f) most common s/s of PE = dyspnea, chest pain, tachypnea, syncope, cough 2) d-dimer: a) d-dimer = fibrin degradation product - small protein fragment detectable in blood after a blood clot is degraded by fibrinolysis. D-dimer assays are fast, accurate, and readily available. Useful only for ruling out DVT if results -ve; +ve results are not diagnostic as many conditions eg/ impaired renal function, blood loss, pregnancy, AF, can cause d-dimer levels to rise b) clinical diagnosis + raised d-dimer testing has a sensitivity of 80%

Describe the clinical presentation, morphology and pathological consequences of carcinoma of the pancreas

1) clinical presentation - a) painless obstructive jaundice b) 75% tumours in body & tail present with epigastric pain (radiates to back, relieved by sitting forward) c) palpable gallbladder (Courvoisier's law) d) hepatomegaly + splenomegaly e) lymphadenopathy + ascites f) may have - anorexia, weight loss, diabetes, acute pancreatitis, pruritis 2) morphology - a) damage to pancreas = release of proteolytic enzymes - causes necrosis of acini & thrombosis in vessels, leading to further damage b) damage to epithelium lining duct = enzyme diffusion into surrounding tissue, and small areas of necrosis appear radiating outwards from ducts - can progress to panlobular pancreatitis c) form poorly defined firm fibrotic masses d) white-yellow & obscure the normal lobular architecture of the pancreas e) degree of gland formation can vary from well-formed glands, to partially formed glands, to focal intracellular mucin production by poorly oriented cells infiltrating singly, to solid sheets of neoplastic cells f) infiltrates in a haphazard pattern 3) pathological consequences - a) 1st metastasises to regional LN, then liver (less commonly to lungs) b) can directly invade surrounding visceral organs eg/ duodenum, stomach, colon, or metastasize to any surface in abdominal cavity via peritoneal spread c) may spread to the skin as painful nodular metastases d) haemorrhage - may lead to Grey-Turner's & Cullen's signs NB/ in gallstone disease, bile refluxes up pancreatic duct (irritant), alcoholic disease - ampulla spasms, ducts plugged by viscous secretion & alcohol is directly toxic to acinar cells

Discuss the effect of inhalation of coal dust on lung function and its relation to pneumoconiosis

1) coal - predominantly the element carbon, with oxygen, hydrogen, and a trace of sulfur. Anthracite has the highest % carbon (>91%) 2) brisk inflammatory response at sites of deposition - macrophages generate fibrogenic proteins and growth factors that stimulate collagen elaboration (IL-1, platelet-derived growth factor, and fibronectin) 3) activation and lytic effect on alveolar macrophages. Freshly fractured coal, or coal of a higher rank (e.g., anthracite), has more free radicals and causes the release of more inflammatory cytokinins. Coal also causes the same serum immunological changes as described for silica. 4) gives restricted spirometry pattern - dec reduced FVC, normal FEV1/FVC ratio, dec slow vital capacity, reduced total lung capacity (TLC), reduced lung diffusion capacity testing (DLCO). 5) may have obstructive changes: reduced FEV1, reduced FEV1/FVC ratio, increased residual volume/TLC ratio, reduced DLCO, if hx of mineral dust exposure and cigarette smoking 6) may show mixed pattern 7) coal workers' pneumoconiosis develops after years & is a severe state, initially have milder form of the disease known as anthracosis

List the symptoms and signs of large bowel obstruction

1) colicky abdominal pain - increasing constant pain, pain on movement, coughing, or deep breathing may imply perforation or impending perforation 2) abdominal distention 3) tympanic abdomen 4) change in bowel habits - failure to pass faeces (complete obstruction) or successful passing of some flatus or faeces (partial obstruction) NB/ dec stool calibre/diameter and loose motions may be associated with a diverticular stricture or colorectal malignancy 5) hard faeces - may indicate faecal impaction when noted on digital rectal examination 6) empty rectum - implies proximal obstruction 7) soft stools - suggests partial obstruction 8) recent weight loss - suggests an underlying malignancy 9) rectal bleeding - suggests an underlying malignancy 10) abnormal bowel sounds - inc in frequency with absent sounds in more advanced stages of obstruction 11) palpable rectal mass - suggests rectal carcinoma 12) palpable abdominal mass - may indicate malignant disease or diverticular mass 13) positive faecal occult blood test - mandates exclusion of a more proximal large bowel malignancy

Describe the symptoms that may suggest a diagnosis of irritable bowel syndrome

1) colicky abdominal pain/discomfort relieved by defecation or associated with altered stool form or bowel frequency (constipation/diarrhoea may alternate) and 2+ of: a) urgency b) incomplete evacuation c) abdominal bloating/distention d) mucous PR e) symptoms worse after food 2) can also have - nausea, backache, bladder symptoms, headaches, halitosis, poor sleep 3) symptoms are chronic (>6 months) and exacerbated by stress, depression, menstruation + sex, gastro-enteritis, anxiety, trauma, Abx, eating disorders 4) may have psychological element - pts otherwise well constitutionally, with no abnormalities found on exam (except bloating + some tenderness). 5) female:male presentations = 3 to 1 6) infectious diarrhoea precedes 7-30% of IBS cases 7) Rome III diagnostic criteria - in previous 3 months should be at least 3 days per month of recurrent abdominal pain or discomfort with 2+ of: a) improvement with defecation b) onset associated with change in stool frequency or change in form of stool

List the clinical features that would raise suspicion of a carcinoma and indicate urgent patient referral

1) colorectal - a) presence of risk factors: APC mutation, Lynch syndrome (hereditary non-polyposis colorectal cancer, HNPCC), MYH-associated polyposis, hamartomatous polyposis syndromes, IBD, obesity b) inc age - <5% pts are <44 yo, mean age at diagnosis is 71 years c) rectal bleeding d) change in bowel habit - inc frequency or looser stools, particularly combined with rectal bleeding, is common in left-sided cancers e) rectal mass f) anaemia - 90% pts with right-sided colon cancer are anaemic at diagnosis 2) anal - a) risk factors - HPV, HIV, men who have sex with men, immunosuppression, and anal trauma b) rectal bleeding, pain, or mass

Describe the natural history of carcinomas affecting the large bowel

1) colorectal cancers mostly arise from dysplastic adenomatous polyps 2) multistep process involving the inactivation of a variety of tumour-suppressor and DNA repair genes + simultaneous activation of oncogenes - this confers a selective growth advantage to the colonic epithelial cell and drives the transformation from normal colonic epithelium to adenomatous polyp to invasive colorectal cancer 3) germline mutations underlie the well-described inherited colon cancer syndromes, whereas sporadic cancers arise from a step-wise accumulation of somatic genetic mutations. A single germline mutation in the adenomatous polyposis coli (APC) tumour suppressor gene is responsible for the dominantly inherited syndrome familial adenomatous polyposis. Clinical expression of the disease is seen when the inherited mutation of one APC allele is followed by a second hit mutation or deletion of the second allele 4) spread of colorectal cancer is to local lymph nodes, via enteric venous drainage to the liver and haematogenously to the lungs, and - less commonly - to bone & brain

Define the term chronic obstructive pulmonary disease (COPD) and list recognised risk factors

1) common progressive disorder characterised by airway obstruction - FEV1/FVC<0.7 with little or no reversibility 2) includes: a) chronic bronchitis - cough/sputum production on most days for 3 months of 2 successive years (symptoms improve if stop smoking) + b) emphysema - enlarged air spaces distal to terminal bronchioles with destruction of alveolar walls 3) usually have COPD or asthma, not both 4) abnormal inflammatory response of lungs to noxious particles or gases 5) risk factors - a) >35yrs old b) smoking (passive or active) - related to nos of cigarettes smoked, or pollution related c) genetics (alpha-1-antitrypsin deficiency = rare) d) occupation (particularly mining, as in Mansfield)

List the common pathogens that may cause acute diarrhoea in the community and in travellers to tropical and sub-tropical countries. Outline the management of this problem

1) community diarrhoea - a) eg/ Rotavirus, Enterotoxogenic Escherichia coli, Shigella, Campylobacter jejuni, Cryptosporidium b) protozoa: Giardia lamblia, Entamoeba histolytica, Isospora belli. 2) traveller's diarrhoea - a) eg/ enterotoxigenic Escherichia coli, enteroaggregative E coli (EAEC), Shigella , Campylobacter jejuni , Salmonella, Yersinia ,Vibrio b) protozoa: Giardia , Entamoeba , Cryptosporidium , Cyclospora 3) mx - a) 1st line - fluoroquinolone (eg/ ciprofloxacin) or azithromycin or rifaximin b) +/- loperamide (unless blood in stool or dysentery or fever) c) +/- rehydration and/or bismuth subsalicylate and/or probiotics d) if diarrhoea not responsive to abx: switch to antiparasitic: tinidazole or metronidazole

Outline the management of a venous ulcer

1) compression - 3 or 4 layer compression at 40mmHg (ankle) a) low-adherent dressings are applied and replaced weekly b) alternative dressings may be considered, to help with pain (hydrocolloid), heavy exudate (alginate), or slough (hydrogels) c) graduated compression dec venous reflux & ankle oedema & inc venous blood flow, improving microcirculation and encouraging healing d) no compression if ABPI <0.8, active phlebitis, DVT, or cellulitis 2) superficial venous surgery 3) skin grafting 4) cleansing and debridement - ulcer irrigated at each dressing change with warm tap water or saline & dried, also - slough, necrotic, fibrous, or excess granulation tissue removed 5) pentoxifylline - effective adjunct to compression bandaging for venous leg ulcers, may be effective in absence of compression 6) self help - a) keep mobile with regular walking - exercise calf muscle pump function b) elevate legs when immobile c) avoid trauma and wear appropriate (well-fitting) footwear d) use an emollient frequently e) examine legs regularly for broken skin, blisters, swelling, or redness f) healthy lifestyle eg/ lose weight, balanced diet (malnutrition impairs ulcer healing), and drink alcohol within recommended levels. Stop smoking NB/ can be managed in community by specialist in ulcers & dressings eg/ district nurse or tissue viability nurse

Describe the symptoms of patients with umbilical hernia. Perform a physical examination of patients with umbilical hernia, differentiating reducible and non-reducible hernia; recognise the signs of strangulation

1) congenital - seen when baby is crying, laughing, or straining to use the bathroom, telltale symptom = swelling or bulge near umbilical area (not present when baby is relaxed), usually asymptomatic 2) following symptoms may indicate more serious situation that requires medical treatment: a) baby is in obvious pain b) baby is vomiting c) bulge (in both children and adults) is tender, swollen, or discoloured - untreated, will rupture leading to fatal peritonitis NB/ umbilical - neck is wide so rarely obstructs, most disappear spontaneously by 2yrs old, paraumbilical hernia - neck less wide so risk of strangulation, contents = omentum, transverse colon & SI, may be obstructive symptoms & adhesions 3) exam typically reveals an easily reduced hernial sac with a well-defined rim of fascia and central defect of variable diameter. Overlying skin may appear stretched 4) reducible vs non-reducible - reducible hernia: bulge flattens out when you lie down or push against it gently - not an immediate danger to a person's health, although may be painful and worsen over time if untreated. Nonreducible hernia: loop of intestine becomes trapped, lose ability to make the bulge flatten out - very painful and requires prompt medical attention 5) signs of strangulation - if an unrepaired umbilical hernia presents with discomfort or as a tender umbilical mass, incarceration with or without strangulation should be suspected. Signs - redness, warmth, swelling, pain. Most congenital umbilical hernias are easily reducible NB/ paraumbilical hernias less likely to be reducible and more likely to strangulate due to smaller neck + inc incidence of adhesions

Classify the causes of valvular heart disease

1) congenital eg/ bicuspid aortic valve, Lutembacher's syndrome, congenital mitral stenosis, Ebstein's anomaly (tricuspid regurg), Fallot's tetralogy (ventricular septal defect, pulmonary stenosis, right ventricular hypertrophy, an overriding aorta) 2) rheumatic - reaction to infection with Group A strep, history of RF found in 50% of pts with mitral stenosis & regurgitation 3) degenerative - senile calcification (most common cause of aortic stenosis), and rupture of chordae tendinae after MI 4) cardiac remodelling - may occur in SLE, Marfan's or Ehler-Danlos, in syphilis or akylosing spondylitis, resulting in dilation of valve rings & valvular insufficiency 5) IE - bacteria may erode into valves & cause valvular insufficiency, leading to regurgitation

List 3 patient groups most at risk of developing an acute hydrocephalus

1) congenital hydrocephalus - present prior to birth. Mostly due to aqueductal stenosis (narrow passage between the third and fourth ventricles in the brain is blocked or too narrow to allow sufficient CSF to drain. Fluid accumulates in the upper ventricles, causing hydrocephalus) 2) neural tube defects, arachnoid cysts, Dandy-Walker syndrome, and Arnold-Chiari malformation 3) acquired - consequence of CNS infections, meningitis, brain tumors, head trauma, toxoplasmosis, intracranial hemorrhage (subarachnoid or intraparenchymal) and is usually painful

Outline the treatment of painful diabetic neuropathy

1) conservative - manage BMs, insulin, diet, exercise, good foot care 2) medication - a) 1st line: pregabalin or gabapentin or duloxetine b) 2nd line: antidepressants eg/ amitryptiline, venlafaxin c) 3rd line: opiods eg/ tapentadol or tramadol or oxycodone d) 4th line: capsaicin topical e) 5th line: TENS, PENS or acupuncture f) spinal cord stimulation

Discuss smoking cessation methods

1) contact GP or an NHS stop smoking adviser for advice & to be referred to an NHS stop smoking service 1) nicotine replacement therapy (NRT) - provides you with a low level of nicotine, without the tar, carbon monoxide and other poisonous chemicals present in tobacco smoke a) can help dec withdrawal effects eg/ bad moods and cravings, which may occur when you stop smoking b) available as: skin patches, chewing gum, inhalators (which look like plastic cigarettes), tablets, oral strips and lozenges nasal and mouth spray - patches release nicotine slowly, inhalators, gum and sprays act more quickly and may be better for alleviating cravings, often best to combine a patch with a faster acting form such as gum, inhalator or nasal spray c) tx usually lasts 8-12 weeks, before you gradually reduce the dose and eventually stop 2) Varenicline (Champix) - dec cravings for nicotine like NRT + blocks the rewarding and reinforcing effects of smoking a) most effective medication for helping people stop smoking b) only available on prescription c) tx usually lasts around 12 weeks, but it can be continued for longer if necessary 3) Bupropion (Zyban) - originally used to treat depression, found to help people quit smoking a) unclear how it works, but it's thought to have an effect on the parts of the brain involved in addictive behaviour b) only available on prescription c) tx usually lasts 8 weeks 4) E-cigarettes - electronic device that delivers nicotine in a vapour, giving nicotine without most of the harmful effects of smoking, as the vapour contains no tar or carbon monoxide

Outline the principles of treatment of confirmed cases and the principles of contact tracing for TB

1) contact screening - a) offer Mantoux testing to diagnose LTBI in household contacts or close work or school contacts (aged 5+) of all pts with active TB b) offer to pts with significant exposure in previous 1 to 2 years, repeat test for LTBI (TB skin test or interferon-gamma release assay) recommended 8 to 10 weeks after last exposure if initial evaluation was performed prior to this and the initial test result was negative c) Mantoux test may be +ve if have had BCG vaccine, so use interferon gamma testing 2nd-line if Mantoux test is +ve 2) mx of active TB - a) notify Public Health b) most cases can be managed as outpatients but occasional admission may be needed (in side-room) c) abx - I) 2 months of - isoniazid + rifampicin + pyrazinamide + ethambutol II) 4 further months of - isoniazid + rifampicin III) isoniazid, rifampicin and pyrazinamide are associated with liver toxicity so need LFTs before starting tx IV) compliance can be an issue so may need DOT with a nurse 3) mx of latent TB - a) either 6 months of isoniazid or 3 months of rifampicin + isoniazid

Discuss the management of atrial fibrillation, taking into account rate versus rhythm control strategies. Outline assessment for the need of anticoagulation using a recognized risk scoring system

1) control of CV risk factors 2) rate control - a) step 1 = B-blocker or verapamil/diltiazem (or digoxin if sedentary) b) step 2 = B-blocker or diltiazem, + digoxin c) never give B-blocker + verapamil! 2) offer rate control as 1st line except when - AF has reversible cause (tx this), or secondary heart failure, or new onset AF, or atrial flutter suitable for ablation, if rhythm control would be better based on clinical judgement 3) rhythm control - a) cardioversion - medical or electrical b) ablation 4) cardioversion - a) pts may have 'pill in pocket' therapy for paroxysmal AF (unless CHF) eg/ flecainide, propafenone b) if AF lasts >24hrs need electrical cardioversion c) can pre-treat with amiodarone for 4 weeks before d) medical = amiodarone (IV or PO) 5) stroke prevention - CHA2DS2-VASc score: if >1 (women) or >0 (men) then consider DOACs (apixaban, dabigatran, rivaroxaban) or warfarin, assess against HAS-BLED score & address all modifiable criteria

Describe the clinical syndrome that would arise from; Cord transection at C3 and at T10; Cord hemisection; Posterior cord lesion

1) cord transection at C3 - C3, 4, 5 keep the diaphragm alive. Symptoms include: a) limited range of motion b) loss of diaphragm function c) requirement of a ventilator for breathing d) paralysis in arms, hands, torso, and legs e) trouble controlling bladder and bowel function 2) cord transection at T10 - a) good upper body control depending on the level of cord damage b) lack of function in the legs and/or torso, resulting in paraplegia c) bowel and/or bladder incontinence d) possible reduced ability to control the trunk of the body or abdominal muscles e) good balance while in a sitting position f) ability to stand in a specialized frame, or walk with braces 3) cord hemisection - a) Brown-Séquard syndrome is caused by incomplete spinal cord lesion which causes damage to ½ spinal cord, resulting in paralysis and loss of proprioception ipsilaterally to injury or lesion, and loss of pain and temperature sensation contralaterally = ipsilateral hemiplegia and contralateral pain and temperature sensation deficits. The loss of sensation on the opposite side of the lesion is because the nerve fibers of the spinothalamic tract (which carry information about pain and temperature) crossover once they meet the spinal cord from the peripheries b) Brown-Séquard syndrome is characterized by loss of motor function (i.e. hemiparaplegia), loss of vibration sense and fine touch, loss of proprioception (position sense), loss of two-point discrimination, and signs of weakness on the ipsilateral (same side) of the spinal injury 4) posterior cord lesion/ posterior spinal artery syndrome (PSA) - type of incomplete spinal cord injury. Originates from an infarct in the posterior spinal artery and is caused by lesions on the posterior portion of the spinal cord, specifically the posterior column, posterior horn, and posterolateral region of the lateral column eg/ trauma to the neck, occlusion of the spinal artery, tumors, disc compression, vit B12 deficiency, syphilis, or MS 5) general symptoms include ipsilateral loss of proprioceptive sensation, fine touch, pressure, and vibration below the lesion; deep tendon areflexia; and in severe circumstances, complete paralysis below the portion of the spinal cord affected

Discuss the potential underlying causes of angina

1) coronary artery disease - atheroma, causes dec blood flow in areas of myocardium 2) valvular heart disease - inc cardiac workload 3) cardiomyopathy (tachyarrhythmia) - interferes with cardiac contractility 4) anaemia- dec oxygenation

Define the terms cost-benefit analysis, cost-effectiveness analysis, cost utility analysis, QALY, (efficiency and effectiveness)

1) cost-benefit analysis - systematic approach to calculating and comparing the benefits and costs of a course of action in a given situation - want to determine which option returns the optimal ratio of benefits to savings 2) cost-effectiveness analysis - form of economic analysis that compares the relative costs and outcomes (effects) of different courses of action. Distinct from cost-benefit analysis, which assigns a monetary value to the measure of effect. Cost-effectiveness analysis is often used in the field of health services, where it may be inappropriate to monetize health effect. CEA is expressed in terms of a ratio where the denominator is a gain in health from a measure (years of life, premature births averted, sight-years gained) and the numerator is the cost associated with the health gain - most commonly used outcome measure is QALY 3) cost utility analysis - estimate the ratio between the cost of a health-related intervention and the benefit it produces in terms of the number of years lived in full health by the beneficiaries eg/ intervention A allows a patient to live for 3 additional years than if no intervention had taken place,with a quality of life weight of 0.6, then the intervention confers 3 * 0.6 = 1.8 QALYs to the patient. If intervention B confers two extra years of life at a quality of life weight of 0.75, then it confers an additional 1.5 QALYs to the patient. The net benefit of intervention A over intervention B is therefore 1.8 - 1.5 = 0.3 QALYs 4) QALY - generic measure of disease burden, including both quality & quantity of life lived. Assesses value for money of medical interventions. 1 QALY = 1yr perfect health. QALYs can be used to inform personal decisions, to evaluate programs, and to set priorities for future programs NB) efficiency & effectiveness can be derived from these and other measures described above

Describe the common clinical presentation of lung cancer

1) cough 2) haemoptysis 3) dyspnoea 4) chest pain (pleuritic with invasion of pleura or ribs?) 5) recurrent or slowly resolving pneumonia (infection) 6) lethargy 7) anorexia + malaise 8) weight loss 9) anaemia 10) clubbing 11) hypertrophic pulmonary osteoarthritis (wrist pain) 12) enlarged supraclavicular or axillary nodes 13) can have consolidation, pleural effusion, chest collapse 14) metastatic - bone pain, hepatomegaly, confusion, fits, focal CNS signs 15) small-cell carcinomas has endocrine burden in 10% (overall) a) ACTH secretion = adrenal hyperplasia = Cushing's syndrome b) ADH secretion = retention of water = dilutional hyponatraemia c) PTHrP secretion = hypercalcaemia

Elicit physical signs of COPD

1) cough 2) sputum 3) wheeze (mild) 4) dyspnoea 5) tachypnoea (severe) 6) use of accessory muscles of respiration 7) hyperinflation + prolonged expiration (severe) 8) dec cricosternal distance (<3cm) + expansion 9) resonant or hyperresonant note on percussion 10) quiet breath sounds (eg/ over bullae) 11) cyanosis - cor pulmonale 12) intercostal muscles may be indrawn during inspiration & lips pursed 13) hypercapnia = bounding pulse, peripheral vasodilation & flapping tremor 14) elderly men may develop barrel chest due to weakening of spine & loss of height

Describe the principles of cross matching blood

1) crossmatch routinely used as final step of pretransfusion compatibility testing 2) detects: irregular antibodies; errors in ABO grouping, clerical errors in pt identification + result recording 3) 2 types of cross-matches: Major cross-match and Minor cross-match a) major crossmatch - testing pt's serum with donor cells to determine whether pt has an antibody which may cause a hemolytic transfusion reaction or dec cell survival of donor cells. This is the most important cross-match b) minor crossmatch - testing pt's cells with donor plasma to determine whether there is an antibody in the donor's plasma directed against an antigen on the pt's cells NB/ mixing part of pt's sample with donor blood units, to ensure no abnormal reaction occurs (Coomb's test)

List relevant investigation for lung cancer and interpret results

1) cytology - pleural fluid + sputum 2) CXR - peripheral nodule, hilar enlargement, consolidation, lung collapse, pleural effusion, bony secondaries 3) fine needle aspiration or biopsy 4) CT - stage tumour & guide biopsy 5) bronchoscopy - give histology & assess operability 6) F-deoxyglucose PET or PET/CT - staging 7) radionuclide bone scan - if suspected metastasis 8) lung function tests - assess suitability for lobectomy

Describe the pathology of the coronary arteries in pts presenting with angina

1) damage to endothelium (trauma, stress) = entry of cholesterol-rich LDL to intima 2) lipid taken up by macrophages in intima 3) normal receptor mediated uptake of lipid can be bypassed by oxidation of LDL which is taken up by a receptor independent pathway - excessive lipid accumulation in intimal macrophages form a visible pale bulge = fatty streak 4) development & inc lipid accumulation causes macrophages to release lipid into intima 5) cytokines secreted by macrophages stimulate proliferation of intimal cells with features of myofibroblasts - these secrete collagen (hyaline) & plaque becomes fibrotic 6) lesions are raised & yellow (lipid plaques) 7) as lesions develop = pressure atrophy of media & elastic lamina disrupted 8) inc secretion of collagen forms dense fibrous cap to plaque which is now hard & white (fibrolipid plaque) 9) advanced plaque shows free lipid as well as lipid in macrophages 10) collagenisation affects media & weakens arterial wall - endothelium is fragile & often ulcerates, allowing platelet aggregation & thrombosis NB/ factors released by macrophages eg/ TNF-a, IL-1 and MMPs may lead to destruction of fibrous cap & predispose to superficial thrombus formation and calcification at lower sheer stress levels

List the complications associated with vascular catheterisation and discuss the emergency management of each

1) damage to vessels - a) damage to central veins or arteries eg/ injury, bleeding and hematoma b) veins - emergent surgical intervention is often needed c) arteries - more often affected, leaving the arterial catheter in place with prompt repair carries less morbidity and mortality than catheter removal with pressure 2) pulmonary complications - a) pneumothorax - O2, observation, may need chest drain b) injury to trachea. - of little significance, unless the patient is mechanically ventilated c) injury to the laryngeal nerve (controls your voice box) - may heal over time d) air embolism - high flow O2, pt in L lateral decubitus position. Small air embolisms are typically of little significant and should not distract from safely securing the line, can be self-resolving or managed with supplemental O2 and increased systemic pressures to help "washout" the air bubble. In extreme cases, hyperbaric O2 can be attempted to help reabsorb the air 3) cardiac complications - a) arrythmia or cardiac arrest - both need prompt initiation of advanced cardiac life support (ACLS) b) direct valvular (particularly tricuspid) injuries - rarely require surgery, normally managed medically 4) device dysfunction - cracking or breaking of the catheter, thrombosis, obstruction of the catheter- normally require surgery to remove catheter, or mx of thrombosis 5) infection - can lead to blood poisoning, shock or even death a) central line infections become established when a thin slimy film, called biofilm, collects on the internal or external surface of the catheter eg/ Staphylococcus aureus or Staphylococcus epidermidis b) if suspected - 2 blood cultures from separate sites, catheter tip sent for culture & IV abx promptly

Discuss treatment of bronchiectasis with postural drainage and physiotherapy and antibiotics for infective exacerbation

1) damaged lung can't be repaired so need to prevent or slow down further deterioration 2) tx of any underlying cause & co-morbidities 3) healthy diet & physical exercise 4) smoking cessation 5) vaccines - influenza and pneumococcus 6) physiotherapy - airway clearance techniques & exercise, physio should be performed by pts on a daily basis if chronic productive cough, and intermittently if productive cough during exacerbations a) eg/ conventional tipped position b) sterile water inhalation may be used before airway clearance to facilitate clearance c) inhaled B2 agonists should be used prior to treatment to enhance sputum clearance d) non-invasive ventilation or intermittent positive pressure breathing may be used if pt becoming fatigued e) pulmonary rehabilitation should be offered to pts whose breathlessness is affecting their ADL 7) abx - for acute exacerbation (after sputum sample sent off for culture) eg/ amoxicillin a) if 3+ exacerbations per year requiring abx, or fewer exacerbations that are causing significant morbidity, consider long-term abx 8) bronchodilators - B2 agonists may help some pts, likewise, theophylline and aminophylline are occasionally used if a trial of therapy has demonstrated improvement 9) don't give - a) corticosteroids - inhaled or oral - unless co-existent asthma b) mucolytics c) LTRA 10) O2 may be needed 11) non-invasive ventilation can improve QOL in some pts with chronic respiratory failure 12) surgery - lung resection surgery may be considered if localised disease & symptoms are not controlled by medical treatment a) bronchial artery embolisation +/- surgery is first-line therapy for mx of massive haemoptysis b) lung transplantation may be considered for end-stage disease if FEV1 <30% of predicted

Outline the pathological basis of classical Parkinson's disease

1) dec dopamine in basal ganglia - loss of nigrostriatal dopaminergic neurons, esp in substantia nigra pars compacta 2) presence of Lewy bodies in many remaining neurons 3) inc inhibition of thalamus + dec excitation of motor cortex

Describe the important principles of the dietary and lifestyle interventions for the treatment of diabetes

1) dec sat fats + sugar 2) inc carboydrate + protein foods made specially for diabetics aren't needed obese pts with T2DM - controversial but thought low-carbohydrate ketogenic diets (<20g carbs daily) could be beneficial vs low-glycaemic index diets 3) protein - 1g/kg, fat - <35% total intake (sat+trans fat<10%), CHO - 40-60% (low GI foods), fruit, veg, some alcohol allowed, salt < 6g (<3g a day in hypertensive diabetic patients), recommend fibre

List the structural lesions and predisposing factors which may predispose toward Deep intracerebral haemorrhage and Lobar cerebral haemorrhage

1) deep intracranial - a) HTN b) blood thinners eg/ heparin, and warfarin c) AVM - a tangle of abnormal arteries and veins with no capillaries in between d) aneurysm - a bulge or weakening of an artery wall e) head trauma - fractures to skull and penetrating wounds (gunshot) can damage an artery and cause bleeding f) bleeding disorders - haemophilia, sickle cell anaemia, DIC, thrombocytopenia g) tumours - if highly vascular eg/ angiomas and metastatic tumours can bleed into the brain tissue h) amyloid angiopathy - buildup of protein within the walls of arteries i) drug usage - alcohol, cocaine and other illicit drugs j) spontaneous 2) lobar - a) idiopathic b) cerebral amyloid angiopathy c) HTN - primary or secondary to drug use (e.g. cocaine, amphetamines) d) cerebral aneurysm - saccular and mycotic e) arteriovenous malformation f) cerebral cavernoma g) brain tumour eg/ glioma, brain metastasis h) bleeding disorders or coagulopathies or anticoagulation i) vasculitis j) cerebral venous thrombosis

Specify the definition, approximate prevalence and the risk factors for obesity

1) definition - BMI greater than or equal to 30, class 1 = moderate (30-35), 2 = severe (35-40), 3 = very severe/morbid (>40) 2) approximate prevalence in UK ~25% 3) risk factors: a) genetics - may affect amount of body fat you store, fat distribution, metabolism b) inactivity c) unhealthy diet (inc calories) d) medical problems eg/ Prader-Willi, Cushing's, arthritis, hypothyroidism, hypothalamic damage e) certain medications eg/ some antidepressants, anti-seizure meds, diabetes meds, antipsychotics, (cortico)steroids, B blockers f) social & economic issues - avoiding obesity is difficult if don't have safe areas to exercise. May not have been taught healthy ways of cooking, or have money to buy healthier foods. More likely to become obese if you have obese friends or relatives g) age - inc age = hormonal changes, less active, body muscles dec = dec metabolism h) pregnancy - woman's weight inc, some find this weight difficult to lose i) quitting smoking associated with weight gain j) lack of sleep - hormonal changes inc appetite. Crave foods high in calories & carbs

Congestive cardiac failure - definition, causes

1) definition - cardiac output inadequate for body's requirements. Prognosis is poor with ~25-50% 5yr survival. Failure of both sides of heart 2) common causes: a) systolic failure - IHD (35%), MI, cardiomyopathy (30%) b) diastolic failure - constrictive pericarditis, tamponade, restrictive cardiomyopathy, hypertension (15%) c) low output failure - i. pump failure - systolic/diastolic HF, dec HR (B-blockers, heart block, post MI), -ve inotropic drugs (eg/ antiarrhythmics) ii. excess preload - mitral regurgitation, fluid overload (eg/ NSAIDs) iii. chronic excess afterload - aortic stenosis, hypertension d) high output failure -pregnancy, Paget's disease, arteriovenous malformation, beri beri, infection, alcohol & drugs, pericardial disease and hyperdynamic circulation (anaemia, hyperthyroid, haemochromatosis)

Define an umbilical hernia and relate it to the embryological origin of the umbilicus. List the factors that predispose to the development of umbilical and para-umbilical hernia

1) definition - if abdominal wall behind navel is damaged navel may bulge outwards. Bulge consists of abdominal fat from greater omentum or occasionally parts of SI 2) embryology - result when persistence of a patent umbilical ring occurs 3) 3 forms of umbilical hernia occur: a) exomphalos - rare, failure of gut (mid-gut) to return to abdominal cavity following embryological rotation that occurs outside of the body. Bowel is contained within a translucent sac which runs through the defective anterior abdominal wall b) congenital umbilical hernia - results from failure to completely close the umbilical cicatrix. It allows for herniation during periods of inc intra-abdominal pressure c) acquired para-umbilical hernia - occurs just above or below umbilicus along linea alba due to weakening 4) predispose - most common in infants: a) premature babies b) low birth weight babies c) black infants d) equally likely between boys & girls adults: a) being overweight b) having multiple pregnancies c) coughing for a long period of time (applies pressure to abdominal wall) d) para-umbilical hernias more common in women, umbilical hernias in men e) genetic factors

Define status epilepticus and describe initial investigations and components of management, including airway protection; use of anti-convulsants

1) definition - life-threatening neurological condition defined as 5+ minutes of either continuous seizure activity or repetitive seizures without regaining consciousness 2) ix - a) anticonvulsant drug blood level - check for poor adherence in pts with known epilepsy b) toxicology screen - may be positive for alcohol or potentially epileptogenic substance c) comprehensive metabolic panel - severe electrolyte imbalance, hypoglycaemia, and hyperglycaemia are treatable and reversible causes of seizures and SE d) FBC - low platelets may indicate intracranial haemorrhage, or raised white cells may indicate infection as the cause e) ECG - arrhythmias or cardiac ischaemia may result from SE f) EEG - evaluating non-convulsive SE & monitoring response to tx when patient is rendered iatrogenically comatose g) consider - ABG (metabolic disturbances may be a complication of SE), CT head (rule out structural lesions, haemorrhage, ischaemic strokes, oedema and tumour, hydrocephalus), LP (when suspicion of encephalitis or meningitis, after CT) 3) mx - a) generalised convulsive status epilepticus - I) 1st line - supportive care + oxygen: BLS including securing the airway, delivering 100% oxygen by mask or nasal cannula, and correcting hypotension, hyperthermia, glucose and electrolyte abnormalities, and potential thiamine deficiency. ECG should be monitored II) hypoglycaemia: + thiamine + glucose: thiamine IV, 50% glucose 50 mL IV III) + benzodiazepine: lorazepam: 0.1 mg/kg (max) 4 mg/dose) IV or diazepam: 0.15-0.2 mg/kg (max 10 mg/dose) IV or midazolam: >40 kg: 10 mg IV, repeat all (except midazolam when necessary) IV) 2nd line - fosphenytoin/phenytoin or valproic acid or levetiracetam V) 3rd line - phenobarbital VI) 4th line - general anaesthesia: midazolam or propofol b) non-convulsive status epilepticus - I) focal impaired awareness (complex partial): 1st line - supportive care + oxygen: as above II) + benzodiazepine; hypoglycaemia: thiamine + glucose III) 2nd line - fosphenytoin/phenytoin or valproic acid or levetiracetam; 3rd line - phenobarbital; 4th line - general anaesthesia c) absence - 1st line - I) 1st line - supportive care + oxygen II) hypoglycaemia: + IV glucose (50% in 50mL) III) + anticonvulsant: lorazepam: 0.1 mg/kg (max 4 mg/dose) IV or diazepam: 0.15-0.2 mg/kg (max 10 mg/dose) IV or valproic acid: 40 mg/kg (max 3000 mg/dose) IV, repeat all but valproic acid d) focal aware (simple partial) status epilepticus - I) 1st line - tx guided by underlying aetiology II) 1st line - anticonvulsants

Define lymphoedema

1) definition - localised fluid retention & tissue swelling caused by compromised lymphatic system (normally returns interstitial fluid to thoracic duct, then bloodstream). Legs most commonly affected 2) primary causes: a) can be inherited or caused by a birth defect eg/ Milroy's disease 3) secondary causes: a) cancer treatments - surgery + radiotherapy b) parasitic infections c) severe cellulitis d) inflammatory conditions eg/ rheumatoid arthritis, dermatitis, and eczema e) CV diseases f) injury & trauma eg/ severe skin burns or anything which results in excessive scarring NB/ chronic, non-pitting oedema, progresses with age. Chronic disease may cause a secondary cobble-stone thickening of the skin. Best place to establish presence of lymphoedema = second toe; this is known as Stemmer's sign NB2/ treatment - compression stockings and physical massage. In recurrent cellulitis, long-term antibiotics may be considered, as each infection will further damage vasculature. Surgery should be avoided

Describe pathological changes in the lungs and the natural history of disease in a typical patient, and genetics

1) dehydration of the airway surfaces reduces mucociliary clearance and favours bacterial colonisation b) local bacterial defences are impaired by local salt concentrations c) bacterial adherence is inc by changes in cell surface glycoproteins d) inc bacterial colonisation & dec clearance produce inflammatory lung damage due to an exuberant neutrophilic response involving mediators such as IL8 and neutrophil elastase 2) projected life expectancy for patients ~40 a) females and those people from lower socio-economic classes have a worse prognosis 3) genetics - autosomal recessive disease caused by mutations in the CF transmembrane conductance regulator (CFTR) gene, on chromosome 7 a) most common mutation in Caucasians = d-F508 (DF508) b) CFTR is an ATP-responsive chloride channel that also affects other cellular activities, such as sodium transport across the respiratory epithelium, composition of cell surface glycoprotein and antibacterial defences NB/ B. cepacia, P. aeruginosa, Mycobacterium and S. aureus are the most important infectious agents in CF pts

Describe the difference between dementia and delirium

1) dementia develops over time, with a slow progression of cognitive decline, but delirium occurs abruptly, and symptoms can fluctuate during the day 2) hallmark separating delirium from underlying dementia is inattention - pt cannot focus on one idea or task 3) delirium often is unrecognized by healthcare professionals because changes in behavior in persons with dementia (such as agitation or sun downing) may be attributed to the dementia disease process, versus an acute problem

Describe dermatomes, myotomes and the root values of the deep tendon reflexes

1) dermatome: area of skin mainly supplied by a single spinal nerve. There are 8 cervical nerves (C1 being an exception with no dermatome), 12 thoracic nerves, 5 lumbar nerves and 5 sacral nerves. Each nerve relays sensation (including pain) from a particular region of skin to the brain 2) myotome: group of muscles that a single spinal nerve innervates 3) deep tendon reflexes: a) biceps = C5+6 b) brachioradialis/supinator = C6 c) triceps = C7 d) patellar = L4 e) achilles tendon = S1

Describe the clinical examination on a semi-conscious/unconscious patient, with specific reference to the initial assessment of ABC, and subsequent neurological and cardiological exams

1) determine unresponsiveness: verbal stimulus eg/ 'Can you hear me?' or 'Are you OK?', tactile stimulus to either the hands or face, noxious stimulus (intense but not cause injury) eg/ pressure on the supra-orbital ridge or nail-bed pressure 2) primary review - ABCDE including exam of resp & CV & GI systems 3) GCS score 4) assessment of the cranial nerves and motor response to pain should be performed 5) pupil examination can provide useful clues as to the aetiology: a) small pupils (8 mm) - can be due to drug toxicity (amphetamines, cocaine) or an oculomotor nerve pathology or pontine lesion b) unilateral fixed pupil - due to a third cranial nerve lesion c) midsize pupils (4-6 mm) unresponsive to light = midbrain lesion d) maximally dilated pupils (>8 mm) = drug toxicity eg/ anticholinergic overdose 6) fundoscopy may reveal key diagnostic findings eg/ papilloedema in patients with hypertensive crisis and posterior reversible encephalopathy syndrome 7) may smell alcohol on the breath of an unconscious patient 8) look for potential drug injection sites (groins, arms) or sites for subcutaneous insulin injections 9) breathing pattern abnormalities - a) Cheynee Stokes breathing - can occur with many underlying pathologies and is not helpful in differentiating between diagnoses in the unconscious patient b) ataxic breathing (Biot's respiration) - abnormal pattern of breathing, characterized by groups of quick, shallow inspirations then regular or irregular periods of apnoea, and indicates a lesion in the lower pons c) central neurogenic hyperventilation - abnormal pattern of breathing, characterized by deep and rapid breaths at 25+ breaths per minute, indicates a lesion in pons or midbrain

Fundoscopy appearances in diabetes, HTN (macular degeneration), papilloedema

1) diabetes - a) NPDR - microaneurysms, exudates, retinal haemorrhages, cotton wool spots (axoplasmic accumulations), vascular dilatations, calibre variations, intraretinal microvascular abnormalities (IRMA) b) severe NPDR - many dark haemorrhages, irregular calibre variation (beading), dilatation of retinal veins, IRMAs c) PDR - neovascularisation on optic disc or retina from significant retinal ischaemia (likely to bleed = pre-retinal +/- vitreous haemorrhage I) PDR late changes - retinal fibrosis, traction retinal detachment (blurred vision), iris neovascularisation (rubeosis iridis) and neovascular glaucoma 2) HTN - a) 3) 2 types of AMD- a) dry - atrophy of photoreceptors in retinal pigment cells, drusen +/- geographical atrophy b) wet - choroidal neovascularisation 3) papilloedema - a) s/s - transient visual obscurations, headaches, inc blind spot, progressive filed loss b) splinter haemorrhages, exudates, cotton wool spots & retinal folds near disc

Describe the diagnostic criteria for diabetes mellitus and glucose intolerance with reference to laboratory glucose and HBA1c measurement

1) diabetes mellitus: chronic hyperglycaemia + disturbances in carb, fat and protein storage due a dec/defect in action or release of insulin from the endocrine pancreas a) symptoms (polyuria, polydipsia, unexplained weight loss, visual blurring, genital thrush, lethargy) + random venous plasma glucose concn ≥ 11.1 mmol/l or fasting plasma glucose concn ≥ 7.0 mmol/l or 2x raised venous plasma glucose - fasting, random, or OGTT (plasma glucose concn ≥ 11.1 mmol/l 2hrs after 75g anhydrous glucose) or HbA1C >48mmol/L (6.5%) 2) impaired glucose tolerance: fasting plasma glucose <7.0 mmol/l + OGTT of 7.8-11.0 mmol/l 3) impaired fasting glycaemia: fasting plasma glucose between 6.1-6.9 mmol/l a) pts identified with IFG should have OGTT to rule out IGT

List the two major hyperglycaemic complications of diabetes

1) diabetic ketoacidosis 2) Hyperosmolar Hyperglycaemic State 3) osmolality = 2[Na]+2[K]+[glucose]+[urea] (normal = 280-300)

Describe the appearances of background retinopathy, proliferative retinopathy and maculopathy and recognise the appearance of these and of cataract on direct ophthalmoscopy. Outline the treatment of proliferative retinopathy and maculopathy

1) diabetic retinopathy - retinal vasculopathy due to DM: 2) 3 types of diabetic retinopathy - a) non-proliferative DR (NPDR) or 'background' DR (BGR) - no proliferation of BV b) proliferative (PDR) - abnormal new BV growing c) diabetic maculopathy - special retinopathy, may occur with either NPDR or PDR 3) NPDR/BR - a) microaneurysms, exudates, retinal haemorrhages, cotton wool spots, vascular dilatations, calibre variations, intraretinal microvascular abnormalities (IRMA) NB/ haemorrhages: dot (small), blot (large) from venous end of retinal capillaries deep in retina; flame shaped haemorrhage (more arterial) NB2/ exudates: yellowish-white deposits with well-defined edges; precipitation of leaked lipoproteins from diseased retinal vasculature NB3/ cotton wool spots (CWS): greyish white, poorly defined fluffy edged lesions, axoplasmic accumulations adjacent to retinal nerve fibre microinfarcts b) severe NPDR - many dark haemorrhages, irregular calibre variation (beading), dilatation of retinal veins, IRMAs. Most progress to PDR in 1yr 4) PDR - a) neovascularisation on optic disc or retina from significant retinal ischaemia I) v fine irregularly branching BV arising from veins II) start flat but enlarge & move forward into vitreous III) fragile, likely to bleed with slight traction = pre-retinal +/- vitreous haemorrhage b) PDR late changes - I) retinal fibrosis II) traction retinal detachment (blurred vision) III) iris neovascularisation (rubeosis iridis) and neovascular glaucoma 5) diabetic maculopathy - a) focal - focal leakage from microaneurysms or dilated capillaries from focal retinal thickening & surrounding exudates b) diffuse - diffuse leak from dilated capillaries = diffuse retinal oedema, maybe some retinal haemorrhages but usually no exudates c) ischaemic - closure of perifoveal capillary network; diffuse oedema + dark haemorrhages. Fluorescein angiography confirms ischaemia 8) mx of DR - a) control of diabetes & risk factors b) laser photocoagulation - stops focal leaks as in focal maculopathy c) future: anti-VEGFs etc d) vitrectomy

Describe the pathophysiology and diagnosis of B12/folate deficiency causing a macrocytic anaemia

1) diagnosis: a) dec Hb + WCC + platelets + serum B12 b) inc MCV c) reticulocytes dec or normal as production impaired d) hypersegmented polymorphs e) megaloblasts in marrow f) pernicious anaemia tests - parietal cell antibodies, intrinsic factor antibodies 2) pathophysiology: a) general - symptoms of anaemia, lemon tinge to skin due to combination of pallor + mild jaundice, glossitis, angular cheilosis b) neuropsychiatric - irritability, depression, psychosis, dementia c) neurological - paraesthesia, peripheral neuropathy d) classical triad = extensor plantars (UMN), absent knee jerks (LMN), absent ankle jerks (LMN)

Discuss the management strategies for obesity, including surgical options

1) diet and exercise advice, possibly drugs (eg/ Orlistat) 2) if that fails to work surgery - a) according to NICE considered if all criteria met I) BMI >40 or >35 if significant comorbidities that could improve with dec weight II) failure of non-surgical management to achieve & maintain beneficial weight loss for >6 months III) fitness for surgery + anaesthesia IV) part of integrated programme that includes guidance on diet, physical activity, and psychosocial concerns as well as lifetime monitoring V) pt must be well informed & motivated 3) laproscopic adjustable gastric banding (LAGB) - creates pre-stomach pouch by placing silicone band around top of stomach - serves as new smaller stomach 4) Roux-en Y gastric bypass - portion of jejunum attached to small stomach pouch to allow food to bypass distal stomach, duodenum + proximal jejunum - works by restriction + malabsorption 5) vertical banded gastroplasty - anastomose GOJ with GDJ & staple stomach NB/ need input from many health professionals eg/ OT, physio, podiatrist, specialist nurses, doctors etc

Discuss differential diagnosis of hypertension and causes of secondary hypertension

1) differential diagnoses: a) amphetamine toxicity b) anxiety disorders c) cocaine-related cardiomyopathy d) heart failure/MI e) hyperthyroidism f) primary aldosteronism g) stroke - haemorrhagic or ischemic 2) causes of secondary hypertension: a) renal disease - most common cause, 75% due to intrinsic renal disease eg/ glomerulonephritis, polyarteritis nodosa, systemic sclerosis, chronic pyelonephritis, polycystic kidneys, diabetic nephropathy. 25% due to renovascular disease eg/ atheromatous b) endocrine disease - Cushing's, Conn's, phaeochromocytoma, acromegaly, hyperparathyroidism, adrenal hyperplasia c) others - coarctation of aorta, pregnancy, steroids, MAOI, OCP, sympathomimetics, NSAIDs NB/ 8-10% of pregnancies, women suffer hypertension

Describe the circumstances in which preliminary consideration of screening would be reasonable and how screening should then be evaluated

1) disease - needs to be serious eg/ causes death, disability, or discomfort; natural history should be understood; disease must have a latent period during which it can be detected before symptoms appear 2) diagnosis & tx - facilities need to be adequate; there is an available, effective, acceptable, and safe treatment; early treatment should be more effective than later 3) screening test - should be sensitive (and ideally also specific), and have good predictive value (see below); simple and cheap; safe and acceptable; reliable 4) evaluating - test has to provide an advantage in distinguishing eg/ men with and without prostate cancer; also needs to demonstrate that early identification and treatment of the disease results in some improvement eg/ dec probability of dying of disease, or inc survival, or some measurable improvement in outcome

Outline the disease modifying therapies available in MS, and their effect on relapse rate and long term disability

1) disease modifying drugs (DMDs) are a group of treatments for people with relapsing multiple sclerosis. They reduce the number of relapses you might experience as well as reducing the severity of any relapses you do have 2) DMDs available - teriflunomide, interferon beta 1a or 1b, glatiramer acetate, fingolimod, alemtuzumab, cladribine, ocrelizumab, peginterferon beta 1a, dimethyl fumarate, natalizumab 3) benefits - a) fewer relapses b) less severe relapses c) dec build-up of disability which can occur if you don't recover completely from relapses d) DMDs work with different parts of the immune system to dec inflammation caused by MS to nerve cells in the brain and spinal cord - helps dec number and severity of relapses 4) still debate over whether DMDs slow down the accumulation of disability or the onset of progressive MS, although the preliminary results are encouraging

Discuss the anatomy of haemorrhoids

1) disrupted & dilated anal cushions. anus lined with discontinuous masses of spongy-vascular tissue (anal cushions) which contribute to anal closure - at 3, 7 and 11 o'clock 2) attached by SM and elastic tissue, but prone to displacement or disruption 3) effects of gravity (+age), inc anal tone (stress) + straining at stool make them become bulky & loose - protrude to form piles 4) vulnerable to trauma (eg/ hard stool) & bleed readily from capillaries of underlying lamina propria as have impaired venous return (engorged) 5) no sensory fibres above dentate line so not painful unless thrombose when protrude & gripped by anal sphincter, blocking venous return NB/ anal cushions - aside from sphincter function, provide important sensory information, allowing differentiation between solid, liquid and gas NB2/ engorgement may be furthered by straining, pregnancy, low-fibre diet & prolonged time on toilet

List the main causes of raised intracranial pressure

1) disturbance of CSF circulation - obstructive hydrocephalus, communicating hydrocephalus 2) bleeding into the brain, localised mass lesions: traumatic haematomas (extradural, subdural, intracerebral) 3) oedema - a) focal - secondary to trauma, infarction, tumour b) diffuse - encephalitis, meningitis, diffuse head injury, subarachnoid haemorrhage, Reye's syndrome, lead encephalopathy, water intoxication, near drowning 4) aneurysm 5) blood pooling in some part of the brain 6) brain or head injury 7) brain tumor - glioma, meningioma, metastasis 8) infections such as encephalitis or meningitis 9) hydrocephalus 10) high blood pressure 11) stroke 12) abscess 13) obstruction to major venous sinuses - depressed fractures overlying major venous sinuses, cerebral venous thrombosis 14) idiopathic intracranial hypertension

Distinguish on examination between an epigastric hernia and a divarication of the rectus abdominus muscle, and describe why one requires surgical treatment whereas the other doesn't

1) divarication = gap of ~ 2.7cm+ between 2 sides of rectus abdominis muscle 2) complication - children = development of an umbilical or ventral hernia (rare), medical attention when infant displays signs of vomiting, redness or pain in the abdominal area, weakening of abdominal muscles and the reduced force transmission from the stretched linea alba may also make it difficult to lift objects, and cause lower back pain, weakened pelvic alignment, altered posture 3) hernia ruled out using ultrasound NB/ epigastric hernias may occur anywhere between xiphoid process + umbilicus. Divarication of rectus muscles occurs along linea alba due to its stretching and weakness. Increased IAP causes L + R muscles to separate. USS may be useful for establishing anatomy of lesions. when pt does a sit-up, rectus muscles are only muscles being used so a midline bulge + muscle separation will present in both epigastric hernia and divarication. when pt coughs, all abdominal muscles used, so muscle separation & midline bulge will only occur in hernia

Discuss the differential diagnosis of rectal bleeding

1) diverticular disease 2) IBD 3) noninfectious gastroenteritis and colitis 4) benign anorectal diseases 5) haemorrhoids 6) anal fissure 7) fistula-in-ano 8) neoplasia 9) coagulopathy 10) arteriovenous malformations (AVMs) 11) ischaemic colitis 12) infants, children, and adolescents - Meckel diverticulum, intussusception, polyposis syndromes, IBD 13) colonic/rectal varices 14) small bowel ulceration

List the symptoms and signs of acute appendicitis

1) dull periumbilical pain that moves to RIF where is sharper (guarding, percussion tenderness, PR painful of right) 2) nausea + vomiting 3) fever + tachycardia 4) furred tongue 5) lying still 6) coughing hurts 7) foetor + flushing 8) shallow breaths 9) anorexia 10) Rovsing's sign (pain > in RIF than LIF) 11) psoas sign (pain on extending hip if retroceacal appendix) 12) cope sign (pain on flexion & internal rotation of R hip if appendix in close relation to obturator internus) 13) peritonitis may be present in perforation - associated pyrexia, tachycardia, tachypnoea NB/ rebound tenderness over McBurney's point (1/3rd of the way from ASIS to umbilicus).

Causes of dull percussion on different areas of the chest

1) dullness - inc tissue density eg/ consolidation, fluid, tumour, collapse 2) stony dullness - pleural effusion, bone 3) hyper-resonance - dec tissue density eg/ pneumothorax, COPD, asthma attack

List the symptoms of peptic ulcer disease

1) duodenal: a) epigastric pain before meals or at night, relieved by eating or drinking milk b) 50% asymptomatic, others experience recurrent episodes c) epigastric tenderness 2) gastric: a) asymptomatic or b) epigastric pain related to meals and relieved by antacids c) dec weight 3) both: a) pain worse/only noticed at night b) can radiate to the back c) nausea + anorexia (& weight loss) may accompany pain d) bleeding & perforation may occur and may be noticed in vomit/faeces though vomiting is not always present e) perforation into adjacent structures eg/ transverse colon, may result in fistulae. f) natural history of disease may take 2 major paths: I) relapse & remittance over years due to atrophic gastritis & dec in acid secretion II) sudden presentation as anaemia from blood loss, abrupt haematemesis or acute perforation g) DUs tend to occur in younger demographic -45-55yrs, with GUs in 55-65yr olds h) DUs 2-3x more common than GUs

List the investigations of peptic ulcer disease

1) duodenal: a) upper GI endoscopy - stop PPIs 2 weeks before b) test for H. pylori c) measure gastrin concn when off PPIs if zollinger-ellison syndrome suspected 2) gastric: a) upper endoscopy to exclude malignancy b) multiple biopsies from ulcer rim + base & brushings 3) both: a) purge stress b) avoid aggravating food and drink c) H. pylori eradication (triple therapy) d) drugs to dec acid - PPIs e) for drug-induced ulcers - stop drug, PPIs best for treating & preventing gi ulcers & bleeding in pts on NSAID or antiplatelet drugs 4) endoscopy can visualise ulceration & required in all pts with alarm symptoms (ALARMS55 - anorexia, loss of weight, anaemia, rapid onset, melaena/haematemesis, swallowing problems, age>55) 5) all GUs should be biopsied to exclude cancer 6) 13C urease test is sensitive & specific (>96%) for identification of H Pylori infection - pt ingests 13C urea & breathes out 13CO2

Clinical features associated with parietal lobe disease

1) dysgraphia 2) apraxia 3) sensory inattention

Discuss the microvascular complications of diabetes affecting the nerves and outline their relationship to diabetic control and disease duration

1) earliest change in diabetic neuropathy = dec in nerve conduction velocity caused by damage to Schwann cells, due to direct metabolic damage or microvascular (vasa nervorum) damage 2) 5 varieties of neuropathy: symmetrical distal sensorimotor polyneuropathy ('walking on cotton wool'); acute painful neuropathy; mononeuropathy and mononeuritis; diabetic amotrophy; autonomic neuropathy symmetrical distal sensorimotor polyneuropathy 1) vibration, temperature & pain sensation lost (deep before superficial) 2) involvement of motor nerves = interosseous wasting, giving high arch & clawing of toes a) leads to abnormal distribution of pressure when walking = callous formation at 1st metatarsal head & ulceration b) Charcot's joints may develop in the ankle c) pattern of loss = "glove & stocking" loss, though stocking loss more common d) loss is slow and progressive (rising) acute, painful neuropathy 1) less common - pt describes a burning or crawling pain in feet, up the leg 2) symptoms worse at night, usually remits spontaneously after 3-12 months if good control maintained mononeuropathy 1) may be painful, commonly affects CN3 + 6, but any nerve may be targeted 2) in diabetic CN3 lesions, pupillary reflexes maintained as fibres are spared 3) full spontaneous recovery 3-6 months later 4) carpal tunnel syndrome more common in diabetic pts amyotrophy 1) asymmetrical, painful wasting of muscles - quadriceps or shoulders 2) area will be tender, reflexes may be absent, alongside extensor plantar response 3) usually associated with periods of poor control, resolving in time with better control autonomic neuropathy 1) rarely symptomatic, but can affect SNS + PNS, potentially causing disabling postural hypotension 2) other possible effects: GI + CV disturbance, bladder involvement, erectile dysfunction

Classify asthma into early and late onset, and discuss the characteristics of a typical patient who is likely to present with each type

1) early onset = childhood, late onset = adulthood 2) symptoms of both: a) wheezing b) coughing c) congestion d) chest pain/pressure e) inc mucus secretion in airways f) SOB after physical activity g) difficulty sleeping h) delayed recovery from a respiratory infection eg/ flu or cold 3) both can be triggered by allergens eg/ smoking, mould, feather bedding, pollution, animal dander 4) differences - children: a) intermittent symptoms b) allergens can set off attack - children typically more sensitive to allergens so more prone to an asthma attack because their bodies are still developing c) may find that asthma symptoms almost completely disappear or are less severe during puberty, but they may recur later in life 5) differences - adults: a) symptoms are typically persistent - daily treatment often required to keep asthma symptoms & flare-ups under control NB/ extrinsic asthma = young pts with atopy, inc IgE, eczema and/or rhinitis, may disappear at 15 but may recur NB2/ intrinsic asthma = pts >30yo, no history of atopy, skin tests usually -ve & IgE normal, remission is rare - may present with SOB & tight chest

Formulate a differential diagnosis for acute appendicitis

1) ectopic pregnancy - pregnancy test 2) UTI (test urine) 3) mesenteric adenitis/lymphadenitis 4) cystitis 5) cholecystitis 6) diverticulitis 7) salpingitis/PID 8) dysmenorrhoea 9) Crohn's disease (IBD) 10) perforated ulcer 11) food poisoning 12) Meckel's diverticulum 13) ovarian cyst rupture, 14) caecal volvulus, 15) psoas abscess 16) cancer

Describe the main clinical features, immunopathology, investigation and principles of management of the following conditions: eczma, urticaria and angioedema; pernicious anaemia, Addison's disease, autoimmune liver disease and bullous skin diseases

1) eczema - a) s/s - I) risk factors eg/ allergic rhinitis, asthma, age <5 years, fmx eczema II) pruritus III) xerosis (dry skin) IV) others - erythema, scaling, vesicles, papules, keratosis pilaris, excoriations, lichenification, hypopigmentation b) pathology - I) impairment of the skin's barrier function leads to an inc sensitisation to cutaneous antigens. May have some genetic link II) in the acute phase of eczema, the immune response following sensitisation is predominantly Th2 mediated, with over-expression of IL-4, IL-5, and IL-13. These interleukins lead inc IgE and peripheral eosinophilia III) persistent inflammation and scratching can eventually lead to chronic eczema, with thick, lichenified skin. Lesions demonstrate a different complement of immune cells and cytokines, with a predominant Th1 response, and increased levels of IL-12 c) ix - usually by clinical diagnosis, don't need any tests I) consider - allergy testing (eg/ dust mites, pollens and grasses, moulds, animal dander, milk, and egg), IgE levels (inc), skin biopsy d) mx - I) 1st line - emollients: applied 3x/day II) + intermittent topical corticosteroids: hydrocortisone topical: (0.2-2.5%) apply sparingly to the affected area(s) twice daily III) +/- topical calcineurin inhibitor eg/ pimecrolimus; +/- topical crisaborole IV) evidence of cutaneous infection or suspected colonisation: +/- oral abx V) persistent pruritus or symptoms resulting in sleep disturbance: +/- antihistamine or doxepin: chlorphenamine or diphenhydramine 2) urticaria & angioedema - a) s/s - I) erythematous oedematous lesions II) pruritus III) resolution within 24 hours IV) swelling of face, tongue, or lips V) blanching lesions b) pathology - mast cells are the primary effector cells in urticaria. Mast cells are distributed throughout the body, including within the dermis, sub-dermis, and mucosal surfaces. When mast cells are activated, de-granulation leads to a rapid release of pre-formed vasoactive mediators such as histamine, leukotriene C4, and prostaglandin D2, causing vasodilation and inc vascular permeability. This manifests clinically as oedema and pruritus. A second, delayed release of inflammatory cytokines (TNF, interleukin-4, interleukin-5) accounts for an inflammatory infiltrate and longer-lasting lesions. Urticaria is confined to the dermal layer, whereas angio-oedema involves the deeper sub-dermal layers and mucosal sites c) ix - clinical diagnosis I) consider - FBC with differential, complete metabolic panel, urinalysis, ESR & CRP (inc), anti-IgE receptor antibody and related tests (+ve in autoimmune-related chronic urticaria), TSH & antithyroid antibodies, antinuclear antibodies (ANA +ve in many rheumatological diseases), skin biopsy, C4 & C1-esterase inhibitor level & C1q levels (dec in hereditary and acquired angio-oedema) d) mx - airway involvement I) 1st line - adrenaline (epinephrine) + airway protection: adrenaline 0.5mg 1:1000 IM every 1-2 hours when required II) + H1-receptor antagonists (antihistamines): diphenhydramine: 25-50 mg PO every 4-6 hours III) + IV systemic corticosteroids: methylprednisolone sodium succinate IV) + trigger identification and avoidance without airway involvement - 1st line - trigger identification and avoidance I) 1st line -trigger identification & avoidance eg/ dietary modifications, meds, physical stimuli II) + H1-receptor antagonists (antihistamines): loratadine or desloratadine or cetirizine or levocetirizine or fexofenadine III) + systemic corticosteroids: prednisolone IV) +/- adrenaline (epinephrine): 0.3 mg (1:1000) IM every 1-2 hours when required 3) pernicious anaemia - a) s/s - insidious I) untreated can lead to neurological complications, and in serious cases, death II) many s/s due to anaemia eg/ triad of tingling or other skin sensations (paresthesia) + tongue soreness (glossitis) + fatigue and general weakness III) can have depressive mood, low-grade fevers, diarrhea, dyspepsia, weight loss, neuropathic pain, jaundice, angular cheilitis, pale and dehydrated IV) can also have difficulty in proprioception, memory changes, mild cognitive impairment, psychoses, impaired urination, loss of sensation in the feet, unsteady gait, difficulty in walking, muscle weakness and clumsiness V) anaemia may also lead to tachycardia, cardiac murmurs, yellow waxy pallor altered BP, SOB b) pathology - anaemia from lack of intrinsic factor, most commonly due to an autoimmune attack on the cells that create it in the stomach. It can also occur following the surgical removal of part of the stomach or from an inherited disorder. Other causes of low vitamin B12 - dec dietary intake (eg/ vegan diet), celiac disease, or tapeworm infection c) ix - I) blood smear - shows large, fragile, immature erythrocytes (= megaloblasts) II) FBC - shows inc mean corpuscular volume (MCV - macrocytic anaemia) & normal mean corpuscular hemoglobin concentration (MCHC) III) presence of antibodies to gastric parietal cells and intrinsic factor IV) inc gastrin levels V) diagnosis of atrophic gastritis type A confirmed by gastroscopy and stepwise biopsy d) mx - I) vit B12 replacement - PO or IV cyanocobalamin/hydroxocobalamin II) IM cobalamin: cyanocobalamin (CN-Cbl), hydroxocobalamin (OH-Cbl) or methylcobalamin 4) Addison's disease - a) s/s - I) risk factors eg/ female, adrenocortical autoantibodies, adrenal haemorrhage, use of anticoagulants II) fatigue III) anorexia IV) weight loss V) hyperpigmentation VI) others - N&V, hypotensionb) b) pathology - disease results from dec production of adrenocortical hormones (aldosterone, cortisol, and dehydroepiandrosterone) either from destruction of the 3 layers of the adrenal cortex (glomerulosa, fasciculata, and reticularis) or disruption of hormone synthesis. Infiltrating diseases such as TB or metastasis can also cause destruction of the adrenal medulla c) ix - I) serum electrolytes - low sodium; inc potassium; rarely, inc calcium II) blood urea - may be inc III) FBC - anaemia, eosinophilia IV) morning serum cortisol - blood drawn between 8-9 a.m, when cortisol levels peak. Low serum cortisol: <83 nanomols/L V) consider - ACTH stimulation test - serum cortisol <497 nanomols/L (<18 micrograms/dL) VI) serum ACTH - >22 picomols/L (>100 picograms/mL) VII) plasma renin activity - inc VIII) plasma aldosterone & plasma dehydroepiandrosterone (DHEA) & plasma DHEA sulphate (DHEAS) - suppressed IX) adrenal antibodies - may be present X) adrenal CT or MRI - commonly normal or atrophic adrenals; may be enlarged with or without calcifications XI) insulin hypoglycaemia test & overnight single-dose metyrapone test - abnormal in primary or secondary adrenal insufficiency d) mx - adrenal crisis - I) 1st line - glucocorticoid and supportive therapy: dexamethasone sodium phosphate: 4 mg IV OD for 1-3 days or hydrocortisone sodium succinate: 50-100 mg IV every 6-8 hours for 1-3 days II) + IV fluids - 1 L rapidly and a further 2-4 L over 24 hours, to correct hypotension. Careful monitoring of BP, fluid status, and serum sodium and potassium levels should be maintained III) +/- glucose to correct hypoglycaemia IV) +/- tx of cause of crisis stable and/or after treatment of acute episode - I) 1st line - glucocorticoid + mineralocorticoid: cortisone: 10-37.5 mg/day orally in 2 divided doses (2/3rds in morning, 1/3rd afternoon), or hydrocortisone: 15-30 mg/day orally as above, or dexamethasone etc II) +/- temporary switch to stress dosing of glucocorticoid: cortisone: 50-100 mg/day for 3 days III) women with decreased libido: +/- androgen replacement - dehydroepiandrosterone 5) autoimmune liver disease - a) s/s - I) risk factors eg/ female, genetic pre-disposition, immune dysregulation II) fatigue/malaise III) anorexia IV) abdominal discomfort V) hepatomegaly VI) jaundice VII) others - pruritus, arthralgia, nausea, fever, spider angiomata b) pathology - in a genetically pre-disposed person, an environmental agent can trigger a pathogenic process leading to liver necrosis and fibrosis. Auto-antigens initiate the cascade of events in AIH. Appears to be a common susceptibility determinant in the HLA-class II binding groove crucial to antigen recognition. Most of the evidence supports a central role for an alteration in T-cell function, although abnormalities in B-cell function may also be important for the escape from suppressive mechanisms and the development of the necro-inflammatory process of AIH c) ix - I) AST & ALT - inc ~200-300 international units/L II) bilirubin & gamma-GT & alkaline phosphatase - mildly to moderately inc III) serum globulin - markedly inc in patients with untreated AIH IV) serum albumin - dec V) prothrombin time - prolonged: good indicator of hepatocellular function VI) consider - ANA & smooth muscular antibodies (anti-nuclear antibodies): titres 1:80 or higher in adults associated with type 1 AIH, anti-SLA/LP (anti-soluble liver antigens or liver/pancreas) - +ve if type 1 AIH, p-ANCA (perinuclear anti-neutrophil cytoplasmic antibodies - atypical), anti-ssDNA (can be +ve), anti-LKM-1 or 3 (inc is associated with type 2 AIH), anti-LC1 (can be +ve), AMA (anti-mitochondrial antibodies), IgM anti-HAV (-ve), anti-HBc (-ve), hep C antibodies and viral RNA (-ve), ceruloplasmin (normal), alpha-1 antitrypsin (normal), serum iron and TIBC (normal), thiopurine methyltransferase (TPMT) activity (dec), abdominal ultrasound (biliary dilatation), liver biopsy (periportal lesion or interface hepatitis) d) mx - non-severe disease I) asymptomatic - observation and monitoring only; otherwise - II) observation and monitoring III) expected tx <6 months or immunosuppressant contraindication: +/- prednisolone IV) expected tx >6 months with no immunosuppressant contraindication: +/- prednisolone + azathioprine or mercaptopurine severe disease - I) expected tx <6 months or immunosuppressant contraindication: prednisolone II) expected tx >6 months with no immunosuppressant contraindication: prednisolone + azathioprine or mercaptopurine 6) bullous skin diseases - a) s/s - I) risk factors eg/ male, aged 60-90 II) pruritus III) tense blisters on normal or erythematous skin IV) others - erythematous or urticarial plaques b) pathology - auto-antibodies directed against 2 hemidesmosomal proteins, designated BP180 and BP230. While BP230 localises intracellularly and associates with the hemidesmosomal plaque, BP180 is a transmembrane glycoprotein with an extracellular domain. In most bullous pemphigoid sera, circulating antibodies to BP180NC16A are detected with serum levels correlating with disease activity. Bullous pemphigoid often provokes blood and tissue eosinophilia, which suggests chemoattractants may modulate the eosinophil infiltration. Eotaxin and interleukin (IL)-5 are strongly associated with the tissue eosinophilia of bullous pemphigoid. These findings suggest that eotaxin and IL-5 may be important for eosinophil migration in bullous pemphigoid lesions and that therapies that aim to inhibit production of eotaxin and IL-5 may improve inflammation and blister formation c) ix - I) skin biopsy for histopathological evaluation with light microscopy - sub-epidermal blister with dermal inflammatory cell infiltrate rich in eosinophils II) skin biopsy for direct immunofluorescence testing - linear band of IgG and/or C3 (rarely other Ig classes) along the basement membrane zone III) indirect immunofluorescence test on serum - +ve titre for antibodies to bullous pemphigoid antigens IV) consider - ELISA test (to determine specific region of the bullous pemphigoid antigens) d) mx - localised lesions - topical corticosteroids or topical tacrolimus +/- sedating antihistamines widespread lesions - I) 1st line - oral corticosteroids: prednisolone II) +/- sedating antihistamines: hydroxyzine or diphenhydramine III) 2nd line - oral corticosteroids plus antibiotic therapy: prednisolone + nicotinamide + tetracycline or doxycycline or minocycline or erythromycin base IV) +/- ciclosporin; +/- sedating antihistamines inadequate response to initial therapy - I) 1st line - prednisolone + azathioprine or mycophenolate mofetil or ciclosporin or cyclophosphamide or methotrexate + folic acid or calcium folinate II) +/- sedating antihistamines: hydroxyzine or diphenhydramine III) 2nd line - plasmapheresis or IVIG and/or rituximab

Describe an appropriate investigation plan for a patient with recurrent syncope

1) electrocardiography 2) echocardiography - if clinical heart disease is suspected 3) exercise stress testing - if syncope on exertion in the absence of ventricular outflow tract obstruction 4) ambulatory electrocardiographic monitoring - to diagnose an arrhythmia as the cause of syncope 5) electrophysiological testing - can assess various arrhythmias 6) tilt table testing - identifies syncope caused by neurally mediated reflexes 7) coronary angiography - if known ischaemia and syncope

Describe the rationale for adjuvant radiotherapy, chemotherapy, hormonal therapy and biological therapy in the treatment of breast cancer

1) endocrine therapy - tamoxifen if pre-menopausal, aromatase inhibitors eg/ letrozole if post-menopausal a) oestrogen receptor +ve tumour b) down-staging primary lesions c) definitive tx in old, infirm pts 2) irradiation - a) wide local excision b) large lesion, high grade or marked vascular invasion following mastectomy 3) immunotherapy eg/ Herceptin (Trastuzamab) - a) cancers expressing specific growth factor receptors eg/ HER-2 4) chemotherapy eg/ FEC regime - flucoracil, epirubicin, cyclophosphamide - a) down staging advanced lesions to facilitate beast conserving surgery b) pts with grade 3 lesions or axillary nodal disease

List the complications of gallstones and describe the history, physical examination and laboratory findings for each

1) endoscopic retrograde cholangiopancreatography (ERCP)-associated pancreatitis - a) s/s - I) risk factors eg/ middle-aged women, young- to middle-aged men, gallstones, alcohol, hypertriglyceridaemia, certain meds, ERCP procedure, HIV/AIDS, SLE, and Sjogren's syndrome II) N&V III) abdominal pain - mid-epigastric pain that radiates to the back IV) tachycardia b) ix - serum lipase: 3x upper limit 2) iatrogenic bile duct injuries 3) post-sphincterotomy bleeding 4) Bouveret syndrome - gastric outlet obstruction secondary to impaction of a gallstone in the pylorus or proximal duodenum a) s/s - variable and nonspecific but often includes: I) N&V II) epigastric pain III) hx of gallstone disease in a minority of patients b) ix - I) plain radiograph - Rigler triad: bowel obstruction + pneumobilia + ectopic gallstone II) US - cholecystoenteric fistula, residual gallstones and gastric outlet obstruction 5) gallstone ileus - a) s/s - I) long history of recurrent RUQ pain, in keeping with chronic cholecystitis with repeated inflammatory events II) can acutely present as colicky abdol pain & distension in the course of small bowel obstruction b) ix - I) x-ray - Rigler triad: bowel obstruction + pneumobilia + ectopic gallstone, most frequently, stones become impacted in the distal ileum II) CT 6) cholecystitis - a) s/s - I) previous episode of biliary pain II) RUQ pain III) positive Murphy's sign - palpation of R subcostal region reveals tenderness. During deep inspiration, the tenderness suddenly becomes worse and produces inspiratory arrest IV) abdominal mass V) others - right shoulder pain, anorexia, nausea, fever b) ix - I) LFTs - show a cholestatic picture: inc alkaline phosphatase, gamma-GT, and bilirubin II) RUQ US - pericholecystic fluid, distended gallbladder, thickened gallbladder wall, gallstones, positive Murphy's sign 7) ascending cholangitis - a) s/s - I) risk factors eg/ 50+, hx of cholelithiasis, primary or secondary sclerosing cholangitis, stricture of the biliary tree (benign or malignant), or post-procedure injury of bile ducts (surgical, endoscopic, or radiological intervention with resulting inadequate biliary drainage) II) RUQ/upper abdominal pain and tenderness III) jaundice IV) fever V) others - acholic stools (putty/clay coloured stools), pruritus b) ix - FBC, U&E, LFT, US abdo, ERCP 8) acute biliary pancreatitis 9) Mirizzi syndrome - a) s/s - varies from no symptoms to severe cholangitis I) may present with recurrent episodes of jaundice and cholangitis II) may be associated with acute cholecystitis b) ix - I) US - can reveal gallstones and cholecystitis, evidence of Mirizzi syndrome such as an atrophic gallbladder and ectatic common hepatic duct with a normal distal CBD, or edematous gallbladder caused by acute cholecystitis II) CT - useful for detecting the cause and location of biliary obstruction III) MRCP -preferred imaging modality, can delineate the typical characteristics of Mirizzi syndrome IV) ERCP - gold standard for diagnosis of Mirizzi syndrome

List the indications for enteral and parenteral nutritional support; for each condition, identify the appropriate route for administering support. Contrast the risks and benefits of enteral and parenteral nutritional support. List the trace elements that must be replaced in a patient on long-term parenteral nutrition

1) enteral - preferred if gut is working normally, may have to be delivered into the gut through a tube, or may be possible for them to take this by mouth a) indications - I) by mouth - for patients who are unable to eat enough food, either from a poor appetite, eating is difficult or because their body requires additional energy because of illness. These products provide more energy and nutrition than normal food, so patients don't have to consume a large amount II) by tube - when nutrition cannot be taken normally by mouth, but the gut is otherwise working eg/ stroke or neurological conditions which impair swallowing, some operations on the face, neck, throat, gullet or stomach, blockages or radiotherapy to gullet or stomach b) risks - major limitation with dysfunctional GIT - feeding intolerance due to dysmotility & malabsorption = frequent interruptions and suboptimal delivery rates c) benefits - preferred for pts with a functioning GIT, fewer infectious complications, dec cost, earlier gut function, dec length of stay 2) parenteral - if patients gut cannot be used to absorb nutrients, then nutrition must be delivered into the patient's blood stream, bypassing the gut a) indications - obstruction of gut or ileus, perforations of the gut where feeding will result in worsening infections, short bowel syndrome, functional short bowel b) risks - complications rising from the intravenous presence of a foreign body and the ability for greater (and therefore more unphysiological) manipulation of nutrient intake, inc cost, need more intensive monitoring, inc risk of complications c) benefits - provision of adequate nutrients when GI dysfunction, also useful with fluid restriction as caloric density can be increased with a central venous catheter beyond that tolerated by the enteral route. General indication is pts who cannot, will not, or should not eat, enterally or orally or who cannot eat enough or be fed adequately by tube d) trace elements - zinc, copper, manganese, and chromium

Describe the typical examination features of a pleural effusion and describe the aetiology and clinical features of an empyema

1) exam - a) can have: dyspnoea, pleuritic chest pain b) dec expansion + mediastinum shifted away c) stony dull percussion d) diminished breath sounds (NB/ 2-4 only on affected side) e) tactile vocal fremitus & vocal resonance dec f) may be bronchial breathing above effusion (lung compressed) g) large effusions - may be tracheal deviation away from effusion h) look for aspiration marks & signs of associated disease: I) malignancy - cachexia, clubbing, lymphadenopathy II) stigmata of chronic liver disease III) cardiac failure IV) hypothyroidism V) rheumatoid arthritis VI) butterfly rash of SLE 2) aetiology - a) pus in pleural space - suspected if pt with resolving pneumonia develops recurrent fever b) complication of bacterial pneumonia or TB, rarely from transfer of subphrenic abscess through diaphragm or infection of haemothorax 3) s/s - fever, rigors, malaise, pleuritic pain, SOB, signs similar to those of an effusion 4) clinical features - CXR indicates pleural effusion, aspirated pleural fluid is yellow & turbid with pH <7.2, dec glucose, inc LDH

Describe the physical findings in pts with reducible inguinal hernias, including examination of the external ring and descent to the scrotum, and incarcerated inguinal herniae including the signs of bowel obstruction and possible strangulation

1) exam of external ring and descent to scrotum - place a fingertip into the scrotal sac and advance up into the inguinal canal. Locate deep inguinal ring (midway between ASIS + pubic tubercle), press firmly on lump and, starting inferiorly, try and lift it up & compress it towards the DIR. Once reduced slide fingers up & maintain pressure over DIR. Ask pt to cough - if hernia reappears = direct, otherwise = indirect. Release and watch hernia reappear (indirect will slide down obliquely, direct will project forwards) 2) incarcerated inguinal hernia - dead intestine releases toxins into blood - causes septicaemia and death. Warning might come on very suddenly from nothing at all. 1st = acute pain that can rapidly become severe and nothing relieves it. Common for this onset to be measured in mins. More likely in smaller hernias. May also have - vomiting, blood in faeces, constipation, malaise with or without a fever, burning or hot sensation around hernia 3) signs of bowel obstruction - depends on location. Higher up GI tract = vomiting early, constipation later, lower down = constipation early & vomiting later. Both have abdominal pain + distension NB/ scrotal continuation of a hernia is more common in indirect hernia but may occur in either. Incarceration will cause bowel obstruction, characterised by constipation, distension, vomiting, pain. On examination = inc bowel sounds. Strangulation & ischaemia associated with 4 signs of inflammation (pain, redness, swelling, warmth) and tenderness

List the causes of a raised serum calcium concentration including hyperparathyroidism and malignancy

1) excess PTH secretion: primary (or tertiary) hyperparathyroidism, adenoma, carcinoma, ectopic 2) malignant disease: myeloma, bony mets, PTHrP, osteoclastic factor release 3) excess vitD: vitamin tablets, granulomatous disease, lymphoma 4) excess calcium intake: milk-alkali syndrome 5) other endocrine disease (mild hypercalcaemia): Thyrotoxicosis, Addison's disease 6) drugs: thiazides, vitamin A, lithium (long term) 7) misc: long-term immobility, familial hypocalciuric hypercalcaemia NB/ technique for correcting calcium - add 0.02 for every 1 albumin below 40

State the aetiological factors of haemorrhoids

1) excessive straining due to either chronic constipation or diarrhoea - causes downward stress on the vascular haemorrhoidal cushions, leading to the disruption of the supporting tissue elements with subsequent elongation, dilation, and engorgement of the haemorrhoidal tissues 2) inc intra-abdominal pressure eg/ pregnancy or ascites 3) space-occupying lesions within the pelvis 4) age - incidence peaks between 45-65 yo & declines after age 65 years

Outline the features of specific pathogens that contribute to their pathogenicity (eg. exo and endo-toxins) and their propensity to spread

1) exotoxins: heat labile proteins secreted by certain species of bacteria which diffuse into the surrounding medium a) found in both gram +ve and -ve bacteria b) highly antigenic c) toxoids can be made by tx with formalin d) highly toxic, fatal in µg quantities e) bind to specific receptors f) eg/ Staph aureus, Bacillus cereus, Strep pyogenes, Bacillus anthrcis g) diseases - tetanus, diphtheria, botulism 2) endotoxins: heat stable lipopolysaccharide-protein complexes which form structural components of cell wall of Gram -ve bacteria and liberated only on cell lysis or death of bacteria a) only gram -ve bacteria b) weakly immunogenic c) toxoids cannot be made d) moderately toxic e) no specific receptors f) eg/ E.coli, Salmonella Typhi, Shigella, Vibrio cholera(Cholera toxin g) diseases - meningococcemia, sepsis

Describe the role of the anal sphincters in maintaining faecal continence

1) external + internal anal sphincters surround anal canal 2) IAS - SM ring at upper end of anus a) when anal canal is empty, it is contracted, when it receives something from the gut, it stretches b) don't consciously control this muscle ring - work is done automatically 3) EAS - ring of muscle lower down & around anus a) under voluntary control (can squeeze) 4) severe faecal incontinence when the contractility of both the IAS and the EAS is affected

Differentiate external from internal haemorrhoids

1) external - origin below dentate line (external rectal plexus) a) lined with epithelium (ectoderm) b) innervated by cutaneous branches of pudendal nerve - become painful c) drain via middle & inferior rectal veins into systemic circulation d) may cause problems around entire circumference of anus - this region marks 1 of 4 major porto-systemic anastomoses 2) internal - origin above dentate line (internal rectal plexus) a) covered with columnar epithelium (endoderm), and are not painful b) drain via superior rectal vein into portal venous system c) cushions make up internal haemorrhoids 3) mixed - both external + internal plexuses, above + below dentate line

List the risk factors for carcinoma of the breast

1) family history for some types of breast cancer 2) alcoholic drinks 3) oestrogen-progestogen contraceptives 4) hormone replacement therapy (HRT) (diethylstilbestrol) 5) X radiation and gamma-radiation 6) body fatness - obesity 7) adult attained height 8) female gender 9) late menopause/early menarche 10) age (inc) 11) nulliparity 12) BRCA 1 & 2 genes 13) previous benign disease

Discuss common investigations to exclude a secondary cause of hypertension

1) fasting metabolic panel with estimated GFR - inc risk of hypertension if features of the metabolic syndrome 2) fasting lipid panel - metabolic syndrome: may show high LDL, low HDL, or high triglycerides 3) Hb - anaemia accompanies chronic renal failure, polycythaemia may be seen with phaeochromocytoma 4) TSH - indicated if signs/symptoms of hypo- or hyperthyroidism 5) plasma renin activity (PRA) - indicated when unprovoked hypokalaemia present, low renin suggests hyperaldosteronism 6) plasma aldosterone - high aldosterone or failure to suppress with salt loading suggests hyperaldosteronism 7) renal duplex ultrasound/MRA renal arteries/CT angiography - may show renal artery stenosis, renal scarring, or lesions 8) 24-hour urine phaeochromocytoma screen - inc catecholamines if phaeochromocytoma 9) plasma fractionated metanephrines - inc metanephrines if phaeochromocytoma 10) 24-hour urine free cortisol - inc in Cushing's disease

Describe the typical clinical features and common infecting organisms in infective endocarditis

1) fever, malaise and endurance fatigue, weight loss 2) new or changing heart murmur 3) coughing 4) vascular phenomena: septic embolism (= problems such as stroke or gangrene of fingers), Janeway lesions (painless haemorrhagic cutaneous lesions on palms & soles), intracranial or conjunctival or splinter haemorrhages, kidney or splenic infarcts 5) immunologic phenomena: glomerulonephritis (blood & albumin in urine), Osler's nodes (painful subcutaneous lesions in the distal fingers), Roth's spots on retina, +ve serum rheumatoid factor 6) other signs: night sweats, rigors, anaemia, spleen enlargement NB/ cerebral emboli, mycotic aneurysms, cardiac failure = serious complications 7) organisms - a) bacterial: Strep viridans (>35%) - mouth or valve disease, Enterococci - gut, Staph aureus/epidermis - intra-vascular device/injection or valve disease, diptherioids, microaerophilic streps, rarely HACEK organisms, Coxiella burnetii, Chlamydia, staphy 2) fungi: Candida, Aspergillus, Histoplasma NB/ valves in immunocompromised pts may be colonised by opportunistic pathogens, such as Candida

Identify and describe the major types of breast lump (fibroadenoma, fibroadenosis, cyst, carcinoma). Outline the natural history of benign breast lumps

1) fibroadenoma a) s/s - lump in breast with rubbery texture, smooth to touch & moves easily under the skin. Painless, but may feel tender, particularly just before a period. Can be seen on USS or mammography, very common among younger women b) natural history - start after puberty. Partially hormone-related and frequently regress after menopause. Higher intake of fruits and vegetables, higher number of live births, use of oral contraceptives and moderate exercise associated with lower frequency 2) fibroadenosis a) s/s - fibrous tissue and a lumpy, cobblestone texture in breasts. Lumps are smooth with defined edges, and free-moving. Bumps can sometimes be obscured by irregularities in the breast associated with the condition. Lumps most often in upper, outer sections of breast (near armpit). May have a persistent or intermittent breast aching or breast tenderness related to periodic swelling. Breasts and nipples may be tender or itchy. Symptoms peak in the days before each period and decrease afterwards. At peak, breasts may feel full, heavy, swollen, and tender to the touch b) natural history - exact mechanism not fully understood, though tied to hormone levels, as subsides after menopause & related to menstrual cycle. Post-menopausal women placed on HRT also report symptoms of FBC 3) cyst a) s/s - fluid-filled sac within breast. Can have 1+ cysts. Round or oval lumps with distinct edges - feels like a soft grape or water-filled balloon, but sometimes feels firm. Can be painful. Most common in pre-menopausal women in 30s-40s. Disappear after menopause, but may persist or reappear with hormone therapy. Pain and swelling worse in 2nd half of menstrual cycle or during pregnancy b) natural history - develop naturally with age, due to changes in hormone levels NB/ fibroadenoma - in a small breast may be large deviation so may remove for cosmetics NB2/ in benign breast lumps, axillary lymph nodes should be impalpable

Describe the principles of management of fibroadenoma, cyst, nipple discharge and breast pain

1) fibroadenoma - normally doesn't need any tx (as many will re-occur, and not harmful), may have removed surgically if: small breasts and large lump, causing pain, pt overly worried eg/ lumpectomy or excisional biopsy; or cryoablation 2) cyst - if asymptomatic can leave, if pt distressed or causing symptoms aspirate using US 3) nipple discharge - a) non-suspicious and breast mass not present - reassurance with observation b) non-suspicious and breast mass present - referral to oncologist c) suspicious - referral and surgery, if discharge is bloody and very profuse, it may be necessary to perform either microductectomy (removal of single symptomatic milk duct) or central duct excision 4) breast pain - a) 1st line - supportive measures: reassurance (explanation of the effects of menstrual hormonal cycling and oestrogen effects is effective in relieving patient anxiety), bra that provides good support can help, some may restrict caffeine or sodium, if on HRT - modification of therapy regimens may help b) +/- analgesia - paracetamol, or ibuprofen, or aspirin c) +/- evening primrose oil d) 2nd line - hormonal therapy: tamoxifen or bromocriptine - consider if breast pain is severe, has persisted for more than 6 months, and significantly interferes with daily activities; +/- analgesia

Outline appropriate investigations for a patient with confirmed iron deficiency anaemia

1) find & treat underlying cause 2) investigate for GI blood loss a) sigmoidoscopy/endoscopy/colonoscopy b) barium enema c) stool microscopy (hookworm) 3) ask about diet 4) systems enquiry for multi-system disorder or other chronic disease 5) give oral iron and see if Hb normalises - if it doesn't, this may suggest continued bleeding, chronic disease, or malabsorption (or simple lack of adherence)

Outline a classification of epilepsy and describe the differential diagnosis of epilepsy

1) firstly define seizures by how they begin in the brain - a) focal seizures - start in an area or network of cells on one side of the brain b) generalized seizures - engage or involve networks on both sides of the brain at the onset c) focal to bilateral seizure - starts in one side or part of the brain and spreads to both sides 2) awareness - a) focal aware - if awareness remains intact, even if the person is unable to talk or respond b) focal impaired awareness - If awareness is impaired or affected at any time during a seizure c) generalized seizures - all presumed to affect a person's awareness or consciousness in some way. Thus no special terms are needed to describe awareness in generalized seizures 3) Describing Motor and Other Symptoms in Focal Seizures - a) focal motor seizure - some type of movement occurs during the event eg/ twitching, jerking, or stiffening movements of a body part or automatisms (such as licking lips, rubbing hands) b) focal non-motor seizure - has other symptoms that occur first, such as changes in sensation, emotions, thinking, or experiences c) auras 4) Describing Generalized Onset Seizures - a) generalized motor seizure - generalized tonic-clonic seizure describes seizures with stiffening (tonic) and jerking (clonic) b) generalized non-motor seizure - primarily absence seizures, involve brief changes in awareness, staring, and some may have automatic or repeated movements like lipsmacking 5) summary - a) focal - aware or not aware - motor: automatisms, atonic, clonic, epileptic spasms, hyperkinetic, myoclonic, tonic; non-motor: autonomic, behaviour arrest, conitive, emotional, sensory b) generalised - motor: tonic-clonic, clonic, tonic, myoclonic, myoclonic-tonic-clonic, myotonic-atonic, atonic, epileptic spasms; non-motor (absence): typical, atypical, myoclonic, eyelid myoclonic 6) DD - stroke/TIA, hypovolemia (e.g., blood loss, diuretics), dec arterial or venous tone (e.g., vasodilators, autonomic dysfunction), limited cardiac output (e.g., aortic stenosis, arrhythmias), inappropriate baroreceptor reflexes (e.g., emotional situations, Valsalva maneuver), migraine, AEDs, sleep disorders, toxic-metabolic disturbances, psychogenic nonepileptic seizures, inc intracranial pressure eg/ tumour, abscess, psychogenic non-epileptic seizures

Describe the clinical presentation of a chronic subdural haemorrhage

1) fluctuating levels of consciousness 2) insidious physical/mental slowing 3) sleepiness + confusion 4) headache 5) personality changes 6) unsteadiness 7) focal deficits + epilepsy develop 8) stupor, coma and coning may follow NB/ interval between trauma and s/s may be weeks NB2/ pathology: layer of fluid & partially clotted blood forms between dura & arachnoid, which both thicken reactively. Pressure atrophy of the brain

Discuss the complications of small bowel obstruction and their recognition

1) fluid & electrolyte shifts - fluid loss from emesis, bowel oedema, and loss of absorptive capacity = dehydration. Emesis = loss of gastric potassium, hydrogen, and chloride ions, significant dehydration stimulates renal proximal tubule reabsorption of bicarbonate and loss of chloride, perpetuating metabolic alkalosis 2) vascular compromise -diminished blood flow leaves cells without enough oxygen, which causes the cells to weaken and die. If damage is severe enough, infection, gangrene and eventually a hole (perforation) in the wall of the intestines can occur. If untreated, intestinal ischemia can be fatal 3) sepsis - ongoing dilation of intestine increases luminal pressures. When luminal pressures > venous pressures, loss of venous drainage causes inc oedema and hyperemia of bowel - may eventually lead to compromised arterial flow to the bowel, causing ischemia, necrosis, and perforation

Provide a differential diagnosis of a semi/unconscious patient

1) focal brain dysfunction a) brain tumour b) vascular events (CVA) c) demyelination d) infection, such as cerebral abcess e) focal head injury 2) diffuse brain dysfunction a) infection, such as meningitis or encephalitis b) epilepsy c) hypoxia and hypercarbia d) drugs, poisoning and overdoses ( including alcohol) e) metabolic/endocrine causes, such as diabetic coma, hepatic or renal failure, hypothyroidism, severe electrolyte disturbances f) hypotension, or hypertensive crisis g) diffuse head injury h) subarachnoid haemorrhage i) hypothermia, hyperthermia j) psychiatric

Discuss follow up, recurrence and metachronous tumours

1) follow-up - a) purpose of follow-up is to monitor for the presence of treatment-related complications, to detect recurrence at the primary site, to detect and remove metachronous colorectal polyps, and to monitor for potentially resectable metastases in suitable patients b) recommend 3- to 6-monthly patient clinic review for the first 2 years and then yearly until 5 years post-treatment c) for stage I cancers, a colonoscopy should be performed 1 year post surgery, repeated in 3 years, and then again at 5 years d) for stage II and III cancers - combination of history and physical exam, carcinoembryonic antigen, CT imaging, and colonoscopy at varied intervals is recommended 2) recurrence - a) outcome of colorectal cancer depends on the stage at diagnosis b) 5-year survival rates for colorectal cancer - I) 95% for stage I disease II) 80% for stage II disease III) 44-83% (depending on nodal involvement) for stage III disease IV) <8% for stage IV disease 3) metachronous tumours - a) multiple primary carcinomas often occur in the rectum and colon b) the time lag between the first and second malignant transformation is variable c) 2+ primary carcinomas can coexist at the time of diagnosis (synchronous), or develop consequently (metachronous), sometimes years after resection of the first primary

Discuss the incidence of incisional hernia according to risk factors of patient comorbidity and previous surgery, and risks of complications

1) following breakdown of muscle after surgery (defect in scar from previous surgery) - if obese repair is not easy, most common in midline laparotomy scars 2) mesh repair has dec recurrence but inc infection over sutures 3) likelihood of development depends on: a) pts pre-operative status (chronic cough, obesity, cachexia, protein or vitamin C deficiency) b) closure during operation (poor technique, weak suture material) c) post-operative factors (cough, distension, infection, haematoma) 4) complications - a) fluid buildup at site of mesh placement, sometimes requiring aspiration (draining off) b) postoperative bleeding, though seldom enough to require repeat surgery c) injury to intra-abdominal organs eg/ intestines, liver etc. d) nerve injury e) fever, usually related to surgical wound infection f) intra-abdominal abscess (complete breakdown of wound = loop of bowel protrudes into abdominal wall) g) bowel obstruction & strangulation NB/ usually a wide neck so strangulation is rare

List the types of amputation of the lower limb and contrast their rehabilitation potential

1) foot, including toes or partial foot - rarely requires the skilled rehabilitation of physical therapy, but occasionally therapy will be involved in the rehabilitation process if balance or gait issues are limiting functional mobility 2) at the ankle (ankle disarticulation) 3) below the knee (transtibial) - knee-joint is spared, and walking with a prosthesis is typically more successful 4) at the knee (knee disarticulation) 5) above the knee (transfemoral) - may lead to significant loss of muscular control in the hamstrings and quadriceps muscles, making walking with a prosthesis difficult 6) at the hip (hip disarticulation) - walking after a hemipelvic amputation is difficult since there is no residual limb on which a prosthesis can be fitted. physical therapist and doctor can work with you to fit you with the most appropriate device to help maximize your ability to walk

Discuss the potential complications associated with wound drains and outline the steps taken to prevent them

1) fragmentation of the drain in the abdomen 2) pain - analgesia 3) infection (overproduction of exudate may signal bacterial contamination or inflammation) - check site regularly, abx if needed 4) loss of function due to obstruction - check drain regularly for blockage 5) perforation of visceral organs 6) too little exudate might indicate a systemic problem like shock or dehydration 7) problems regarding drain withdrawal

Describe the common fluid and electrolyte and acid base abnormalities in patients with the following problems: excessive gastric losses; high volume pancreatic fistula; jejunal fistula; ileal fistula; bile fistula; diarrhoea; closed head injury; hypovolaemic shock due to GI blood loss or major trauma

1) gastric losses - a) gastric fluid typically contains 120-160 mmol/L Cl−, balanced by K+, Na+, and H+ b) Cl- drops through either vomiting or nasogastric, with variable H+ loss as well. Any H+ loss abruptly inc serum [HCO3−] while dec signal to the pancreas to secrete HCO3− into the duodenum - causes metabolic alkalosis, initiated by H+ loss but sustained by disproportionate loss of Cl c) initial loss of Na+ is rapidly followed by a rise in K+ excretion, leading to secondary K+ depletion and hypokalaemia 2) pancreatic or bilious fistula - usually the volume loss is low, so, despite loss of a HCO3−-rich fluid, significant metabolic acidosis does not occur. In the rare setting in which drainage volume >2 L, metabolic acidosis will develop and be maintained by concomitant volume depletion 3) jejunal or ileal fistula - causes a metabolic acidosis by inc strong ion difference, due to the loss of a large amount of strong cations (Na + K) with a relatively small loss of chloride 4) diarrhoea - for acid-base and electrolyte abnormalities to occur, the volume of fluid lost must be sufficiently large to overcome the kidney's ability to adjust excretion to maintain acid-base equilibrium. With larger losses, any form of diarrhoea will lead to a significant fall in extracellular fluid volume, reducing GFR and limiting the ability of the kidney to help correct the abnormalities. This most commonly occurs with secretory diarrheas, and the typical presentation is hypotension, acute renal failure, hyperchloremic metabolic acidosis, and hypokalemia. When diarrhea is severe, lactic acidosis may supervene as a result of tissue hypoperfusion 5) closed head injury - a) potassium derangements after severe traumatic brain injury implies that the normal buffering mechanisms that maintain the ionic homeostasis needed for establishment of the resting membrane potential have been overcome - can be inc or dec b) can also have derangement of Na causing hyponatraemia or hypernatraemia depending on injury 6) hypovolemic shock - hypovolaemia, hyperkalaemia, hypocalcaemia

List diagnostic methods used to investigate pts with suspected gastric neoplasia

1) gastroscopy + multiple ulcer edge biopsies - aim to biopsy all gastric ulcers 2) endoscopic ultrasound can evaluate depth of invasion 3) CT/MRI for staging - staging laproscopically recommended for locally advanced tumours 4) cytology of peritoneal washings can help identify peritoneal metastases

Describe the treatment of pancreatic pseudocyst

1) goal of therapy is avoidance of complications eg/ infection, rupture 2) most pseudocysts resolve without interference and only require supportive care 3) drainage if - complications, symptoms, concern about possible malignancy 4) need MDT - gastroenterologists, surgeons, and invasive radiologists 5) diet and activity - low-fat diet as tolerated, if abdo pain may need parenteral or enteral nutrition through a percutaneously or endoscopically placed jejunal tube 6) outpatient monitoring - if endoscopically placed stents monitor via serial CT scans to observe resolution of the cyst 7) surgical drainage - catheter drainage, endoscopic drainage, transpapillary drainage, transmural drainage

Outline the main clinical patterns of peripheral nerve damage, and describe typical findings on examination

1) gradual onset of numbness, prickling or tingling in your feet or hands, which can spread upward into your legs and arms 2) sharp, jabbing, throbbing, freezing or burning pain 3) extreme sensitivity to touch 4) lack of coordination and falling 5) muscle weakness or paralysis if motor nerves are affected 6) f) if autonomic nerves are affected, signs and symptoms might include: a) heat intolerance and altered sweating b) bowel, bladder or digestive problems c) changes in blood pressure, causing dizziness or lightheadedness 7) peripheral neuropathy can affect one nerve (mononeuropathy), two or more nerves in different areas (multiple mononeuropathy) or many nerves (polyneuropathy). Carpal tunnel syndrome is an example of mononeuropathy. Most people with peripheral neuropathy have polyneuropathy

Describe the clinical presentation of acute pancreatitis

1) gradual or severe sudden epigastric or central abdominal pain (radiates to back, sitting forward may relieve) 2) V+N prominent (may occur after large meal or 6-12 hrs post-alcoholic binge) sometimes: 3) tachycardia 4) fever 5) jaundice 6) shock 7) ileus 8) rigid abdomen +/- local/general tenderness 9) periumbilical bruising (cullen's sign) or flanks (grey turner sign) from blood vessel autodigestion and retroperitoneal haemorrhage

Identify normal granulation tissue in a wound and describe its significance in terms of epithelialisation and defence against contamination

1) granulation tissue is new connective tissue and microscopic blood vessels that form on the surfaces of a wound during the healing process. Granulation tissue typically grows from the base of a wound and is able to fill wounds of almost any size. Examples of granulation tissue seen in pyogenic granulomas and pulp polyps. Its histological appearance is characterized by proliferation of fibroblasts and new thin-walled, delicate capillaries (angiogenesis), infiltrated inflammatory cells in a loose extracellular matrix 2) during the migratory phase of wound healing, granulation tissue is: light red or dark pink as perfused with new capillary loops or "buds"; soft to the touch; moist; bumpy (granular) in appearance, due to punctate hemorrhages; pulseful on palpation; painless when healthy 3) the extracellular matrix of granulation tissue is created and modified by fibroblasts. Initially, it consists of a network of type-III collagen, a weaker form of the structural protein that can be produced rapidly. This is later replaced by the stronger, long-stranded type-I collagen, as evidenced in scar tissue 4) immunity - main immune cells active in the tissue are macrophages and neutrophils. These work to phagocytize old or damaged tissue, and protect the healing tissue from pathogenic infection. This aids the healing process and protects against invading pathogens, as the wound often does not have an effective skin barrier to act as a first line of defense 5) vascularisation - need a network of blood vessels to be established ASAP to provide the growing tissue with nutrients, to take away cellular wastes, and transport new leukocytes to the area. Fibroblasts, the main cells that deposit granulation tissue, depend on oxygen to proliferate and lay down the new extracellular matrix. In vascularisation endothelial cells quickly grow into the tissue from older, intact blood vessels, which branch out in a systematic way, forming anastomoses with other vessels 6) it is essential to protect the granulation tissue to allow the epithelialisation process to proceed in order to close the wound

Describe the morphology and pathological consequences of cirrhosis

1) gross - cirrhotic liver appears nodular, "hub-nailed", on the external surface and nodular on the cut surface 2) liver is usually indurated shrunken and yellowish-tan but it may be enlarged and yellow as in alcoholic fatty cirrhosis, rusty as in hemochromatosis or large and green as in biliary obstruction 3) microscopic - presence of nodules and fibrous septa with effacement of the lobular architecture. The nodules are of two types: Dissection type and Hyperplastic Regenerative type

Describe, identify and manage complications secondary to venepuncture or arterial puncture

1) haematoma - most common complication of venepuncture a) caused by blood leaking into the tissues during or after venepuncture - occurs when the area around the puncture sites begins to swell indicating blood is leaking into the tissues, which will result in a bruise due to partial insertion into the vein b) a common sign is swelling at or near the venepuncture site c) mx - release the tourniquet immediately, withdraw the needle, and apply firm pressure, then recheck to ensure bleeding has stopped d) to avoid haematoma formation from venepuncture, puncture only the uppermost wall of the vein just under the skin, remove the tourniquets before removing the needle, use the major superficial veins, make sure the needle fully penetrates the uppermost wall of the veins because partial puncture may allow blood to leak into the tissue just under the skin and apply pressure to the puncture site 2) infection - a) may be localized giving rise to thrombophlebitis or systemic leading to septicemia. b) causes - inadequate cleansing or poor technique 3) nerve damage - a) haematoma formation can potentially cause nerve damage, rare problem but potentially serious b) mx - immediately discontinue venepuncture, pt may need physical therapy 4) haemoconcentration - a) can result from prolonged tourniquet application, massageing, squeezing or probing a site, long-term intravenous fluid therapy, scleroses or occluded veins, dehydration and certain diseases 5) extravasations - a) occurs when a cannula pulls out of the vein or becomes partly occluded by venous construction causing backflow of the infusate through the puncture site into the surrounding tissues (cannula enters the tissues rather than the vein) b) mx - cannula should be removed immediately 6) syncope & fainting - a) pts may become dizzy and faint at the thought or sight of blood b) caused because of rapid fall in the BP & ANS reaction (psychomatic trigger) usually based on fear 7) petechiae - a) may be due to coagulation problems or abnormalities b) make sure bleeding stops prior to leaving the patient 8) excessive bleeding - a) pt should not be left alone until bleeding has stopped 9) others - oedema, thrombosis, anaemia 10) arterial puncture - artery can be punctured instead of the vein a) mx - needle should be removed immediately and pressure applied over the site 11) pain - needle should be removed immediately if pt complains of excessive or severe pain

Discuss the clinical features, diagnosis and management of inherited bleeding disorder including haemophilia and von Willebrands disease

1) haemophilia - a) s/s - I) risk factors eg/ fmx (from maternal side usually positive), and male sex, hx of recurrent or severe bleeding II) bleeding into muscles & haemarthrosis - pain & swelling, with dec range of motion, erythema, and inc local warmt III) mucocutaneous bleeding - epistaxis, bleeding from gums following minor dental procedures etc IV) others - excessive bruising/haematoma, fatigue, menorrhagia and bleeding following surgery or childbirth, extensive cutaneous purpura (acquired haemophilia) b) ix - I) FBC - to rule out thrombocytopenia as a cause of bleeding II) activated partial thromboplastin time (aPTT) III) plasma factor VIII and IX assay - Factor VIII +/- factor IX assay + mixing study requested to confirm diagnosis if aPTT is prolonged - assays dec or absent if haemophilia IV) prothrombin time (PT) - evaluates Extrinsic & common pathways of coagulation - normal in haemophilia V) plasma von Willebrand factor assay - normal VI) plasma factor V, VII, XI, XII assay - all normal VII) others - closure time/bleeding time and platelet aggregation studies (normal), serum liver aminotransferases (AST, ALT - normal), Plain x-rays of specific bony sites, antenatal factor VIII or IX mutation analysis by amniocentesis or chorionic villus sampling if mothers are carriers, head or neck CT or MRI/abdopelvic US or CT (to evaluate acute bleeding), c) mx - life-threatening bleed or low-titre factor inhibitor - I) factor concentrate: octocog alfa for haemophilia A, coagulation factor IX for haemophilia B II) + supportive care + subspecialty consultations III) +/- antifibrinolytic agent: aminocaproic acid or tranexamic acid IV) 2nd line - bypassing agent: eptacog alfa or factor VIII inhibitor bypassing fraction congenital: non-life-threatening bleed into joint or muscle - I) 1st line - factor concentrate: octocog alfa or efmoroctocog alfa for haemophilia A, coagulation factor IX for haemophilia B II) + analgesics + physiotherapy evaluation: paracetamol, codeine, celecoxib, adjunctive measures include rest, ice, compression, and elevation (RICE), analgesia, and physiotherapy evaluation III) +/- orthopaedic + pain team evaluation congenital: non-life-threatening nasal or oral bleeding - I) 1st line - desmopressin + supportive care: general observation and RBC transfusion if needed II) +/- antifibrinolytic agent: aminocaproic acid or tranexamic acid acquired - I) 1st line - bypassing agent or recombinant porcine factor VIII: eptacog alfa or factor VIII inhibitor bypassing fraction or antihemophilic factor (factor VIII, recombinant), porcine II) + immunosuppression: prednisolone + cyclophosphamide or rituximabONGOING inhibitors to factor VIII or IX - I) 1st line - immune tolerance induction (ITI): octocog alfa or coagulation factor IX II) after successful ITI: + prophylaxis: octocog alfa for haemophilia A, coagulation factor IX or eftrenonacog alfa for haemophilia B III) with recurrent bleeds into single joint: + orthopaedic evaluation for radioactive synovectomy no VIII/IX inhibitors: severe haemophilia - prophylaxis + orthopaedic evaluation for radioactive synovectomy 2) von Willebrand's - a) s/s - I) risk factors eg/ fmx and consanguineous relationships II) bleeding from minor wounds III) postoperative bleeding IV) easy and excessive bruising V) menorrhagia VI) others - GI bleed, epistaxis b) ix - I) prothrombin time (PT) - measures extrinsic pathway, normal II) activated partial thromboplastin time (APTT) - prolonged if factor VIII is decreased sufficiently III) FBC - usually normal IV) von Willebrand factor antigen - diagnostic for VWD if <0.30 international units (IU)/mL V) von Willebrand factor function assay VI) factor VIII activity - may be dec but often within normal range VII) consider - von Willebrand factor multimer analysis, platelet aggregometry, factor VIII - von Willebrand factor binding assay, TFTs, serum protein electrophoresis c) mx - VWD type unknown with active severe haemorrhage - I) 1st line - von Willebrand factor (VWF)-containing concentrate II) +/- platelet transfusion III) 2nd line - cryoprecipitate; +/- platelet transfusion all types VWD with severe bleeding or before high-risk bleeding procedures (including where sustained high levels of VWF required for several days) - I) 1st line - VWF-containing concentrate II) +/- antifibrinolytic therapy: tranexamic acid or aminocaproic acid III) known desmopressin responder: + desmopressin IV) known desmopressin non-responder : + VWF-containing concentrate V) +/- antifibrinolytic therapy: tranexamic acid or aminocaproic acid

Describe the classical symptoms, examination findings and treatment of hypothyroidism

1) hair & eyebrow loss 2) cold pale skin 3) characteristic face (toad-like) 4) lethargy 5) constipation 6) menorrhagia 7) hoarse voice 8) dec mood 9) inc weight 10) bradycardic - reflexes relax slowly, ataxia, dry think hair/skin, ascites, CCF inc: TSH, dec: T4 11) tx - a) healthy & young - levothyroxine (T4) 50-100ug/24hr PO, review every 6 weeks for rest of life b) elderly or ischaemic heart disease - 25ug/24hr - inc dose by 25ug/4 weeks according to TSH c) if diagnosis in question & T4 already given - stop T4, check TSH in 6 weeks NB/ complete suppression of TSH not recommended to avoid risk of AF & osteoporosis, annual thyroid function testing recommended

Describe the symptoms of hyperthyroidism and the typical examination findings

1) hair loss 2) bulging eyes 3) pretibial myxoedema 4) onycholysis 5) diarrhoea 6) weight loss but inc appetite 7) heat intolerance & sweats - warm peripheries 8) palpitations - pulse fast + irregular (tachycardia or AF) 9) irritability 10) palmar erythema 11) may be goitre 12) oligomenorrhoae +/- infertility 13) lid lag + lid retraction, proptosis inc: T4/3, dec: TSH

Describe the use of different investigations in diagnosing venous disease

1) handheld Doppler ultrasound identifies backward reflux of blood 2) colour duplex scans (using B mode and Doppler ultrasounds) produce a colour picture of blood flow - blue forward, red back - and thus the presence of valvular incompetence NB/ Trendelenburg test is no longer a recognised method

Describe the symptoms and physical signs of pancreatic cancer on the basis of location of tumour within gland

1) head of pancreas - painless obstructive jaundice 2) body/tail - epigastric pain that radiates to the back & relieved by sitting forward 3) as with other malignancy, weight loss may be present 4) duodenal obstruction in late disease may lead to obstructive symptoms (vomiting, anorexia, constipation) 5) enlargement of body (25%) + tail (15%) of pancreas may result in interaction with gastric expansion, but more commonly with vague symptoms eg/ back pain, weight loss 7) cancers in body + tail may present with secondary metastases or malignant ascites 8) head of pancreas gives uncinate process which sits posteriorly to superior mesenteric vessels, and as such may constrict them 9) <15% pancreatic tumours are resectable, however in body & tail this drops to 3%

Outline clinical situations where a CT scan of the head is indicated

1) head trauma 2) stroke 3) headaches 4) initial evaluation for space-occupying lesions 5) unexplained change in mental status 6) seizures 7) suspected hydrocephalus 8) suspected intracranial hematoma 9) psychiatric disorders 10) dizziness 11) vascular occlusive disease and aneurysm evaluation

Describe the clinical features of a headache arising from temporal arteritis & other associated s/s

1) headache - usually located over the temporal or occipital areas. It may be accompanied by scalp tenderness 2) PMR s/s - aching and stiffness in the neck, shoulders, hips, and proximal extremities that worsen after a period of inactivity and with movement. Pain and swelling of the distal joints may occur 3) extremity claudication - pain may occur with use of the upper extremities 4) other cranial artery abnormalities - other vessels, such as the occipital, postauricular, or facial arteries, may be enlarged or tender 5) loss of vision - partial or complete loss of vision (painless) in one or both eyes in 20% pts, can progress to total blindness. Other vision symptoms may include amaurosis fugax and diplopia 6) jaw claudication - may have pain with chewing that is unilateral or bilateral 7) superficial temporal artery tenderness, thickening, or nodularity - may have tenderness, thickening, and nodularity of the frontal or parietal branches of the superficial temporal arteries may be present 8) may have absent temporal artery pulse 9) may have abnormal fundoscopy if ischaemia of the optic nerve - pallor and oedema of the optic disc, occasionally, cotton-wool spots and small haemorrhages are evident 10) systemic symptoms - may have low-grade fever, malaise, fatigue, and weight loss

List the common infective causes of acute hepatitis and outline risk factors

1) hepA - food & drink transfer, vaccination recommended if travelling, doesn't progress to chronic, may cause infection in childhood but 80% are asymptomatic a) risk factors - travel/work in regions with high rates of hepA, around many child, gay sex, HIV +ve, clotting-factor disorder eg/ haemophilia, use injected or noninjected illicit drugs, live with a person who has hepA, have oral-anal contact with someone with hepA 2) hepB - spread via blood, vaccination for high risk groups, usually asymptomatic a) risk factors - unprotected sex with multiple partners or with someone infected with HBV, share needles during IV drug use, gay sex, live with someone who has chronic HBV infection, infant born to infected mother, have a job that exposes you to human blood, travel to regions with high infection rates of HBV eg/ Africa, Asia, Eastern Europe, haemophiliacs, haemodialysis 3) hepC - most common type in UK, spread via blood, usually asymptomatic a) risk factors - health care worker exposed to infected blood, IV or inhaled illicit drugs, HIV +ve, had a piercing or tattoo in unclean environment using unsterile equipment, blood transfusion or organ transplant before 1992, clotting factor concentrates before 1987, hemodialysis treatments for a long period of time, born to a woman with hepC, being in prison, born between 1945-1965, male, older, alcoholics, HBV 4) hepD - affects people already infected with hepB (needs hepB virus to survive in body), spread via blood or sex a) risk factors -IV drug use, infected mother to neonate, having hepB virus, gay men, receiving many blood transfusions 5) hepE - food & drink transfer, most common cause of acute hepatitis in UK a) poor sanitation, ingestion of undercooked meat or meat products from infected animals, transfusion of infected blood products, vertical transmission from pregnant woman to fetus NB/ bacterial seeding from ascending infection, systemic septicaemia & portal pyaemia may lead to infectious hepatitis through abscess development. Protozoa & helminths (esp cyst causing entamoeba histolytica, schistosoma, hydatid) may be causative

Describe the causes of acute and of chronic hepatitis, including infection

1) hepatitis A virus - most commonly transmitted by consuming food or water contaminated by faeces from a person infected with hepatitis A 2) hepatitis B - transmitted through contact with infectious body fluids eg/ blood, vaginal secretions, or semen, containing the hepatitis B virus (HBV). Injection drug use, having sex with an infected partner, or sharing razors with an infected person increase your risk of getting hepatitis B can be acute or chronic 3) hepatitis C virus - transmitted through direct contact with infected body fluids, typically IVDU and sexual contact can be acute or chronic 4) hepatitis D virus - contracted through direct contact with infected blood. Hepatitis D is a rare form of hepatitis that only occurs in conjunction with hepatitis B infection. The hepatitis D virus can't multiply without the presence of hepatitis B 5) hepatitis E virus - waterborne disease mainly found in areas with poor sanitation and typically results from ingesting faecal matter that contaminates the water supply 6) alcohol and other toxins - excessive alcohol can cause liver damage and inflammation, sometimes called alcoholic hepatitis. The alcohol directly injures the cells of your liver. Over time, it can cause permanent damage and lead to liver failure and cirrhosis, a thickening and scarring of the liver. Other toxic causes of hepatitis - overuse or overdose of meds & poisons can be acute or chronic 7) autoimmune system response - immune system mistakes the liver as a harmful object and begins to attack it. It causes ongoing inflammation ranging from mild to severe, often hindering liver function

Describe the features of drug induced liver injury including paracetamol overdose

1) hepatitis eg/ isoniazid, methyldopa, NSAIDs 2) necrosis - often caused by more severe hepatitis eg/ paracetamol 3) cholestasis (decreased secretion and/or flow of bile) eg/ steroids 4) steatosis (accumulation of fat in the liver) eg/ valproate, MTX 5) cirrhosis (advanced scarring of liver) - chronic hepatitis, cholestasis, fatty liver 6) mixed disease eg/ hepatitis + necrosis of liver cells, hepatitis + steatosis, cholestasis + hepatitis 7) fulminant hepatitis with severe, life threatening liver failure 8) blood clots in liver veins 9) tumours eg/ OCP, steroids 10) inc blood levels of liver enzymes (ALT, AST, AP) - many drugs cause mild elevations in blood levels of liver enzymes without symptoms or signs of hepatitis eg/ statins, some antibiotics, some antidepressants, and some medications used for treating diabetes, tacrine, aspirin, and quinidine 11) paracetamol = predictable, dose dependent liver damage 12) idiosyncratic drug reactions are unpredictable, may occur at any time and may persist on drug withdrawal. 10-15% fulminant liver failure eg/ valproate, NSAIDs, amiodarone, diclofenac, methyldopa, isoniazid, minocycline, halothane and methotrexate 13) some drugs cause cholestasis: chlorpromazine, oestrogens (+ steroids), coamoxiclav/flucloxacillin, chlorpropamide

Outline the features and causes of inherited red cell membrane defects and of red cell enzymopathies

1) hereditary spherocytosis - a) cause - inherited disorder resulting in defects in the skeletal proteins of the red-cell membrane which normally maintain the red-cell shape and deformability b) s/s - I) risk factors eg/ fmx of anaemia, jaundice, splenectomy, known HS, Northern European ancestry II) pallor III) jaundice IV) splenomegaly V) fatigue 2) hereditary elliptocytosis - a) cause - abnormalities of proteins involved in the red cell membrane horizontal skeletal network including the spectrin dimer-dimer interaction or the spectrin-actin-protein b) s/s - often asymptomatic or: I) mild haemolytic anaemia II) jaundice, splenomegaly and gallstones 3) hereditary stomatocytoses - c) cause - genetic disease characterised by red cell dehydration, due to the loss of the cation content, in particular K+ and cell water d) s/s - I) well-compensated mild to moderate anaemia II) can have splenomegaly and cholelithiasis 4) glucose-6-phosphate dehydrogenase deficiency - a) cause - genetic, on the long arm of the X chromosome so seen in males b) s/s - I) risk factors eg/ male, fmx of G6PD deficiency; African, Mediterranean, or Asian origin; recent exposure to drugs or broad beans; and recent infection II) jaundice, pallor, dark urine, nausea 5) pyruvate kinase deficiency - a) cause -mutations in the PKLR gene b) s/s - I) haemolytic anaemia II) icterus, fatigue, lethargy, recurrent gallstones, jaundice, and pallor

Describe the findings on physical examination of pts with ventral hernias including mass and tenderness in those with incarcerated hernias, and estimation of size of defect in pts with reducible hernias

1) hernia that appears in the abdomen at the site of a previous surgery 2) soft bulge in abdomen that becomes larger with coughing, crying or straining, and becomes smaller with relaxation. Bulging area can be tender 3) mass and tenderness in those with incarcerated hernias. If the intestines become blocked then vomiting, abdominal pain, and abdominal distension occur 4) estimation of size of defect in pts with reducible hernias - size of defect may be estimated by reducing the hernia then feeling for the borders NB/ elongated gap between rectus muscles in elderly, wasted pts (divarication of recti)

Describe the diagnosis of a breast lump and the concept of triple assessment, including mammography, US and cytology/biopsy (core and open)

1) history & exam 2) cytology - looking at cells taken from biopsy 3) mammography - x-ray of breasts 4) USS - sound waves to asses if lump is solid or fluid filled 5) biopsy (core & open) - core = removing a small amount of suspicious tissue from the breast with a larger "core" (meaning "hollow") needle, open = minor operation to remove the whole breast lump NB/ pathway: history, exam, mammography/ultrasound, biopsy (trucut/open), cytology (FNA)

Discuss the possible physical signs and investigation of a patient with chronic renal failure

1) history - establish possible cause a) previous UTIs, LRTIs b) PMH of inc BP, DM, IHD c) check drug history & family history e) current state - uremic symptoms (anorexia, vomiting, restless legs, fatigue, weakness, pruritis, bone pain) f) check for oliguria, dyspnoea, ankle swelling 2) exam - pallor, uraemic tinge to skin (yellow), purpura, excoriations, inc BP, cardiomegaly, signs of fluid overload & possible cause eg/ ballotable polycystic kidneys 3) tests: a) blood - Hb (normocytic anaemia, normochromic), ESR, U&E, glucose (DM), dec Ca2+, inc phosphate + AP, inc PTH if CKD stage 3+ b) urine - dipstick, MC&S, albumin:creatine ratio or protein:creatine ratio c) imaging - USS to check size, anatomy + corticomedullary differentiation (kidneys normally small but can become enlarged in infiltrate disorders (amyloid, myeloma), CT + MRI d) histology - consider renal biopsy if rapidly progressing, unclear cause + normal sized kidneys

Name the location of the causative lesion in; Homonymous hemianopia; Homonymous quadrantanopia; Bitemporal hemianopia; Monocular visual field defect

1) homonymous hemianopia - affects half visual field contralateral to lesion in each eye. Lesions lie beyond chiasm in tracts or occipital cortex eg/ stroke, abscess, tumour 2) homonymous quadrantanopia - optic radiations: opposite side, top or bottom quadrant depending on if top (Meyer's loop) or bottom radiation is cut 3) bitemporal hemianopia - optic chiasm compression eg/ pituitary adenoma, craniopharyngioma, internal carotid artery aneurysm 4) monocular visual field defect - optic nerve defect before chiasm NB/ lesion on visual cortex = macular sparing - loss of hemifield on opposite side, apart from vision in fovea, or bilateral blind spots

Discuss the diagnosis of the likely cause of peripheral neuropathy using relevant aspects of history, examination, electrophysiological and laboratory investigation

1) hx - identify factors mentioned previously eg/ diabetes, alcoholism, vit deficiencies, autoimmune conditions, exposure to toxins, meds, infections, trauma, pmx a) predominantly sensory neuropathies eg/ diabetes, thiamine deficiency, malignancy, leprosy, hereditary sensory neuropathy, amyloid, uraemia, sarcoid b) predominantly motor neuropathies eg/ GBS, CIDP, porphyria, diphtheria, botulism, lead, CMT 2) ix - a) bloods in all if cause unclear - sugar, LFTs, vitamin B12, paraprotein screen, TFTs, FBC, ESR, vasculitic screen b) CSF analysis - useful to distinguish immune mediated neuropathies such as CIDP or chronic immune mediated axonal neuropathies where the CSF protein is frequently notably raised c) genetic testing d) nerve biopsy 3) focal neuropathy - nerve conduction study shows no subclinical lesions = diabetes 4) multifocal neuropathy - NCS shows if: a) axonal - vasculitis? Needs biopsy b) demyelinating eg/ CIDP, hereditary neuropathy 5) generalized neuropathy - a) acute - NCS to find if: I) axonal eg/ GBS, toxins, porphyria II) demyelinating eg/ GBS b) chronic - NCS to find if: I) axonal eg/ CMT, metabolic II) demyelinating - conduction block - yes = CIDP, no = CMT

Describe the assessment of a patient with renal failure including fluid balance

1) hx esp drugs 2) exam a) limited CV - check BP, JVP, check for pulmonary oedema and pericardial rub, sacral oedema and leg oedema b) palpate kidneys - should be non-palpable c) check arms for fistulas - are they pulsating? can you hear blood flow? recent or old? d) palpate bladder e) auscultate for renal bruits - epigastric and femoral 3) fluid status - see OSCE document 4) bloods including: U&E, eGFR, creatinine, Hb (anaemia) 5) MSU dipstick +/- culture

Outline the nature of the allergic reaction underlying extrinsic allergic alveolitis and how this is used to establish the diagnosis

1) hypersensitivity pneumonitis (HP) aka extrinsic allergic alveolitis, is the result of non-IgE mediated immunological inflammation 2) caused by repeated inhalation of non-human protein, which can be of natural plant or animal origin or can be the result of a chemical conjugated to a human airway protein, such as albumin 3) inflammation of HP manifests itself in the alveoli and distal bronchioles 4) clinical syndromes - acute, sub-acute, and chronic HP - present differently & depend on exposure length 5) immunopathogenic mechanisms not fully understood. The potentially important role of cellular adaptive immunity as the cause of HP is supported by the fact that individuals with hypogammaglobulinaemia can develop HP 6) cellular infiltrate consists primarily of lymphocytes, plasma cells, and neutrophils, also have non-caseating granulomas and activated foamy macrophages 7) in acute HP, the fever, tachypnoea, dyspnoea, pulmonary infiltrates, restrictive PFTs, and reduced diffusing lung capacity of carbon monoxide (DLCO) are all due to the lymphocytic alveolitis. 8) in the sub-acute phase, the inflammation is not as intense, and there is often a fair amount of fibrosis resulting in insidious development, over many weeks, of malaise, dyspnoea, cough, a mixed PFT picture, and reduced DLCO 9) in chronic HP, there is very little inflammation; the fibrosis that characterises this form of HP results in dyspnoea, weight loss, malaise, the mixed PFT pattern, and decreased DLCO

Outline the pathophysiology underlying the clinical features of cirrhosis including hypoproteinaemia, abnormal clotting, secondary hyperaldosteronism and portal hypertension

1) hypoproteinaemia - synthetic function of liver compromised so dec albumin produced 2) abnormal clotting - disturbed balance between pro and anti-coagulants = inc bleeding + thrombotic risk. Liver is responsible for production of vitK dependent clotting factors (also factor 5) 3) secondary hyperaldosteronism - cirrhosis causes alterations in oncotic pressures due to the loss of proteins which in turn leads to decreased cardiac output which leads to the activation of RAAS, causing retention of water and Na - this leads to secondary hyperaldosteronism. Also, liver can't metabolise the aldosterone in the circulation 4) portal hypertension scar tissue blocks the flow of blood through the liver

Outline other causes of hypothalamic-pituitary disturbance

1) hypothalamus - a) birth defects of the brain or hypothalamus (eg/ holoprosencephaly, septo-optic dysplasia) b) genetic disorders (eg/ prader-willi syndrome, growth hormone deficiency) c) eating disorders (eg/ anorexia, bulimia), or malnutrition d) tumors (eg/ craniopharyngiomas, germinomas, meningiomas, gliomas, ependymomas, and gliomas of the optic nerve), or cranial radiation or syrgery e) head trauma (eg/ boxing and varied injuries, birth trauma) f) bacterial, viral, or fungal infections g) autoimmune disorders (eg/ sarcoidosis) h) excess iron 2) pituitary - a) tumours b) certain medications eg/ benzos, medication for Parkinson's c) bleeding inside or close to the pituitary d) head trauma, infection, sarcoidosis, autoimmune, radiation, surgical removal of pituitary, or SE of pituitary surgery

Outline the principles on management of patients with umbilical hernia, including distinguishing those needing operative repair from those who do not

1) if <1 cm diameter, nearly always closes without tx by 5 yo, unless the child has a large proboscoid hernia with thin, hyperpigmented skin or is undergoing an operation for other reasons or if the hernia causes familial or social problems 2) if >1.5 cm or child aged >4 yo, usually requires repair 3) hernia is repaired surgically with preservation of umbilicus, after removing causative factors such as ascites 4) incarceration of umbilical hernias is rare in kids

Describe the appropriate care and management of a surgical drain

1) if active, the drain can be attached to a suction source (and set at a prescribed pressure) 2) ensure the drain is secured (dislodgement likely when transferring patients after anaesthesia) - inc risk of infection and irritation to the surrounding skin 3) accurately measure and record drainage output. 4) monitor changes in character or volume of fluid. Identify any complications resulting in leaking fluid (eg/ bile or pancreatic secretions) or blood 5) use measurements of fluid loss to assist intravenous replacement of fluids. 6) prevent clots - "strip" or "milk" 2-3x per day to keep the tubing clear 7) need to be cleaned aseptically after drainage 8) may need to empty the drain 2-3x per day (or more), depending on the amount of output 9) if instructed by your doctor, you may shower with the drains in place. Attach them to an old belt or cloth strap to limit the amount of pulling on the skin and to avoid the risk of pulling the drain out accidentally

Describe the causes, symptoms and signs of acute and chronic ureteric obstruction and discuss their management

1) if have s/s they depend on where the obstruction occurs, whether it's partial or complete, how quickly it develops, and whether it affects one or both kidneys a) loin or suprapubic pain (possible radiation to testes) b) palpable kidney (hydronephrosis) or bladder c) changes in the amount of urine produced/difficulty urinating d) haematuria e) repeated UTIs f) hypertension g) quick obstruction that distends kidney = renal colic - pain may extend into a testis or vaginal area. May have N+V h) slowly progressive obstruction can = hydronephrosis. No symptoms, or attacks of dull, aching discomfort in the flank on the affected side 2) causes - a) congenital duplication of the ureter. Causes 2 ureters to form on the same kidney. Second ureter can be normal or only partially developed. If either ureter doesn't function properly, urine can back up into the kidney and cause damage b) abnormality where ureter connects to bladder or kidney, which blocks urine flow. Abnormal connection at ureteropelvic junction may cause the kidney to swell & eventually stop working - congenital or develops with age, injury or scarring. Abnormal connection at ureterovesical junction may cause urine to back up into kidneys c) ureterocele - if ureter is too narrow urine doesn't flow normally & a tiny bulge in the ureter (ureterocele) may develop (usually close) to the bladder. Can block urine flow and cause urine to back up into kidney, possibly leading to kidney damage d) retroperitoneal fibrosis - rare, occurs when fibrous tissue grows in the area behind the abdomen. Fibers encircle & block ureters, causing urine to back up into kidneys e) ureteral stones f) severe constipation - happens primarily in children but also occurs in adults g) cancerous and noncancerous tumours h) internal tissue growth, such as endometriosis in females i) long-term swelling of ureter wall due to eg/ TB or schistosomiasis NB/ intraluminal: renal calculi, tumour, casts, intramural: stricture, extramural: tumour, retroperitoneal fibrosis, diverticulitis, fibroids, BPH, AAA, pregnancy 3) mx - a) may need abx to clear associated infections b) drainage procedures - may need immediate procedure to remove urine from your body and temporarily relieve the problems caused by a blockage. May perform: ureteral stent insertion, percutaneous nephrostomy, insert a catheter c) surgery - endoscopic surgery, open surgery, laparoscopic surgery

Describe the investigation and early clinical management of a patient presenting with acute hypercalcaemia

1) if the person has severe hypercalcaemia (adjusted serum calcium concentration >3.40 mmol/L) or severe symptoms of hypercalcaemia, admit immediately to hospital for IV hydration and bisphosphonates along with treatment of the underlying disease 2) if asymptomatic mild or moderate hypercalcaemia (adjusted serum calcium concentration <3.40 mmol/L): a) repeat serum calcium concentration after 1 week to exclude rapidly increasing hypercalcaemia, suggestive of cancer which may require hospital admission b) if cancer is suspected, refer urgently (2ww) to the appropriate specialist c) if primary hyperparathyroidism is suspected on the basis of serum parathyroid hormone (PTH) results, refer to endocrinology d) if familial hypocalciuric hypercalcaemia is suspected, refer to endocrinology e) stop or reduce any drugs that may contribute to hypercalcaemia eg/ lithium (psych input), vitamin D f) if sarcoidosis or TB is suspected, refer to respiratory g) if kidney disease refer to renal

Describe the ways in which the immune system may be manipulated therapeutically

1) immunosuppression - for autoimmune diseases, organ transplantation a) immunosupressive drugs - anti-inflammatory (NSAIDs, corticosteroids), cytotoxic (azathioprine, cyclophosphamide), noncytotoxic (cyclosporin A, tacrolimus, rapamycin) b) monoclonal antibodies c) antitumor therapy d) gene manipulation (CRISPR-Cas9, siRNA) 2) immunostimulation - a) Levamisole, antihelmintic, leukocyte activator (colon cancer), Thalidomide - inc cytokine production, activates NK cells (multiple myeloma), BCG, live microbes (bladder cancer), interferons, antiviral effects (hepatitis, melanoma, Kaposi sarcoma), IL-2 (kidney carcinoma, melanoma) 3) immunisation - a) vaccination, immunoglobulins

Describe the local symptoms that result from a large pituitary adenoma and outline the clinical consequences of pituitary adenoma producing prolactin, growth hormone (acromegaly) or ACTH (Cushing's disease)

1) impingement/pressure of structures causes: a) headache (bony structures + meninges) b) visual field defects - bilateral temporal hemianopia (optic chiasm - visual pathways) c) palsy of CN 3, 4, 6 (cavernous sinus) d) diabetes insipidus e) disturbance of hypothalamic centres - sleep, appetite, thirst, precocious puberty f) hydrocephalus (ventricles) g) CSF rhinorrhoae (sphenoid sinus) 2) consequences - a) prolactinomas - galactorrhoea, amenorrhoea, hypogonadism, erectile dysfxn b) somatotroph adenomas - acromegaly/gigantism c) corticotroph adenomas - Cushing's disease d) tumours secreting LH, FSH and TSH are rare e) pituitary adenoma may result in hyposecretion of some hormones (hypopituitarism) - short stature, pale skin and lethargy. Many pituitary tumours capable of producing more than one hormone (eg/ GH + Prolactin) f) pituitary apoplexy - rapid pituitary enlargement from a bleed in a tumour: I) CV collapse - acute hypopituitarism II) death III) suspect if acute onset of headache, meningism, dec GCS, opthalmoplegia/visual field defect, especially if known tumour

Outline the rehabilitation of a patient with a below- or above-knee amputation

1) in the hospital, the staff changes the dressings on the wound or teaches the patient to change them - monitor wound healing and any conditions that might interfere with healing, such as diabetes or hardening of the arteries 2) prescribed pain medication and abx 3) can prescribe meds & counselling as needed for other s/s eg/ phantom pain 4) physical therapy, beginning with gentle, stretching exercises, often begins soon after surgery. practice with the artificial limb may begin as soon as 10 to 14 days after surgery 5) wound should fully heal in about 4-8, but physical and emotional adjustment to losing a limb can be a long process 6) long-term recovery and rehabilitation will include: a) exercises to improve muscle strength and control b) activities to help restore the ability to carry out daily activities and promote independence c) use of artificial limbs and assistive devices d) emotional support, including counseling, to help with grief over the loss of the limb and adjustment to the new body image

Describe the management of the splenectomised patient

1) inc risk of septicaemia from invasive organisms eg/ Streptococcus pneumoniae, Hib and meningococcus. Also at risk from malaria, babesiosis (tick bites), Ecoli and Capnocytophaga canimorsus (dog bites) 2) to combat this need vaccinations: ideally 4-6 weeks before elective splenectomy or initiation of chemo and/or radiotherapy. Where this is not possible they can be given up to two weeks before or at least two weeks post-splenectomy - Hib + MenC, PPV (boosted 5yrs from last dose) +/- MenACWY 3) also annual infenzua vaccine 4) need prophylactic abx - oral Phenoxymethylpenicillin (Penicillin 'V') or Clarithromycin (penicillin allergic), if not possible need IV benzylpenicillin or clarithromycin 5) animal bite or scratch that breaks the skin - Co-amoxiclav or Azithromycin 6) encourage patients to wear medic-alert bracelet 7) advised to present themselves immediately to their GP or tA&E if they have rigor or an acute febrile illness - if admitted to hospital presenting with a non-specific infective illness it is essential to treat immediately with an appropriate abx eg/ Cefotaxime 2g IV six hourly

Define incarcerated, obstructed, strangulated femoral hernia and Richter's hernia

1) incarcerated - a hernia so occluded that it cannot be returned by manipulation; it may or may not become strangulated 2) obstructed - lumen of herniated part of intestine is obstructed 3) strangulated - blood supply of hernia contents is cut off, leading to ischemia, lumen of intestine may be patent or not 4) femoral - protrusion of loop of intestine into femoral canal, a tubular passageway that carries nerves and blood vessels to the thigh; occurs more often in women 5) Richter's (partial enterocele) - protrusion and/or strangulation of only part of the circumference of the intestine's antimesenteric border through a rigid small defect of the abdominal wall, discrete from other types of abdominal hernias as only one intestinal wall protrudes through the defect, so the lumen of the intestine is incompletely contained in the defect, while the rest remains in the peritoneal cavity

Outline the principles of management of patients with hernia (inguinal)

1) incarcerated or strangulated hernia - a) 1st line - urgent open surgical treatment: I) incarcerated hernia may be reduced with the patient sedated, but care should be taken to avoid pushing a non-viable loop of the bowel from the hernia sac back into the peritoneal cavity (hernia en masse) II) strangulated hernia is a surgical emergency as bowel may necrose if underlying obstruction is not relieved promptly III) + fluid resuscitation, nasogastric intubation, and urethral catheterisation IV) viable bowel found during surgery can be reduced and repaired with a mesh. If non-viable or contaminated (gangrenous) bowel is found during surgery, laparotomy and bowel resection required, along with primary-tissue (non-mesh) repair of the hernia V) mesh repair avoided if the bowel is contaminated due to the risk of mesh infection 2) small, asymptomatic hernia - a) 1st line - watchful waiting: pts warned about s/s of incarceration or obstruction (e.g., abdominal pain, N&V, constipation) and advised to seek medical attention immediately if they occur 3) large or symptomatic uncomplicated hernia - a) surgical candidate: unilateral primary hernia I) 1st line - open-mesh repair: low-risk, done under spinal, general, or local anaesthesia with mild sedation II) + prophylactic abx: beta‐lactam/beta‐lactamase inhibitors and first‐generation cephalosporins b) surgical candidate: recurrence (after open-mesh repair) or bilateral primary hernia I) 1st line - laparoscopic mesh repair or open preperitoneal mesh repair: use of abx prophylaxis for laparoscopic repair remains uncertain c) non-surgical candidate, limited life expectancy, or refuses repair I) 1st line - non-surgical treatment (e.g., truss) or observation: a truss (or a wearable device that compresses the tissues over the inguinal canal) may be used for patients in whom surgical intervention represents a very significant risk, whose life expectancy is limited, or who refuse repair - applied after the hernia is reduced and when symptoms alleviated (keeps hernia reduced & relieves pain and discomfort)

List the indications for insertion and removal of a nasogastric tube and describe alternative methods of gastric intubation

1) indications - a) feeding purposes - NICE: NG tubes only used in people who are malnourished or at risk of malnutrition and have inadequate or unsafe oral intake - inaccessible GI tract eg/ neurological conditions causing dysphagia/unsafe swallow (stroke), lowered consciousness level eg/ coma, PVS following upper GI surgery where a high anastomosis must be protected in the initial post-operative period b) medication delivery c) removal of gastric contents eg/ initial & continued gastric decompression in endotracheal intubated patients, symptom relief & bowel rest in bowel obstruction ("drip and suck" conservative management - aspiration of stomach contents in conjunction with IV fluid administration), aspirating ingested toxic material d) diagnostic uses - assessment of presence or volume of upper GI bleeding, administration of radiographic contrast 2) alternate methods of gastric intubation - nasoduodenal or nasojejunal if delivery into stomach is contraindicated. These are more rare NB/ if pt's nutritional needs aren't being met or GI is not functioning normally, Total Parenteral Nutrition (TPN) considered. NG tubes are only a short term measure NB2/ PEG tube (percutaneous enterogastrostomy) if need feeding for 30+ days

List the indications and contraindications for the passage of a urethral catheter

1) indications - a) acute urinary retention eg/ meds (anesthesia, opioids, paralytics), nerve injury b) acute bladder outlet obstruction eg/ severe prostate enlargement, blood clots, or urethral compression c) accurate measurements of urinary output if critically ill d) to assist in healing of open sacral or perineal wounds in incontinent patients e) to improve comfort for end of life, if needed f) pt requires strict prolonged immobilization (eg/ potentially unstable thoracic or lumbar spine, multiple traumatic injuries such as pelvic fracture) g) selected peri-operative needs: urologic surgery or surgery on adjacent structures of genitourinary tract, anticipated long surgery (catheters should be removed in PACU), large volume infusions or diuretics anticipated during surgery, need for intraoperative monitoring of urinary output 2) contraindications - a) in presence of acute prostatitis, unless there is complete urinary retention b) inability to pass a urethral catheter due to an obstruction c) trauma to the pelvis or urinary tract d) pt's inability to tolerate a urethral catheter e) following pelvic or urinary tract surgery f) to minimise the risk of urethral trauma g) need for long term catheterisation

List the indications and contraindications for peritoneal lavage; describe the characteristics of a positive or negative lavage in a patient who has sustained trauma

1) indications - a) can be used to evaluate both blunt and penetrating abdominal trauma in patients who are hemodynamically unstable or who require urgent surgical intervention for associated extra-abdominal injuries as can rapidly confirm or exclude the presence of intraperitoneal haemorrhage b) additionally, DPL can be used in nonemergency circumstances as a means of detecting solid-organ injury or HVI requiring laparotomy 2) a negative result on peritoneal aspiration allows the clinician to proceed to alternative management steps and allows the patient to forgo unnecessary laparotomy. 3) contraindication - an obvious need for laparotomy is the only absolute contraindication a) lack of training or familiarity with performing DPL, prior abdominal surgery, abdominal-wall infections, coagulopathy, morbid obesity, and second- or third-trimester pregnancy are all relative contraindications

Outline the indications for and discuss the contraindications of liver biopsy in hepatitis. Be aware of other investigations including fibroscan, ultrasound and blood tests

1) indications - a) unexplained liver test abnormalities b) alcoholic liver disease or nonalcoholic steatosis c) chronic hepatitis (viral or autoimmune) d) suspected rejection or another complication after liver transplantation 2) CI - a) absolute CI - I) pt unable to remain still and to maintain brief expiration for the procedure II) suspected vascular lesion (eg, hemangioma) III) bleeding tendency (eg, INR > 1.2 despite receiving vitamin K, bleeding time > 10 min) IV) severe thrombocytopenia (< 50,000/mL) b) relative CI - profound anaemia, peritonitis, marked ascites, high-grade biliary obstruction, subphrenic or right pleural infection or effusion 3) fibroscan - type of US to measure the degree of inflammation in liver 4) US - used for screening for hepatocellular carcinoma & AFP 5) blood tests - LFTs (elevated aminotransferases (ALT/AST), alk phos, and bilirubin; low albumin), FBC (microcytic anaemia and/or thrombocytopenia), U&E (hyponatraemia, high urea), coagulation profile (PT/INR - normal or elevated)

Outline the indications for drain placement, advancement and removal

1) indications - used in a wide variety of different types of surgery. Generally speaking, the intention is to decompress or drain either fluid or air from the area of surgery eg/ prevent accumulation of fluid (blood, pus and infected fluids), prevent accumulation of air (dead space), to characterise fluid (eg early identification of anastomotic leakage) a) examples - breast surgery (to prevent collection of blood and lymph), T&O (associated with greater blood loss), chest drain, chest surgery, infected cysts (to drain pus), pancreatic surgery (to drain secretions), biliary surgery, thyroid surgery (concern over haematoma and haemorrhage around the airway), neurosurgery (if risk of raised intracranial pressure), urinary catheters, nasogastric tubes 2) removal - drains removed once drainage has stopped or < 25 ml/day. Drains can be 'shortened' by withdrawing them gradually (by 2 cm per day) and so, in theory, allowing the site to heal gradually. Usually drains that protect postoperative sites from leakage form a tract and are kept in place longer (usually for about a week)

Discuss the importance of monitored oxygen therapy in treatment and the indications for assisted ventilation, outlining how this is undertaken

1) indications for assisted ventilation - a) visual observations of skin colour showing central cyanosis b) RR >30 per minute c) low sats d) signs of respiratory distress e) bradypnea or apnea with respiratory arrest f) acute lung injury and the acute respiratory distress syndrome g) vital capacity less than 15 mL/kg h) minute ventilation greater than 10 L/min i) arterial partial pressure of oxygen (PaO2) with a supplemental fraction of inspired oxygen (FIO2) of less than 55 mm Hg j) alveolar-arterial gradient of oxygen tension (A-a DO2) with 100% oxygenation of greater than 450 mm Hg k) respiratory muscle fatigue l) obtundation or coma m) acute partial pressure of carbon dioxide (PaCO2) > 50 mm Hg with an arterial pH less than 7.25 n) neuromuscular disease 2) monitoring - a) 4hrly if on continuous oxygen, 8 hrly if on intermittent oxygen b) critically ill patients should have their oxygen saturations monitored continuously and recorded every few minutes whereas patients with mild breathlessness due to a stable condition will need less frequent monitoring c) O2 sats <90% in patients not at risk of hypercapnoeic respiratory failure, with or without oxygen, noisy or laboured breathing or respiratory rate of less than 8 or more than 25 should be reported immediately to the medical staff where available or transferred to acute care 3) normally, the initial mode of ventilation should be the assist-control mode, in which a tidal volume and rate are preset and guaranteed. The patient can affect the frequency and timing of the breaths. If the patient makes an inspiratory effort, the ventilator senses a decrease in the circuit pressure and delivers the preset tidal volume. In this way, the patient can dictate a comfortable respiratory pattern and may trigger additional machine-assisted breaths above the set rate. If the patient does not initiate inspiration, the ventilator automatically delivers the preset rate and tidal volume, ensuring minimum minute ventilation. In the assist-control mode, the work of breathing is reduced to the amount of inspiration needed to trigger the inspiratory cycle of the machine. This trigger is adjusted by setting the sensitivity of the machine to the degree of pressure decrease desired in the circuit

Define indirect and direct inguinal hernia. List the factors that predispose to the development of inguinal hernia

1) indirect - pass through deep + superficial inguinal rings. Superficial = 1cm above & superolateral to pubic tubercle, deep = couple of cm above midpoint of inguinal ligament 2) direct - push their way directly through posterior wall of inguinal canal (transversalis fascia), into a defect in the abdominal wall (Hesselback's triangle; medial to inferior epigastric vessels & lateral to rectus abdominus) - passes through superficial ring. Breach is commonly medial to inferior epigastric vessels 3) predisposing factors - a) males (8:1) b) chronic cough c) constipation d) urinary obstruction e) heavy lifting f) ascites g) past abdominal surgery (eg/ damage to iliohypogastric nerve during appendicetomy) or weakness of transversalis fascia (previous hernia, age) NB/ anything that inc intra-abdominal pressure eg/ squats, pregnancy, obesity

Describe the symptoms and signs of urinary tract infection including the factors that may predispose to urinary tract infection

1) infants - a) unexplained fever may be only symptom b) crying or another indication that urination is painful c) odd-smelling or cloudy or bloody urine d) unexplained, persistent irritability e) vomiting f) refusal to eat g) diarrhea 2) adults - a) loin pain/tenderness b) burning feeling on urination c) frequent or intense urge to urinate d) cloudy, dark, bloody, or strange-smelling urine e) feeling tired or shaky f) fever or chills NB/ LRTI = frequency, urge, dysuria, and haematuria. URTI = loin pain, fever, rigors. Older patients may have confusion and/or incontinence 3) predisposing factors - a) female anatomy - women have shorter urethra b) sexual activity - sexually active women have more UTIs, new sexual partner also inc risk c) spermicide birth control eg/ diaphragms + spermicidal agents d) menopause - dec in circulating oestrogen e) urinary tract abnormalities that don't allow urine to leave the body normally or cause urine back up f) blockages in urinary tract - kidney stones or enlarged prostate can trap urine in the bladder g) suppressed immune system eg/ diabetes h) pregnancy i) catheter use j) recent urinary procedure - urinary surgery or exam of urinary tract that involves medical instruments

Outline common predisposing factors of TB

1) infection with HIV 2) diabetes mellitus 3) low body weight 4) head or neck cancer, leukemia, or Hodgkin's disease 5) some medical treatments eg/ corticosteroids, certain meds used for autoimmune or vasculitic diseases eg/ rheumatoid arthritis, lupus, which suppress the immune system 6) silicosis, a respiratory condition caused by inhaling silica dust 7) social deprivation, alcoholics, IVDU 8) inc age 9) return visits to countries with high incidence of cases & ethnic minorities eg/ dub-Saharan African, South Asian 10) overcrowding, close contact

Define bacterial endocarditis and who is at risk

1) inflammation of inner tissues of heart (endocardium), usually of the valves, but can be of large intrathoracic vessels, and intracardiac foreign bodies (eg/ pacemaker leads, valves). It is caused by infectious agents, or pathogens - largely bacterial 2) susceptible pts - generally bacteraemia + damaged endocardium required: a) artificial heart valves b) intracardiac devices, such as Implantable cardioverter-defibrillators c) unrepaired cyanotic congenital heart defects d) history of infective endocarditis e) chronic rheumatic heart disease (response to repeated Strep pyogenes) f) age-related degenerative valvular lesions g) haemodialysis h) coexisting conditions, especially if suppress immunity. Diabetes mellitus, alcohol abuse, HIV/AIDS, and IV drug use all fall in this category i) poor dental hygiene j) dental treatment, IV cannulas and cardiac surgery may introduce bacteria

Describe the pathology of simple and complicated coal workers pneumoconiosis

1) inhaled coal dust reaches terminal bronchioles - carbon engulfed by alveolar + interstitial macrophages 2) phagocytosed coal particles travel by macrophages up mucocililary elevator & expelled in mucus or through lymph system 3) when system is overwhelmed, dust-laden macrophages accumulate in alveoli & may trigger an immune response (lungs must be expose for a significant amount of time to dust particles 2-5um diameter) 4) fibroblasts secrete reticulin which entraps macrophages - if macrophages lyse fibroblastic response is augmented & more reticulin laid down 5) macules may continue to enlarge & form nodules that produce progressive massive fibrosis when they coalesce 6) simple produces fine micronodular shadowing on CXR 7) severity of syndrome is graded, into simple (cat 1, 2, 3) and progressive massive fibrosis (PMF) 8) category 1 = small round opacities present but few in numbers (unlikely to be compensated + progress to PMF) 9) cat 2 - opacities numerous but normal lung markings visible (7% progress to PMF) 10) cat 3 - very numerous and lung markings obscured (30% progress to PMF) 11) PMF - pts develop round; black, fibrotic masses in apices of lungs, several cm in diameter. Necrotic central cavities may develop 12) mixed restrictive & obstructive ventilatory defect, with dec gas transfer 13) pts display effort dyspnoea & may produce black sputum. Disease may progress or develop once exposure to coal dust has been withdrawn NB/ patients exposed to coal and asbestos can obtain industrial compensation

List the laboratory tests that would be helpful in the diagnosis of bleeding disorders

1) initial - a) Prothrombin Time (PT - I, II, V, VII, X) and Partial Thromboplastin Time (PTT - factors I (fibrinogen), II (prothrombin), V, VII, VIII, IX, X, XI, XII b) FBC for no of platelets present and to determine if bleeding has led to anaemia (may do blood smear) 2) further tests as needed - a) coagulation factors b) D-dimer c) factor inhibitors d) fibrinogen e) platelet function tests f) Ristocetin cofactor and von Willebrand factor antigen g) thrombin time h) thromboelastography

Describe the morphology and pathological consequences of asthma

1) initial pathology - type 1 hypersensitivity - Th2 cells activate plasma cells to release antibodies alongside eosinophil release of inflammatory mediators 2) antibodies cross-link antigens & activate mast cells to release inflammatory mediators 3) type 4 hypersensitivity may occur simultaneously & Th2 cells may activate SM receptors 4) final common inflammatory result: bronchial SM contraction, bronchial oedema, mucus plugging 5) histological changes reflect combination of allergic reaction & muscular hypertrophy (result of prolonged spasm) a) mucus secreting glands present & viscous secretion may be seen to contain eosinophils (with Charcot-Leyden crystals), Curschmann's spirals (white granules) + desquamated epithelium cells b) submucosa appears oedematous & vessels congested c) SM is hypertrophied & basement membrane is thickened + hyalinated

Describe the clinical features of obstructive jaundice and outline their pathophysiology

1) injury to RBCs can result in spur-cell hemolytic anemia 2) hypercholemia (inc serum bile salt concn) - liver cells retains bile salts, resulting in down-regulation of new bile acid synthesis and dec in total pool size. Bile salts are regurgitated from the hepatocyte = inc concn of bile salts in peripheral circulation 3) pruritus - several links to pathogenesis have been proposed eg/ role of bile acids, endogenous opioid and serotonins, and lysophosphatidic acid 4) hyperlipidemia - bile is the normal excretory pathway for cholesterol, and with dec bile formation, cholesterol is retained 5) xanthomas - deposition of cholesterol into dermis. More characteristic of obstructive cholestasis than of hepatocellular cholestasis 6) failure to thrive - malabsorption, anorexia, poor nutrient use, hormonal disturbances, and secondary tissue injury due to dec delivery of bile salts to the intestine, which results in inefficient digestion and absorption of fats 7) malabsorption of fat-soluble vitamins can result in vitamin deficiency states. Vitamins E, D, K, and A are all malabsorbed in cholestasis, and in that order

CV exam - causes of abnormalities

1) inspection - a) malar flush - may suggest mitral stenosis b) legs - scars from saphenous vein harvest for CAGB, peripheral oedema, missing limbs or toes 2) hands - a) clubbing - endocarditis, cyanotic congenital heart disease b) splinter haemorrhages - bacterial endocarditis c) cyanosis - hypoxia d) temperature - cool peripheries may suggest poor cardiac output/hypovolaemia, sweaty/clammy- can be associated with ACS e) Janeway lesions - bacterial endocarditis f) Osler's nodes - infective endocarditis g) tar staining - smoking = risk factor for CV disease h) xanthomata - hyperlipidaemia i) inc CRT - hypovolaemia 3) pulse - a) irregular pulse b) narrow pulse pressure - aortic stenosis; wide pulse pressure - aortic regurgitation c) slow-rising pulse - aortic stenosis d) collapsing pulse - aortic regurg e) radio-radial delay - coarctation of aorta 4) JVP - a) inc - fluid overload, RV failure, tricuspid regurgitation b) positive hepatojugular reflux - RHS heart failure +/- tricuspid regurgitation 5) face - a) conjunctival pallor - anaemia b) corneal arcus +/- xanthelasma - hypercholesterolaemia 6) mouth - a) central cyanosis b) angular stomatitis - iron deficiency c) high arched palate - Marfan syndrome - ↑ risk of aortic aneurysm/dissection d) dental hygiene - important if considering sources for infective endocarditis 7) chest - a) thoracotomy - minimally invasive valve surgery b) sternotomy - CABG / valve surgery c) clavicular - pacemaker d) L mid-axillary line - subcutaneous implantable cardioverter defibrillator (ICD) e) chest wall deformities - pectus excavatum or carinatum f) visible pulsations - forceful apex beat may be visible - hypertension/ventricular hypertrophy 8) palpation - a) apex beat displaced - cardiomegaly b) heaves - parasternal = RV hypertrophy c) thrill - palpable murmur 9) auscultation - a) carotid bruits - radiation of an aortic stenosis murmur b) aortic regurgitation (sit forward) c) mitral regurg/stenosis - LHS 10) lungs - a) crackles - pulmonary oedema (eg/ secondary to LV failure) 11) sacral/pulmonary oedema - RV failure

Describe the various types of perianal infections

1) inter-sphincteric abscesses are located in the space between the internal and external anal sphincter 2) perianal abscesses occur in the superficial soft tissues overlying the inter-sphincteric space 3) perirectal abscesses are found in the ischio-rectal or post-anal spaces 4) supra-levator abscesses occur above the anorectal ring in the supra-levator space

List the clinical manifestations of chronic peripheral arterial occlusive disease and describe its investigation and management

1) intermittent claudication - pain with physical activity, usually lower leg, which resolves on rest (2-5 mins), pain is reproducible within same muscle groups 2) most common location of arterial lesions = distal superficial femoral artery, which corresponds to claudication in the calf muscle area 3) thigh/buttock muscle claudicationwith atherosclerosis in aortoiliac area. Can also have male impotence = "Leriche syndrome" 4) can occur in both legs but often worse in one 5) more severe disease associated with rest pain, defined as severe unremitting pain in foot which stops pt from sleeping (during sleep, CO dec, and also lying flat stops blood draining to the foot through gravity). Partially relieved by hanging foot out of the bed or standing on a cold floor 6) even more severe disease may cause ulceration or gangrene 7) signs - cold, dry skin, hair loss. Pulses diminished + peripheries discoloured 8) Buerger's test will be +ve & angle of positivity should be established 9) bruits may be heard over major arteries 10) ix - lab studies helpful only for identifying accompanying silent alterations in renal function and elevated lipid profiles. Angiography is the recommended imaging study. Other studies that may be considered: CT angiography (CTA), magnetic resonance angiography (MRA), and duplex ultrasonography a) Standard Angiography - criterion standard arterial imaging study for diagnosis of PAOD, but usually reserved for when an intervention (endovascular procedure or a traditional open surgical procedure) is planned b) Magnetic Resonance Angiography - useful for imaging large and small vessels, MRA can be used to diagnose & help plan type of indicated intervention c) Computed Tomography Angiography - another modality used to image arterial disease, but needs a large amount of contrast media, and an upgraded CT d) Duplex Ultrasonography - evaluates status of a patient's vascular disease, noninvasive and requiries no contrast media or radiation 11) Mx - a) tx of claudication is medical, except in severe cases b) goal of medical mx of PAOD is to impede progression of disease c) if medical and exercise therapy fail or if have lifestyle-limiting claudication symptoms, surgical treatment options are the next line of therapy - I) Primary amputation preferred in nonambulatory patients with a limited life expectancy and extensive necrosis or gangrene II) Consider surgical bypass for ambulatory patients with a patent infrapopliteal artery that has direct flow to the foot and an adequate autologous venous conduit III) Endovascular mx - balloon angioplasty for clinically significant infrapopliteal arterial disease; consider stents for tibial arterial disease refractory to treatment with balloon angioplasty d) Smoking Cessation e) Exercise - daily walking program of 45-60 minutes is recommended. Pt walks until claudication pain occurs, rests until the pain subsides, and then repeats the cycle f) Pharmacologic Therapy - I) daily aspirin (no consensus on most effective dose) II) Pentoxifylline shows promise III) Consider clopidogrel bisulfate and enoxaparin sodium IV) Consider cilostazol in intermittent claudication V) Consider statins VI) Tx of concurrent conditions eg/ T2DM, HTN e) Pts tx medically should be seen every 4-6 months to assess the effects of therapy

Describe the clinical presentation of intracerebral haemorrhage, initial investigations and immediate patient management

1) intracerebral haemorrhage (ICH) = bleeding within the brain tissue itself — a life-threatening type of stroke, mostly from HTN, arteriovenous malformations, or head trauma 2) s/s - headache, N&V a) lethargy or confusion b) sudden weakness or numbness of the face, arm or leg, usually on one side c) loss of consciousness d) temporary loss of vision e) seizures 3) ix - a) CTA or MRA to detect brain bleeding b) CT angiogram c) bloods - FBC, U&E, clotting, CRP 4) mx - a) tracheal intubation indicated if dec level of consciousness or risk of airway obstruction b) IV fluids are given to maintain fluid balance, using isotonic not hypotonic fluid c) if small haemorrhage (<10 cm3) and minimal deficits tx medically d) if cerebellar haemorrhages >3 cm3 & deteriorating or who have brainstem compression and hydrocephalus - tx surgically to remove the hematoma as soon as possible e) if large lobar haemorrhages (50 cm3) & deteriorating - surgical removal of haematoma f) medical tx - pt on stroke unit or ICU for close monitoring - I) reverse blood thinners if applicable II) BP managed to dec risk of more bleeding yet provide enough blood flow (perfusion) to the brain III) controlling ICP if large bleed IV) removing CSF from ventricles helps control pressure. A ventricular catheter (VP shunt) may be placed to drain CSF fluid and allow room for the haematoma to expand without damaging the brain V) hyperventilation also helps control ICP. In some cases, coma may be induced with drugs to bring down ICP VI) frozen plasma, vitamin K, protamine, or platelet transfusions may be given in case of a coagulopathy VII) fosphenytoin or other anticonvulsant given if seizures or lobar haemorrhage VIII) H2 antagonists or PPIs given to prevent stress ulcers, a condition linked with ICH g) surgical tx - goal is to remove as much of the blood clot as possible and stop the source of bleeding if from an identifiable cause eg/ AVM or tumor I) craniotomy - inc risk to brain, so used only when the hematoma is close to the surface of the brain or if it is associated with an AVM or tumor that must also be removed II) stereotactic clot aspiration - minimally invasive surgery for large hematomas deep inside the brain III) catheter may be passed into the brain vasculature to close off or dilate blood vessels, avoiding invasive surgical procedures

List the commonest presentations of chronic GI blood loss

1) iron deficiency anaemia s/s eg/ lethargy, SOB, pallor, weakness, dizziness, angina 2) may have occult bleeding in stool 3) may have obvious bleeding eg/ meleana, haematemesis

Describe the investigations and management of pulmonary oedema

1) ix - a) ECG - signs & symptoms of MI, dysrhythmias b) echocardiography c) CXR - cardiomegaly, signs of pulmonary oedema - look for shadowing (usually bilateral), small effusions at costophrenic angles, fluid in lung fissures, Kerley B lines, hazy hila, prominent vasculature d) troponin e) FBC, U&Es, LFTs and urine dip - helpful in diagnosis of cause 2) mx - a) preload reducers eg/ furosemide IV - dec pressure caused by fluid going into heart and lungs. Due to inc urine may temporarily need a urinary catheter. SE = mild GI disturbances, pancreatitis, hepatic encephalopathy, postural hypotension. Need daily weighing to check if working b) morphine orally - can relieve SOB and anxiety. SE = GI disturbance, irregular heartbeat, blurred vision, headache c) afterload reducers eg/ nitroprusside IV - vasodilation + dec pressure on L ventricle. SE = hypotension, cyanide toxicity NB/ sitting upright helps relieve breathlessness, high flow O2 to restore sats and reduce respiratory workload, and ECG should be monitored

Describe the acute investigation and management of a suspected extradural haemorrhage

1) ix - a) EDH is a subperiosteal haematoma located on the inside of the skull, between inner table of the skull and parietal layer of the dura mater (the periosteum), so EDHs are usually limited in their extent by the cranial sutures, as the periosteum crosses through the suture continuous with the outer periosteal layer. This is therefore helpful in distinguishing EDHs from subdural haematomas, which are not limited by sutures b) CT head - see EDH in almost all cases, typically bi-convex (or lentiform) in shape, and most frequently beneath the squamous part of the temporal bone. EDHs are hyperdense, somewhat heterogeneous, and sharply demarcated. Depending on their size, secondary features of mass effect (e.g. midline shift, subfalcine herniation, uncal herniation) may be present c) MRI - can clearly demonstrate the displaced dura that appears as a hypointense line on T1 and T2 sequences which is helpful in distinguishing it from a subdural haematoma d) angiography - can be used to evaluate nontraumatic cause (i.e. AVM) of EDH 2) tx - a) prognosis, even with a relatively large haematoma, is in general quite good, as long as the clot is evacuated promptly b) smaller haematoma without mass effect or swirl sign can be treated conservatively, sometimes resulting in calcification of the dura. c) blood may be removed surgically to remove the mass and reduce the pressure it puts on the brain - haematoma is evacuated through a burr hole or craniotomy. If transfer to a facility with neurosurgery is prolonged trephination may be performed in the emergency department

Colorectal carcinoma - ix, mx & define appropriate resection levels according to lymphatic drainage

1) ix - a) FBC - anaemia, LFTs (normal), U&Es (normal) b) colonoscopy - ulcerating or exophytic mucosal lesion that may narrow the bowel lumen, or CT colonography c) double-contrast barium enema - mass lesion in colon +/- as a characteristic 'apple core' lesion d) CT scan of thorax, abdomen, and pelvis - colonic wall thickening, enlarged lymph nodes, liver metastases, ascites, lung secondaries 2) rectal cancer mx - a) stage I, low risk (<3 cm; <30% circumference of the bowel; moderately or well differentiated; localised) = local excision or radical excision b) stage I, high risk = radical resection +/- preoperative radio ± chemotherapy c) stage II - III = radical resection +/- preoperative or postoperative chemotherapy d) stage IV = surgical resection + chemo e) if not suitable for surgery = chemo +/- stenting +/- monoclonal antibodies eg/ bevacizumab 3) colon cancer mx - a) stage I - III = surgical resection + postop chemo b) stage IV = surgical resection + preop chemo +/- monoclonal antibodies +/- c) if not suitable for surgery = chemo + monoclonal antibodies +/- stenting

Ix, and mx of IBS

1) ix - a) FBC - normal; anaemia suggests non-IBS disease b) stool studies - done if diarrhoea, normal; WBCs in stool or presence of parasites suggests non-IBS disease c) anti-endomysial antibodies - +ve if coeliac d) anti-tTG antibodies - +ve if coeliac disease e) plain abdominal x-ray - normal; abnormal bowel pattern suggests obstruction f) flexible sigmoidoscopy - abnormal mucosa = IBD, can also detect polyps or carcinoma g) colonoscopy - detection of IBD or neoplasm h) hydrogen breath test - normal; abnormal if bacterial overgrowth or lactase deficiency i) faecal calprotectin - <40 micrograms/g makes IBD unlikely (and IBS more likely) j) serum CRP - <0.5 mg/L makes IBD unlikely (and IBS more likely) 2) mx - a) constipation-predominant - I) 1st line - lifestyle & diet: FODMAPP diet; avoid precipitants eg/ caffeine, lactose, or fructose; add fibre; consider probiotics II) + laxatives eg/ ispaghula or lactulose or if cannot tolerate - lubiprostone or linaclotide III) if pain or bloating - antispasmodics eg/ dicycloverine or hyoscyamine IV) if pain or bloating - SSRI eg/ paroxetine or citalopram V) +/- CBT b) diarrhoea predominant - I) lifestyle & diet as above II) + antidiarrhoeals eg/ loperamide or colestyramine III) + antispasmodics IV) + TCA eg/ amitriptyline V) +/- CBT c) alternating constipation and diarrhoea - combine the above

Coeliac disaese ix and mx

1) ix - a) FBC and blood smear - iron deficiency anaemia b) IgA-tTG - higher titres have inc +ve predictive value c) EMA - more expensive than IgA-tTG with greater specificity but lower sensitivity d) skin bx - if skin lesions suggestive of dermatitis herpetiformis e) IgG DGP or IgA/IgG DGP (deamidated gliadin peptide) - inc titre f) IgG-tTG - inc titre g) HLA typing - +ve HLA-DQ2 or -DQ8 h) small bowel - atrophy and scalloping of mucosal folds; nodularity and mosaic pattern of mucosa, presence of intra-epithelial lymphocytes, villous atrophy, and crypt hyperplasia i) gluten challenge - pt placed back on a gluten-containing diet, with serological tests and small bowel histology assessed after 2 to 8 weeks on the gluten-containing diet - inc coeliac serological tests and presence of intra-epithelial lymphocytes, villous atrophy, and crypt hyperplasia on small intestinal biopsy j) future tests - point-of-care tTG testing, saliva coeliac genetic test 2) mx - a) 1st line - gluten-free diet b) + calcium and vitamin D supplementation ± iron eg/ ergocalciferol + calcium carbonate c) refractory coeliac disease + referral to nutritionist or gastroenterologist d) coeliac crisis - I) rehydration + correction of electrolyte abnormalities + corticosteroid eg/ budesonide or prednisolone

Outline the principles of management of patients with symptomatic haemorrhoids including investigation and differential diagnosis appropriate to patient factors including history and age

1) ix - a) anoscopic examination - see haemorrhoids b) colonoscopy/flexible sigmoidoscopy - normal in haemorrhoids c) FBC - may demonstrate microcytic/hypochromic anaemia d) stool for occult haem - positive, only if no obvious haemorrhoids 2) DD - anal fissure, IBD, colorectal cancer, anal fistula, rectal prolapse 3) mx - a) 1st line - dietary and lifestyle modification eg/ inc fibre and fluids , dec time spent passing stool, moist, gentle cleaning following a bowel movement is advised b) if suspicious symptoms eg/ altered bowel habit, abdo pain, weight loss, iron-deficiency anaemia, or passage of blood clots +/- mucus, lower GI endoscopy is performed c) grade 1 haemorrhoids: + topical corticosteroids eg/ rectal hydrocortisone d) grade 2 prolapsing internal haemorrhoids: + rubber band ligation or sclerotherapy or infrared photocoagulation or haemorrhoid arterial ligation or stapled haemorrhoidopexy e) grade 3 prolapsing internal haemorrhoids: + rubber band ligation, or stapled haemorrhoidopexy or haemorrhoid arterial ligation or surgical haemorrhoidectomy f) grade 4 internal, external, or mixed internal and external haemorrhoids: + surgical haemorrhoidectomy

Anal carcinoma - ix, mx & define appropriate resection levels according to lymphatic drainage

1) ix - a) anoscopy - anal mass seen b) biopsy of tumour - cancerous cells seen on pathological examination c) anal ultrasound - anal tumour; enlarged perianal lymph nodes d) abdominopelvic CT scan - primary tumour seen; metastatic lesions may be visualised e) CXR or chest CT scan - to rule out lung metastases f) PET scan - primary tumour seen; metastatic lesions may be able to be visualised g) inguinal node needle biopsy 2) mx - a) 1st line = combined-modality treatment (CMT) - radiotherapy + fluorouracil + mitomycin b) +/- pre-combined-modality treatment (CMT) diverting colostomy - if fecal incontinence or an anal fistula may need to have a pre-treatment diverting colostomy c) 2nd line = salvage combined-modality treatment (CMT) - radiotherapy + fluorouracil + cisplatin d) 3rd line = abdominoperineal resection - if response is still less than complete 6 weeks after completing salvage combined-modality treatment (CMT), results in a permanent colostomy

Outline the investigation of a patient with acute renal failure. Outline the principles of treatment. Outline the indications for referral for a specialist opinion/renal replacement therapy

1) ix - a) basic metabolic profile (including urea and creatinine) - inc creatiine may be initial or only sign of decline in renal function, may have inc K+, metabolic acidosis b) ratio of serum urea: creatinine - 20:1 or inc supports pre-renal azotaemia c) urinalysis - glomerular disease typically present with proteinuria and microscopic haematuria with HTN and oedema, may show infection d) urine culture - may show infection e) FBC - anaemia is suggestive of possible CKD, blood loss, leukocytosis supports infection, thrombocytopenia can be seen in cryoglobulinaemia, HUS, or thrombotic thrombocytopenic purpura f) fractional excretion of sodium and urea g) venous blood gases - diagnostic for metabolic acidosis and certain intoxications h) fluid challenge - diagnostic and therapeutic for pre-renal azotaemia i) bladder catheterisation - diagnostic and therapeutic for bladder neck obstruction j) urine osmolality, urine sodium concentration k) renal ultrasound l) CXR (see if renal failure is associated with heart failure), ECG (may occur with severe hyperkalaemia) m) consider - ANA + anti-DNA (SLE), complement (C3, C4, CH50), anti-glomerular basement membrane (eg/ Goodpasture's and anti-glomerular basement membrane syndrome), anti-neutrophil cytoplasmic antibodies (ANCA - vasculitic syndromes eg/ granulomatosis with polyangiitis, eosinophilic polyangiitis, and microscopic polyangiitis), acute hepatitis profile, HIV serology, cryoglobulins, ESR, anti-streptolysin-O antibody (supports infectious glomerulonephritis), abdo CT or MRI (check for obstruction), nuclear renal flow scan, cystoscopy, renal biopsy 2) mx - a) general components - I) with volume overload: +/- diuretic: furosemide: 20-40 mg IV II) with uraemia, severe metabolic acidosis, hyperkalaemia refractory to medical management, or volume overload unresponsive to diuretics: +/- renal replacement therapy: need nephrologist consultation - conventional haemodialysis for 4-6 hours in haemodynamically stable patients., otherwise other forms of dialysis, or transplant if ineffective b) pre-renal azotaemia - I) 1st line - volume expansion and/or RBC transfusion: crystalloid (normal saline or lactated Ringer's) sufficient in most cases for volume expansion, haemorrhage requires blood product replacement II) with severe hypotension: + vasopressor: dopamine or adrenaline or noradrenaline or phenylephrine c) intrinsic renal failure - I) 1st line - treatment of underlying condition: stop nephrotoxic agents, pt referred to a nephrologist if specific treatment, such as dialysis, mx of fluids/acid-base status, severe hyperkalaemia, or immunosuppression is required II) with pre-existing pre-renal azotaemia: +/- volume expansion d) obstructive renal failure - I) 1st line - bladder catheterisation II) 2nd line - relief of obstruction above bladder neck: ureteral stenting: if ureteral stricture, stone or extrinsically obstructing mass; or lithotripsy: stones at ureteropelvic junction causing obstruction may require lithotripsy or surgical removal; or exploratory laparotomy: compressing tumours may require surgical removal; or percutaneous nephrostomy: placement of a catheter into the renal pelvis percutaneously for drainage of urine from a distal obstruction

Outline methods of evaluating and managing patients with carotid stenosis

1) ix - a) duplex ultrasonography - I) formula used to calculate the degree of stenosis is: % stenosis = [1 - (minimum diameter/distal diameter)] ×100 II) elevated blood flow velocities along with visualisation of details of the plaque within the carotid arteries; the velocities are translated into ranges of stenosis, generally categorised as mild (<50%), moderate (50%-69%), high-grade (70%-79%), and critical (80%-99%); a very tight and long stenosis (from the carotid bifurcation to the base of the skull) in the range of >95% is sometimes called a string sign b) consider - CT angiography (CTA) or MRA of head, neck, and chest, (shows narrowed artery), cervical angiography, CT or MRI brain 2) mx - a) asymptomatic carotid stenosis - I) 1st line - antiplatelet therapy and cardiovascular risk reduction eg/ aspirin, or clopidogrel or ticlopidine; risk factors such as smoking, hypercholesterolaemia, and HTN must be managed according to appropriate guidelines; inc physical activity & health diet II) if >70% - +/- carotid endarterectomy or stenting b) symptomatic - I) ipsilateral carotid stenosis ≥50% - 1st line - carotid endarterectomy + antiplatelets & CV risk reduction symptomatic patients include those with TIA, stroke, and amaurosis fugax aspirin or clopidogrel or aspirin/dipyridamole II) ipsilateral carotid stenosis <50% - 1st line - antiplatelets & CV risk reduction c) bilateral carotid stenosis - I) 1st line - carotid endarterectomy or stenting based on merits of each carotid artery + antiplatelet therapy & CV risk reduction

Outline the laboratory features of microangiopathic anaemia and list the common causes

1) ix - a) red cell fragments = schistocytes, seen b light microscopy b) isolated inc in serum bilirubin c) -ve Coombs test d) thrombocytopenia 2) causes - a) DIC b) HELLP syndrome c) thrombotic thrombocytopenic purpura-hemolytic uremic syndrome d) malignancy e) malignant hypertension f) scleroderma renal crisis g) malfunctioning cardiac valves (called the "waring blender syndrome") h) kasabach-merritt syndrome i) insertion of foreign bodies j) drugs (e.g. cancer chemotherapy) k) others diseases: eclampsia, renal allograft rejection, paroxysmal nocturnal hemoglobinuria, scleroderma, and vasculitides such as polyarteritis nodosa and granulomatosis with polyangiitis, antiphospholipid syndrome

Outline the investigation necessary to confirm the diagnosis and outline the treatment options including the role of immunosuppressive therapy for some forms of GN

1) ix - a) urinalysis - haematuria, proteinuria, dysmorphic RBCs, leukocytes, RBC casts b) comprehensive metabolic profile - inc creatinine = severe or advanced disease, inc liver enzymes suggests hep B or C, hypoalbuminaemia suggests NS c) glomerular filtration rate (GFR) - normal or reduced d) full blood count - often normocytic normochromic anaemia e) lipid profile - hyperlipidaemia or normal f) spot urine albumin:creatinine ratio (ACR) - if ACR is >220 mg/mmol, patients classified as having nephrotic-range proteinuria and may have full nephrotic syndrome (hyperlipidaemia, hypoalbuminaemia, oedema, nephrotic-range proteinuria) g) US kidneys - small kidneys or normal, excludes other causes h) consider - ESR or CRP (inc = inflammation eg/ vasculitis), complement levels (low or normal C3 in immune complex diseases), RF (+ve in RA), anti-neutrophil cytoplasmic antibody (+ve = pauci-immune or anti-glomerular basement membrane disease), anti-glomerular basement membrane (GBM) antibody (+ve = anti-GBM disease or Goodpasture's syndrome), antistreptolysin O antibody +/- antihyaluronidase +/- anti-DNase (high or rising titres = post-streptococcal GN), anti-double-stranded DNA +/- antinuclear antibody (+ve = SLE), cryoglobulins (+ve = cryoglobulinaemia), hep C and B & HIV serology, electrophoresis (inc gamma-globulin associated with eg/ lymphoma, amyloidosis, and SLE), drug screen, renal biopsy, antiphospholipase A2 receptor antibodies (+ve = idiopathic membranous GN), CTCA (may be important to exclude malignancy in older patients) 2) mx - a) mild disease - I) 1st line - tx underlying cause + supportive measures: eg/ abx (, antivirals, withdrawal of the causative drug, dec intake of salt and water II) abx options if post-streptococcal GN: phenoxymethylpenicillin or benzylpenicillin sodium or amoxicillin or amoxicillin/clavulanate or cefalexin or cefuroxime or azithromycin or clarithromycin b) moderate-severe disease - I) 1st line - ACEI or ARB: ramipril or enalapril or lisinopril or losartan or candesartan etc II) +/- abx - as above III) +/- furosemide IV) with nephrotic syndrome: prednisolone ± other immunosuppressant eg/ cyclophosphamide or azathioprine or mycophenolate mofetil or rituximab V) with nephrotic syndrome: +/- prophylactic trimethoprim/sulfamethoxazole c) rapidly progressive - I) anti-GBM 1st line - plasmapheresis + prednisolone + cyclophosphamide +/- prophylactic trimethoprim/sulfamethoxazole +/- abx (as above) II) immune complex (no SLE) - 1st line - corticosteroids: methylprednisolone or prednisolone; +/- abx (as above) III) immune complex (with SLE) - 1st line - immunosuppressants: cyclophosphamide or mycophenolate mofetil or rituximab +/- prednisolone +/- prophylactic trimethoprim/sulfamethoxazole +/- abx (as above) IV) pauci-immune - 1st line - corticosteroids + other immunosuppressants: methylprednisolone + cyclophosphamide or azathioprine or mycophenolate mofetil or rituximab; +/- prophylactic trimethoprim/sulfamethoxazole +/- abx (as above)

Describe the immediate investigation and management of suspected acute hydrocephalus

1) ix - US, CT, MRI, or pressure-monitoring techniques depending on pt's age, clinical presentation, and the presence of known or suspected abnormalities of the brain or spinal cord 2) mx - a) most often treated by surgically inserting a shunt system to divert flow of CSF from the CNS to another area of the body where it can be absorbed as part of the normal circulatory process b) shunt is a flexible but sturdy plastic tube, consists of shunt, a catheter, and a valve. One end of the catheter is placed within a ventricle inside the brain or in the CSF outside the spinal cord. The other end of the catheter is commonly placed within the abdominal cavity, but may also be placed at other sites in the body such as a chamber of the heart or areas around the lung where the CSF can drain and be absorbed. A valve located along the catheter maintains one-way flow and regulates the rate of CSF flow

Describe the acute management of stroke, with particular attention to Examination; Investigations; Consideration or initiation of Antiplatelet therapy, Anticoagulation, Thrombolysis, Blood pressure control, Statin therapy

1) ix - brain CT to exclude bleed, FBC, U+Es, LFTs, Coag, glucose, cholesterol, ESR, TFT, sickle cell, thrombophilia & vasculitic screen if young a) exam - general inspection, GCS, ABCDE, CV s/s I) NIHSS (national institute of health stroke scale) - tool to rapidly assess stroke pts, /40, 20 = severe 2) mx - a) specialist stroke wards b) scanned within 1hr (within 12) if high NIHSS score - CT brain to exclude haemorrhage (eg/ haem = white, isch = dark) c) ECG - check for MI, AF, LVF, may echo eg/ for valvular heart disease (SBE) d) homeostasis: I) O2 if sats <95% II) maintain blood glucose between 4-11 e) antiplatelets - aspirin (300 mg) ASAP after haemorrhage excluded f) long-term vascular prevention if ischaemic stroke or TIA without AF - clopidogrel or aspirin 75 mg daily + dipyridamole 200 mg 2x/day, dual antiplatelet therapy for 1yr, then just aspirin g) thrombolytics - unless CI eg/ alteplase within 4.5 hrs if no haemorrhage h) no drugs depressing CNS (eg/ anxiolytics) i) encourage pts mobilise ASAP j) pts with TIA assessed in specialist service within 7 days k) surgery - if supratentorial haemorrhage or posterior fossa/cerebellar haematoma: mechanical clot retrieval, surgical decompressive hemicraniectomy l) swallowing assessment before commence E&D m) anticoagulation - heparin or warfarin if venous sinus thrombosis, continued for 3 to 6 months, as aspirin and alteplase (r-tPA) are not indicated in these patients n) long term - BP control, statins

Describe the normal functions of the kidneys, ureters, and bladder

1) kidneys - retroperitoneal on posterior abdominal wall (T12-L3), R kidney slightly lower, fluid homeostasis, remove salts & wastes of protein metabolism from blood while returning nutrients + chemicals to blood (acid-base + electrolyte balance), BP control, production of EPO + vitD a) anatomy - medial margins have renal sinus + renal pelvis, renal hilum (medial border) = entrance to space in kidney = renal sinus (mainly fat) - has renal pelvis, calices, vessels & nerves. At hilum: renal vein, artery, pelvis 2) ureters - connect kidneys and bladder - allows waste products to be transported in the form of urine, run inferiorly to kidneys, passing over pelvic brim at bifurcation of common iliac arteries, run along lateral wall of pelvis & enter urinary bladder a) anatomy - superior expanded surface = renal pelvis - formed by merging of 2+ major calices (formed by merging of minor calices). Each minor calyx indented by apex of renal pyramid - renal papilla. Abdominal parts of ureters adhere closely to parietal perineum & retroperitoneal. Run inferomedially anterior to psoas major + tips of transverse processes (L), cross external iliac artery beyond bifurcation of common iliac artery, run along lateral wall of pelvis to enter urinary bladder. Normally constricted at: junction of ureters + renal pelves, where cross brim of pelvic inlet, during passage through wall of urinary bladder 3) bladder - stores urine to allow infrequent + voluntary urination a) anatomy - hollow muscular distendable organ, sits on pelvic floor, capacity = 300-500mL

Discuss the indications for and sequence of investigations for suspected pseodocyst

1) lab - a) amylase & lipase often inc but may be within reference ranges b) bilirubin & LFTs may be inc if biliary tree involved c) analysis of cyst fluid may help differentiate pseudocysts from tumors d) carcinoembryonic antigen (CEA) and carcinoembryonic antigen-125 (CEA-125) tumor marker levels - low in pseudocysts, inc in tumors 2) fluid viscosity - low in pseudocysts, inc in tumors 3) US - abdo US may visualise cystic fluid collections in and around the pancreas 4) endoscopic US - important in planning therapy, particularly if endoscopic drainage is contemplated, transmural drainage may be performed only when a symptomatic pseudocyst is positioned next to the gut wall 5) CT or MRI 6) Endoscopic Retrograde Cholangiopancreatography 7) histology

Outline basic investigations that may be performed in the investigation of a suspected myopathy

1) lab - a) creatine kinase (CK) with isoenzymes - inc (enzyme released when muscle fibres degenerate) b) electrolytes (calcium, magnesium) c) serum myoglobin d) serum creatinine and blood urea nitrogen (BUN) e) urinalysis - myoglobinuria is indicated by positive urinalysis with few red blood cells (RBCs) on microscopic evaluation f) FBC, ESR, TFTs, LFTs (esp AST) 2) ECG - findings suggestive of hypokalemia include: a) diffuse nonspecific ST-T wave changes b) inc PR interval c) U waves d) wide QRS 3) others - genetic testing, ANA 4) muscle biopsy - characteristic degeneration and attempted regeneration of muscle fibres 5) electromyography - shows differences in the electrical patterns of normal muscle, myopathy, and chronic denervation, such as in the spinal muscular atrophies

Recognise and differentiate traumatic wounds that can be safely closed primarily and those that cannot

1) lacerations - caused when trauma exceeds intrinsic tissue strength eg/ skin torn by blunt injury over a bony prominence such as the scalp. Tissue damage may not be extensive, and primary suturing may be possible. Sterile skin closure strips may be appropriate in some circumstances eg/ pretibial laceration, as suturing causes increased tissue tension, with the swelling of early healing and inflammation leading to more tissue loss 2) contusions - caused by more extensive tissue trauma after severe blunt or blast trauma. Overlying skin may seem to be intact but later become non-viable. Large haematomas under skin or in muscle may coexist; if they are superficial and fluctuant they can be evacuated with overlying necrosed skin. Ultrasound scanning or magnetic resonance imaging may define a haematoma amenable to evacuation. Extensive contusion may lead to infection (abx prophylaxis should be considered in open wounds) and compartment syndromes (fasciotomy will be needed to preserve a limb) 3) large open wounds - may be left to heal "by secondary intention" (wound heals from the base upwards, by laying down new tissue) or with delayed skin grafting, depending on the extent of the residual defect. Exploration of a traumatic wound is needed if there is a suspicion of blood vessel or nerve damage, with attention to fractures and debridement of devascularised tissue and removal of foreign material 4) abrasions - superficial epithelial wounds caused by frictional scraping forces. When extensive, they may be associated with fluid loss. Such wounds should be cleansed to minimise the risk of infection, and superficial foreign bodies should be removed (to avoid unsightly "tattooing")

Describe the morphology and pathological consequences of a nodular goitre

1) lack of iodine = dec production of thyroid hormones 2) dec thyroid hormones = inc TSH release - has trophic effects on gland 3) gland enlarges in an attempt to maximise the amount of iodine it is able to extract from the blood, to maximise hormone production 4) nodular goitres include: multinodular, solitary nodular, fibrotic (rare, "Reidel's thyroiditis) 5) multinodular - most common, esp in older patients a) pt may be euthyroid, or hyperthyroid with suppressed TSH but normal T4 and T3 b) extra-endocrine complications - tracheal and/or oesophageal compression, laryngeal nerve palsy c) nodules of hyperplasia interspersed with inactive tissue 6) solitary nodule goitres considered for malignancy - fine needle biopsy a) rapid enlargement & LN growth separate malignancy from benign goiter b) may be associated with T3 toxicosis - autonomous production of T3 by adenoma = drop in TSH & rest of gland will become inactive 7) histologically, follicles may be small or normal size 8) fibrotic goitres may produce a "woody" gland 9) clues malignancy status - systemic symptoms of inflammation & markers

Outline the main complication that arises from untreated or missed temporal arteritis

1) large vessel stenosis - stenosis of branches of aortic arch, esp subclavian & axillary arteries occur in ~15% pts 2) aortic aneurysm - esp thoracic, inc risk of abdominal as well 3) glucocorticoid related adverse effects eg/ glucocorticoid induced diabetes, inc BP, corticoid induced bone loss 4) vision loss - once blindness has occurred it's irreversible

Outline the risk factors for recurrence of hernias after repair

1) larger size 2) obesity 3) chronic cough 4) female gender 5) direct inguinal hernias at primary procedure 6) operation for a recurrent inguinal hernia ( 7) smoking 8) emergency admission 9) CT composition 10) degradation 11) fmx

Discuss the frequency of each according to location of the carcinoma, particularly with rectal and carcinoma of the caecum. Identify the pathological differences between colorectal and anal cance

1) largest proportion of bowel cancer cases occur in the rectum, with slightly smaller proportions in the sigmoid colon and caecum, and a much smaller proportion in the ascending colon 2) proportions of cases in the rectum and sigmoid colon are higher in males (31.5% and 23.1%, respectively) than females (23.1% and 20.4%, respectively). In the caecum and ascending colon, the proportions are higher in females (17.2% and 9.8%, respectively) than males (12.2% and 7.3%, respectively), and there are no marked sex differences in other parts of the bowel 2) colon - a) ascending - 7% men, 10% women affected by colonic cancer b) transverse - 5% men, 6% women affected by colonic cancer c) descending - 3% men & women affected by colonic cancer d) caecum - 12% men, 17% women affected by colonic cancer e) sigmoid - 23% men, 20% women affected by colonic cancer 3) rectum - 32% men, 23% women affected by colonic cancer 4) pathology - a) most rectal cancers are adenocarcinomas, with neuroendocrine tumors being the second most common histologic type b) adenocarcinomas are distinguished by the presence of a neoplastic columnar epithelium invading the lamina propria or beyond and show either a pure gland-forming growth pattern or a mixed growth pattern with glandular, solid, and cribriform areas c) most anal cancers are squamous cell carcinomas. The dentate line, which corresponds to the transition between rectal and anal mucosa, represents a landmark for differentiating histologic subtypes of squamous cell carcinoma arising in and around this region. Tumors originating in the transitional zone itself are frequently basaloid SCC, composed of nests of tumor cells with a high nuclear-to-cytoplasmic ratio and small amounts of cytoplasm, which imparts a more poorly differentiated appearance, and these tumors may or may not show keratinization; those originating distal to the dentate line are more often well-differentiated keratinizing squamous cell carcinomas

Outline the investigation of a patient with suspected TB

1) latent - do a Mantoux test + consider interferon-gamma testing 2) active - take sputum samples (3+) & send for MC&S for acid-fast bacilli on Ziehl-Neelson staining 3) if not respiratory get other samples eg/ urine, pus, pleural fluid, BM, CSF 4) PCR - allows rapid identification of rifampicin resistance 5) histology - hallmark = presence of caseating granulomata 6) CXR (ZN) - consolidation/collapse, cavitation (apices), fibrosis, calcification, abscess, pneumonia, infarct, pleural effusion, miliary shadowing

Discuss general measures and medical therapy for GORD. Outline the surgical measures used to treat GORD

1) lifestyle - most pts require further therapy: a) weight loss b) smoking & alcohol cessation c) raise the head of the bed at night d) take small, regular meals e) avoid hot drinks, alcohol and eating during the three hours before bed f) avoid drugs which affect oesophageal motility (nitrates, anticholinergics, TCAs) or damage the mucosa (NSAIDs, potassium salts, alendronate) 2) meds - a) pts with reflux symptoms but no red flags - give full-dose PPI for 1/12 b) if symptoms return after tx, and long-term acid suppression is required, use step-down strategy to the lowest dose of PPI that provides effective relief of symptoms 3) referral for endoscopy - a) inadequate response to tx b) new emergent symptoms c) pts undergoing endoscopy should be free from PPI or H2RA for 2/52 d) if endoscopy shows oesophagitis is present, a healing dose of PPI should be prescribed for 2/12 4) surgery - if tx not working may consider laparoscopic fundoplication surgery (laparoscopic insertion of a magnetic bead band) 5) sudden or progressive worsening of symptoms in a pt >55 yo, or development of dysphagia, anaemia, persistent vomiting or weight loss at any age, merits urgent referral for endoscopy (2/52 rule)

List the medical and surgical approaches to treating vasospastic disorders

1) lifestyle changes eg/ stopping smoking, avoid caffeine, dec stress, avoid cold objects, dressing warmly in cold eg/ gloves, avoid vibrating tools 2) meds that vasodilate eg/ CCBs (nifedipine) or diltiazem 3) tx of underlying condition if present 4) emergency measures in attack - running warm water over affected area until healthy colour returns 5) in severe cases, a sympathectomy can be performed 6) infusions of prostaglandins eg/ prostacyclin, may also be tried 7) botox has been used in severe cases 8) biofeedback has occasional been helpful in selected cases

Compare and contrast the pathology and natural histories of liver neoplasia, abscesses and cysts

1) liver neoplasia a) aetiology - I) aetiology of HCC is multifactorial II) major risk factors - chronic hep B&C, heavy alcohol consumption, DM; also fmx of liver cancer in a first-degree relative appears to be a significant risk factor III) ¼ patients have no known risk factor b) pathophysiology - I) chronic inflammation and cirrhosis play important roles in hepatocellular carcinogenesis II) acquisition of hep B virus early in life results in longer duration of disease that may lead to chronic inflammation and cirrhosis. HBV is a direct carcinogen and may cause methylation of the P16 gene, known to cause hepatocellular carcinogenesis leading to HCC. In addition, mutation of the X gene of HBV has also been found to play a role III) hypothesised that high levels of oestrogen and lipid peroxidation in patients with chronic hep C virus (HCV) may result in the development of the carcinoma. [18] The HCC related to HCV is almost always due to cirrhosis IV) chronic HBV and HCV infection with metabolic disorders may lead to generation of reactive oxygen species from oxidative stress that can cause damage to DNA, resulting in mutation of cancer-related genes such as p53 V) aflatoxin is a mycotoxin naturally produced by Aspergillus , and commonly contaminates soya beans. In those patients who have had long-term exposure to aflatoxin, it has been found to cause mutations in the p53 tumour-suppressor gene VI) pathogenesis of HCC usually begins with the development of dysplastic nodules, which can be classified as high-grade or low-grade dysplasia. High-grade dysplasia is a precancerous condition leading to a high rate of conversion to HCC 2) liver abscess - a) aetiology - I) often caused by a polymicrobial infection eg/ E. coli , Klebsiella, Strep constellatus , S anginosus , S intermedius II) recurrent pyogenic cholangitis (associated with hepatolithiasis) due to Salmonella typhi may cause pyogenic liver abscess b) pathophysiology - I) liver abscesses form by spread of infection from 1 of the following sources: biliary tree, portal or hepatic veins, extension of contiguous infection, penetrating trauma II) can be due to IBD, pancreatitis, appendicitis, diverticulitis, or a migrated foreign body 3) liver cyst - a) aetiology - result of a malformation in the bile ducts, although the exact cause of this malformation is unknown. Some people are born with liver cysts, whereas others don't develop cysts until they're much older b) pathology - single, unilocular cyst, usually subcapsular or in falciform ligament, variable amounts of clear amber fluid (may contain blood, bile, mucus, pus)

Describe the role of the liver in normal clotting, including the role of vitamin K in the synthesis of some clotting factors

1) liver plays a major role in haemostasis, as most of the coagulation factors, anticoagulant proteins & components of fibrinolytic system are synthesized by hepatic parenchymal cells 2) reticuloendothelial system of liver helps to regulate coagulation and fibrinolysis by clearing these coagulation factors from the circulation 3) makes coagulation factors - I (fibrinogen), II (prothrombin), V, VII-XI, XIII, as well as protein C, protein S and antithrombin (dec in any of these = inc INR) 4) vitK - fat-soluble vitamin necessary for synthesis of coagulation factors II, VII, IX, and X (in intrinsic + common pathways of coagulation). Also synthesizes Protein C, S, and Z, anticoagulant proteins that degrade specific coagulation factors, preventing excessive thrombosis following the initial coagulation cascade 5) vitK involved in carboxylation of glutamate residues at GIa domains on clotting factors II (prothrombin), VII, IX and X, protein C and protein S. It is absorbed in upper ileum and stored in the liver. It has a short half life 6) activated protein C inactivates factors V and VIII, stopping further thrombin generation 7) vitK can be inhibited by anticoagulant drug warfarin - antagonist for vitK NB/ vitK requires bile salts for absorption - cholestasis will reduce vit K supply

Identify the usual initial anatomic location of deep venous thrombosis & describe the pathophysiology of chronic venous insufficiency and the post-phlebitic syndrome

1) location - DVT often develops in the calf veins and "grows" in the direction of venous flow, towards the heart. Veins in the calf or thigh are most commonly affected, including the femoral vein, the popliteal vein, and the iliofemoral vein 2) Chronic venous insufficiency - condition that occurs when the venous wall and/or valves in the leg veins are not working effectively, making it difficult for blood to return to the heart from the legs. CVI causes blood to "pool" or collect in these veins (stasis) a) Veins return blood to the heart - needs calf muscles and the muscles in the feet to contract with each step to squeeze the veins and push the blood upward. To keep the blood flowing up, and not back down, the veins contain one-way valves b) Chronic venous insufficiency occurs when these valves become damaged, allowing blood to leak backward c) Valve damage = aging, extended sitting or standing or a combination of aging and reduced mobility d) When the veins and valves are weakened to the point where it is difficult for the blood to flow up to the heart, BP in the veins stays inc for long periods of time, leading to CVI e) CVI most commonly occurs as the result of a DVT in the deep veins if the legs, can also be - pelvic tumors and vascular malformations, idiopathic f) Chronic venous insufficiency that develops as a result of DVT is also known as post-thrombotic syndrome 3) Post-phlebitic syndrome - develops as a consequence of long-standing venous hypertension, combination of reflux due to valvular incompetence and venous hypertension due to thrombotic obstruction is thought to contribute to post-thrombotic syndrome eg/ acute DVT 4) Inc inflammatory cytokines or adhesion molecules, such as interleukin-6 and intercellular adhesion molecule-1, have also been linked with the development of post-thrombotic syndrome

Classify anaemia in terms of red cell indices and list common causes of each type of anaemia

1) low MCV = microcytic anaemia a) iron deficiency anaemia b) thalassaemia - definitive diagnosis = DNA analysis (inborn error of Hb synthesis) c) sideroblastic - bone marrow is unable to incorporate iron into RBCs properly 2) normal MCV = normocytic anaemia a) acute blood loss b) anaemia of chronic disease (or dec MCV) - aplastic or haemolytic anaemia c) bone-marrow failure (dec WCC or platelets) d) renal failure e) hypothyroidism (or inc MCV) f) pregnancy g) inflammation & malignancy h) malnutrition 3) high MCV = macrocytic anaemia a) B12 or folate deficiency (megaloblastic): I) B12 - lack of intake (vegetarians), lack of IF (pernicious anaemia, gastrectomy), failure of ileal absorption (disease of term ileum, resection, tropical sprue) II) folate - lack of intake (elderly, alcoholics), not meeting demand (pregnancy, haemolytic anaemias), failure of absorption (coeliac, tropical sprue, anticonvulsant drugs) b) alcohol excess or liver disease c) reticulocytosis d) cytotoxins eg/ hydroxycarbamide e) myelodysplastic syndromes f) marrow infiltrate g) hypothyroidism h) antifolate drugs eg/ phenytoin NB/ haemolytic anaemias don't fit in classification as may be normocytic or if many young (hence larger) RBCs & reticulocytes, macrocytic

Outline the counter-regulatory hormone responses to hypoglycaemia

1) main counterregulatory hormones: glucagon, adrenaline, cortisol, GH 2) non-diabetics: a) pancreas decreases insulin output b) a-cells of pancreas secrete glucagon - signals liver to release more glucose c) adrenal glands secrete adrenaline - signals liver + kidneys to produce more glucose, keeps certain body tissues eg/ muscle, from using as much glucose from bloodstream, acts to dec insulin secretion d) when glucagon + adrenaline fail to adequately raise blood glucose levels, body releases cortisol & growth hormone - also inc blood glucose levels 3) established T1DM - many lose most of these defenses against hypoglycemia NB/ glucagon response to hypoglycaemia in diabetes is normally inadequate, A response may be absent in long history of diabetes (loss of warning symptoms). GH & cortisol released over a longer time to give small inc in blood glucose levels

Outline the pathology of malabsorption of key nutrients and consequent presentation and management for each

1) malabsorption syndrome - refers to a number of disorders in which the small intestine can't absorb enough of certain nutrients and fluids 2) causes eg/ damage to the intestine from infection, inflammation, trauma, or surgery; prolonged use of abx; celiac disease, Crohn's disease, chronic pancreatitis, or cystic fibrosis; lactase deficiency, or lactose intolerance; congenital defects eg/ biliary atresia etc 3) s/s - based on which nutrient is deficient: a) fats - steatorrhoea (light-colored, foul-smelling stools that are soft and bulky and float) b) protein - dry hair, hair loss, or oedema c) certain sugars - bloating, gas, or explosive diarrhea d) certain vitamins - anaemia, malnutrition, low BP, weight loss, or muscle wasting 4) tx - a) start tx by addressing symptoms such as diarrhea b) replace the nutrients and fluids which are deficient c) monitor for signs of dehydration - may need PO or IV fluids d) tx cause eg/ if lactose intolerance avoid milk and other dairy products or take a lactase enzyme tablet e) may need dietician referral f) may need enzyme supplements g) may need vitamin supplements may need diet changes

Describe the range of clinical presentation and associated pathology of pulmonary embolic disease

1) massive PE - a) pts in shock and can have sudden death, have systemic hypotension, poor perfusion of extremities, tachycardia, and tachypnoeic. Pts appear weak, pale, sweaty, oliguric and develop impaired mentation b) signs of pulmonary hypertension eg/ RV S3 gallop, systolic murmur louder on inspiration at L sternal border (tricuspid regurg), heave LICS2, ECG showing RBBB + anterior T wave inversion 2) acute pulmonary infarction - a) acute onset of pleuritic chest pain, breathlessness, and haemoptysis b) chest pain may be clinically indistinguishable from ischemic myocardial pain - normal ECG + no response to nitroglycerin rules out myocardial pain c) dec excursion of involved hemithorax, palpable or audible pleural friction rub, pleural effusion, and even localized tenderness d) signs - dullness to percussion and diminished breath sounds 3) acute embolism without infarction - a) nonspecific physical signs that may easily be secondary to another disease b) tachypnea + tachycardia frequent, pleuritic pain may be present, crackles in area of embolization 4) multiple pulmonary emboli or thrombi - a) physical signs of pulmonary hypertension and cor pulmonale b) may have inc JVP, RV heave, palpable impulse in LICS2, RV S3 gallop, systolic murmur over L sternal border louder during inspiration, hepatomegaly, ascites, pitting oedema c) findings not specific for PE and require a high index of suspicion for pursuing appropriate diagnostic studies NB/ prognosis dependent on size of thrombus & previous cardiorespiratory status

Describe the s/s and what's seen on physical exam in pts with femoral hernia

1) may have none in small-moderate hernias 2) large - may be more noticeable and can cause some discomfort a) bulge may be visible in groin area near upper thigh - bulging may become worse and can cause pain on standing, lifting heavy objects, or strain in any way b) are often located very close to the hip bone - may cause hip pain 3) severe symptoms: a) can signify that the hernia is obstructing intestines = strangulation - causes intestinal & bowel tissue to die b) severe stomach pain c) sudden groin pain d) N+V NB/ femoral ring is tight, so strangulation is likely 4) neck of hernia (bulge) felt inferior & lateral to pubic tubercle (attachment of inguinal ligament) - below inguinal ligament NB/ DDx for lump in groin includes: LN, femoral artery aneurysm, saphena varix, hernia, lipoma, psoas abscess

Discuss the various ways of measuring ill health

1) measures of health status - a) signs - BP, temperature, X-ray, tumour size b) symptoms - disease specific checklists c) co-morbidity - Charlson Index, ICED- index of co-existing disease (looks at both disease severity and functional severity), adverse events - pain, bleeding, readmission, complications (e.g. using Clavien-Dindo Classification of Surgical Complications) 2) measures of quality of life - a) QOL is a measure of the difference between the hopes and expectations of the individual and the individual's present experience. Health-related QOL is primarily concerned with those factors which fall within the spheres of influence of health care providers and health care systems b) measured by eg/ asking the patient directly or through various instruments c) HRQoL measures can be combined with measures of time in a particular health state, to form Quality Adjusted Life Years (QALYs) I) generic tools for measuring HRQoL include: short form (SF)-36, WHOQOL, EuroQoL (EQ5D), Nottingham health profile (NHP), Sickness Impact Profile (SIP) d) disease specific tools include: I) Asthma Quality of Life Questionnaire II) Mandatory PROMS from April 2009 - PROMS measures for hip and knee replacement and varicose veins mental health measures, such as the Warwick Edinburgh score

Outline the immediate 'first aid' treatment of a patient having a generalised seizure plus the drugs used to control an acute seizure

1) meds - a) 1st line - IV or rectal benzodiazepine + supportive care: lorazepam: 0.1 mg/kg followed by a second dose in 10-15 minutes only if necessary, or diazepam (rectal), or midazolam (intranasal) b) 2nd line - fosphenytoin/phenytoin + supportive care: I) fosphenytoin: 20 mg PE/kg IV as a loading dose, followed by a second dose of 5-10 mg PE/kg if necessary II) phenytoin: 20 mg/kg IV as a loading dose, followed by a second dose of 5-10 mg/kg only if necessary 2) first aid - a) protect them from injury (remove harmful objects from nearby) b) cushion their head c) look for an epilepsy identity card or identity jewellery - it may give you information about their seizures and what to do d) time how long the jerking lasts e) aid breathing by gently placing them in the recovery position once the jerking has stopped (see picture)

List common primary sites for metastatic tumour to the liver

1) men - stomach, lung, colon 2) women - breast, stomach, colon, uterus 3) liver mets common - in 40% of pts dying with cancer 4) with exception of CRC deposits almost always multiple & not amenable to resection 5) most common form of cancer in liver is secondary - spread through to peritoneum & result in malignant ascites 6) 8-10% pts undergoing curative resection of CRC have isolated liver mets amenable to resection 50% have mets at time of diagnosis; most of rest develop mets over 3 years 7) without surgical resection, 5yr survival is 0, compared with 30% with resection 8) fit healthy person will regenerate a 75% liver resection within 3 months

List the common sites of non-pulmonary TB infection and outline the pathological features of non-pulmonary TB including involvement of meninges, kidneys, spine, peripheral joints, gastrointestinal tract, pericardium & adrenal glands

1) meninges - fever, headache, vomiting, abdominal pain, drowsiness, meningism, delirium, seizures, tremor, papilloedema, cranial nerve palsies - LP + TB PCR stage 1 = meningism, stage 2 = hemiparesis or focal neurological signs, stage 3 = coma 2) kidneys - dysuria, frequency, loin/back pain, haematuria, sterile pyuria - EMUs +/-renal US, nocturia. May mimic tumour in testes or present as infertility in women. May be some calcification 3) spine - look for vertebral collapse + Pott's vertebra (chronic back pain, may present without typical fever & night sweats) 4) peripheral joints - long weight-bearing bones usually affected - knee or hip arthritis is relatively common, presents as cold abscess (15%), with swelling + mild erythema and pain - may be misdiagnosed as a tumor, earliest manifestation is pain - may precede signs of inflammation for weeks or months, established through arthrocentesis and mycobacterial culture; however, synovial biopsy is often needed, gives neurological sign (extradural collection), psoas spasm, should be suspected in monoarthritis in elderly 5) GI tract - most commonly ileocaecum, may simulate appendicitis, obstruction or Crohn's, though ascites and RIF mass common. Abdominal pain, GI upset - look for AFB in ascites, laparotomy may be needed 6) pericardium - acute (like primary exudative allergic lesion), chronic + constrictive pericarditis (associated with AF, elevated JVP, Kussmaul's sign, pulsus paradoxus) - fibrosis/calcification - may have spread to pericardium, fluid shows lymphocyte rich exudate 7) adrenal glands - can mimic Addison's disease, insidious onset of symptoms or during an adrenal crisis (Tb can = hypoadrenalism through destruction of glands), normal signs/symptoms of Addison's, tests - low basal levels of cortisol, low ACTH in plasma and poor response to rapid ACTH stimulation tests, NB/ TB = dec CD4, Hb, albumin & Na+ Miliary disease - major symptoms are B symptoms (fever, night sweats, weight loss), alongside meningism. In cryptic cases there will be no xray change. Miliary TB should always be on DDx for pyrexia of unknown origin

List the important contra-indications to an MRI scan

1) metallic implants eg/ vascular clips, foreign bodies (metal), artery stents, prosthetic heart valves and annuloplasty rings, cardiac occlude devices, vena cava filters and embolization coils, haemodynamic monitoring and pacing devices, permanent contraception devices 2) tattoos and cosmetics 3) claustrophobia 4) pregnancy & postpartum 5) renal insufficiency - contrast

Define and discuss transluminal angioplasty as used in coronary, visceral and peripheral vascular arterial beds. List the indications for pulmonary arteriography

1) minimally invasive endovascular procedure to widen narrowed or obstructed arteries or veins, typically to treat arterial atherosclerosis 2) a deflated balloon attached to a catheter is passed over a guide-wire into the narrowed vessel and then inflated to a fixed size. The balloon forces expansion of the blood vessel and the surrounding muscular wall, allowing an improved blood flow. A stent may be inserted at the time of ballooning to ensure the vessel remains open, and the balloon is then deflated and withdrawn 3) coronary angioplasty - therapeutic procedure to treat stenotic coronary arteries of the heart found in coronary heart disease 4) peripheral angioplasty - open a blood vessel outside the coronary arteries eg/ atherosclerotic narrowings of the abdomen, leg and renal arteries caused by peripheral artery disease. Often, peripheral angioplasty is used in conjunction with guide wire, peripheral stenting and an atherectomy 5) venous angioplasty - angioplasty is occasionally used to treat venous stenosis, such as stenosis of the subclavian vein caused by thoracic outlet syndrome 6) indications for pulmonary arteriography - a) diagnosis and treatment of PE in certain circumstances b) treatment of pulmonary arteriovenous malformations or fistulas c) treatment of iatrogenic or postinfectious pseudoaneurysm d) indicated as part of a complete embolization for severe cavitary or inflammatory lung lesions e) evaluation of the pulmonary arteries for tumor encasement f) retrieval of foreign objects eg/ embolized catheter fragments or IVC filters

Describe the morphology and pathological complications of infective endocarditis

1) morphology - a) microscopic appearance - vegetations contain no fibroblasts develop rapidly, with no evidence of repair. Large amounts of polymorphonuclear leukocytes & organisms are present in an ever-expanding area of necrosis. This process rapidly produces spontaneous rupture of the leaflets, of the papillary muscles, and of the chordae tendineae b) 2D echocardiogram (long-axis view) can display vegetations on the valve. Infection occurs along the edge of the heart valves, which is generally where damage takes place. Endocardial damage allows for adhesion should there be a bacteraemia c) once attached, bacteria may become encased in a fibrin/platelet mesh, allowing for rapid and protected multiplication d) in atrial lesions vegetations appear on atrial side, in ventricular on ventricular side (of the valve) 2) pathology - a) valve destruction with heart failure, embolic disease, glomerulonephritis b) acute: virulent pyogenic bacteria adhere to valve, regardless of structural abnormality. Bacteria proliferate & form large vegetations in combination with fibrin, platelets + leucocytes. Vegetations are large and crumbly, and locally cause rapid degeneration of cusps and chordae, while spreading to adjacent heart muscle - leading to acute heart failure c) subacute: less virulent organisms, structurally abnormal valve (eg/ post-Rheumatic fever). Large, secure vegetations formed, fairly dense with few leucocytes. Gradually inc in size to damage the cusps, causing gradual heart failure. d) systemic effects: pyrexia, leucocytosis, and embolism. Emboli may be cast off to brain & kidney to cause infarction and/or pyaemic abscesses e) glomerulonephritis due to immune complexes becoming trapped in the kidney

Outline the morphology and pathological consequences of bronchiectasis

1) morphology - marked dilated affected bronchi & bronchioles - long tube like = cylindrical bronchiectasis, or fusiform or saccular a) suppurative yellow-green or haemorrhagic exudate fill lumen with necrotic black oedematous ulcerated mucosa b) pseudo-stratified columnar or squamous metaplasia of remaining epithelium c) patchy emphysema & atelectasis in lung parenchyma d) may progress to lung abscess or extend to pleura - suppurative pleuritic 2) pathological consequences: a) dilation & thickening of bronchi due to chronic inflammation elicited by host response to micro-organisms colonising airways b) persistent airway inflammation leads to development of bronchial wall oedema & inc mucus production c) inflammatory cells eg/ neutrophils + T lymphocytes recruited to airways & subsequently release inflammatory cytokines, proteases + reactive oxygen mediators implicated in progressive destruction of airways d) initial insult to airways by primary infection leads to inc inflammation = bronchial damage, and serves as a nidus for subsequent colonisation of the airways - vicious cycle ensues that predisposes to persistent bacterial colonisation & subsequent chronic inflammatory reaction that eventually leads to progressive airway damage and recurrent infections e) factors predisposing those with an initial infection to develop bronchiectasis remain unclear 3) systemic complications - cerebral abscess = suppuration into pulmonary vasculature & seeding, development of systemic amyloid disorder = amyloid impaction in glomerular capillaries & tubule basement membranes, and lastly cor pulmonale & diabetes 4) mucociliary transport mechanisms damaged & mucostasis occurs = frequent bacterial infections 5) continuous inflammation = squamous metaplasia + loss of cilia

Describe the morphology and pathological consequences of Crohn's disease and ulcerative colitis

1) morphology of UC - continuous pattern of changes, beginning with the rectum and progressing proximally, terminating anywhere in the colon up to the ileocecal valve. Mucosa may be hyperemic and nodular and may exhibit fine granularity. Erosions and ulcerations may be seen. Ulcers are typically broad-based and may be linear or geographic; blood, pus, or mucus may overlie the mucosa. Colon is diffusely involved up to the point where the mucosa transitions to normal regions. Generally, submucosa, muscularis propria, and serosa are normal, because UC is not a transmural process and remains limited to the mucosa. Rather than cobblestones, the surviving patches of mucosa form pseudo-polyps 2) morphology of Crohn's - rigid thickening of involved segment of bowel wall. Hyperplasia of subserosal and mesenteric adipose tissue, leading to the appearance of "creeping fat". Strictures, especially single, long segment strictures in the terminal ileum (leading to the string sign of Kantor on CT scan). Patchy lesional distribution with sharply delineated areas of disease surrounded by normal mucosa. Cobblestone change of the mucosa. Deep ulcers, vertical fissures, and fistula tracts are commonly seen. Skip lesions. Granulomas are present in 2/3rds of cases 3) pathology of both - idiopathic, chronic, relapsing, inflammatory conditions that are immunologically mediated. Exact aetiologies remain uncertain, but though to be heterogeneous diseases characterized by various genetic abnormalities that lead to overly aggressive T-cell responses to a subset of commensal enteric bacteria. Onset and reactivation of disease triggered by environmental factors that transiently break the mucosal barrier, stimulate immune responses or alter the balance between beneficial and pathogenic enteric bacteria

Discuss the role of anal crypts in perianal infection

1) most anorectal abscesses result from infections of the anal glands (cryptoglandular infections) 2) anal canal has 6-14 glands that lie in the plane between the internal and external anal sphincters, ducts from these glands pass through the internal sphincters and drain into the anal crypts at the dentate line 3) glands may become infected when a crypt is occluded by impaction of food matter, by oedema from trauma secondary to a hard stool or foreign body, or as a result of an adjacent inflammatory process such as Crohn's disease

List the three most common adult primary brain tumours and outline their prognosis

1) most common brain tumors are gliomas - begin in the glial (supportive) tissue. Several types of gliomas, including: a) astrocytomas - arise from small, star-shaped cells called astrocytes. May grow anywhere in the brain or spinal cord. Most often arise in the cerebrum. Grade IV astrocytoma = glioblastoma multiforme I) prognosis - low grade (grade 2) = 40% 10 yr survival rate, most live 7 yrs after surgery; high grade (grade 3) = 27% 5 yr survival; glioblastoma (grade 4) = 5% 5 yr, 25% 1 yr b) oligodendrogliomas - arise in the cells that produce myelin. Usually arise in the cerebrum. Grow slowly and usually do not spread into surrounding brain tissue I) prognosis - low grade (grade 2) = 75% 5 yr survival rate, high grade/anaplastic (grade 3) = 35% 2) meningiomas - grow from the meninges. Usually benign. Grow very slowly, so brain may be able to adjust to their presence; meningiomas may grow quite large before they cause symptoms. Occur most often in women between 30 and 50 years of age a) prognosis - low grade (grade 1) = >80% 5 yr survival rate, high grade (grade 3) = <60% 3) others - a) germ cell tumors - arise from primitive (developing) sex cells, or germ cells. Most frequent type is a germinoma b) pineal region tumors - occur in or around the pineal gland. Can be slow growing (pineocytoma) or fast growing (pineoblastoma) c) schwannomas - benign tumors that arise from Schwann cells, which produce myelin. Acoustic neuromas are a type of schwannoma. Affect women twice as often as men

Describe the nature of a Meckel's diverticulum and its possible pathological effects

1) most common congenital abnormality of GI tract - remnant of the yolk stalk 2) affects 2-3% of population 3) diverticulum (pouch) projects from ileal wall ~60cm from ileocecal valve, pouch usually 3-6cm long & always appears on antimesenteric border of ileum 4) usually asymptomatic, but 50% contain gastric mucosa capable of HCl secretion - may cause peptic ulceration & bleeding 5) acute inflammation of diverticulum clinically indistinguishable from appendicitis 6) wide communication with bowel precludes stasis & inflammation - if diverticulum is still in contact with anterior abdominal wall via vitello-intestinal duct, may predispose to SI volvulus 7) Meckel's diverticulum may be displayed using gamma camera scan after IV administration of [99mTc] pertechnetate - has affinity for gastric mucosa 8) may be surgically removed, often laproscopically

Describe the clinical presentation of a subarachnoid haemorrhage, with specific reference to features in the history and examination including the rate of onset of symptoms, and signs arising from the event

1) most common premonitory symptoms: a) headache + dizziness + orbital pain 2) classic presentation: a) sudden onset of severe headache (classic feature) b) accompanying N+V c) symptoms of meningeal irritation - neck stiffness, Kernig's sign, photophobia d) visual changes e) focal neurologic deficits f) seizures + sudden loss of consciousness at ictus 3) physical exam may be normal or include: a) BP + temp + pulse inc b) papilledema c) retinal hemorrhage d) global or focal neurologic abnormalities e) may mimic MI on ECG due to adrenaline release f) Brudzinski's signs - flexion of elbow if pressure to cheek, flexion of hip if flexion of neck, abduction + flexion of hip if pubic symphysis pressure NB/ most commonly - saccular, berry aneurysms (70%) AV malformations (10%) NB2/ can lead to coma and death

Discuss the management of trauma to the bladder (both accidental and surgical)

1) most extraperitoneal bladder leaks can be effectively managed with maximal bladder drainage per urethral or suprapubic catheter - bladder drained for 10-14 days, then assessed for healing via cystogram - will heal naturally over this time 2) intraperitoneal bladder rupture - need surgery to repair 3) extraperitoneal extravasation - surgical repair 4) in any trauma setting follow ATLS as needed; with the patient stabilized in anticipation of surgical intervention, broad-spectrum abx administered.

Describe the pathological lesions of multiple sclerosis, the common sites of involvement in the nervous system and outline the pathogenesis of the disease

1) multiple sclerosis (MS) - inflammatory demyelinating disease characterised by episodic neurological dysfunction in at least 2 areas of the central nervous system (brain, spinal cord, and optic nerves) separated in time and space 2) substantial evidence supporting both grey and white matter involvement, appears to have both inflammatory and degenerative components that may be triggered by an environmental factor or factors in persons who are genetically susceptible 3) 20-40x more common in first-degree relatives, genes in the human leukocyte antigen (HLA) region and interleukin region are likely to be involved 4) environmental factors postulated to be involved in MS include toxins, viral exposures, and sunlight 5) relapses sometimes triggered by infections or postnatal hormonal changes. Surgical procedures may also trigger relapses 6) precise pathogenesis of MS is unknown - no specific or sensitive antigen or antibody, and there is some debate if MS represents a single disease or a syndrome of pathogenically heterogeneous patient subgroups 7) during the initial stage of the inflammatory phase, lymphocytes with encephalitogenic potential are activated in the periphery by factors such as infection or other metabolic stress. These activated T cells seek entry into the CNS via attachment to a receptor on endothelial cells. This interaction, mediated by production of matrix metalloproteinases, allows a breach in the blood-brain barrier, leading to further upregulation of endothelial adhesion molecules and additional influx of inflammatory cells. The T cells produce inflammatory cytokines that cause direct toxicity and also attract macrophages that contribute to demyelination. Lesions thus develop eg/ in periventricles of brain 8) demyelination disrupts axonal support and leads to destabilisation of axonal membrane potentials, which causes distal and retrograde degeneration over time 9) pathologically, MS is characterised by multifocal areas of demyelination, loss of oligodendrocytes, and astrogliosis with loss of axons primarily in the white matter of the CNS, although cortical lesions may also play a significant role

Outline the clinical management of sickle cell crisis and the importance of sickle cell

1) mx - a) vaso-occlusive crisis - I) 1st line - analgesia eg/ paracetamol, ibuprofen, ketorolac (IM or IV), naproxen, codeine, oxycodone, morphine II) +/- antihistamine eg/ diphenhydramine (pruritis) III) + supportive care + correction of cause eg/ O2, correcting acidosis etc IV) +/- hydration - IV or oral depending on level of dehydration V) +/- abx if infection VI) +/- blood transfusion (simple or exchange) - for life-threatening vaso-occlusive events, symptomatic anaemia, acute organ dysfunction, high-risk procedures (including general anaesthesia), and in selected pregnancies b) acute chest syndrome - I) 1st line - O2 + incentive spirometry II) + analgesia III) +/- antihistamine IV) + abx if infection V) +/- blood transfusion VI) +/- hydration 2) importance of sickle cell - a) sickling occurs when RBC containing sickle Hb become rigid and distorted into a crescent shape. In sickle cell anaemia, valine replaces glutamic acid at the sixth amino acid of the beta globin chain, as a result of a recessive single gene mutation b) polymerisation of sickle Hb in RBCs can be triggered by hypoxia and acidosis, which causes cells to become rigid and deform into a sickle (crescent) shape. These deformed cells may cause vaso-occlusion in the small vessels or adhere to vascular endothelium, resulting in intimal hyperplasia in larger vessels and slowing blood flow. These deformed cells are also prone to haemolysis, which contributes to anaemia. c) known precipitants include acidosis, dehydration, cold temperatures, extreme exercise, stress, and infections d) when anaemia is severe, high blood flow may result in cardiomegaly. Splenic sequestration or temporary bone marrow aplasia can cause circulatory failure and become life-threatening in children. From an early age, splenic dysfunction increases vulnerability to serious infections.

Describe the clinical presentation of myasthenia gravis and outline the immunological basis of disease

1) myasthenia gravis (MG) is a disorder of neuromuscular transmission a) results from binding of autoantibodies to components of the neuromuscular junction, most commonly the acetylcholine receptor, leading to muscular weakness with easy 'fatiguability', worse on exercise and improves with rest b) in most patients, IgG1-dominant antibodies to acetylcholine receptors cause fatigable weakness of skeletal muscles. In the rest, a variable proportion possesses antibodies to MuSK c) can sometimes be inherited, although there is a predisposition for autoimmune diseases to run in families 2) s/s - a) almost all will have ocular manifestations at some point during the course of their disease b) clinical presentation varies from mild weakness of limited muscle groups (class I, or ocular, MG) to severe weakness of multiple muscle groups (class V, or severe generalised, MG) c) muscle fatigues more readily after exercise I) eg/ get pt to count up to 50. As the patient nears 50 their voice becomes less audible as they are fatiguing II) drooped upper eyelids is typical with weakness of external ocular muscles producing diplopia III) weakness is more marked in proximal muscles and isolated weakness of limb muscles is the presenting feature in a minority of patients IV) weakness of the following muscles may also be seen: small muscles of the hands (finger extensors), deltoid and triceps, bulbar muscles (nasal & slurred speech), facial muscles (abnormal horizontal smile with a furrowed brow that compensates for ptosis), muscles involved in chewing V) symmetrical weakness of a number of other muscles may produce difficulty with walking, sitting or even holding the head up d) no muscle wasting or fasciculation. Tone is normal. Sensation is unimpaired and tendon reflexes normal e) seizures may occur

Outline the clinical features and differential diagnoses of myelofibrosis, polycythaemia rubra vera and essential thrombocythaemia

1) myelofibrosis - a) s/s - I) risk factors eg/ hx of exposure to radiation and industrial solvents II) constitutional symptoms - weight loss, night sweats, low-grade fever, cachexia, fatigue, and pruritus III) splenomegaly ± hepatomegaly b) dd - polycythaemia vera, essential thrombocythaemia, chronic myelogenous leukaemia, myelodysplastic syndrome (MDS), Hodgkins's and non-Hodgkin's lymphoma 2) polycythaemia rubra vera - a) s/s - I) risk factors eg/ age >40, Budd-Chiari syndrome and affected family members II) features of thrombosi - stroke, MI, PE, superficial thrombophlebitis, or DVT III) others - features of haemorrhage, asymptomatic, headache, generalised weakness/fatigue, pruritus, erythromelalgia (tenderness or painful burning and/or redness of fingers, palms, heels, or toes), splenomegaly b) dd - secondary polycythaemia owing to hypoxia or inc erythropoietin, or inc testosterone, essential thrombocythaemia, chronic myelogenous leukaemia (CML), congenital polycythaemia 3) essential thrombocythaemia - a) s/s - I) erythromelalgia - burning pain and dusky congestion of extremities, pain of erythromelalgia inc with exposure to heat and improves with cold II) splenomegaly III) arterial and venous thrombosis - stroke, TIAs, retinal artery or venous occlusions, coronary artery occlusion, PE, hepatic or portal vein thrombosis, DVT, and digital ischaemia (Raynaud's phenomenon) IV) bleeding V) livedo reticularis - purplish mottled discoloration of skin, usually on legs, described as lacy or net-like in appearance, seen in several CT diseases eg/ lupus, antiphospholipid syndrome, or Sneddon's syndrome VI) others- age 50-70, female, no symptoms, headache, dizziness, lightheadedness, and paraesthesias b) dd - secondary (reactive) or spurious thrombocytosis, familial essential thrombocythaemia, myelodysplastic syndrome (MDS), polycythaemia vera, primary myelofibrosis, CML

Distinguish between myeloid and lymphoid cell lineages in the classification of acute leukaemia; highlight differences between childhood and adult-onset leukaemia

1) myeloid: structure that originated in the bone marrow - refers to cancers relating to the bone marrow and blood 2) lymphoid: originates in lymph or lymphatic system - refers to cancers relating to the lymphatic tissues + bone marrow 3) children vs adults - a) ALL more common in children, AML more common in adults b) adult cancers typically develop from epithelial tissue (adenocarcinomas), which come from the ectoderm or endoderm. Childhood cancers develop from the mesoderm (primitive cells or embryonic cells) c) childhood cancers develop either because of genetics or an event in early fetal development, whereas adult cancers typically result from an accumulation of mutations caused by external sources, such as smoking or sun overexposure

List the differential diagnoses of MS

1) myelopathy due to cervical spondylosis 2) fibromyalgia 3) postural orthostatic tachycardia syndrome with or without cervicogenic migraine 4) sleep disorders 5) Sjogren's syndrome 6) vitamin b12 deficiency 7) ischaemic stroke 8) peripheral neuropathy 9) lymphoma 10) inherited disorders such as mitochondrial diseases and leukodystrophies 11) Guillain-Barre syndrome 12) amyotrophic lateral sclerosis (ALS) 13) systemic lupus erythematosus (SLE)

Discuss the difference between wants, demands and needs for health care

1) need - necessity, something required to survive or to sustain eg/ food, water, shelter etc 2) want - desires, things without which we can survive, but we need them for higher satisfaction 3) demand - willing and able to buy

Contrast the indications for surgical and medical treatment of acute arterial occlusion

1) need surgery if have s/s of 5Ps (acute arterial occlusion) - severe pain, polar sensation (coldness), paresthesias (or anesthesias), pallor, and pulselessness 2) class I: viable - pain, No paralysis or sensory loss: anti-coagulation, angiography and elective revascularzation 3) class 2: threatened but salvageable - 2A: some sensory loss, no paralysis: anti-coagulation, angiography and elective revascularzation 4) 2B: sensory and motor loss: needs immediate treatment, early angiographic evaluation and intervention, exception: suspected common femoral emboli 5) class 3: non-viable - profound neurologic deficit, absent capillary flow, skin marbling, absent arterial& venous signal: amputation

By performing the requisite tasks and outlining them in written orders, demonstrate knowledge of the daily catheter care and maintenance necessary for preventing stricture formation and retrograde infection

1) need to perform catheter care every day including cleaning the catheter, changing the drain collection bags, and washing the drainage bags 2) can shower with catheter but avoid baths to dec risk of infection - clean catheter in shower: men should retract the foreskin & clean the area, including the penis, women should separate the labia, and clean the area from front to back, both need to clean the urethra & all along the catheter 3) change drainage bag twice a day, cleaning the catheter as you do 4) leg bags - always wear below the knee to help it drain 5) clean drainage bags when changing 6) preventing infection - a) keep the drainage bag below the level of your bladder and off the floor at all times b) keep the catheter secured to your thigh to prevent it from moving c) shower daily to keep the catheter clean d) clean your hands before and after touching the catheter or bag e) drink plenty 7) preventing strictures - change catheter regularly to stop strictures forming

Create a treatment plan including specification of observations, general supportive measures, appropriate antibiotic regimens, analgesia and physiotherapy

1) need tx within 4hrs of presenting at hospital 2) abx - a) CURB 65 0-1 = amoxicillin for 5/7 b) CURB 65 2 = amoxicillin + clarithromycin for 7-10/7 3) monitoring - regular obs 4) conservatove - stop smoking, rest, fluids 5) may need O2 for hypoxia; ventilation if severe hypoxia 6) analgesics - NSAIDs + paracetamol for mild pleuritic pain; more severe pain may require opiate analgesia but care is needed not to aggravate CO2 retention 7) neb saline may help expectoration 8) chest physio 9) chest x-ray should be performed 6 weeks after tx to ensure there's no underlying malignancy

Define the nephrotic syndrome and describe its relationship to conditions causing abnormal proteinuria

1) nephrotic syndrome - a) syndrome comprising signs of nephrosis, chiefly proteinuria, hypoalbuminemia, and oedema b) due to loss of selective permeability of glomeruli to plasma proteins (itself due to thickening), causing massive proteinuria (>3g/day) c) component of glomerulonephrosis d) proteinuria = low protein levels in blood (hypoproteinemia including hypoalbuminemia), so water drawn into soft tissues (oedema) e) severe hypoalbuminemia can cause a variety of secondary problems eg/ ascites, pericardial effusion, pleural effusion, hyperlipidemia, loss of molecules regulating coagulation (inc risk of thrombosis) f) pts may suffer more infections due to loss of antibodies in urine & hypercoagulability due to net loss of coagulation inhibitors (Proteins C and S). Urine is full of protein & frothy 2) persistent proteinuria - albumin:creatinine ratio >30mg/mmol or albumin concentration >200mg/l or urine protein:creatinine ratios >45 mg/mmol (2g protein for each 1g creatine), raised protein in urine on 2+ occasions 3) conditions - a) most common cause in children is minimal change disease b) in adults, primary glomerular diseases are more frequent in males (55%), whereas secondary glomerular disease is more frequent in females (72%) c) most common cause in younger adults is FSGS, followed by minimal change nephropathy. d) membranous nephropathy is the most common cause in older people I) secondary causes - hepatitis B, autoimmune disease, malignancy, and adverse drug reactions (including gold, penicillamine, and NSAIDs) e) can also be diabetic nephropathy in adults with a history of long-standing diabetes f) can also develop in patients with IgA nephropathy, membranoproliferative glomerulonephritis, and post-infectious glomerulonephritis, however pts usually have a nephritic pattern with haematuria and red cell casts as the predominant feature g) can also happen secondary to amyloidosis

List the classes of drugs used to treat angina, their mechanisms of actions, and their common side effects

1) nitrates most common - vasodilators, 2-3 mins. SE = headaches, dizziness/light-headedness, flushing or a warm feeling in the face 2) B-blockers - dec heart work load so dec likelihood of ischaemia & symptoms. SE = dizziness, tiredness, blurred vision, cold hands and feet, slow heartbeat. CI - asthma + COPD 3) calcium channel blockers - prevent calcium entering heart muscle cells + SM cells causing them to relax. Rate limiting eg/ verapamil and diltiazem inhibit conduction through AVN = bradycardia, dihydropyridine blockers dec contractility but may cause reflex tachycardia (use with B-blockers). SE = headache, dizziness, flushing, foot and ankle swelling. Verapamil tends to cause constipation 4) ACEI - dec BP. SE = dizziness, tiredness or weakness, persistent, dry cough 5) oral antiplatelets eg/ aspirin - prevent blood clots. SE uncommon, but can include irritation of stomach or bowel, indigestion and feeling sick 6) anticoagulants (blood thinners) eg/ warfarin - prevent blood clots. SE = excess bleeding, diarrhoea or constipation, N+V 7) potassium channel agonists eg/ Nicorandil - act similarly to nitrates to reverse angina pain. SE = headaches, flushing and reflex tachycardia

Outline the clinical indications for screening for thrombophilia and how this is done

1) no definitive guidelines for screening - screening of general population not recommended 2) may be appropriate in selected groups eg/ family members with a known familial thrombophilia, or at high risk for venous thromboembolism (VTE) eg/ pregnant women, or about to start oral contraceptive pills or oestrogen therapy; patients on chemotherapy; patients who are to undergo high-risk surgery; patient with thrombosis <50 yo with no obvious precipitant; women with recurrent miscarriages 3) if performed, genetic counselling should be provided before the screening tests to enable informed consent and explanation of risks/benefits of the test 4) recommended that a full thrombophilia screen be done on patients who fall into the at risk categories already defined. This consists of : proteins C + S, antithrombin, APCR and Factor V Leiden, protein gene mutation, lupus anticoagulant

List and discuss the treatment options for breast cancer

1) node and HER2/neu-negative, invasive ductal, lobular, mixed, or metaplastic carcinoma a) 1st line - surgery: breast conserving (lumpectomy or wedge) or modified radical mastectomy or skin-sparing mastectomy I) + sentinel LN biopsy or axillary node dissection or completion axillary node dissection II) CI to breast-conserving surgery: multi-centric disease, diffuse mammographic micro-calcifications = suggestive of multi-centricity, hx of prior radiotherapy, continued positive margins despite reasonable re-excisions, and pregnancy b) + chemotherapy as an adjunct to surgery (anthracycline or non-anthracycline-based): I) FAC - fluorouracil + doxorubicin + cyclophosphamide II) FEC-100 - fluorouracil + epirubicin + cyclophosphamide III) CEF - cyclophosphamide + epirubicin + fluorouracil c) + radiotherapy - essential component of local therapy following breast-conserving surgery: 2) node +ve: surgery + chemo (more aggressive eg/ FEC followed by docetaxel) + radiotherapy 3) HER2/neu +ve: surgery + trastuzumab-based chemotherapy (eg/ doxorubicin + cyclophosphamide + docetaxel + trastuzumab) + radiotherapy 4) oestrogen receptor (OR)-positive and/or progesterone receptor (PR)-positive - a) no previous ongoing tx: pre-menopausal - I) 1st line - endocrine therapy or oophorectomy: tamoxifen or oophorectomy or goserelin NB/ endocrine therapy after chemo (if warranted). Tamoxifen for 5 years if pt remains pre-menopausal during the 5-year period. If the patient is confirmed to be post-menopausal at year 2 or year 3, she may switch to aromatase-inhibitor therapy for the remaining years of the 5-year period II) + osteoporosis prophylaxis: weight bearing exercise + calcium carbonate + ergocalciferol +/- bisphosphates (eg/ alendronic acid) b) no previous ongoing treatment: post-menopausal - Primary options I) 1st line - endocrine (hormone) therapy: anastrozole or exemestane or letrozole, secondary: tamoxifen NB/ treatment should continue for 5 years II) + osteoporosis prophylaxis +/- bisphosphonates or denosumab c) completed 5 years of tamoxifen (or 2-3 yrs): now confirmed post-menopausal - Primary options I) 1st line - endocrine (aromatase-inhibitor) therapy: exemestane or anastrozole or letrozole II) + osteoporosis prophylaxis +/- bisphosphonates or denosumab

Describe the management of a suspected cord syndrome

1) nonsurgical a) admission to ICU after initial injury b) early immobilization of the cervical spine with a neck collar for ~6 weeks until the individual experiences a reduction in pain and neurological symptoms c) inpatient rehabilitation is initiated in hospital, followed by outpatient physical therapy and occupational therapy to assist with recovery d) autonomic dysreflexia - proper medical management of the skin, bowel, and bladder should prevent most occurrences. If mechanical means do not resolve the syndrome, medical management needed to dec BP eg/ nifedipine and transdermal nitroglycerin e) neurogenic bladder - insertion of Foley catheter for drainage. Later need bladder training f) spasticity - proper bed positioning, regular stretching program, can try baclofen g) neuropathic pain - removal of possible exacerbating factors (eg/ infections, new pressure ulcers), then medication h) neurogenic bowel - pts started on a regular bowel program to avoid incontinence. Also need adequate fluid intake to avoid constipation/faecal impaction. The use of evacuants and/or manual removal by way of digital stimulation or other methods should be instituted 2) surgical - usually given to those with inc instability of their cervical spine, which cannot be resolved by conservative management alone. Further indications for surgery include a neurological decline in spinal cord function in stable patients, and those who require cervical spinal decompression

Contrast changes in liver function tests in obstructive and hepatocellular jaundice

1) obstructive jaundice - cholestasis: inc ALP & GGT 2) hepatocellular jaundice - AST & ALT inc, ALT is more specific for liver damage than AST, AST: ALT =1 (associated with CCF and ischaemic necrosis and hepatitis), AST: ALT >2.5 (associated with Alcoholic hepatitis), AST: ALT <1 (specific for Hepatocellular damage, viral hepatitis, ischaemic necrosis, toxic hepatitis) 3) radiology in obstructive jaundice - a) US - 1st line for assessment of jaundice. Effective in diagnosing biliary dilatation, can also help characterise lesions and determine the level of obstruction b) MRCP (MR cholangiopancreatography) - specific type of MRI of abdomen. Looks specifically at the fluid within the biliary tree and is very helpful when determining the level of biliary obstruction c) CT - used for the assessment of extrahepatic mass lesions that cause biliary obstruction and also to determine if there is distant spread in suspected malignancy 4) endoscopy in jaundice - Endoscopic Retrograde Cholangiopancreatography (ERCP) is a common investigative procedure that uses a side-viewing duodenoscope to view from the mouth to the duodenum. Here instruments are passed down the tube to the ampulla of Vater (where the common bile duct empties into the duodenum). Contrast material is injected into the biliary tree and pancreatic ducts so they can be viewed by the physician and x-rays taken. ERCP is used primarily in the diagnosis and management of bile duct stones, and other conditions such as strictures, leaks and malignancies eg/ can be used to stage pancreatic cancer

Outline the possible long-term complications of GORD

1) oesophagitis/ulcer 2) anaemia 3) oesophageal stricture 4) Barrett's oesophagus (risk related to length of time with GORD)

Discuss the microvascular complications of diabetes affecting the kidneys and outline their relationship to diabetic control and disease duration

1) often due to glomerular disease, manifesting 15-25 years after diagnosis (affects 30% pts diagnosed <30) - leading cause of premature death in young diabetic pts a) due to ischaemia resulting from hypertrophy of afferent & efferent arterioles or b) due to ascending infection 2) diabetic kidney hypertrophies - associated with inc GFR 3) as kidney becomes damaged afferent arteriole becomes vasodilated more than efferent = inc intraglomerular filtration pressure, leading to damage to glomerular capillaries 4) inc in pressure also leads to shearing forces, contributing to mesangial cell hypertrophy & deposition of matrix material 5) eventually - sclerosis + thickening in basement membrane (hyaline nodules become present = "Kimmelstiel-Wilson lesions") 6) progressive leak of large molecules into urine - earliest evidence = microalbuminuria, undetectable to regular dipsticks - predictive marker of progression to nephropathy in T1DM and of CV risk in T2DM 7) intermittent proteinuria leads to persistent proteinuria, at which point pts are 5-10 years from end-stage renal failure 8) proteinuria may produce transient nephrotic syndrome - pts show a normochromic, normocytic anaemia +raised ESR, rise in plasma creatinine is a late finding that progresses with decline in GFR

Discuss the differences between closed suction and open drains

1) open drains drain fluid on to a gauze pad or into a stoma bag. They are likely to increase the risk of infection. 2) closed drains are formed by tubes draining into a bag or bottle eg/ chest, abdominal and orthopaedic drains. Generally, the risk of infection is reduced

In an exsanguinating patient who has received a massive blood transfusion, identify the acute aetiological factors that may be responsible for the bleeding disorder

1) open wound 2) thrombocytopenia 3) haemophilia A or B (Christmas disease) 4) factor II or V or VII or X deficiency 5) hypovolaemic shock 6) fracture including skull fracture and hip fracture 7) certain animal bites 8) DIC 9) bleeding oesophageal varices 10) peptic ulcer 11) dissection of the aorta 12) IBD 13) Von Willebrand Disease or acquired platelet function disorder 14) Goodpasture syndrome 15) intussusception 16) multiple myeloma 17) dengue fever, ebola, yellow fever 18) less likely - haemorrhoids (aka piles), menorrhagia, threatened miscarriage, dysfunctional uterine bleeding

Describe the treatment of hypoglycaemia

1) oral sugar + long-lasting starch (eg/ toast) 2) if can't swallow - 25-50ml 50% glucose IV (dextrose) or glucagon IM 1mg if no IV access 3) if episodes are often advise small high-starch meals 4) if post-prandial glucose dec give slowly absorbed carb (high fibre) 5) diabetics - rationalise insulin therapy

List the organs that are commonly transplanted and outline the main indications for transplantation

1) organs - a) chest - heart, lung b) abdomen - kidney (most common), liver, pancreas, intestine, stomach, testis, penis 2) often transplantation happen due to end state organ failure, such as liver and heart failure. Although end stage renal disease patients can be treated through other renal replacement therapies, kidney transplantation is generally accepted as the best treatment both for quality of life and cost effectiveness

Outline the principles of treatment of: immuno -suppression; symptomatic management and rehabilitation of spasticity, bladder problems, pain, sensory symptoms, weakness, fatigue and depression

1) other symptoms - a) with fatigue - + lifestyle modification and/or non-pharmacological therapies: I) most MS patients benefit from regular exercise programmes, including progressive resistance training II) good sleep hygiene practices should also be encouraged III) +/- amantadine or modafinil or armodafinil if other measures not working b) with urinary frequency - + lifestyle modification ± mind-body therapies ± medication: I) lifestyle - avoid caffeine and 'vitamin waters'; consider mind-body therapies, such as yoga and relaxation II) if not working consider - oxybutynin or tolterodine or solifenacin or darifenacin or fesoterodine or botulinum toxin type A III) presence of urinary tract infection should be excluded c) with sensory symptoms (pain and dysaesthesia) - + low-dose anticonvulsants: I) gabapentin or pregabalin or carbamazepine or oxcarbazepine d) with inc muscle tone (with or without spasms) - + physiotherapy ± antispasticity medications: I) gentle stretching II) meds - baclofen or tizanidine or clonazepam or gabapentin or botulinum toxin type A III) tertiary options - baclofen intrathecal e) with tremor - + medical therapy: I) propranolol or primidone or clonazepam f) with gait impairment - + physiotherapy and/or progressive resistance training (PRT) +/- fampridine g) depression - SSRIs, CBT 2) relapsing-remitting MS - a) 1st line - immunomodulators: I) interferon beta 1a 30 micrograms IV once weekly, or interferon beta 1b, or peginterferon beta 1a, or glatiramer or teriflunomide or dimethyl fumarate II) disease-modifying therapy should be offered to all patients with relapsing-remitting MS (RRMS), although some may have such a benign course or be in an age group where the benefits of disease-modifying therapy may be less clear 3) secondary-progressive MS - a) 1st line - methylprednisolone 4) primary progressive MS - a) 1st line - consideration for medical therapy: I) ocrelizumab (humanised anti-CD20 monoclonal similar to rituximab), others - cyclophosphamide, methotrexate, and azathioprine

Describe the common measures of health care used (structure process, outcome and quality measures)

1) outcome measures - these reflect the impact on the patient and demonstrate the end result of your improvement work and whether it has ultimately achieved the aim(s) set a) eg/ reduced mortality, reduced length of stay, reduced hospital acquired infections, adverse incidents or harm, reduced emergency admissions and improved patient experience 2) process measures - these reflect the way your systems and processes work to deliver the desired outcome a) eg/ length of time a patient waits for a senior clinical review, if a patient receives certain standards of care or not, if staff wash their hands, recording of incidents and acting on the findings and whether patients are kept informed of the delays when waiting for an appointment 3) structure measures - these reflect the attributes of the service/provider such as staff to patient ratios and operating times of the service. These are otherwise known as input measures

List the physiological limits of normal blood gases

1) pH - normal = 7.35 to 7.45 2) PaO2 - normal = 10.5 - 13.5 kPa (70 - 100 mmHg) 3) SaO2 - normal = >94% 4) PaCO2 - normal = 4.7 - 6.0 kPa (35 - 45 mmHg) 5) HCO3 - normal = 22 - 26 mEq/L 6) BE normal = -2 to +2 ml litre (-ve BE indicates a base deficit in the blood)

Outline the symptoms of patients with perianal infections

1) pain - usually constant, throbbing, and worse when sitting down 2) skin irritation around anus, including swelling, redness, and tenderness 3) discharge of pus 4) constipation or pain associated with bowel movements 5) deeper anal abscesses may also be associated with: fever, chills, malaise NB/ if infection results in abscess formation it's a surgical emergency, presenting with severe pain and swelling (often with a mass)

Acute coronary syndrome - s/s

1) palpitations 2) severe pain - crushing, squeezing, or burning sensation across precordium, may radiate to neck, shoulder, jaw, back, upper abdomen, or either arm - longer duration than angina pectoris 3) exertional dyspnea that resolves with rest + dec exercise tolerance 4) diaphoresis (sympathetic discharge) 5) nausea (vagal stimulation) 6) autonomic symptoms 7) elderly or long-term diabetes - accompanying symptoms may be absent 8) syncope/collapse may occur 9) symptoms - acute central chest pain lasting >20 mins, often associated with nausea, sweatiness, dyspnoea, palpitations 10) may be 'silent' (no chest pain) in elderly + diabetics - symptoms: syncope, pulmonary oedema, epigastric pain, vomiting 11) signs - a) distress/anxiety b) pallor + sweatiness + tachycardia = cardinal signs c) BP inc or dec d) intense or augmented 4th heart sound e) may be signs of heart failure - inc JVP, 3rd heart sound, basal crepitations f) may have pansystolic murmur - ECG essential g) may have fever h) later develop pericardial friction rub - pulmonary +/- peripheral oedema i) CXR may be normal or show features of pulmonary oedema j) widened mediastinum suggests aortic dissection NB/ Killip 1 = no crackles and no 3rd heart sound, Killip 2 = crackles in <50% of lung or 3rd heart sound, Killip 3 = crackles in >50% of lung fields, Killip 4 = cardiogenic shock

List common pancreatic neoplasms; describe the pathology of each with reference to cell type and function

1) pancreatic cancer is a disease of older people - peaks at 65-75 yo 2) locations - a) 65% tumours are located within head of pancrea b) 15% are in the body c) 10% are in the tail d) 10% are multifocal 3) LN metastases are common (50%), as well as perineural and vascular invasion 4) distant metastases - liver, lung, skin, and brain 5) 3 precursor lesions - pancreatic intraepithelial neoplasia (PanIN), intraductal papillary mucinous neoplasm, and mucinous cystic neoplasm 6) classification - a) tubular adenocarcinoma (most common form). b) adenosquamous carcinoma c) colloid (mucinous non-cystic) adenocarcinoma: almost always arises in association with intraductal papillary mucinous neoplasm and is characterised by well-differentiated neoplastic epithelial cells within large pools of mucin infiltrating stroma d) hepatoid carcinoma e) medullary carcinoma - poor differentiation with micro-satellite instability, a syncytial growth pattern, and pushing borders f) signet ring cell carcinoma g) undifferentiated carcinoma - highly malignant epithelial neoplasms without a more definite direction of differentiation, typically non-cohesive h) undifferentiated carcinoma with osteoclast-like giant cells - mix of atypical pleomorphic cells and multinucleated giant cells with uniform nuclei

Classify thyroid cancer and outline the clinical presentation, diagnosis and principles of treatment

1) papillary thyroid cancer (80%) - often young females - excellent prognosis. May occur in women with familial adenomatous polyposis or Cowden syndrome a) histology - small papillary structures (acini), uniform, optically clear (Orphan Annie) nuclei, and Psammoma bodies (concentric calcifications) 2) follicular thyroid cancer (15%) - occasionally in pts with Cowden syndrome, more common in women past middle age, good prognosis. a) present in 2 forms - minimally invasive + well encapsulated tumours + difficult to differentiate from normal thyroid tissue other than by invasion into vasculature, tumour of varying degrees of differentiation + spreads widely & invades venules - blood spread occurs to lungs + bone 3) medullary thyroid cancer (7%) - cancer of parafollicular cells, often part of multiple endocrine neoplasia type 2 - familial, poor prognosis (but course is indolent) a) cancers arise from calcitonin releasing C-cells - may cause high blood levels of calcitonin. May secrete other hormones = carcinoid or Cushing's syndrome 4) poorly differentiated thyroid cancer 5) anaplastic thyroid cancer (<5%) - not responsive to treatment and can cause pressure symptoms, often distant metastases, occur in elderly & may cause stridor 6) B cell lymphoma - sometimes in long term autoimmune thyroiditis, especially Hashimoto's 7) others - thyroid lymphoma, squamous cell thyroid carcinoma, sarcoma of thyroid clinical presentation & diagnosis of malignant tumours in the thyroid 1) clinical presentation - >75% in women, 90% present with goitre/thyroid nodule, painless, palpable, solitary thyroid nodule (hard & fixed) a) solitary nodules: most likely to be malignant in pts >60 & <30 years b) inc rate of malignancy in males c) nodular growth d) rapid growth = ominous sign e) sudden onset of pain associated with benign disease 2) diagnosis - a) history taking + physical exam (neck etc) b) labs c) fine-needle aspiration biopsy (FNAB) d) imaging can be adjuncts NB/ may have cervical lymphadenopathy, or lung/liver/brain/bone metastases NB2/ previous head/neck radiation, thyroid disease, & family history are risk factors NB3/ tumours rarely associated with hormone release, but 90% secrete thyroglobulin (can act as tumour marker)

List the causes of intestinal obstruction in children and outline the diagnostic tests most appropriate for each

1) paralytic ileus - a) plain film x-ray - may show signs of trapped gas and possibly the obstruction b) CT or US - identifies area of ileus 2) adhesions - CT or MRI 3) foreign bodies - x-ray, CT 4) hernias - CT 5) impacted stool - x-ray, sigmoidoscopy 6) intussusception - a) AXR - supine & upright: absence of air in RLQ + RUQ, RUQ soft tissue density b) US & CT - US = target & pseudokidney signs, AUS = classic target sign of intussusceptum inside intussuscipiens, CT = ying-yang sign 7) tumors blocking the intestines - CT or MRI 8) volvulus (twisted intestine) - a) AXR - intestinal obstruction, duodenal = double-bubble sign, upper GI series (gold standard), US (shows inversion of SMA & SMV), AXR of sigmoid volvulus = coffee bean sign

Outline the pathology and causation of the main muscular diseases

1) pathology - a) Duchenne muscular dystrophy - muscle degeneration due to lack of dystrophin protein, causing a disruption of the membrane covering the muscle fibre, causing entry of excess amounts of calcium ions into the cell and cell degeneration b) Becker MD - similar to DMD but some functional dystrophin is produced so less severe c) myasthenia gravis - autoimmune antibodies cause the destruction of acetylcholine receptors of the neuromuscular junction d) polymyositis - infiltration of the striated muscle by WBC, mainly lymphocytes, which collect between the muscle fibres and around small blood vessels and appear to damage the muscle fibres. Vascular damage also is a major feature, particularly in the childhood form of dermatomyositis e) myotonic diseases - slow relaxation of muscles may be due to delayed disengagement of the thick and thin filaments of myosin and actin, & due to continuing electrical activity of the sarcolemma (membrane of striated muscle fibres). A single nerve action potential causes multiple firing of the sarcolemma, thereby continuing muscular contraction 2) cause - a) Duchenne MD - X-linked recessive so only boys affected, females can be carriers I) s/s - boys, difficulty in walking from toddler age, loss of the ability to walk ~ 11 yo, and death before the age of 20, usually because of respiratory failure or pulmonary infections; have pseudohypertrophic calves).. b) Becker MD - different damage to the same gene on the X chromosome that causes Duchenne muscular dystrophy; less severe, live to be older c) Facioscapulohumeral muscular dystrophy - autosomal dominant inheritance, so pt gets gene from one parent and has 50% chance of passing the disease to his children I) s/s - starts in muscles of face, around the shoulder blades, and upper arms. Progresses more slowly than DMD, and normally have normal life-span. Leg weakness frequently causes "foot drop" and a waddling gait d) myasthenia gravis - acquired autoimmune disorder that involves a failure in the transmission of nerve impulses to the muscles I) s/s - persistent muscular weakness and a tendency of muscles to be easily fatigued, weakness, particularly of the face, limbs, and neck. Can have double vision, difficulty swallowing and breathing, and excessive muscle fatigue during exercise with partial recovery after rest e) toxic myopathies - damage to striated muscle eg/ IM bupivacaine causes damage to the muscle fibres by disrupting the membrane and allowing calcium to enter and destroy the cell; chloroquine disrupt the internal biochemistry of the muscle fibre; corticosteroids affect the muscle metabolism f) muscles can be invaded by bacteria, viruses, protozoa and helminths I) s/s - infection - fever, inc WCC, local inflammation, reddening, swelling, and warmth g) polymyositis - autoimmune muscle diseases of unknown cause I) s/s - inflammation of the skin eg/ eyelids, cheeks, knuckles, elbows, knees, and backs of the hands. Polymyositis + dermatitis = dermatomyositis. Muscle weakness can be proximal or diffuse. Frequently, swallowing is difficult and the neck is weak. Frequently occurs with other autoimmune diseases eg/ RA and progressive systemic sclerosis, and it can be associated with cancer h) myotonic diseases - difficulty in relaxing a muscle after contraction: due abnormal ion channels or ion pumps in the sarcolemma I) s/s - difficulty in relaxing the hand after a handshake, cold exacerbates, also have weakness which is more pronounced after inactivity, with a rapid "warm up" on commencing exercise II) eg/ myotonic dystrophy - most common myotonic disorder. Autosomal dominant disorder affecting many systems of the body in addition to muscle. Symptoms include premature balding, cataract formation, mental impairment, gonadal atrophy, endocrine deficiencies, gastrointestinal tract dysfunction, and muscle fibre degeneration. While the disease has manifested itself by the age of 25 years in most cases, some affected individuals may escape developing significant symptoms throughout their lives

Describe the pathology and natural history of oesophageal malignancy

1) pathology - a) superficial lesions appear as plaques or small ulcers, while advanced lesions are often deep ulcerations or fungating masses b) dysplasia - changes begin at base of epithelium (low-grade) and progress to surface (high-grade). Cytological features: large, dark staining nuclei with coarse chromatin (hyperchromasia), pleomorphism, loss of epithelial order, mitotic activity above the basal layer. Atypical cells trailing off into the lamina propria or deep aberrant keratinization are signs of invasion c) squamous cell carcinoma - polygonal cells with abundant eosinophilic (pink) cytoplasm, intercellular bridges, and variable amounts of keratinization. Nucleus is large, dark, and contains a prominent nucleolus d) frequency of lymphatic & blood vessel invasion incs with inc depth of invasion 2) natural history - a) adenocarcinoma: Barrett's oesophagus (chronic acid reflux) to dysplasia to adenocarcinoma. Occur in lower 3rd of oesophagus. Tumours metastasise early to LN b) squamous cell carcinoma: mid oesophagus at level of carina (may invade to cause fistula), less commonly at the level of cricoid cartilage (Plummer Vinson syndrome) c) tumours may narrow lumen or cause a polypoid mass. In both tumour grows proximally + distally in submucosal lymphatics -surgeon has to remove tumour with wide margins d) risk factors: smoking, high alcohol intake, diet low in fruit and fish, breast cancer radiotherapy, strictures, achalasia, obesity and celiac disease

Describe the pathology underlying coeliac disease and list clinical conditions that may be associated with it

1) pathology - a) loss of immune tolerance to gliadin peptide antigens derived from wheat, rye, barley, and related grains b) peptides are resistant to human proteases, so remain intact c) in the intestinal submucosa these peptides trigger both innate and adaptive immune activation d) in the submucosa, gluten peptides are de-amidated by tissue transglutaminase (tTG), which allows for binding to the coeliac-associated HLA peptides (DQ2 or DQ8) found on antigen-presenting cells, and second, activation of helper T (Th) cells e) cell death and tissue remodelling with villous atrophy and crypt hyperplasia are induced by Th1-derived cytotoxic T lymphocytes. Th2 triggers plasma cell maturation and subsequent anti-gliadin and anti-tTG antibody production 2) other conditions - T1DM, autoimmune thyroid disease

Outline current theories regarding the pathophysiology of IBS

1) pathology - dysfunction of motor & sensory aspects of digestive tract - a) altered gut reactivity (motility & secretion) in response to various stimuli, which may be environmental (personal life stresses or abuse) or luminal (certain foods, bacterial overgrowth or toxins, or gut distension or inflammation) b) altered reactivity leads to pain, constipation or diarrhoea c) gut hypersensitivity with enhanced perception of visceral type pain and sensations. d) dysregulation of brain-gut axis - may be associated with greater stress reactivity and with modulation or perception of afferent signals from the enteric nervous system

Outline the pathology, presentation and management of achalasia

1) pathology - oesophageal motor disorder of unknown aetiology, characterised by oesophageal aperistalsis and insufficient lower oesophageal sphincter (LOS) relaxation in response to swallowing a) results from loss of inhibitory nitrinergic neurons in the oesophageal myenteric plexus 2) s/s - a) dysphagia b) posturing to aid swallowing c) retrosternal pressure/pain d) regurgitation e) gradual weight loss 3) mx - a) surgery - pneumatic dilation or laparoscopic cardiomyotomy b) not a surgical candidate - I) meds - isosorbide dinitrate or nifedipine or verapamil II) injection of botulin toxin A III) gastrostomy

List the main clinical features of dementia including memory loss and global intellectual deterioration and relate these to different lobes of the cerebral hemisphere. Classify dementia according to cause

1) pathophysiology - a) Alzheimer's (~50%) - degeneration & atrophy of cerebral cortex, neurofibrillary tangles, amyloid plaques & dec Ach made from affected neurons, can be genetic b) vascular dementia (~25%) - brain damage due to cerebrovascular disease: major stroke, multiple smaller unrecognised strokes (multi-infarct) or chronic changes in smaller vessels (subcortical dementia) c) dementia with Lewy bodies (~15%) - deposition of abnormal protein within neurons in brain stem & neocortex d) frontotemporal dementia (<5%) - specific degeneration/atrophy of frontal & temporal lobes of brain eg/ Pick's disease (Pick's bodies histologically) 2) presentation - a) cognitive impairment: difficulties with memory, language, attention, thinking, orientation, calculation, and problem-solving b) psychiatric or behavioural disturbances: changes in personality, emotional control, and social behaviour; depression, agitation, hallucinations, delusions c) difficulties with ADL eg/ driving, shopping, eating d) progressive deterioration from previous higher level of functioning, consciousness not clouded (is in delirium). Short term memory impacted e) mild cognitive impairment (MCI) - decline in cognitive function > expected, taking account of age & education, which is not interfering with ADL f) subtle differences in presentation of different types of dementia eg/ I) Alzheimer's - insidious onset II) vascular dementia - stepwise inc in symptom severity III) DLB - fluctuating consciousness, hallucinations, sleep disorders, falls and Parkinsonism's IV) Parkinson's - Parkinsonian features predate dementia by 1+ years V) frontotemporal - behavioural changes (eg/ disinhibition or apathy) & language disturbances VI) dementia vs acute confusional state - ACS = recent onset, may have reversible cause g) diagnosis - cognitive or behavioural symptoms which: affect ability to function in normal activities, represent dec from previous level, not delirium or other psych disorder, established from hx (pt & informant) & MSE/MOCA h) impairment of 2+ of: ability to acquire & remember new info; judgement, ability to reason or handle complex tasks; visuospatial ability; language; personality & behaviour

Describe the pathophysiology of arterial dissection and outline clinical presentation, medical and surgical treatment, complications and causes of death

1) pathophysiology - a) intimal tear, then subsequent degeneration of the medial layer of the aortic wall, blood passes through the media, propagating distally or proximally and creating a false lumen b) as the dissection propagates, flow through the false lumen can occlude flow through branches of the aorta, including coronary, brachiocephalic, intercostal, visceral and renal, or iliac vessels c) intimal tears most commonly occur just above the sinotubular junction or just distal to the left subclavian artery d) tears can have both a retrograde and antegrade extension of the dissection - retrograde dissections starting in ascending aorta can lead to aortic incompetence by separating the aortic valve from the aortic root 2) s/s - a) features of Marfan's/Ehlers-Danlos syndromes b) acute severe chest pain - severe tearing or ripping chest pain suggests aortic dissection c) interscapular pain if dissection of descending aorta d) left/right BP differential - BP differential between 2 arms is suggestive and a hallmark feature e) pulse differential/deficit between 2 legs f) diastolic murmur - crescendo pattern, indicating aortic incompetence, common in proximal dissections, but uncommon in distal dissections 3) classification of aortic dissection- a) Type A dissection - ascending aorta +/- involvement of the arch and descending aorta. b) Type B dissection - only the descending thoracic aorta (distal to the left subclavian artery) +/- abdominal aorta 4) tx - a) hemodynamically unstable - hemodynamic support eg/ NA, dobutamine, IV fluids, + O2, + ALS b) confirmed aortic dissection - I) B-blockade eg/ labetaol (1-5mg/min IV) II) + opioid analgesia eg/ morphine sulfate: 2-5 mg IV PRN III) If B-blockade insufficienct - + vasodilators eg/ nitroprusside, diltiazem c) type A; or type B with complications (rupture, visceral/extremity ischaemia, expansion, or persistent pain) - as above + open surgery or endovascular stent-graft repair I) Open surgery for type A dissection, with replacement of the ascending aorta, immediately upon diagnosis. Depending on the extent of retrograde extension, the aortic valve may or may not need to be repaired or replaced II) For complicated type B dissections, both open and endovascular therapies are acceptable. Endovascular approach with endovascular stent-grafting is gaining preference over the open technique for patients presenting with complications III) For uncomplicated type B aortic dissections, surgical repair has no proven superiority over medical treatment in stable patients IV) Static or dynamic side-branch obstruction can be relieved with additional endovascular stents d) after hospital discharge - antihypertensives eg/ metoprolol +/- enalapril, with additional antihypertensives such as diuretics or calcium-channel blockers used if necessary 5) Complications - pericardial tamponade, aortic imcompetance, MI, aneurysmal degeneration/rupture, regional ischaemia, left arm ischemia/vertebral steal

Describe the pathophysiology of intermittent claudication; differentiate claudication from other causes of leg pain

1) pathophysiology - caused by peripheral artieal occlusive disease: a) atherosclerosis - single or multiple arterial stenoses lead to alterations in the distal perfusion pressures available to affected muscle groups b) resting blood flow in pts with PAOD is similar to healthy pts. But blood flow cannot maximally increase in muscle tissue during exercise, because proximal arterial stenoses prevent compensatory vasodilation. When the metabolic demands of the muscle exceed blood flow, claudication symptoms ensue. Longer recovery period is required for blood flow to return to baseline once exercise is terminated 2) differential diagnoses: a) spinal canal claudication (pulses present and LMN signs) b) osteoarthritis (pain at rest) c) sciatica (shooting down leg) d) popliteal artery entrapment (young patients, normal pulses) e) venous claudication (bursting pain, history of DVT) f) fibromuscular dysplasia and Buergers disease (young men, heavy smokers)

Classify cardiac trauma (penetrating and non-penetrating)

1) penetrating - worry about cardiac tamponade, rarely affects cardiac function 2) non-penetrating - mainly cardiac contusions

List and discuss the common complications following appendicectomy and explain how each can be prevented

1) perforation - common if faecolith present and in young children as diagnosis delayed a) treat with IV Abx + drain abscess, then appendectomy 4-6 weeks later 2) appendix mass - may happen when inflamed appendix becomes covered with omentum, USS/CT may help diagnosis a) NBM + Abx, if mass resolves perform delayed appendectomy 3) appendix abscess - if appendix mass fails to resolve but enlarges a) drainage, Abx alone may work 4) wound infection or dehiscence 5) stump appendicitis (rare) 6) haematoma 7) port site hernia 8) adhesions 9) death (rare)

Examine different modalities of sensation and describe distribution of common types of sensory loss including peripheral neuropathy and spinal cord lesions

1) peripheral neuropathy (polyneuropathy) - a) symmetrical damage to peripheral nerves, relatively common disorder with many causes eg/ toxic, metabolic, inflammatory or infectious - most commonly DM & malnutrition of alcoholism, Lyme disease, syphilis, or HIV b) symmetrical symptoms, glove & stockings distribution c) initial symptoms often sensory & longest nerves are affected first (ones most exposed to toxic or metabolic insult) eg/ nerves in feet "stocking" d) later on, as the symptoms reach the mid-calf, the fingers are involved and a full "stocking-glove" loss of sensation develops. Even later, when the trunk begins to be involved, sensory loss is noted first along the anterior midline e) vibration perception is often the earliest affected modality as they're the largest, most heavily myelinated and most metabolically demanding fibers. Loss of pin-prick, temperature, and light-touch perception follow, with conscious proprioception (joint position sense) being variably affected f) peripheral deep-tendon reflexes are depressed early, particularly Achilles reflex g) symptomatic motor involvement is late and, when it occurs, it affects intrinsic muscles of the feet first 2) radiculopathy (nerve root damage) - a) relatively common result of intervertebral disc herniation or pressure from narrowing of the intervertebral foramina due to spondylosis (arthritis of the spine) b) most commonly sharp, shooting pain along the course of the nerve root in a dermatomal distribution, brought on by stretching nerve root, or manouvers that load the intervertebral discs & compress intervertebral foramina c) damage to a single nerve root, even if severe, usually does not have any sensory loss. May be slight loss, often accompanied with paresthesias (tingling or pins & needles) in small areas of the distal limbs where the sensory dermatomal overlap is not great d) nerve root damage in the cauda equina produces a "saddle" distribution of sensory loss by affecting the lower sacral nerve roots (also seen in anterior spinal cord damage) 3) spinal cord - a) both sensory & motor symptoms at the level of involvement, as well as below in dermatomal & myotomal distribution b) affects the tracts running through the cord c) compression of the spinal cord from the anterior side first involves the spinothalamic paths from the sacral region, and a "saddle" loss of pain and temperature perception is the first symptom even with lesions high in the spinal cord. As symptoms progress with greater degrees of compression, symptoms progressively ascend the body up toward the level of the actual cord damage d) intramedullary lesions of the spinal cord (eg/ syrinx, ependymoma, or central glioma) may present with a very unusual pattern of "suspended sensory loss". This consists of an isolated loss of pain and temperature perception in the region of the expanding lesion because of damage to the crossing spinothalamic tract fibers, with "sacral sparing" of pain and temperature because the more peripheral spinothalamic fibers (ones from the sacrum) are the last to be 4) brain stem - a) characterized by long-tract and segmental (CN) motor and sensory abnormalities and is localized by the segmental sign b) ipsilateral cranial nerve abnormality and contralateral long-tract dysfunction 5) thalamus - a) lesions associated with contralateral hemihypesthesia b) initially, lesion produces considerable sensory loss bordering on anesthesia, but some recovery is expected over time, especially of touch, temperature, and pain perception - vibration and proprioception remain more severely affected 6) cerebral cortex - a) cortical lesions tend to leave minimal deficits in basic sensation b) if the parietal lobe is damaged, may be striking contralateral deficits in higher perceptual functions eg/ stereognosis and graphesthesia

Describe the clinical presentation and natural history of other neurological complications including ocular nerve palsies, diabetic amyotrophy, foot drop, impotence and autonomic neuropathy

1) peripheral neuropathy - affects the feet and legs first, then hands and arms a) s/s often worse at night b) numbness or reduced ability to feel pain or temperature changes c) tingling or burning sensation d) sharp pains or cramps e) inc sensitivity to touch — for some, even the weight of a bedsheet can be painful f) muscle weakness g) loss of reflexes, especially in the ankle h) loss of balance and coordination i) serious foot problems, such as ulcers, infections, and bone and joint pain 2) autonomic neuropathy - autonomic nervous system controls heart, bladder, stomach, intestines, sex organs and eyes a) hypoglycemia unawareness b) bladder problems eg/ UTIs or urinary retention or incontinence c) constipation, uncontrolled diarrhea or both d) gastroparesis, causing N&V, bloating and loss of appetite e) difficulty swallowing f) increased or decreased sweating g) problems controlling body temperature h) changes in the way your eyes adjust from light to dark i) increased heart rate at rest j) sharp drops in blood pressure after sitting or standing that may cause you to faint or feel lightheaded k) erectile dysfunction l) vaginal dryness m) decreased sexual response 3) ocular nerve palsies eg/ CN3, often with severe headache; diabetic foot drop 4) radiculoplexus neuropathy (diabetic amyotrophy) - affects nerves in the thighs, hips, buttocks or legs a) s/s usually unilateral b) severe pain in a hip and thigh or buttock that occurs in a day or more c) eventual weak and shrinking thigh muscles d) difficulty rising from a sitting position e) abdominal swelling, if the abdomen is affected f) weight loss g) most people improve at least partially over time, though symptoms may worsen before they get better

Outline the different forms of renal replacement therapy

1) peritoneal dialysis - a) uses the peritoneum as a natural semi-permeable membrane for diffusive removal of solutes, as effective as haemodialysis b) not widely used due to logistical and practical considerations - needs surgical insertion of a peritoneal dialysis catheter, requiring the additional involvement of a surgical team c) frequently complicated by catheter leakage and malfunction 2) intermittent hemodialysis: gold standard - a) haemodialysis - process of solute clearance based on diffusion across the membrane driven by a concentration gradient between the blood and dialysate, total amount of solute transported per unit of time (clearance), depends on the molecular weight of the molecule, membrane characteristics (dialysance), dialysate flow, and blood flow b) often prescribed for 3-6 h per treatment, 3x/week c) slow efficiency daily dialysis (SLEDD) or EDD (extended daily dialysis) is a variant of IHD where the duration of dialysis is extended to 8-12hrs, blood flow is lowered, fluid removal is more gradual, and solute clearance slower - means less haemodynamic instability 3) continuous renal replacement therapy (CRRT) - a) any renal replacement therapy applied for 24 h per day in an ICU b) CRRT provides slower solute clearance per unit time as compared with intermittent therapies but over 24 h may even exceed clearances with IHD c) solute removal either by convection (hemofiltration), diffusion (hemodialysis), or a combination of both these methods (hemodiafiltration) I) haemodialysis most efficiently removes small molecular weight substances such as urea, creatinine, and potassium II) middle and larger molecular weight substances are more efficiently removed using hemofiltration d) most common = continuous venovenous hemofiltration (CVVH), continuous venovenous hemodialysis (CVVHD), and continuous venovenous hemodiafiltration (CVVHDF)

List the complications of a perforated appendix

1) peritonitis: severe continuous abdominal pain, N+V, pyrexia, tachycardia, SOB, ascites 2) abscesses (may occur all the way up paracolic gutter) 3) fistula (bladder/rectum) 4) adhesions 5) pyaemia (via portal system) 6) sepsis & death

List the three main primary renal causes of NS and outline briefly the key pathological features

1) persistent proteinuria - a) primary renal disease: glomerular (eg/ glomerulonephritis) or tubular b) secondary renal disease: DM, hypertension, connective tissue diseases, vasculitis, amyloidosis, myeloma, congestive cardiac failure 2) nephrotic syndrome - a) primary: kidney diseases eg/ minimal-change (kids - immunologically mediated) or membranous nephropathy, focal segmented or membranoproliferative glomerulosclerosis b) secondary: systemic diseases eg/ DM, lupus erythematosus, amyloidosis. Drugs of abuse eg/ heroin, malaria, syphilis, hepatitis viruses, HIV, penicillamine, gold, NSAIDs, malignancy, obesity and pre-eclampsia 3) pathophysiology - a) each kidney has ~1 million glomeruli, which are the sites of blood filtration I) layers of the glomeruli - fenestrated endothelium of the capillary, glomerular basement membrane, foot processes of the podocytes, main barrier to filtration is the connection between adjacent podocyte foot processes called slit diaphragms b) 3 categories of proteinuria: glomerular, tubular, and overflow I) glomerular proteinuria - develops when the components of the filtration barrier are disrupted by disease, leading to nephrotic syndrome, pts hypoalbuminaemic due to urinary loss of albumin, liver tries to compensate so eventually get hypercholesterolaemia and hypertriglyceridaemia

Discuss the pharmacokinetics and clinical use of warfarin including laboratory tests used to monitor clinical effect

1) pharmacokinetics - antagonises vitK, prevents factor II, VII, IX, X production in liver (mnemonic = 1972) 2) use - anticoagulant, commonly used to treat blood clots eg/ DVTs, PEs, and to prevent stroke in people who have AF, valvular heart disease or artificial heart valves. Less commonly used following STEMI and orthopedic surgery 3) monitoring - INR = Prothrombin Time Ratio (Patient : Normal), adjusted for international comparison, includes fudge factor to enable comparison between hospitals & countries, aim for INR 2-3 normally

Discuss the pharmacokinetics and clinical use of heparin including laboratory tests used in monitoring heparin therapy. Outline the clinical management of over-anticoagulation with heparin

1) pharmacokinetics - inc activity of antithrombin by 1000x as deactivates Factor X and Factor II 2) use - anticoagulant used to treat and prevent DVT, PE, and arterial thromboembolism.Also used in the treatment of heart attacks and unstable angina 3) monitoring - APTT = Activated Partial Thromboplastin Time (usually 22 - 32 secs) measures activity of intrinsic pathway 4) overdose - protamine sulfate has been given to counteract the anticoagulant effect of heparin (1 mg per 100 units of heparin that had been given over the past four hours)

Pink puffers vs blue bloaters

1) pink puffers - primary pathology = emphysematous - destruction of alveolar SA + pulmonary vascular bed, less of ventilation/perfusion mismatch exists a) due to dec in overall gas transfer RR must inc to compensate - ensures CO2 levels remain normal & pts do not become cyanosed (but breathless) - may have T1RF b) dec in CO and eventually muscle wasting & weight loss 2) blue bloaters - primary pathology = chronic bronchitis - obstruction due to mucus & inflammation a) vascular bed is not damaged b) unlike pink puffers, body responds to obstruction by inc CO + dec RR causes V/Q mismatch, leading to hypoxaemia & polycythaemia c) over time, due to obstruction, lung volume inc d) develop cyanosis - low PaO2 & high PaCO2 (not breathless) - may develop T2RF + cor pulmonale e) retain fluid f) respiratory centres insensitive to CO2 - rely on hypoxic drive to retain respiratory effort

Define placebo, nocebo and contextual responses to treatment and describe possible mechanisms that explain these effects

1) placebo - substance or treatment of no intended therapeutic value, made to resemble an active medication or therapy so that it functions as a control; this is to prevent the recipient(s) and/or others from knowing (with their consent) whether a treatment is active or inactive 2) nocebo - negative expectations of pt regarding a treatment cause the treatment to have a more negative effect than it otherwise would have eg/ pt anticipates a SE of a medication, so suffer that effect even if the "medication" is actually a placebo NB/ placebo & nocebo are psychogenic phenomenon as mental states such as beliefs and expectations can strongly influence the outcome of disease, the experience of pain, and even success of surgery

Describe the natural history of pleural plaques, mesothelioma and asbestosis

1) pleural plaques - when exposed to asbestos, inhaled fibers can accumulate in the lining of the lungs and irritate the lung tissue. Fibers reach the pleural space through the lymphatic system 2) pleural macrophages - leading theory suggests asbestos fibers cause an immune response that summons pleural macrophages, which can trigger a chain of events that leads to fibrosis, in which specialized cells replace normal, healthy lung tissue with scar tissue made of collagen fibers 3) pleural plaque development - slowly develop and grow as fibrotic scar tissue accumulates 4) asbestos-exposed patients with pleural plaques may also develop pleural thickening, which involves more extensive fibrous thickening. Because pleural thickening affects a larger area, it can prevent the lungs from fully expanding and cause shortness of breath 5) mesothelioma - asbestos exposure is considered the primary causal facto - fibres result in recruitment and activation of alveolar macrophages and neutrophils, with subsequent generation, possibly iron-catalysed, of reactive oxygen and nitrogen species. Chronic inflammation and oxidative stress may culminate in DNA damage, alterations in gene expression (proto-oncogenes and tumour suppressor genes), and eventual malignant transformation 6) asbestosis - when asbestos fibres are inhaled, they deposit at alveolar duct bifurcations and cause an alveolar macrophage alveolitis. Activated macrophages release cytokines eg/ TNF and IL-1, which initiate a process of fibrosis. The initial interstitial fibrosis typically occurs in the lower lobes and may progress to extensive fibrosis and honeycombing. Peri-bronchial fibrosis with a cellular infiltrate may narrow the airway and cause reduced air flow

Pneumothorax - describe its underlying pathology and investigations

1) pneumothorax = gas accumulates in pleural space which expands & compress underlying lung, which can then collapse 2) spontaneous = passage of air into pleural sac from abnormal connection between pleura + bronchial system due to bullous emphysema (rupture of pleural bleb/bulla - usually apical) or some other lung disease eg/ congenital defects in connective tissue of alveolar walls 3) tension = life-threatening, result of trauma, lung infection, medical procedures eg/ high-pressure mechanical ventilation, chest compression during CPR or thoracoscopy a) unlike spontaneous air trapped in pleural space cannot escape so with each breath air + pressure accumulate in chest b) when lung collapses heart, BVs & airways pushed to centre of chest, compressing other lung - leads to dec BP (cardiorespiratory distress), consciousness & breathing - may lead to shock & death NB/ normally, pressure in pleural space is -ve (lungs sucked towards thoracic wall), once punctured, vacuum-like state between pleural layers causes air to be sucked in & lung no longer sucked out, meaning pleural space fills with air & lung collapses by intrinsic recoil 4) ix - a) CXR - expiratory film to look for area devoid of lung markers, peripheral to edge of collapsed lung (ensure suspected pneumothorax is not a large emphysematous bulla) b) check ABG in dyspnoeic/hypoxic pts and those with chronic lung disease NB/ don't do CXR if suspected pneumothorax as delays treatment

Define portal hypertension and classify its causes

1) portal hypertension - inc in BP within portal venous system (>10-12mmHg) veins coming from stomach, intestine, spleen & pancreas merge into portal vein - branches into smaller vessels & travels through liver if vessels in liver are blocked due to liver damage, blood cannot flow properly through the liver = high pressure in portal system develops 2) pre-hepatic: thrombosis (portal or splenic vein), congenital atresia/stenosis, extrinsic compression 3) intra-hepatic: cirrhosis, schistomiasis, sarcoid, myeloproliferative disease, congenital hepatic fibrosis, acute alcoholic liver disease, sclerosis, drugs, veno-occlusive disease 4) post hepatic: budd-chiari syndrome, R heart failure, constrictive pericarditis, veno-occlusive disease NB/ most common causes in the UK = alcoholic & viral cirrhosis, worldwide = Schistosomiasis

Describe systemic non-metastatic manifestations including paraneoplastic syndromes

1) post-obstructive pneumonia/hypoxia 2) SVC syndrome - bulky mediastinal adenopathy or medial extension of R upper lobe tumour can compress SVC, impeding return of blood from face & arms to heart a) s/s - facial & upper extremity oedema, dyspnoea, cough, orthopnoea, facial plethora, distended neck, chest wall, occasional abdo wall veins b) medical emergency - O2, head elevation, radio/chemo 3) paraneoplastic syndromes 4) others - hoarseness, confusion, personality changes, N&V, headache, dysphagia, weakness/paraesthesia/pain in C8/T1 distribution, seizures, cervical or supraclavicular adenopathy, Horner's (ptosis, miosis, anhidrosis), finger clubbing, hypertrophic pulmonary osteoarthropathy

Classify renal failure into pre-renal, renal and post-renal causes, outlining the pathology of the common diseases that may cause each type

1) pre-renal (55%), all = dec GFR: a) due to renal hypoperfusion eg/ hypotension (any cause including hypovolaemia, sepsis) b) renal artery stenosis - due to atheromatous plaque development, or congenitally with fibromuscular dysplasia. Thrombosis of the renal artery can occur in AF, endocarditis, and post-MI with mural thrombi & ventricular aneurysms (also supra-renal aortic aneurysms c) cardiogenic + septic shock d) drugs eg/ ACEI - dec renal pressure maintenance mechanisms (prostaglandins + AGII) 2) intrinsic (30%): may need renal biopsy for diagnosis - a) tubular - acute tubular necrosis = commonest renal cause of AKI, due to: I) nephrotoxins - drugs eg/ aminoglycosides, radiological contrast II) pre-renal damage III) crystal damage eg/ ethylene glycol poisoning, uric acid IV) myeloma V) inc calcium VI) ischaemia, DIC, septicaemia VII) globinaemias & liver failure b) glomerular - autoimmune eg/ SLE, infection, primary glomerulonephritis c) interstitial - infiltrates eg/ lymphoma, infection, tumour lysis syndrome following chemotherapy d) vascular - vasculitis, malignant inc in BP, thrombus or cholesterol emboli from angiography, large vessel occlusion eg/ dissection or thrombus e) acute tubulointerstitial nephritis - develops 10-14 days following drug exposure; pts become febrile & develop haem+proteinuria following immune response to drug f) acute pyelonephritis- may lead to papillary necrosis or abscess, necrosis is maximal at kidney poles. Inflammation starts at pelvis & spreads into papillae. Intense inflammation leads to venous thrombosis, necrosis, and destruction of CDs and calyces g) chronic pyelonephritis = fibrosis, endarteritis, urinary back-pressure. These features combine to cause fibrosis + distortion of kidney parenchyma, with loss of nephrons. Deep scars form in the kidney. h) diabetic nephropathy - microvascular complication of DM i) pre-eclampsia - hypertension + unselective proteinuria & oedema during pregnancy. Placental ischaemia = vasoconstriction and DIC j) hypertension - benign = hyalinisation of afferent arterioles, leading to patchy glomerular ischaemia. Malignant = characteristic fibrinoid necrosis in afferent arterioles. RBCs escape into capsular space & thrombosis occurs k) polycystic kidney disease - 2 main forms: autosomal dominant due to PKD-1 (85%) + PKD-2 (15%, milder) genes accounts for 5-10% of cases of CKD 3) post-renal (15%): caused by urinary tract obstruction - a) luminal - stones, clots, sloughed papillae (obstruction = back pressure & development of hydronephrosis) b) mural - malignancy (eg/ ureteric, bladder, prostate), BPH, strictures c) extrinsic compression - malignancy (esp pelvis), retroperitoneal fibrosis d) retroperitoneal fibrosis - rare disorder where fibrotic tissue compresses ureters NB/ acute retention - pts experience frequency, nocturia & poor flow due to mechanical impingement on urethra, chronic retention develops, and predisposes to UTIs, hypertrophy, diverticuli, hydronephrosis & bladder stones

Describe the clinical presentation of an extradural haemorrhage

1) precipitated by clearly defined head trauma 2) typically young patient involved in a head strike (either during sport or a result of a motor vehicle accident) 3) may or may not lose consciousness transiently 3) following the injury, they regain a normal level of consciousness (lucid interval) - over the next few hours, they gradually lose consciousness 4) usually have an ongoing and often severe headache 5) haematoma can compress intracranial structures which may impinge on the CN III - fixed & dilated pupil on the side of the injury, eye down and out 6) weakness of the extremities on the opposite side as the lesion (except in rare cases), due to compression of the crossed pyramid pathways, and a loss of visual field opposite to the side of the lesion, due to compression of the posterior cerebral artery on the side of the lesion

Obtain a relevant history from a patient with a pleural effusion

1) presence of risk factors - CHF, pneumonia, and malignancy 2) dyspnoea 3) pleuritic chest pain - pain that is worse with inspiration and may be exacerbated by cough and movement 4) cough - productive cough may be present if the effusion results from pneumonia. However, fluid collection and irritation of the pleural surfaces alone cause a non-productive cough

Describe the classical features of a generalised seizure

1) presence of risk factors - fmx, previous CNS infection or trauma, and prior seizure events 2) focal neurological deficits 3) focal neurological symptoms (before or after seizure) 4) premonitory sensation or experience (fear, epigastric sensation, déjà vu, jamais vu) - mostly if focal-onset epilepsy, most often associated with temporal lobe origin 5) temporary hemiparesis - postictal paralysis or weakness on one side of the body is suggestive of focal-onset epilepsy, and generally indicates that the seizure focus is contralateral to the side of the deficit 6) temporary aphasia - postictal aphasia can also be seen after focal-onset seizures that involve language centres in the dominant hemisphere 7) fever, nuchal rigidity, altered mental status - with infectious causes of seizure

Describe symptoms that may suggest a diagnosis of IBS

1) presence of risk factors eg/ female, <50 yo, previous enteric infection, hx of physical or sexual abuse 2) abdo discomfort - cramping in lower +/- mid-abdomen, mild or severe 3) alteration of bowel habits associated with pain - diarrhoea or more frequent soft stool, constipation, or alternating diarrhoea and constipation, passage of stool relieves abdo pain 4) abdominal bloating or distension - inc during the day & not associated with N+V, improved with defecation or passage of flatus 5) normal examination of abdomen

Describe the typical history of a patient with bronchiectasis and describe how it differs from COPD

1) presence of risk factors eg/ older age, genetics, CF, immunodeficiency, previous infections, congenital disorders of bronchial airways, primary ciliary dyskinesia 2) cough - initial symptom, frequently a morning cough, but becomes constant as disease progresses, acute exacerbation = worsening of cough 3) cough is usually productive & purulent sputum quality may change with exacerbations or superimposed infection, can have haemoptysis. Worsened by lying flat or on one side 4) SOB - initially with exercise but may progress to SOB even at rest 5) crackles, high-pitched inspiratory squeaks (& pops) 6) pts may have difficulty speaking in full sentences 7) foul-smelling breath (halitosis) due to mucostasis & bacterial proliferation 8) strain of constant inflammation & efforts to clear airways can = fever, malaise, weight loss 9) vs COPD - clubbing is common, sputum is not of the same quantity a) similarities - dyspnoea, chronic cough, potential for daily sputum production, airflow obstruction. b) differences - COPD: chronic sputum production associated with coughing, associated with lung irritants eg/ pollution, smoking bronchiectasis: may have dry or wet cough, often result of respiratory infections or lung transplantation

Pneumothorax - describe its typical clinical presentation and the recognised risk factors

1) presentation - a) may be no symptoms if young, fit, small pneumothorax b) sudden onset of dyspnoea and/or unilateral pleuritic chest pain c) pts with asthma or COPD may present with sudden deterioration d) if enlarges dyspnoae progresses, pt may have pallor + tachycardia e) dec chest expansion f) resonant percussion note g) reduced/absent breath sounds h) sudden onset of dyspnoea and/or pleuritic pain i) may be no symptoms (especially if fit, young, small pneumothorax) j) tension pneumothorax - trachea deviated away from affected side 2) risk factors - a) COPD (esp if >40) or asthma b) infection - TB, pneumonia c) carcinoma d) congenital - cystic fibrosis, Marfan's, Ehlers-Danlos e) iatrogenic - subclavian CVP line insertion f) male sex - 6:1 g) pts often tall & thin h) lung abscess or severe fibrosis

Describe the typical presentation of a patient with a community-acquired pneumonia and the features that identify severe pneumonia using CURB-65 as a risk prediction tool

1) presentation - a) fevers, rigors, malaise, anorexia, confusion b) dyspnoea, cough, purulent sputum, haemoptysis, pleuritic pain, cyanosis, tachypnoea c) tachycardia, hypotension d) signs of consolidation (dec expansion, dull percussion, tactile vocal fremitus/vocal resonance, bronchial breathing), pleuritic rub 2) CURB65: Confusion (mental test <8) Urea (>7mmol/L) RR (>30/min) BP (<90 systolic and/or 60 diastolic) >65yrs old 2 = hospital, 3+ = severe (other factors for severity - multilobar involvement, pO2<8, albumin<35, WBC<4 or >20, culture positivity) NB/ most common cause - strep pneumoniae comes with preceding viral infection pt rapidly becomes more ill & febrile, with pleuritic pain and a dry cough 1-2 days later, rusty-coloured sputum is produced and the patient may develop cold sores.

CCF - ix

1) previous MI - Doppler echocardiography - assessment of overall LV systolic + diastolic function, LV wall thickness, valvular disease, estimation of pulmonary artery systolic pressure 2) no previous MI: a) B-type natriuretic peptide (BNP) and N-terminal pro-BNP (NT-proBNP) - released into blood when myocardium is stressed. Measure serum natriuretic peptides: I) high levels (BNP ≥400 pg/ml) = Doppler echocardiography within 2 weeks II) if levels are normal (BNP ≤100 pg/ml), heart failure is unlikely b) 12-lead ECG - identifies potential factors + needed for treatment decisions c) blood tests: FBC, U&E and creatinine, LFTs, glucose, fasting lipids, TFTs; consider cardiac enzymes if an undiagnosed MI is possible in the preceding few days d) CXR - evidence for heart failure and helps exclude other potential causes of breathlessness. Typical findings in heart failure include: I) cardiomegaly (cardiothoracic ratio >50%) II) ventricular hypertrophy III) prominent upper lobe veins (upper lobe diversion) IV) diffuse interstitial or alveolar shadowing - classical perihilar 'bat's wings' or nodular (especially with pre-existing COPD) V) fluid in fissures or pleural effusions VI) Kerley B lines e) urinalysis f) lung function tests (peak flow or spirometry)

List the clinical and investigational features that may distinguish primary from secondary Raynaud's disease

1) primary - a) hands - sharply demarcated color changes of the digits - due to reversible small-vessel vasospasm rather than secondary Raynaud's involving vasospasm plus structural disease in the microcirculation b) precipitated by cold temperatures or emotional stress c) attacks are symmetric and involve all the fingers on both hands d) pallor doesn't extend beyond the metacarpophalangeal joints into the palms; if it does, this is not just a digital arteriolar vasospastic event, but rather one involving compromise of larger vessels, as occurs in Raynaud's from a secondary cause such as scleroderma e) normal nailfold capillaries are consistent with primary Raynaud's phenomenon. But abnormal nailfolds in a patient with no history or physical findings suggestive of a secondary cause of Raynaud's are associated with a 20% chance of developing a rheumatic disease - mostly scleroderma - within the next 2 years f) often show a livedo reticularis pattern of mottled, purplish, lacelike vessels on their skin, especially on the legs 2) secondary - a) can present with digital ulcers, gangrene, or signs of tissue injury b) nailbed - dilated capillaries in nailbed 3) ix - a) ANA - normal in primary RP, in secondary RP often positive with a centromere pattern b) FBC - normal in primary RP, may be abnormal in secondary RP. Any CT disease may cause anaemia. In SLE, cytopenias are often present, may show low WBC, anaemia, and low platelet count in secondary RP c) ESR - normal in primary RP, may be elevated in secondary RP d) Creatinine - normal in primary RP; creatinine may be elevated in secondary RP e) Urinalysis - normal in primary RP; may show RBCs and/or protein in secondary RP such as in SLE or vasculitis with active glomerulonephritis 4) Secondary Raynaud's - Raynaud's phenomenon + another condition eg/ scleroderma, SLE, RA, Sjögren's syndrome, dermatomyositis, polymyositis, mixed CT disease, cold agglutinin disease, Ehlers-Danlos Syndrome, eating disorders, B-blockers, cytotoxic drugs eg/ chemotherapeutics, jobs involving vibration, exposure to the cold (eg/ working as a frozen food packer)

Outline the management of varicose veins including indications for surgery

1) primary care - a) lifestyle changes - weight loss, exercise, avoid prolonged standing and keep leg(s) elevated when possible b) compression stockings only if interventional treatment is not appropriate (need ABPI before prescribing to rule out arterial disease) using a Doppler 2) referral - a) emergency referral if active bleeding from a varicose vein that has eroded the skin b) pts with progressive leg ulcers or painful ulcers (or other symptoms) despite treatment c) hard, painful veins indicative of superficial venous thrombosis. d) suspected venous incompetence e) venous leg ulcer that has not healed within two weeks, or a healed venous leg ulcer. 3) tx in secondary care - for confirmed varicose veins with truncal reflux, minimally invasive procedures such as radiofrequency ablation, endovenous laser therapy or foam sclerotherapy should be considered first a) endothermal (radiofrequency) ablation - sealing the lumen of the long saphenous vein by delivering radiofrequency energy via a catheter under US guidance b) US-guided foam sclerotherapy - imaging-guided injection of sclerosant in foam form c) avulsion - small incisions are made over each varicosity and that part of the vein is excised using a vein hook or forceps d) stripping - wire, plastic or metal rod is passed through the lumen of the saphenous vein and pulled until the entire vein is stripped out of the leg e) ambulatory phlebectomy - small multiple incisions are made in skin overlying vein, which is hooked out and extracted in a piece-meal fashion f) injection sclerotherapy

Differentiate primary from secondary lymphoedema and explain the pathophysiology and treatment of lymphoedema

1) primary lymphoedema - idiopathic and usually sporadic, with no family history, mostly lower extremities, classified based on the age of onset: a) infancy: most common in males, usually bilateral, Milroy's disease is a known subset with a familial pattern of inheritance. b) childhood: least common time of onset. c) adolescence: most common in females, usually unilateral, Meige's disease is a known subset with a familial pattern of inheritance. d) adulthood: rare form of primary lymphoedema. 2) secondary lymphoedema - occurs after injury to the lymphatic system, usually due to nematode infection (filariasis), malignancy, or cancer-related treatment 3) pathophysiology - lymphatic channels are endothelium-lined vessels derived from outpouching of veins with distal to proximal flow. Superficial and deep lymphatic channels drain proteinaceous fluid (lymph) into regional lymph nodes. The lymphatic system returns interstitial fluid and proteins to the circulatory system. Particulate matter and microorganisms are filtered at lymph nodes for immunological presentation. Dysfunction of channels or nodes leads to lymph accumulation in the superficial interstitial space. Lymphatic stasis leads to fat hypertrophy, with associated thickening of subcutaneous tissue, as well as immunological dysfunction. Inc interstitial protein cause inflammation and fibrosis, leading to a cycle of further damage. 4) tx - a) 1st line - skin care: avoid trauma to the skin, moisturising, regular bathing, pts should wear protective clothing, particularly when outdoors, and avoid walking barefoot b) + static compression bandaging or garments c) + elevation d) + exercise - e) + weight control f) +/- manual lymph drainage g) +/- intermittent pneumatic compression h) + psychosocial support eg/ , National Lymphedema Network: patient support groups (US) i) with filariasis + pharmacotherapy eg/ diethylcarbamazine, ivermectin, albendazole j) with significant morbidity and/or refractory to conservative measures + surgery (excisional) + postoperative static compression garment

Outline the major pathological classification of lung cancers and their prognosis

1) primary tumour (T): a) TX - malignant cells in bronchial secretions & no other evidence of tumour, TIS - carcinoma in situ, T0 - none evident (best) b) T1 = <3cm in lobar or more distal airway c) T2 = >3cm & >2cm distal to carina or any size if pleural involvement or obstructive pneumonitis extending to hilum but not all of lung d) T3 = involves chest wall, diaphragm, mediastinal pleura, pericardium or <2cm from, but not at, carina, T>7cm diameter & nodules in same lobe e) T4 = involves mediastinum, heart, great vessels, trachea, oesophagus, vertebral body, carina, malignant effusion, or nodules in another lobe 2) regional nodes (N): a) N0 = none involved (after mediastinoscopy) b) N1 = peribronchial and/or ipsilateral hilum c) N2 = ipsilateral mediastinum or subcarinal d) N3 = contralateral mediastinum or hilum, scalene, or supraclavicular 3) distant metastases (M): a) M0 = none M1 = nodule in other lung, pleural lesions, malignant effusion or distant metastases present

Outline the principles, benefits and risks of immunisation

1) principles - vaccines work by making us produce antibodies to fight disease without actually infecting us with the disease. If the vaccinated person then comes into contact with the disease itself, their immune system will recognise it and immediately produce the antibodies they need to fight it 2) risks - a) most SE are mild and short-lived eg/ redness or swelling around the injection site, younger children or babies may be a bit irritable or unwell, or have a slight temperature for a couple of days b) in much rarer cases, some people have an allergic reaction soon after a vaccination eg/ rash or itching that affects part or all of the body c) on very rare occasions, a severe allergic reaction may happen within a few minutes of the vaccination - anaphylactic reaction which can be life-threatening 3) benefits - a) protection from potentially life-threatening diseases b) elimination of diseases in a population

Define clinical audit and describe its purpose and what it involves (the audit loop)

1) process used by health professionals to assess, evaluate and improve care of patients in a systematic way a) measures current practice against a defined (desired) standard b) forms part of clinical governance, which aims to safeguard a high quality of clinical care for pts 2) audit loop - a) stage 1 - preparation: choose a topic b) stage 2 - select criteria: in the form of a statement eg/ 'All patients with hypertension who smoke should be offered smoking cessation advice'; define the standard - usually a target (percentage). This may be a minimum standard or an optimal one, depending on the clinical scenario c) stage 3 - measuring level of performance: collect the data, compare actual performance with the set standard, discuss how well the standards were met, if the standards were not met, note the reasons for this (if known) d) stage 4 - making improvements: present the results and discuss them with the relevant teams in your organisation, develop an action plan, specifying what needs to be done, how it will be done, who is going to do it and by when e) stage 5 - maintaining improvements: follow-up - to determine whether the actions taken have been effective, or whether further improvements are needed, involves repeating the audit

Describe the indications for proctoscopy and sigmoidoscopy

1) proctoscopy - a) to visually investigate anorectal conditions for which a DRE does not provide sufficient diagnostic information b) to obtain information on conditions such as internal hemorrhoids or disruption and other pathology of the rectal mucosa, or to examine for an anorectal mass or foreign body in the anal canal c) to obtain samples for cytology as a screening method for anal squamous lesions, particularly in high-risk patients with HIV infection 2) rigid sigmoidoscopy - a) symptoms that suggest anorectal pathology, including colorectal neoplasia b) prior to anorectal procedures in the clinic or operating theater c) biopsy of any bowel condition within the reach of the instrument d) assessing the true height (distance from anal verge) of rectal cancers e) conservative treatment of sigmoid volvulus f) gauging the lower resection margin during anterior resection of the rectum 3) flexible sigmoidoscopy - a) screening for colorectal cancer b) preoperative evaluation before anorectal surgery c) surveillance of a previously diagnosed (treated or untreated) malignancy (or polyp with high-grade dysplasia) in the rectum or the sigmoid colon d) local treatment of ailments such as radiation proctitis Removal of rectal foreign bodies e) biopsy of GI pathology in rectum & sigmoid colon f) therapeutic procedures eg/ endoluminal stent placement for strictures, balloon dilation, and decompression with placement of a decompression tube, however a conventional colonoscopy is often commonly used g) haematochezia necessitating haemostasis

List the blood products available for transfusion and outline the rationale for using fresh frozen plasma, cryoprecipitate and platelets

1) products - RBC, plasma, platelets, cryoprecipitate 2) plasma - reversal of anticoagulant effects: either inherent eg/ factor deficiency, or warfarin; active DIC; microvascular bleeding during massive transfusion; replacement fluid for apheresis in thrombotic microangiopathies, hereditary angioedema 3) cryoprecipitate - haemorrhage after cardiac surgery; massive haemorrhage or transfusion; surgical bleeding 4) platelets - major surgery or invasive procedure with no active bleeding & plts <50; ocular or neurosurgery plts <100; surgery with active bleeding plts <50; stable non-bleeding plts <10; as previous with temp >38 degrees plts <20

List the symptoms suggestive of oesophageal malignancy

1) progressive dysphagia (solids, then with liquids) 2) dec weight (inc metabolic demand & dysphagia) 3) retrosternal chest pain 4) hoarseness 5) cough 6) pain with food impaction 7) lymphadenopathy 8) early satiety + haematemasis may present

Classify pituitary adenomas according to size and function

1) prolactinoma (40-50%): young women = most <10mm, men + elderly = >10mm a) signs - galactorrhoea, amenorrhoea, hypogonadism, erectile dysfxn 2) GH secreting tumour (20%): mm-cm in size a) signs - change in appearance, inc height, acromegaly/gigantism 3) ACTH secreting tumour (10-15%): majority <10mm a) signs - Cushing's disease 4) TSH secreting tumour (rare): most >10mm a) signs - hyperthyroidism 5) non-functioning (20%): most >10mm at presentation a) signs - hypopituitarism, mechanical signs & symptoms 6) craniopharyngioma (enlargement of Rathke's pouch): v large & cystic, skull xray abnormal in >50%, headaches, visual field defects, growth failure (50% occur <20; 15% arise from within sella)

Describe the pathological changes and complications seen in purulent leptomeningitis, lymphocytic meningitis and granulomatous meningitis

1) purulent leptomeningitis - a) diffuse purulent inflammation b) leptomeninges (arachnoida and piamater) contain purulent exudate (pus): leukocytes (neutrophils), fibrin, germs, proteins, necrotic debris c) blood vessels in subarachnoidian space & intracerebral are congested and neutrophil margination is present, causes vasculitis d) infiltration of cranial nerves and spinal roots, causing demyelination, cranial nerve palsies e) oedema may occur in brain tissue f) complications - I) hydrocephalus from obliteration of subarachnoid space, exit foramina or other sites II) oedema and increased intracranial pressure III) CN palsies IV) thrombosis from inflammatory infiltrate in the walls of arteries V) brain abscess or subdural abscess VI) seizures 2) lymphocytic (aseptic) meningitis - a) serous inflammation of the linings of meninges, usually with an accompanying mononuclear pleocytosis b) not caused by pyogenic bacteria, although usually caused by certain viruses eg/ enterovirus c) usually clear without tx, rarely cause brain infection 3) granulomatous meningitis - a) usually affects dogs and cats b) 3 types: I) disseminated - diffuse disease throughout CNS. Accumulation of mononuclear cells and neutrophils around blood vessels (perivascular) of CNS II) focal - presents as a granuloma, which mimics a tumor. Usually is found in the cerebrum or cerebellopontine angle III) ocular - uncommon. Extension of CNS disease as blood vessels of the posterior segment of the eye and anterior uvea have the same infiltrates of inflammatory cells as the intracranial vessels

Outline the investigation of suspected acute ancreatitis, emphasising the timing, interpretation and reliability of currently available tests

1) raised serum amylase (>1000u/mL) or ~3x upper limit (degree of elevation not related to severity of disease) within 24-48hrs of onset (then fall). Cholecystitis, mesenteric infarction & GI perforation can all cause rises. Renally excreted - urinary amylases follow serum levels, but with a 2 day lag 2) serum ALT >3x normal is suggestive of gall stones 3) serum lipase - more sensitive & specific for pancreatitis 4) CRP >150mg/L at 36hrs after admission = predictor of severe pancreatitis 5) ABG - monitor oxygenation & acid-base status 6) AXR - no psoas shadow (inc retroperitoneal fluid), sentinel loop of proximal jejunum from ileus (solitary air filled dilatation) 7) erect CXR - helps exclude other causes eg/ perforation 8) CT - standard choice of imaging for severity & complications - MRI even better 9) US - if gallstones + inc AST 10) ERCP - if LFTs worsen

Describe the clinical presentation of an intracerebral space occupying neoplastic lesion

1) rapid onset of symptoms suggests a cerebrovascular lesion whilst a space-occupying lesion of the brain will usually be more gradual 2) headache - a) new headache with features suggestive of raised ICP eg/ papilloedema, vomiting, posture-related headache, or headache waking the patient from sleep b) focal neurological symptoms, or non-focal neurological symptoms eg/ blackout, and change in personality or memory c) unexplained headache that becomes progressively severe, or in anyone with previous cancer or HIV d) new onset of epileptic seizures 3) generalised symptoms and signs (often due to inc ICP) - a) may be weakness, ataxia or disturbance of gait (cerebellar signs if cerebellum affected) 4) temporal lobe lesions - often present with rather vague psychological problems: a) may be depersonalisation, emotional changes and disturbances of behaviour b) temporal lobe epilepsy - hallucinations of smell, taste, sound and sight; déjà vu c) dysphasia may be noted d) visual field defects involve the contralateral upper quadrant e) other psychological problems - forgetfulness, fugue, functional psychosis and fear or rage; may be inappropriate sexual behaviour 5) frontal lobe - a) can cause anosmia, especially significant if it is unilateral b) may be a change in personality with the person becoming indecent, indiscreet or dishonest c) dysphasia if Broca's area is involved d) hemiparesis or fits may affect the contralateral side 6) parietal lobe - a) may be hemisensory loss b) dec two-point discrimination c) astereognosis (inability to recognise a familiar object placed in the hand) d) extinction & sensory inattention e) dysphasia may occur. f) Gerstmann's syndrome may occur - agraphia or dysgraphia + acalculia or dyscalculia + finger agnosia + L-R disorientation 7) occipital lobe - a) lesion in front of the optic chiasma will affect just one eye b) lesion at the optic chiasma eg/ pituitary adenoma, causes a bitemporal hemianopia c) lesion posterior to the optic chiasm will cause a crossed homonymous field defect (eg/ L optic tract lesion causing a R homonymous hemianopia) d) lesion in the visual cortex will cause congruent contralateral visual field defects 8) cerebellopontine angle - most commonly acoustic neuroma eg/ ipsilateral deafness, tinnitus, nystagmus, dec corneal reflex, facial & trigeminal nerve palsies, ipsilateral cerebellar signs 9) midbrain - unequal pupils, inability to direct the eyes up or down, amnesia for recent events, with confabulation, somnolence

Describe the global picture of TB, its relation to the AIDS epidemic and the causes and consequences of multidrug resistant tuberculosis

1) rates of TB have stabilised in the UK with ~9000 cases reported in 2012, an incidence of 14 per 100,000 population 2) up to 60% of AIDS patients develop TB and the disease accounts for 1/3rd AIDS-related deaths 3) MDR TB (resistant to 2+ drugs) and extensively drug-resistant TB (resistant to 3+ drugs) are global problems with high mortality which threaten to destabilise TB control programmes in several parts of the world, cases with extensively drug-resistant TB may be virtually untreatable, depending on the level of resistance to second-line drugs

List the potential complications of a subarachnoid haemorrhage

1) re-bleeding - commonest cause of death, occurs in 20% often in first few days 2) cerebral ischaemia - due to vasospasm, may cause permanent CNS deficit, commonest cause of morbidity 3) hydrocephalus - blockage of arachnoid granulations, requires ventricular or lumbar drain 4) hyponatraemia - common but should not be managed with fluid restriction -expert help NB/ haemorrhage spreads throughout subarachnoid space, leading to inc in ICP. With progression, there may be local infarction (in territory of rupture artery), arterial spasm (irritation effect of haemorrhage), aneurysm may rupture into brain & mimic intracerebral haemorrhage. Left alone, SAH will lead to death

Outline the theories on the aetiology of diverticulosis of the colon including age, diet and vascular anatomy of the colon

1) reason diverticula form in colon wall is not completely understood 2) maybe: thickening of muscle layer + high pressure inside the colon pushing against weak spots in the colon wall 3) if a diet is low in fiber, the colon must exert more pressure than usual to move small, hard stool - low-fiber diet also can inc time stool remains in bowel, adding to high pressure - muscular hypertrophy 4) pouches of mucosa may form when high pressure pushes against weak spots in colon (through wall) - blood vessels pass through muscle layer of bowel wall to supply blood to the inner wall (diverticula) 5) cholinergic denervation with inc age = hypersensitivity & inc uncoordinated muscular contraction - may lead to pouch formation 6) diverticulitis occurs when faeces obstruct neck of diverticulum causing stagnation & allowing bacteria to multiply. Can lead to perforation, abscess formation, fistulae or generalized peritonitis

Outline the principles of management for local recurrence and metastatic breast cancer

1) recurrence after completion of therapy- need individualised tx: a) rate of ~1-2% per year occur in patients with breast-conservation therapy b) need MDT to determine the role and timing of surgery, radiotherapy, and systemic therapy c) need to determine: if occurred in previously irradiated site, presence or absence of distant metastases, number of sites involved by recurrent disease d) considerations for surgery - consider ability to achieve negative margins and whether the patient will need systemic therapy before resection e) systemic therapy - take into account prior interventions a) if postmenopausal with hormone-receptor-positive breast cancer progressing on non-steroidal aromatase inhibitors, double endocrine treatment with fulvestrant + oestrogen deprivation is no better than treatment with fulvestrant or exemestane alone b) combination of fulvestrant + aromatase inhibitor versus an aromatase inhibitor alone appears to benefit endocrine-therapy-naïve patients in particular c) for completely resected isolated locoregional recurrences of breast cancer, adjuvant chemotherapy should be considered esp if recurrence was triple negative 2) metastatic: without visceral/end organ crisis: ER and/or PR receptor-positive - a) HER2-ve: I) 1st line - endocrine-based therapy ± CDK4/6 inhibitor: anastrozole or letrozole +/- palbociclib or ribociclib II) +/- bisphosphonate or denosumab +/- calcium and vitamin D III) supportive care: pain management; palliative radiotherapy for painful skeletal metastases; MDT support with symptom management IV) 2nd line - exemestane + everolimus or fulvestrant + palbociclib or exemestane monotherapy +/- as above V) consideration of clinical trials can be an option at any step b) HER2-positive: post-menopausal - I) 1st line - pertuzumab + trastuzumab + a taxane; or: trastuzumab + anastrozole or letrozole or exemestane II) +/- bisphosphonate or denosumab +/- calcium and vitamin D + supportive care c) HER2-negative: pre-menopausal - I) 1st line - tamoxifen +/- ovarian ablation (surgical or medical) II) +/- bisphosphonate or denosumab +/- calcium and vitamin D + supportive care d) HER2-positive: pre-menopausal (also if ER & PR -ve) - I) 1st line - pertuzumab + trastuzumab + a taxane; or tamoxifen +/- ovarian ablation (surgical or medical) + trastuzumab II) +/- bisphosphonate or denosumab +/- calcium and vitamin D + supportive care 3) without end organ/visceral crisis: ER and PR receptor-negative - a) HER2-negative - I) 1st line - chemotherapy II) +/- bisphosphonate or denosumab +/- calcium and vitamin D + supportive care

Define the classical features of asthma and outline common precipitants

1) recurrent episodes of dyspnoea, cough (nocturnal), wheeze (widespread & polyphonic) & sputum caused by reversible airway obstruction 2) some have: acid-reflux, nocturnal disturbances (severe asthma), other atopic diseases eg/ eczema, hay-fever 3) symptoms normally worse in morning 4) symptoms reversible with short-acting B-agonist NB/ wheeze may not be present when pt is well 5) precipitants - a) cold air b) exercise c) emotion d) allergens (house dust mites, pollen, fur, work) e) infection f) smoking (& passive) g) pollution h) NSAIDs i) B-blockers j) food/drink k) hormones l) stress m) GORD NB/ need to spot occupational asthma eg/bakers, detergent manufacturers, drug, lab workers, metal workers etc

Outline local metastatic manifestations of lung cancer

1) recurrent laryngeal nerve palsy - Hilar tumours may involve L recurrent laryngeal nerve = hoarseness of voice & bovine cough 2) phrenic nerve palsy - bronchial carcinoma may involve phrenic nerve = unilateral, ipsilateral hemidiaphragmatic paralysis 3) SVC obstruction = early morning headache, facial congestion, upper limb oedema, raised JVP 4) horner's syndrome - interruption of sympathetic chain ganglia - ptosis, meiosis, anhidrosis 5) rib erosion - Pancoast's tumour (apex of lung), erodes ribs & involves lower part of brachial plexus = severe pain of shoulder & down inner surface of arm 6) pericarditis 7) AF 8) pleuritic - infiltration of pleura often results in haemorrhagic effusion 9) oesophagus = dysphagia & pericardial effusion + dysrhythymias.

List the metabolic complications of total parenteral nutrition

1) refeeding syndrome - can occur in severely malnourished patients receiving aggressive PN: when too much food and/or liquid nutrition supplement is consumed during the initial 4-7 days of refeeding, this triggers synthesis of glycogen, fat and protein in cells, to the detriment of blood potassium, magnesium, and phosphorus 2) hyperglycaemia - due to parenteral carbohydrate intake in diabetic patients, in response to post-aggression metabolism or systemic inflammatory response syndrome (SIRS), or due to systemic steroid therapy a) in extreme cases PN can result in a hyperosmolar, hyperglycaemic non-ketotic coma 3) when suddenly discontinuing parenteral intake, rebound hypoglycaemia may occur 4) abnormalities in the acid-base-balance can occur due to PN and may result in significant electrolyte shifts 5) hypertriglyceridemia with dyslipoproteinaemia may occur 6) hepatic complications of PN include: steatosis (fatty liver) and cholestasis 7) long term PN may result in: metabolic bone disease with bone demineralisation and osteoporosis 8) in all forms of PN there is an imminent risk of infectious complications 9) intestinal SE eg/ mucosal atrophy, inc translocation of micro organisms and their toxins

Outline the main pathological processes affecting the glomerulus in glomerulonephritis

1) refers to several kidney diseases (usually affecting both kidneys) 2) many of the diseases are characterised by inflammation either of glomeruli or small blood vessels in the kidneys, but not all diseases have an inflammatory component 3) categorised into several different pathological patterns, which are broadly grouped into non-proliferative or proliferative. Diagnosing the pattern is important for outcome and treatment. Primary causes are intrinsic to the kidney. Secondary causes are associated with certain infections (bacterial, viral or parasitic), drugs, systemic disorders (SLE, vasculitis), or diabetes 4) can be due to immunological damage to glomerulus - either through autoantigens, antigens trapped in nephron, or immune complexes from other regions a) autoantigens - Goodpasture's - antibodies against type IV collagen in glomerular BM. Leads to crescenteric GN, may also occur in SLE, Wegeners, IgA nephropathy b) antigens from elsewhere - acute diffuse proliferative GN occurs 2-3 weeks postinfection with group A strep (pharyngitis). Granular deposition of IgG and C3 may be shown on immunofluorescence c) immune complexes - in Type 1 membranoproliferative GN, immune complexes develop & trigger complement system activation. In IgA nephropathy, IgA immune complexes form in systemic blood and get trapped in kidney. Complement promotes immune complex formation, but also generates an environment that facilitates their destruction - complement deficiency may predispose to immune complex disease d) immune complex & complement presence causes cell damage and neutrophil activation, leading to glomerular injury

Describe the principles of insulin therapy. Classify the different types of oral hypoglycaemic drugs and non-insulin injectables used in Type 2 diabetes and outline their indications and contraindications

1) regimen - a) twice daily insulin - short or rapidly acting insulin mixed with intermediate or long acting insulin once or twice daily = biphasic b) basal-bolus - short or rapidly acting insulin with each meal + separate intermediate or long-acting insulin once or twice daily 2) risks of insulin therapy - risk of accidents/errors, misreading u as number, large dosing range so potential for 10 fold errors 3) Tx - a) if T2DM diet & lifestyle for 3/12 b) biguanides eg/ metformin - I) 1st line in T2DM, esp if overweight II) dec gluconeogenesis & inc peripheral utilisation III) dec CV risk IV) not suitable in renal imapirement - lactic acidosis V) GI SE so titrate dose over weeks VI) no hypos c) sulfonylureas eg/ glicazide - I) 2nd line II) enhances ability of pancreas to secrete insulin III) can cause hypos, and gi disturbance & weight gain (generally mild & infrequent), rarely causes disturbances in liver function d) dipeptidylpeptidase-4 (DPP-4) inhibitors eg/ sitagliptin, vildagliptin - I) inc insulin secretion & dec glucagon release II) low risk of hypos e) thiazolidinediones eg/ pioglitazone - I) dec peripheral insulin resistance II) low risk of hypos III) can cause: weight gain, fluid retention, inc fractures, CVD, may precipitate heart failure f) glucagon-like peptide-1 (GLP-1) mimetic eg/ exenatide, liraglutide - I) inc insulin secretion, inhibits glucagon release, delays gastric emptying (appetite suppression) - so can cause weight loss II) s/c injection III) SE - nausea g) sodium-glucose co-transporter 2 inhibitors (SGLT2) eg/ dapagliflozin, empagliflozin - I) inc glucose excretion in proximal convoluted tubule (pee out glucose) II) weight loss III) dec CV events IV) SE - risk of DKA & UTI (need gd renal function) 4) monitoring blood glucose - a) self-monitoring T2DM not on insulin not needed - causes depression & dec QOL b) do if tx with insulin - prevent hypos, set realistic goals for glucose levels, accept periodic deviation

Describe the sources of variation in clinical measurement including regression to the mean and the importance of observer variation

1) regression to the mean - phenomenon that if a variable is extreme on its first measurement, it will tend to be closer to the average on its second measurement—and if it is extreme on its second measurement, it will tend to have been closer to the average on its first eg/ students takes a 100-item true/false test and all students choose randomly on all questions. Will expect mean of 50. Some will score substantially above 50 and some substantially below 50 by chance. If look at top scoring 10% & give them a second test on which they again choose randomly on all items, the mean score would again be expected to be close to 50, so mean of these students would "regress" all the way back to the mean of all students who took the original test 2) observer variation - failure by observer in a study or test to measure accurately, resulting in error. Inter-observer variation is the amount of variation between the results obtained by 2+ observers examining the same material.

Outline the main immunological barrier to successful transplantation and how this may be overcome by tissue typing and immunosuppressive therapy

1) rejection is an adaptive immune response via cellular immunity (mediated by killer T cells inducing apoptosis of target cells) as well as humoral immunity (mediated by activated B cells secreting antibody molecules), though the action is joined by components of innate immune response (phagocytes and soluble immune proteins) 2) immunosuppressive therapy - short course of high-dose corticosteroids can be applied, and repeated. Triple therapy adds a calcineurin inhibitor and an anti-proliferative agent a) immunosuppressive drugs: I) corticosteroids - prednisolone, hydrocortisone I) calcineurin inhibitors - ciclosporin, tacrolimus II) anti-proliferatives - azathioprine, myocophenolic acid III) mtor inhibitors - sirolimus, everolimus b) antibody-based treatments - antibody specific to select immune components can be added to immunosuppressive therapy eg/ basiliximab, daclizumab, anti-thymocyte globulin (ATG), anti-lymphocyte globulin (ALG), rituximab c) blood transfer - cases refractory to immunosuppressive or antibody therapy are sometimes treated with photopheresis, or extracorporeal photoimmune therapy (ECP), to remove antibody molecules specific to the transplanted tissue d) marrow transplant - can replace the transplant recipient's immune system with the donor's, and the recipient accepts the new organ without rejection e) gene therapy - genes that cause the body to reject transplants would be deactivated. Research is still being conducted, and no gene therapies are being used to date to treat patients. Current research tends to focus on Th1 and Th17 which mediate allograft rejection via the CD4 and CD8 T cells

Define relative risk, absolute risk, attributable risk fraction, population attributable risk, prevalence, incidence, and prognosis

1) relative risk/risk ratio (RR) - ratio of the probability of an outcome in an exposed group to the probability of an outcome in an unexposed group eg/ if 9% of males and 75% of females have an adult height <167cm, the risk ratio for such a final height in women vs men is 0.75/0.09= 8.33 2) absolute risk - probability or chance of an event, used for the number of events (eg/ disease) that occurred in a group, divided by the number of people in that group 3) attributable risk fraction - difference in risk of a condition between an exposed population & unexposed population (assuming exposure is causally related to the disease) eg/ if lung cancer occurs 0.17/1000 man years in non smokers & 4.17/1000 in smokers, the attributable risk in the heavy smokers is 4 per 1000 man years, risk difference = +0.4 4) population attributable risk - indicates number (or proportion) of cases that would not occur in a population if the factor were eliminated (e.g. how many lives would be saved if people no longer smoked?) 5) prevalence - proportion of population who have the disease at the relevant time: no of existing cases of a disease/total population 6) incidence - no of new cases of disease occurring in given population over given period of time: no new cases of a disease during a time period/total population at risk 7) prognosis - prediction of the course of a disease following its onset, refers to the possible outcomes of a disease and the frequency with which they can be expected to occur

Describe the effect of declining renal function on drug clearance and discuss the need to adjust doses according to British National Formulary (BNF) guidelines

1) renal disease alters the effects of many drugs, particularly when active drug moieties are renally cleared 2) drug doses should usually be reduced in renal disease in proportion to the predicted reduction in clearance of the active drug moiety 3) pt factors to consider in adjusting drug doses: degree of renal impairment and patient size 4) drug factors to consider in adjusting doses: fraction of the drug excreted unchanged in urine and the drug's therapeutic index 5) estimates of renal function are useful to guide dosing of renally cleared drugs with medium therapeutic indices, but are not precise enough to guide dosing of drugs with narrow therapeutic indices

Discuss non-surgical management, indications for surgery and list common operations

1) resectable (stages I and II) - a) 1st line - surgical resection eg/ proximal pancreaticoduodenectomy with antrectomy (Kausch-Whipple procedure) or pylorus-preserving pancreaticoduodenectomy (Traverso-Longmire procedure) for tumours in the head of the pancreas b) + pancreatic enzyme replacement - pancreatin Adjunct - c) +/- preoperative biliary stenting d) +/- neoadjuvant radiotherapy or chemoradiotherapy 2) locally advanced unresectable (stage III) - a) 1st line - endoscopic stent insertion or palliative surgery b) + chemotherapy or chemoradiotherapy or stereotactic body radiotherapy eg/ gemcitabine c) + pain management (often need opioids) d) + pancreatic enzyme replacement 3) metastatic (stage IV) - a) 1st line - endoscopic stent insertion or palliative surgery b) + chemotherapy c) + pain management + pancreatic enzyme replacement

Interpret blood gas analyser results from a patient with: metabolic acidosis; metabolic alkalosis; respiratory acidosis; respiratory alkalosis

1) respiratory acidosis eg/ COPD - pH: dec, pCO2: inc, bicarbonate: normal a) compensated respiratory acidosis - pH: normal, pCO2: inc, bicarb: inc 2) respiratory alkalosis eg/ hyperventilation - pH: inc, pCO2: dec, bicarb: normal a) compensated respiratory alkalosis - pH: normal, pCO2: dec, bicarb: dec 3) metabolic acidosis eg/ DKA - pH: dec, pCO2: normal, bicarb: dec a) compensated metabolic acidosis - pH: normal, pCO2: dec, bicarb: dec 4) metabolic alkalosis - pH: inc, pCO2: normal, bicarb: inc a) compensated metabolic alkalosis - pH: normal, pCO2: inc, bicarb: inc 5) mixed acidosis - pH: dec, pCO2: inc, bicarb: dec 6) mixed alkalosis - pH: inc, pCO2: dec, bicarb: inc

Describe the manifestations of MND leading to death

1) respiratory failure - inevitable consequence of ALS. Early symptoms can be effectively treated with non-invasive ventilation, while severe end-stage respiratory failure requires either tracheostomy, with permanent assisted ventilation, or palliative care 2) aspiration pneumonia - if dysphagia & difficulty with airway maintenance, choking and aspiration pneumonia may develop at any time 3) riluzole-related hepatotoxicity; riluzole-related neutropenia

Outline the principles of treatment of massive blood loss (eg/ GI haemorrhage)

1) restore circulating volume - a) insert wide‐bore peripheral cannulae - 14 G or larger b) give adequate volumes of warmed crystalloid c) aim to maintain normal BP & UO & keep pt warm 2) contact key personnel - clinician in charge, anaesthetist, blood bank, haematologist, surgeon 3) arrest bleeding - early surgical or obstetric intervention, or interventional radiology 4) bloods - a) FBC, PT, APTT, fibrinogen; blood‐bank sample, biochemical profile, blood gases or pulse oximetry b) repeat FBC, PT, APTT, fibrinogen every 4 h or after 1/3 blood volume replacement 5) request suitable red cells - a) un‐crossmatched group O Rh negative - max 2 units b) un‐crossmatched ABO group‐specific when blood group known c) fully cross‐matched d) use blood warmer and/or rapid infusion device 6) request platelets & FFP & cryoprecipitate 7) suspect DIC - tx underlying cause if possible, tx shock, hypothermia, acidosis

Name three typical symptoms of gastro-oesophageal reflux disease (GORD). Describe the investigations used to confirm a diagnosis of GORD

1) retrosternal burning pain (discomfort after meals, lying, stooping or straining, relieved by antacids) 2) waterbrash/acid regurgitation 3) dysphagia/odynophagia. 4) other symptoms include night cough (assoc w/ aspiration), nausea, laryngitis and tooth decay NB/ pain may be mistaken for angina if has a spasmodic character 5) endoscopy if: a) symptoms >4 weeks b) persistent vomiting c) GI bleeding/iron deficiency d) palpable mass e) >55 f) dysphagia g) symptoms despite treatment or relapsing symptoms h) dec weight 6) barium swallow may show hernia hiatus (anatomical problems) 7) 24hr oesophageal pH monitoring & manometry when endoscopy normal, pH <4 for more than 6% of monitoring period = GORD. NB/ endoscopy can establish malicious causes & consequences of GORD eg/ hiatus hernia, oesophagitis, Barrett's oesophagus

Describe the indications and methods for providing routine and reverse isolation

1) reverse isolation - used when pts immune system not functioning properly, to prevent them getting an infection which they may then struggle to fight a) often involves the use of laminar air flow and mechanical barriers (to avoid physical contact with others) b) handwashing before and after entering c) full gowns d) visitors to only come if well e) visitors not to bring any outside things in eg/ food, glasses etc 2) routine isolation - if pt unwell with a communicable disease, to prevent others becoming infected a) ensuite room b) precautions depend on microorganism eg/ gown, gloves, mask may need eye protection, respirator etc

Describe the complications of acute myocardial infarction (AMI) and describe their presentation

1) rhythm abnormalities (tachy/bradycardia), late malignant ventricular arrhythmias - 1-3 weeks post MI, AF/VF 2) cardiac failure - RHS = inc JVP, low cardiac output 3) ventricular septal defect - pansystolic murmur, inc JVP, cardiac failure, septal ischaemia 4) ruptured chordae tendinae - causes mitral regurgitation + pulmonary oedema 5) haemopericardium - low CO, pulsus paradoxus, inc JVP, muffled heart sounds 6) Dressler's syndrome - immune mediated recurrent pericarditis, pleural effusions, fever, anaemia, inc ESR 1-3 weeks after MI 7) ventricular aneurysm 8) re-infarction 9) post-infarct angina

List the factors that contribute to infection following a surgical procedure

1) risk correlated to level of cleanliness of wound 2) having surgery that lasts more than 2 hours 3) other medical problems or dec immune system eg/ diabetes, cancer 4) being an elderly adult 5) being overweight 6) smoking 7) having emergency surgery 8) having abdominal surgery 9) malnutrition 10) poor tissue perfusion/previous irradiation 11) foreign bodies 12) inc length of stay in hospital

Describe the radiological and surgical treatment choices for patients with occlusive arterial disease according to affected vessel

1) risk factor management - smoking cessation, optomise blood glucose, normalise hypercholesterolemia, control hypertension 2) pharmacological - daily aspirin - 81-325 mg/day (no consensus on the most effective dose) 3) supervised exercise programmes can improve quality of life and walking distance 4) radiological - 5) surgical - NB/ ix - exam - complete lower-extremity evaluation and pulse examination - abdominal aorta to foot, with auscultation for bruits in abdominal & pelvic regions. When palpable pulses are not present, use a handheld Doppler device to assess circulation 2) ankle-brachial index (ABI) - noninvasive way of establishing PAOD - ratio of SBP at the ankle to that in the arm (normal range = 0.9-1.1; PAOD = <0.9, IC = 0.4-0.9, <0.4 = critical limb ischaemia. ABPI may appear higher than its actual value in calcified/hardened arteries) 3) angiography - standard for arterial imaging in diagnosis of PAOD; usually reserved for when an intervention is planned 4) magnetic resonance angiography (MRA) - useful for imaging large and small vessels 5) computed tomography angiography (CTA) - used to image arterial disease but requires large amount of contrast media and an upgraded CT scanner to reconstruct helpful images 6) duplex ultrasonography - evaluates status of a patient's vascular disease and provides information about haemodynamics; noninvasive & no contrast media

Describe the risk factors, clinical presenting features and pathological causes and consequences of ischaemic and haemorrhagic stroke

1) risk factors - 2) s/s - 3) pathological causes & consequences - a) acute ischemic - vascular occlusion due to thromboembolic disease (or stenosis) 2) normally affects internal carotid/verterbral artery or intracerebral artery 3) emboli commonly from heart, as in AF, IE, mural thrombi/ventricular aneurysm. Emboli may also come away from ulcerated atheroma in carotid artery 4) stenosis (atheromatous) on its own is not a cause of infarction but a small drop in BP can cause watershed infarct/infarct in territory of stenotic artery 5) up to 24 hours - little gross appearance of infarction. After 24 hours - some swelling & blurring of junction between white and grey + microscopic picture of necrosis. After a few days, a line of demarcation seen between necrotic and viable tissue - infarct begins to "organise", macrophages infiltrate, oedema lessens, and capillaries begin to sprout. Weeks later, demolition of necrotic tissue, scarred area shrinks & ventricle enlarges on the same side 6) ischemia = cell hypoxia and depletion of cellular ATP - no energy to maintain ionic gradients across cell membrane & cell depolarization. Influx of sodium + calcium ions and passive inflow of water into cell = cytotoxic oedema 7) acute vascular occlusion = heterogeneous regions of ischemia in affected vascular territory. Local blood flow limited to any residual flow in major arterial source + collateral supply, if any 8) affected regions with cerebral blood flow <10 mL/100 g of tissue/min = core - die within mins of stroke onset 9) zones of dec or marginal perfusion (blood flow < 25 mL/100g of tissue/min) = ischemic penumbra - tissue can remain viable for several hours because of marginal tissue perfusion 10) intracerebral haemorrhage - bleeding directly into brain parenchyma - leakage from small intracerebral arteries damaged by chronic hypertension or: bleeding diatheses, iatrogenic anticoagulation, cerebral amyloidosis, cocaine abuse 11) predilection for certain sites in brain eg/ thalamus, putamen, cerebellum, brainstem. In addition to the area of the brain injured by haemorrhage, surrounding brain can be damaged by pressure from mass effect of haematoma. General inc in ICP may occur 12) mainly pts past middle age 13) microaneurysms develop in pts with hypertension - ruptures = IC haemorrhage 14) sudden headache, then ICP develops, and death often follows 15) haematoma forms, most commonly in MCA branches to BG and internal capsule, but also in pons and cerebellum

Describe the risk factors for venous thrombosis and the use of the Wells score in assessing the probability of DVT and the risk of pulmonary thrombo-embolism

1) risk factors - a) blood-clotting disorders - hypercoagulability and thrombophilia b) prolonged bed rest or sitting eg/ during a long hospital stay, or paralysis c) injury or surgery d) varicose veins e) pregnancy f) birth control pills or hormone replacement therapy g) lifestyle - being overweight or obese, smoking h) cancer and cancer treatment i) heart failure (cardiorespiratory failure, recent MI/CVA) j) IBD k) personal or family history of DVTs or PEs l) age >60 m) endothelial damage n) acute infection o) venous catheter 2) Wells score can be used to assess likelihood of DVT.: a) includes major risk factors + other signs eg/ localised tenderness in deep vein system, swelling of entire leg, pitting oedema greater in symptomatic leg, 'lose' points if alternative diagnosis more likely than DVT (Baker's cyst, cellulitis, muscle damage, superficial venous thrombosis, post phlebitic syndrome, inguinal lymphadenopathy, external venous compression) b) 3+ points = high probability of DVT, 1-2 = moderate probability, -2-0 = low c) Wells for PE - clinical signs and symptoms of DVT, PE is #1 diagnosis OR equally likely, HR > 100, immobilisation for 3+ days OR surgery in lasts 4 weeks, previous diagnosed PE or DVT, haemoptysis, malignancy w/ treatment within 6 months or palliative

List recognised risk factors and physical signs for bronchiectasis

1) risk factors - a) absent or dysfunctional CFTR protein in bronchial cells in CF b) primary ciliary dyskinesia eg/ Kartagener's syndrome c) childhood infections eg/ measles, whooping cough, pneumonia, TB etc d) exposure to chemical irritants e) immunodeficiency disorders f) lung fibrosis g) mechanical obstruction eg/ bronchial carcinoma h) congenital disorder 2) physical signs - a) persistent cough b) copious purulent sputum (khaki, thick, foul smelling) - halitosis may persist c) intermittent haemoptysis d) finger clubbing e) coarse inspiratory crackles over infected areas f) wheeze g) pts often febrile

List risk factors for the symptoms suggestive of gastric cancers

1) risk factors - a) pernicious anaemia b) group A blood type c) H. pylori d) atrophic gastritis e) adenomatous polyps f) lower social class g) smoking h) diet - high nitrate, high salt, pickling, low vitC i) nitrosamine exposure j) male:female ratio is 2:1. k) family history 2) s/s (often non specific) - a) dyspepsia (if >1 month & over 50 = immediate investigation) b) vomiting + nausea + weight loss c) dysphagia + early satiety d) anaemia (occult blood loss) e) epigastric mass/pain (constant & severe, relieved by antacids) f) hepatomegaly g) jaundice h) ascites i) large L supraclavicular (virchow's) node = troisier's sign j) acanthosis nigricans k) most pts in west present with locally advanced (inoperable) or metastatic disease

List the symptoms and signs suggestive of strangulation

1) risk factors - a) previous abdominal surgery b) hernias c) Crohn's disease d) foreign body ingestion e) appendicitis f) intussusception g) malrotation, volvulus, and intestinal atresia in newborns/infants. 2) s/s - a) failure to pass flatus or stool b) abdominal pain - crampy, intermittent, can be severe c) vomiting - may be bilious; occurs after onset of pain d) abdominal distention - classically associated with high-pitched, inc frequency bowel sounds d) abdominal tenderness - associated with intestinal ischaemia; a sign of progressive disease and impending tissue necrosis e) absolute constipation f) peritonitis - if intestinal ischaemia, necrosis, and/or perforation

Describe the clinical presentation of glomerulonephritis

1) risk factors - infections (GABHS, hep B and C, respiratory and gastrointestinal infections, infective endocarditis, HIV), connective tissue diseases (SLE, systemic vasculitides), malignancy (Hodgkin's lymphoma, lung cancer, colorectal cancer, non-Hodgkin's lymphoma, leukaemia, thymoma), haemolytic uraemic syndrome, and drugs 2) haematuria - mainly microscopic 3) oedema - generalised, more specific to nephrotic 4) HTN - dec eGFR + salt & water retention results in systemic hypertension 5) oliguria 6) anorexia & nausea 7) malaise & weight loss & fever 8) skin rash 9) arthralgia 10) haemoptysis 11) abdominal pain 12) sore throat

Describe the symptoms and signs associated with liver abscess. List the investigations that differentiate neoplasia, abscesses and cysts and outline their treatment options

1) s/s - I) risk factors eg/ underlying biliary tract abnormalities, age >50, malignancy, DM, and interventional biliary or hepatic procedures II) fevers and chills III) RUQ tenderness IV) hepatomegaly V) others - weight loss, fatigue, abdo pain, N&V, cough, SOB, chest pain 2) ix - a) FBC - leukocytosis, elevated neutrophil count, anaemia b) serum LFTs - inc alk phos, mildly elevated aminotransferases and bilirubin, hypoalbuminaemia c) blood cultures - may show cause d) PT and activated partial thromboplastin time - usually normal e) liver ultrasound - demonstrates a variably echoic lesion f) contrast-enhanced abdominal CT scan - demonstrates hypodense liver lesions g) Gram stain and culture of aspirated abscess fluid - may show cause h) consider - CXR (may have pleural effusion), serum antibody test for Entamoeba histolytica, stool Entamoeba histolytica antigen detection test, antigen testing or PCR of aspirated abscess fluid, liver MRI, CRP 3) tx - a) bacterial abscess - I) IV abx - switch to oral when possible, total course of 4-6 weeks b) amoebic abscess: - metronidazole, then luminal agent: paromomycin for 7 days b) neoplasia - Barcelona Clinic liver cancer (BCLC) stage 0-A (very early 0 or early disease A) I) 1st line - consider resection II) if not surgical candidate & no comorbidity: 1st line - liver transplantation +/- transarterial chemo-embolisation (TACE) and/or radiofrequency ablation (RFA) bridging therapy III) poor liver function with comorbidity & lesions <2 cm: 1st line - TACE or RFA IV) poor liver function with comorbidity & lesions <2 cm: 2nd line - percutaneous ethanol injection (PEI) BCLC stage B: intermediate disease I) 1st line - TACE II) +/- percutaneous ablation BCLC stage C: advanced disease I) 1st line - sorafenib or lenvatinib II) 2nd line - regorafenib or nivolumab BCLC stage D: end-stage disease I) 1st line - hospice care (± liver transplantation in selected candidates) c) cyst - I) may choose not to treat a small cyst, instead suggesting a wait-and-see approach II) if cyst is larger and causes pain or bleeding may need tx: III) US guided drainage IV) surgical removal of cyst

Describe the clinical presentation, investigation and management of renal artery stenosis

1) s/s - a) presence of key risk factors - smoking, dyslipidaemia, and diabete b) onset of hypertension (HTN) age >55 years c) hx of accelerated, malignant, or resistant HTN +/- unexplained kidney dysfunction +/- multi-vessel coronary artery disease +/- other peripheral vascular disease d) abdominal bruit e) sudden or unexplained recurrent pulmonary oedema 2) ix - a) serum creatinine - normal or elevated b) serum potassium - hypokalaemia or low-normal potassium may suggest RAS due to activation of the renin-angiotensin system c) urinalysis and sediment evaluation - normal in the absence of diabetic nephropathy or hypertensive glomerulosclerosis d) aldosterone-to-renin ratio - <20 excludes primary aldosteronism as cause of hypertension and hypokalaemia or low-normal potassium e) others - duplex ultrasound, gadolinium-enhanced MR angiography (MRA), CTA, conventional angiography, CO2 angiography, non-contrast magnetic resonance angiography, captopril radionuclide renal scan 3) mx - ONGOING a) 1st line - anti0hypertensives eg/ captopril, enalapril, valsartan etc b) + lifestyle changes - weight loss, low-salt diet, and smoking cessation c) + statin eg/ atorvastatin, simvastatin d) + antiplatelet eg/ aspirin e) 2nd line - renal artery stenting + continuation of medical therapy f) + post-stent clopidogrel (DAPT) g) 3rd line - surgical reconstruction of the renal arteries

Describe the symptoms, signs, investigations, differential diagnosis and treatment of chronic mesenteric vascular occlusive disease

1) s/s - a) Postprandial pain - epigastric or periumbilical, develops 10mins to 3hrs after meal, no rebound or guarding b) Fear of eating (sitophobia) & weight loss c) Hx of vascular disease involving other organs (eg/ MI, cerebral vascular disease, or peripheral vascular disease) d) N&V & D/C e) Flatulence f) Abdominal bruit g) Signs of peripheral vascular disease (eg/ carotid bruits, dec pulses, and ischemic feet) 2) ix - lab tests, angiography, CTA, MRI and MRA, US, ECG (rule out cardiac disease) a) lab - I) FBC - may show anemia, leukopenia, or lymphopenia secondary to chronic malnourishment II) U&E - may show electrolyte abnormalities from malnutrition, vomiting, or diarrhea III) Coagulation studies: Prothrombin time (PT), activated partial thromboplastin time (aPTT), and international normalized ratio (INR) IV) LFTs - may show hypoalbuminemia from malnutrition V) Urinalysis - to rule out stones or infection VI) Stool testing - testing for occult blood may return positive results because of the sloughing of dead ischemic bowel; if a patient presents with steatorrhea, stool fat should be sent for examination b) Angiography - criterion standard for the diagnosis of chronic mesenteric ischemia (CMI), typically shows occlusion of 2 visceral branches of aorta, with severe stenosis of the remaining visceral branch, usually celiac trunk or the superior mesenteric artery (SMA) c) CTA - 1st-line alternative to conventional angiography d) MRI and Magnetic Resonance Angiography - can consider e) US - mesenteric duplex ultrasonography is a useful initial screening tool for chronic mesenteric ischemia (CMI) f) Histologic Findings - transected mesenteric vessels show diffuse atherosclerosis, atrophy of the tips of the villi of bowel, which leads to loss of the absorptive surface in the small bowel 3) DD - gastritis, acute mesenteric ischaemia, biliary colic or obstruction, cholangitis, cholecystitis, pancreatitis, pyelonephritis, diverticulitis, gastric cancer, hiatal hernia, peptic ulcer disease, small bowel obstruction 4) Tx - Approach Considerations a) Surgery if - peritonitis, massive haemorrhage, recurrent fever or sepsis, s/s over 3/52, chronic protein losing colopathy, chronic segmental colitis with ulceration, symptomatic ischaemic stricture I) Angioplasty, with or without stent placement II) Surgical revascularization III) Eg/ Transaortic endarterectomy of the celiac trunk or the superior mesenteric artery (SMA), retrograde bypass from the external iliac artery, anterograde bypass, which provides the best orientation of the graft to the aorta b) low-fat diet, similar to that of patients with cardiac disease, small, multiple meals, regular exercise c) meds - heparin and warfarin for anticoagulation and intra-arterial papaverine for vasodilation or nitroprusside d) abx - clindamycin, timentin, metronidazole, aztreonam, cefoxitin, cefotetan, meropenem

Describe the clinical features and laboratory diagnosis of chronic lymphatic leukaemia and outline the principles of treatment

1) s/s - a) SOB & fatigue - anaemia b) lymphadenopathy - painless c) splenomegaly d) less common - hepatomegaly, B symptoms (fever, chills, night sweats, weigh loss), recurrent infections, petechiae (thrombocytopenia) e) risk factors - >60 yo, male, white ethnicity, and positive family history 2) ix - a) WBC count with differential - inc absolute lymphocytosis >5 x 10⁹/L (>5 x 10³/microlitre) b) blood film - smudge/smear cells present; spherocytes and polychromasia can be seen if there is active haemolysis c) Hb - anaemia indicates a poor prognosis d) platelet count - thrombocytopenia indicates a poor prognosis e) flow cytometry - analysis of peripheral blood: +ve for surface immunoglobulin, CD5, CD19, CD20, and CD23; may also show CD38, CD49d, and ZAP-70 f) consider - fluorescent in situ hybridisation (FISH): may show cytogenetic abnormalities such as del(13q14), del(11q), or del(17p), molecular genetic analysis, direct antiglobulin test (DAT if pt anaemic), immunoglobulin levels, bone marrow aspirate and trephine biopsy (may show marrow infiltration by leukaemic cells, reduction in haematopoetic precursor compartment), CT scan (may show hepatosplenomegaly; retroperitoneal or mediastinal adenopathy) 3) mx - a) asymptomatic early stage (Binet A and B; Rai 0-II) - I) 1st line - observation, FBC with differential, flow cytometry, and routine physical exam performed every month for 3 months then every 3 months to follow disease progression b) advanced (Binet C; Rai III-IV) or symptomatic disease - I) 1st line - chemoimmunotherapy or ibrutinib eg/ fludarabine + cyclophosphamide + rituximab, or bendamustine, or ofatumumab, or ibrutinib c) with del(17p)/TP53 mutation - 1st line: ibrutinib or idelalisib or venetoclax I) +/- allogenic stem cell transplant

Describe the clinical features and laboratory diagnosis of multiple myeloma. Outline the associated laboratory abnormalities including changes in blood viscosity, renal function and serum calcium

1) s/s - a) anaemia - can cause fatigue b) bone pain - typically localised to the back c) infections d) renal impairment 2) ix - a) serum/urine electrophoresis - paraprotein spike (IgG >35 g/L [>3.5 g/dL] or IgA >20 g/L [>2.0 g/dL] and light chain urinary excretion >1 g/day); hypogammaglobulinaemia b) skeletal survey - osteopenia, osteolytic lesions, pathological fractures c) whole-body, low-dose computed tomography (WBLD-CT) - osteolytic lesions (≥5 mm in diameter), pathological fractures d) serum free light-chain assay - inc free light chain in serum e) bone marrow aspirate and biopsy - diagnostic test for MM: monoclonal plasma cell infiltration in the bone marrow f) serum calcium - hypercalcaemia g) FBC - normocytic and normochromic anaemia h) creatinine, urea - renal impairment (creatinine >176 mmol/L [>2 mg/dL]) associated with poor prognosis i) serum beta2-microglobulin - corresponds with staging j) serum albumin - combined with beta2-microglobulin levels improve prognostic significance k) consider - whole-body MRI (>1 focal lesions on MRI study; bone marrow infiltration), 18F-fluorodeoxyglucose-positron emission tomography (bone disease and bone marrow infiltration), CRP & LDH (inc means more extensive disease), cytogenetics & FISH analysis (pts with del13, del(17)p, or chromosomal abnormalities such as t(4;14) or t(14;16) are unresponsive to conventional chemotherapy regimens and have a poor prognosis)

Describe the clinical features, investigation and treatment of paracetamol overdose including the importance of monitoring of hepatic and renal function

1) s/s - a) attempted self-harm - may have hx of previous attempted self-harm or overdose. If awake can establish amount, timings etc b) asymptomatic presentation - many patients in first 24 hours will be asymptomatic c) N&V, abdo pain - early s/s are vague d) s/s 2-3 days afterwards: RUQ pain & tenderness, jaundice, can have confusion, decreased consciousness level/coma, and/or asterixis with fulminant hepatic failure 2) ix - a) serum paracetamol level - checked from 4hrs of ingestion onwards, tx depends on level in blood (compared to graph) b) serum AST and ALT - may be elevated c) arterial pH, arterial lactate level, U&Es, INR/prothrombin time - all used to monitor severity of hepatic failure and assist in patient stratification for optimal benefit in orthotopic liver transplantation NB/ monitor hepatic & renal function for signs of hepatic failure, and to assist in patient stratification for optimal benefit in orthotopic liver transplantation 3) tx - a) <8 hours since ingestion (>4 hrs since ingestion) - I) 1st line if blood paracetamol plotted on or above tx line - acetylcysteine: 150 mg/kg IV infusion over 1 hour, then 50 mg/kg infusion over 4 hours, then 100 mg/kg infusion over 16 hours. Repeat bloods for paracetamol after 21hr course of acetylcysteine II) + supportive care - may need intubation and ventilation or vasoactive infusions for blood pressure support, psychological support for mental illness, and education for those with accidental overdoses III) +/- anti-emetic - ondansetron: 4-8 mg IV as a single dose IV) +/- activated charcoal: 50 g orally/nasogastrically as a single dose if within 2hrs of ingestion of >75 mg/kg b) >8 hours since ingestion - I) as above - acetylcysteine + supportive care +/- anti-emetic - ondansetron: 4-8 mg IV as a single dose (not activated charcoal) II) +/- evaluation for liver transplant - if: arterial lactate concentration >3.5 mmol/L after fluid resuscitation; OR arterial pH <7.3, and lactate >3.0 mmol/L after fluid resuscitation; OR PT/INR >100 seconds/6.0 seconds AND encephalopathy grade 3 or more AND creatinine >292 micromol/L (3.3 mg/dL) within 24 hours AND a normal arterial pH

Describe the diagnosis and management of bladder calculi

1) s/s - a) can be asymptomatic, only have s/s if stone irritates the bladder wall or blocks the flow of urine eg/ b) lower abdominal pain c) in men, pain or discomfort in the penis or testicles d) dysuria & frequency e) difficulty urinating or interrupted urine flow f) haematuria or cloudy or abnormally dark-coloured urine 2) ix - a) urinalysis to exclude UTI b) CTKUB c) US pelvis d) sometimes x-ray pelvis . 3) tx - a) may pass naturally b) inc fluids to help small stones pass naturally. c) cystolitholapaxy - cystoscope inserted through your urethra and into bladder to view the stone, then use a laser, US or mechanical device to break the stone into small pieces and flushes the pieces from bladder d) surgical removal - if stones are large or too hard to break up, incision in bladder to directly remove stones

Describe the typical history of a patient with COPD including complications

1) s/s - a) cough - many years of 'smoker's cough' b) production of white/clear sputum c) wheeze d) breathlessness e) colds seem to settle on the chest, frequent exacerbations occur to give purulent sputum f) severe disease - breathlessness inc to things such as getting dressed g) systemic effects: hypertension, osteoporosis, depression, weight loss and loss of muscle mass with weakness h) pts may develop cor pulmonale 2) complications - a) acute exacerbation - infection b) polycythaemia c) pneumothorax (ruptured bullae) d) lung carcinoma e) respiratory failure - pink puffers + blue bloaters

Describe the presenting clinical features, natural history and mx of renal cysts

1) s/s - a) fairly common in older people and typically do not cause any symptoms or harm b) rarely cause complications eg/ fever, pain or tenderness betdddween the ribs and pelvis, upper abdominal pain, changes in urinary habits, haematuria 2) natural history - a) renal cysts are sacs of fluid that form in the kidneys. Most of the time, they are simple kidney cysts, meaning they have a thin wall and only water-like fluid inside. They are fairly common in older people and typically do not cause any symptoms or harm b) cause of renal cysts is not known, although age is a major risk factor. 1/3rd of people >70 have at least one simple renal cyst. It can be normal to have more than one simple cyst in each kidney, especially with increasing age 3) mx - a) generally do not require treatment unless they are causing symptoms or harming kidney function: b) sclerotherapy (aka percutaneous alcohol ablation) - insertion of a long needle through the skin and into the cyst under ultrasound guidance, then drain cyst & fill with an alcohol-based solution that causes the tissue to harden and shrink, reducing the chance of recurrence. Procedure is usually performed on an outpatient basis with a local anaesthetic c) surgery - larger cysts drained via laparoscopically

Summarise the clinical and biochemical features of congenital adrenal hyperplasia

1) s/s - a) genetic predisposition - 21-hydroxylase deficiency (21-OHD, autosomal recessive) b) failure to thrive & weight loss c) vomiting d) hypotension e) ambiguous genitalia - males have small testes in comparison to the phallus, and hyperpigmentation of the scrotum; females may have an enlarged clitoris, fused labia, and a urogenital sinus (classical form) f) hyperpigmentation - may be the only presenting sign in newborn males with classical simple virilising form 2) biochemical - a) inc serum 17-hydroxyprogesterone b) inc serum 11-deoxycortisol c) serum chemistry - hyponatraemic, hyperkalaemic, metabolic acidosis; azotaemia d) rapid ACTH stimulation test

Discuss the clinical presentation, laboratory features and complications of primary hyperparathyroidism. Outline the important clinical associations of hyperparathyroidism

1) s/s - a) hx of osteoporosis or osteopenia - bone loss from bone resorption due to excess PTH, may have associated bone pain b) poor sleep & fatigue & memory loss c) anxiety & depression d) myalgias & paraesthesias & muscle cramps e) constipation 2) ix - a) serum calcium - inc b) serum intact PTH with immunoradiometric or immunochemical assay - inc c) 25-hydroxyvitamin D level - may be low d) serum alkaline phosphatase - may be raised e) serum phosphorus - low or low normal f) 24-hour urinary calcium - high or normal in PHPT; low in familial hypocalciuric hypercalcaemia g) others - dual-energy x-ray absorptiometry (DEXA) scan for bone health, trabecular bone score, Tc-99m sestamibi scanning and ultrasonography, single photon emission CT + sestamibi scanning, CT or MRI neck 3) complications - a) of surgery - neck haematoma, recurrent & superior laryngeal nerve injury, hypocalcaemia, pneumothorax b) osteoporosis & bone fractures c) nephrolithiasis 4) associated with some inherited conditions (lead to hyperfunctioning parathyroid glands) eg/ a) MEN 1 & 2 & 4 b) HPT-jaw tumour syndrome c) familial isolated hyperparathyroidism

Describe the clinical features, investigation and treatment of alcohol intoxication

1) s/s - a) lower doses - mild sedation, tachycardia and poor coordination b) higher doses - slurred speech, trouble walking, and vomiting c) extreme doses - respiratory depression (<8 breaths/min), coma, or death d) complications include seizures, aspiration pneumonia, injuries including suicide, and low blood sugar e) others - euphoria, lowered social inhibition, severe confusion, unpredictable behaviour, stupor, sudden lapses into and out of unconsciousness, s/s of hypoxia f) may notice - smell of alcohol on breath or clothing, large pupils, faster-than-usual eye movements 2) ix - a) definitive diagnosis relies on a blood test for alcohol, usually performed as part of a toxicology screen, can also use breathalyzer units and field sobriety tests b) need to rule out other conditions eg/ hypoglycemia, stroke, use of other intoxicants, mental health issues etc c) may have X-rays or CT scans to check for fractures from falls and/or blows to head 3) mx - a) acute alcohol poisoning is a medical emergency due to the risk of death from respiratory depression or aspiration of vomit b) emergency treatment - stabilize and maintain an open airway and sufficient breathing, while waiting for the alcohol to metabolize, remove vomitus, if unconscious or impaired gag reflex, intubation of the trachea c) fluids, antiemetics, supplementary O2 d) tx hypoglycaemia with dextrose IV e) administer thiamine to prevent Wernicke-Korsakoff syndrome, which can cause a seizure (for chronic alcoholism, but in the acute context usually co-administered to ensure maximal benefit) f) hemodialysis if blood concentration is dangerously high (>400 mg/dL), and especially if there is metabolic acidosis

Describe the presenting clinical features, natural history and mx of transitional cell carcinoma of the renal pelvis

1) s/s - a) microscopic or macroscopic haematuria b) if tumour is located at the pelviureteric junction, then can have symptomatic hydronephrosis causing flank pain and clot-related renal colic may mimic renal calculi c) some only present once metastatic disease becomes symptomatic, with constitutional symptoms eg/ weight loss, or focal symptoms due to a metastatic deposit eg/ pathological fracture 2) natural hx - a) transitional cell carcinomas account for 85% of all uroepithelial tumours of the renal pelvis, remaining 5% made up of squamous cell carcinoma (the majority) and adenocarcinoma (rare) b) metastases are most frequently to liver, bone, and lung 3) mx - a) surgery - nephroureterectomy, taking not only the kidney but also the ureter and a cuff of the bladder at the vesicoureteric junction b) in low stage tumours and especially in patients with bilateral tumours or solitary kidneys - renal-sparing surgery may be attempted, in which tumours are locally excised often endoscopically (percutaneous or transurethral approach) c) most renal pelvis TCCs are low grade so prognosis is typically good, with a 5-year survival of over 90% d) high rate of recurrence due to field effect on the urothelium: ~40% of patients with an upper urinary tract TCC will go on to develop one or more TCCs of the bladder

Outline the clinical features of disseminated intravascular coagulation including laboratory tests used in diagnosis and tx

1) s/s - a) presence of underlying disorders eg/ major trauma/burns/organ destruction, sepsis/severe infection, severe obstetric disorders or complications, solid tumours and haematological malignancies, severe toxic or immunological reactions, and major vascular disorders (large aortic aneurysms or giant haemangiomas) b) oliguria, hypotension, or tachycardia - systemic s/s of circulatory collapse c) purpura fulminans, gangrene, or acral cyanosis - systemic signs of microvascular/macrovascular thrombosis d) delirium or coma - systemic s/s of microvascular/macrovascular thrombosis e) petechiae, ecchymosis, oozing, or haematuria - generalised bleeding, evidenced by at least 3 unrelated sites 2) ix - a) platelet count - dec due to excessive consumption b) prothrombin time (PT) - measures extrinsic and common coagulation pathways, prolonged in 50-70% of patients with DIC c) fibrinogen - dec due to excessive consumption d) D-dimer/fibrin degradation products - inc e) activated partial thromboplastin time (aPTT) - measures intrinsic and common coagulation pathways, prolonged in 50-60% of patients with DIC only f) imaging studies or other tests - dependent on underlying disorder and areas of thrombosis and haemorrhage g) ix to consider - thrombin time, protamine test, factor V, VIII, X, XIII h) emerging tests - D-dimer (monoclonal antibody test), antithrombin III, fibrinopeptide A (FPA), prothrombin fragment 1 and 2, protein Ctreatment 3) mx - a) 1st line - tx underlying disorder: aggressive tx of underlying disorder to remove trigger is the most effective therapy in management of DIC and should be applied to all patients b) + platelets + coagulation factors and coagulation inhibitors when appropriate

Discuss the clinical features and management of shingles

1) s/s - a) risk factors - >50 yo, HIV-positive, chronic corticosteroid use, chemotherapy, malignancies. b) localised pain in a dermatome - I) burning, stinging, itching, or tingling and ranges from mild to severe II) pain, localised to the affected dermatome, can precede the rash by days to weeks III) mostly thoracic and trigeminal nerves c) pruritis - may be present in affected dermatome d) rash - erythematous maculopapular rash, followed by appearance of clear vesicles. The eruption occurs in segments innervated by the affected sensory ganglion, but does not cross the midline. The vesicles eventually pustulate and form crusts e) corneal ulceration - if trigeminal nerve, pain in the affected eye and reduced vision 2) mx - a) immunocompetent - I) 1st line - oral antiviral therapy eg/ famciclovir or valaciclovir for 7 days (dec duration of viral shedding, stop the formation of new lesions, and reduce pain severity), start within 48 to 72 hours of rash onset and administer for 7 days II) mild pain - + simple analgesics ± calamine lotion eg/ paracetamol, ibuprofen + calamine lotion topical: apply to affected area(s) up to four times daily when required III) moderate-to-severe pain - + opioid analgesics ± topical analgesic eg/ oxycodone: 5 mg PO, lidocaine topical: (5% ointment) or calamine lotion topical IV) eye involvement - + prompt referral to ophthalmologist: IV aciclovir is given as needed for retinitis, may need abx ophthalmic ointment to protect the ocular surface, and topical corticosteroids. Systemic corticosteroids may be indicated in moderate-to-severe pain, especially if there is oedema surrounding the orbital area b) postherpetic pain - I) mild pain 1st line - paracetamol or NSAID II) +/- weak opioid analgesic eg/ codeine 15-60mg PO III) 2nd line - topical capsaicin moderate-to-severe pain - 1st line - strong opioid, amitriptyline, or anticonvulsant eg/ tramadol, oxycodone, amitriptyline, gabapentin

Outline the clinical features and laboratory diagnosis of thalassaemia

1) s/s - a) risk factors - country of origin or ancestry (Mediterranean, Middle East, northern Africa, India, and almost all of Southeast Asia), fmx b) asymptomatic - B-thalassaemia trait c) lethargy - major, intermedia: due to anaemia d) abdominal distension - major, intermedia: due to hepatosplenomegaly e) failure to gain weight - major, intermedia f) pallor - major, intermedia: anaemia g) spinal changes - all, major, intermedia: osteopenia related to iron overload h) large head - major, intermedia: frontal and parietal bossing i) chipmunk facies, misaligned teeth - major, intermedia j) hepatosplenomegaly - major, intermedia k) jaundice - major, intermedia, chronic elevations of bilirubin may result in gallstones 2) ix - a) FBC - microcytic anaemia, normal to elevated leukocyte and platelet counts from generalised haematopoietic hyperactivity, all decreasing as the spleen enlarges b) peripheral smear - microcytic red cells, tear drops, microspherocytes, target cells, some fragments, large number of nucleated red cells c) reticulocyte count - inc (variable, corresponding to severity) d) haemoglobin analysis - newborn screening haemoglobin analysis is standard in many resource-rich countries; beta-thalassaemia major: minimal to no Hb A, elevated Hb F and HbA2; beta-thalassaemia intermedia: decreased Hb A, elevated Hb F and HbA2; beta-thalassaemia trait: mostly Hb A, elevated Hb F and HbA2 e) LFTs - if B-thalassaemia major and intermedia, inc total and unconjugated bilirubin, elevated LDH f) plain x-rays of skull - widening of the diploeic space, facial deformity if B-intermedia or major g) abdominal US - liver and spleen enlargement h) plain x-rays of long bones - widening of the diploeic space, evidence of osteopenia i) consider - genetic testing (confirms diagnosis), HLA typing (if stem cell transplantation is a consideration)

Describe the presenting clinical features, natural history, and mx of Wilm's tumour

1) s/s - a) risk factors - fmx, age 2 to 5 years, congenital overgrowth or non-overgrowth syndromes and congenital urogenital anomalies b) upper abdominal/flank mass or swelling - usually painless, non-tender, firm, smooth, and unilateral, retroperitoneal ('ballotable'), and does not move with respirations c) abdominal distension d) may have localised or diffuse abdominal pain due to intra-abdominal spread and/or tumour rupture e) pallor - due to anaemia: normocytic normochromic (more common and is secondary to tumour haemorrhage) or iron-deficiency (secondary to blood loss from haematuria) 2) natural history - multi-step process and thought to develop via a pre-malignant stage. Chromosomes indicated - 11p13 (WT1 and PAX6 gene) and 11p15 (WT2 gene, IGFII, H19, p57kip2), as well as 17q (FWT1), 19q (FWT2), aberrant expression of the IGF-II oncogene, resulting from the loss of imprinting of the maternal IGF-II gene 3) mx under SIOP/UKCCSG criteria - a) stage I-IV - I) preop chemo eg/ vincristine + dactinomycin for 4 weeks II) + surgery - radical nephrectomy III) +/- post chemo (definitely if stage II +) IV) + radiotherapy if stage II+ b) stage V - I) preop chemo depending on local protocol II) + surgery - nephron-sparing surgery eg/ tumourectomy, wedge resection, polar resection, heminephrectomy, nephrectomy on one side, and partial resection, thus avoiding bilateral radical nephrectomies III) + postop chemo - see local protocol c) multiple adverse prognostic factors or multiple relapses - as above, and consider bone marrow transplant

Describe the clinical features and laboratory diagnosis of chronic myeloid leukaemia and outline the principles of management

1) s/s - a) risk factors - middle age, male sex, and exposure to ionising radiation b) splenomegaly c) SOB on exertion d) LUQ discomfort or fullness (due to splenomegaly or splenic infarct) e) epistaxis (due to thrombocytopenia or abnormal platelet function) f) arthralgia (inc uric acid production from extensive cell turnover) g) sternal tenderness (bone marrow expansion of the sternum) h) weight loss i) excessive sweating 2) ix - a) FBC - inc WCC, may have: anaemia, thrombocytosis or normal or thrombocytopenia b) complete metabolic profile - may have elevated potassium, LDH, or uric acid c) peripheral blood smear - mature or maturing myeloid cells, elevated basophils and eosinophils d) bone marrow biopsy - granulocytic hyperplasia e) cytogenetics - Philadelphia chromosome positive t(9,22) f) quantitative reverse transcription PCR (qRT-PCR) including breakpoint analysis - only quantitative method of assessing molecular response to therapy, carried out every 3-6 months, detection of BCR-ABL fusion 3) mx - a) chronic or accelerated phase - I) 1st line - tyrosine kinase inhibitor eg/ imatinib or dasatinib or nilotinib II) not in remission, relapsed, or progression of disease after tyrosine kinase inhibitor - allogeneic haematopoietic stem cell transplant (HSCT) + high-dose induction chemotherapy b) blast phase - tyrosine kinase inhibitor + high-dose induction chemotherapy followed by allogeneic haematopoietic stem cell transplant (HSCT) c) relapse after allogeneic haematopoietic stem cell transplant (HSCT), or allogeneic HSCT contraindicated - consider use of interferon or hydroxycarbamide, or referral for inclusion in clinical trials

Describe the clinical presenting features, diagnosis and management of benign prostatic hyperplasia and possible complications

1) s/s - a) risk factors eg/ >50yo, fmx b) storage symptoms eg/ frequency, urgency, and nocturia c) voiding symptoms eg/ weak stream, hesitancy, intermittency, straining, incomplete emptying, and post-void dribbling 2) ix - a) urinalysis - can have pyuria b) PSA - inc greater than that of age guideline c) International Prostate Symptom Score - self-administered patient questionnaire covering both irritative and obstructive voiding symptoms: mild = <7, moderate = 8-19, severe = 20+; + global bother score d) volume charting - diary of frequency and volume of voiding e) consider - US (may show hydronephrosis, mass, urolithiasis), CTAP (may show mass, hydronephrosis, urolithiasis), cystoscopy (mas show mass, stone, stricture), uroflowmetry (<20 mL/second), urodynamic study (shows abnormal bladder pressure, abnormal bladder voiding) 3) mx - a) non-bothersome symptoms - I) 1st line - watchful waiting: self-monitoring of symptom progression by pt & yearly follow-up by the physician to re-assess the condition II) + behavioural management programme: to dec symptoms & inc QOL eg/ limitation of fluids including caffeine, bladder training focused on timed and complete voiding, and treatment of constipation b) bothersome symptoms with no indications for surgery - I) 1st line - alpha-blocker + behavioural management programme: terazosin or doxazosin or alfuzosin etc II) 1st line - 5-alpha-reductase inhibitor + behavioural management programme: finasteride or dutasteride (takes months for effect) III) 1st line - phosphodiesterase-5 (PDE-5) inhibitor + behavioural management programme: sildenafil or tadalafil or vardenafil IV) 1st line - anticholinergic agent + behavioural management programme: tolterodine or fesoterodine or oxybutynin or solifenacin V) 2nd line - combination alpha-blocker with 5-alpha-reductase inhibitor or PDE-5 inhibitor or anticholinergic agent + behavioural management programme c) bothersome symptoms with indication for surgery: prostate <80 grams - I) 1st line - minimally invasive therapy: transurethral microtherapy (TUMT), transurethral needle ablation (TUNA), and prostatic urethral lift (PUL) II) 1st line - moderately invasive therapy: TURP, transurethral vaporisation of the prostate (TUVP), laser vaporisation d) bothersome symptoms with indication for surgery: prostate ≥80 grams - I) 1st line - open prostatectomy or laser enucleation (HoLEP or ThuLEP) 4) complications - a) BPH progression b) urinary - UTI, haematuria c) bladder - overactive bladder, acute urinary retention, bladder stones d) kidneys - renal insufficiency e) sexual dysfunction

Describe the clinical features and treatment of anaerobic and synergistic gangrene

1) s/s - a) risk factors eg/ DM, atherosclerosis, smoking, hypercoagulable states, drug abuse, malignancy, renal disease, trauma or abdominal surgery, alcoholism, and malnutrition b) pain - hx of chronic claudication-type pain in patients with ischaemic gangrene; sudden onset of pain in infectious gangrene c) oedema with overlying erythema d) sometimes skin discoloration - ecchymoses, purpura, skin blebs, and haemorrhagic bullae may develop in gangrene e) crepitus - with gas gangrene, gentle palpation may demonstrate crepitus f) diminished pedal pulses and ankle-brachial index - ischaemic gangrene g) low-grade fever and chills - infectious gangrene 2) mx - a) necrotising fasciitis awaiting confirmation of microbial culture and sensitivity results - I) 1st line - surgical debridement + intensive supportive care: surgical incisions extend beyond the areas of visible necrosis and the entire necrotic area excised. Intensive haemodynamic support with IV infusion II) + empiric broad-spectrum abx: vancomycin or linezolid + piperacillin/tazobactam or imipenem/cilastatin or meropenem or ertapenem b) confirmed type I necrotising fasciitis (polymicrobial) - I) 1st line - ICU + urgent surgical debridement ± amputation: II) + local irrigation with bacitracin-infused normal saline III) + broad-spectrum IV abx: vancomycin or linezolid + piperacillin/tazobactam or imipenem/cilastatin or meropenem or ertapenem c) confirmed type II necrotising fasciitis (streptococcal) - I) 1st line - ICU + urgent surgical debridement ± amputation II) + IV abx: benzylpenicillin sodium + clindamycin III) streptococcal toxic shock: +/- IVIG d) gas gangrene - I) 1st line - ICU + urgent surgical debridement ± amputation: fasciotomy could be necessary to treat compartment syndrome. Daily debridement is necessary to remove all necrotic and infected tissue II) + IV abx: benzylpenicillin sodium + clindamycin III) +/- hyperbaric oxygen therapy e) ischaemic gangrene - I) 1st line - IV heparin II) with threatened or non-viable extremity: life expectancy >2 years: + surgical revascularisation ± amputation III) with threatened or non-viable extremity: life expectancy ≤2 years: + percutaneous transluminal angioplasty (PTA) ± amputation IV) with viable extremity: + thrombolytic therapy: doesn't have 6 Ps (pain, paraesthesia, paralysis, pulseless, perishingly cold, pale) V) with phlegmasia cerulea dolens: + thrombolytic therapy

Outline the clinical features of aplastic anaemia and describe the laboratory diagnosis and principles of treatment

1) s/s - a) risk factors eg/ paroxysmal nocturnal haemoglobinuria (PNH), hepatitis, and drug exposure (eg/ NSAIDs) b) hx of recurrent infection c) fatigue, pallor d) hx of bleeding or easy bruising e) tachycardia f) dyspnoea g) persistent warts 2) ix - a) FBC with differential - pancytopenia is common, need ≥2 cytopenias among the following: Hb <100 g/L (<10 g/dL), platelet <50 × 10⁹/L, and/or absolute neutrophil count <1.5 × 10⁹/L for diagnosis b) reticulocyte count - dec = anaemia is hypoproductive, as opposed to hyperproductive anaemias such as haemolytic anaemia c) bone marrow biopsy and cytogenetic analyses - hypocellular marrow, no abnormal cell population (such as blasts) and no fibrosis d) consider - serum B12 and folate (severe deficiency may be an alternative cause of pancytopenia), HIV (another cause), LFTs (abnormal in inherited syndrome eg/ dyskeratosis congenita or Shwachman-Diamond syndrome), autoantibody screen, flow cytometry for glycosylphosphatidylinositol (GPI)-anchored proteins, CXR (if malignancy, infection, or lung fibrosis is suspected), abdo US (if malignancy, Fanconi anaemia, or telomeropathy is suspected), genetic tests if suspect congenital syndrome, CT if telomeropathies or dyskeratosis congenita suspected 3) mx if non-severe acquired disease - a) 1st line - monitoring ± immunosuppressive therapy: lymphocyte immunoglobulin, anti-thymocyte globulin or antithymocyte immunoglobulin b) + methylprednisolone c) + ciclosporin d) any potential aetiological agent (eg/ chloramphenicol, NSAIDs) or exposure (eg/ benzene) should be withdrawn e) pts can be monitored conservatively with regular FBCs f) +/- blood product transfusions g) +/- abx & antifungals (when infection develops) 4) mx of severe/very severe acquired disease - a) 1st line - matched related donor allogeneic stem cell transplantation: I) beforehand need conditioning with: high-dose cyclophosphamide with antithymocyte globulin (ATG) or alemtuzumab II) graft-versus-host disease (GVHD) prophylaxis: methotrexate + calcineurin inhibitor (e.g., ciclosporin or tacrolimus) b) any potential aetiological agent (eg/ chloramphenicol, NSAIDs) or exposure (eg/ benzene) should be withdrawn c) +/- blood product transfusions d) +/- abx & antifungals (when infection develops) e) 2nd line - immunotherapy: as above f) 3rd line - matched unrelated donor allogeneic stem cell transplantation g) 4th line - eltrombopag

Describe the diagnostic evaluation of an intra-abdominal abscess

1) s/s - a) risk factors eg/ recent surgery or trauma, recent appendicitis, recent diverticulitis, diabetes, and malignancy b) fever or hypothermia c) abdo pain d) hx of IBD or biliary disease - ? hepatic abscess e) tachycardia f) change in bowel habits/abnormal bowel function g) prolonged ileus h) anorexia/lack of appetite & N&V i) hx of biliary disease 2) ix - a) WBC count - inc with increased proportion of granulocytes (left shift), persistent leukocytosis or bandaemia, or leukopenia b) abdominal CT scan - normally with contrast c) consider - serum CRP & ESR (inc), gram stain of abscess fluid/drainage culture (helps determine pathogenic organism for tx), serum glucose (if also diabetic), serum LFTs (inc aminotransferases +/- alkaline phosphatase), serum albumin (low if longstanding abscess), abdo USS (may be a useful aid in characterising abscess), abdo MRI (pregnant), indium-labelled leukocytes

Describe the pathology, clinical presenting features diagnosis, management and follow-up of transitional cell carcinoma of the bladder

1) s/s - a) risk factors eg/ tobacco exposure, male, age >55, exposure to chemical carcinogens, pelvic radiation, systemic chemotherapy, fmx of bladder cancer b) haematuria - most often gross painless haematuria, episodes of haematuria are typically intermittent, can also be microscopic c) dysuria - associated with aggressive bladder cancer 2) ix - a) urinalysis - RBC casts and crenated red cells are seen with glomerular bleeding b) urine cytology - +ve with carcinoma in situ or high-grade tumours c) renal and bladder ultrasound - bladder tumours and/or upper tract obstruction may be seen d) CT urogram - bladder tumours, upper urinary tract tumours, and/or obstruction may be seen e) cystoscopy - visualises bladder tumours and enables pathological diagnosis f) consider - IV urogram (may show filling defects indicative of bladder tumour), FBC (normal or mild anaemia), chemistry profile (normal or may show elevated alkaline phosphatase), CXR (usually normal), CT or MRI abdomen and pelvis (negative for stone disease; may reveal evidence of primary bladder cancer and/or metastatic disease), MR urogram (normal or may indicate presence of upper urinary tract lesions), bone scan (if alk phosph inc - normal or hot spots indicative of bony mets), urinary markers (+ve) 3) mx - non-muscle-invasive tumours - a) low grade - I) 1st line - transurethral resection: need follow up cystoscopy at 3 months, then every 6 months for 2 years, and then once a year indefinitely II) + immediate post-operative intravesical chemotherapy: mitomycin b) intermediate risk - as low risk I) + delayed intravesical bacilli Calmette-Guerin (BCG) immunotherapy or intravesical chemotherapy c) high risk - as intermediate risk I) 2nd line - radical cystectomy locally invasive tumours - a) organ-contained: T2a or T2b or non-organ contained: T3a or T3b: I) 1st line - radical or partial cystectomy + pelvic LN dissection II) +/- preoperative chemotherapy: methotrexate + vinblastine + doxorubicin + cisplatin III) +/- post-operative chemotherapy or chemo-radiotherapy: as above IV) 2nd line - immunotherapy: atezolizumab or pembrolizumab b) non-organ-contained: T4a or T4b: I) 1st line - chemotherapy: methotrexate + vinblastine + doxorubicin + cisplatin II) +/- radiotherapy III) +/- radical cystoprostatectomy IV) 2nd line - immunotherapy: atezolizumab or pembrolizumab c) metastatic disease - I) 1st line - chemotherapy: cisplatin + gemcitabine + paclitaxel II) +/- surgery or radiotherapy III) 2nd line - immunotherapy: atezolizumab or pembrolizumab 4) follow-up - lifelong risk of tumour recurrence so need lifelong monitoring by cystoscopy: high-risk - cystoscopy every 3 months for 2 years, then every 6 months for 2 to 3 years, then yearly; low-risk - cystoscopy at 3 months, but may progress rapidly to annual examinations if 3- and 9-month examinations are negative

Describe the symptoms and signs of pancreatic cancer depending on location of the tumour within the gland. Outline investigations indicated

1) s/s - a) risk factors eg/ tobacco use, family history, and other hereditary cancer syndromes b) jaundice - suggests biliary obstruction or, very rarely, hepatic or hilar nodal metastases - head of pancreas c) non-specific upper abdominal pain or discomfort d) persistent back pain - retroperitoneal metastases, cancer in body or tail of pancreas e) weight loss and anorexia f) aged 65-75 2) ix - a) abdominal US - may see pancreatic mass, dilated bile ducts, liver metastases b) pancreatic protocol CT - may demonstrate a mass in the pancreas and the extent of local or distant spread c) LFTs - bilirubin, alk phosph & GGT inc in obstructive jaundice; aminotransferases normal or slightly inc d) prothrombin time (PT) - derangement of vit K-dependent clotting factors causes prolonged PT e) FBC - platelets dec in DIC; anaemia in GI bleeding f) cancer antigen (CA)19-9 biomarker - inc, sensitivity of 70-90% & specificity of 90% g) PET h) endoscopic retrograde cholangiopancreatography (ERCP) i) magnetic resonance cholangiopancreatography j) endoscopic ultrasound k) staging laparoscopy (with laparoscopic US) l) biopsy

List the different clinical patterns of MS and describe the different courses that MS can take. Describe commonly encountered clinical features of MS relapses

1) s/s - a) visual disturbance in one eye - graying or blurring of vision in 1 eye (can be described as looking through petroleum jelly). May have pain in moving that eye and describe loss of colour discrimination, particularly reds b) peculiar sensory phenomena - pts can have a patch of wetness or burning, or hemibody sensory loss or tingling. In particular, banding or hemibanding is associated with spinal cord lesions. Lhermitte's sign (electric shock-like sensations extending down the cervical spine radiating to the limbs) and trigeminal neuropathy or neuralgia are other possible sensory findings in MS c) foot dragging or slapping - gradual onset of weakness after walking several streets or several miles such that the foot slaps the ground. This weakness resolves with rest d) leg cramping - involuntary movement in the lower leg with cramping or jerking in the calves, particularly at night or while driving e) fatigue - may be related to MS primarily, but often worsened dramatically by poor sleep hygiene, depression, restless legs, urinary frequency, or underlying sleep apnoea f) urinary frequency - damage to CNS resulting in urinary retention and detrusor instability, UTIs more frequent in patients with urinary retention g) bowel dysfunction - constipation common, h) spasticity/inc muscle tone - damage to CNS, commonly affects legs and can be very unpleasant and painful, disturbing sleep as well as ambulation i) inc deep tendon reflexes - particularly clonus at ankles and often asymmetrical j) imbalance/incoordination - wide-based gait and/or limb ataxia indicate cerebellar dysfunction, which occurs frequently in MS 2) diagnostic criteria according to Schumacher - age of onset 10-50, objective neurological signs present on examination, neurological s/s indicative of CNS white matter disease, dissemination in space - 2+ non-contiguous anatomical areas involved, dissemination in time - 2+ episodes of worsening lasting at least 24 hours separated by one month or more, or progression over at least six months, no better explanation by a physician competent in neurology 3) types of MS - a) relapsing remitting multiple sclerosis (RRMS) - most common type at ~85%, presents with acute or sub-acute onset of neurological symptoms, from which people may recover either completely or partially. Further relapses may then occur at irregular intervals. If recovery from relapses is incomplete people may accrue neurological deficit and disability b) secondary progressive multiple sclerosis (SPMS) - over time RRMS may convert to a disease pattern of gradual progression with accumulating irreversible neurological deficit and disability (SPMS). The proportion of people developing progressive disease increases with length of follow up. People with SPMS may continue to have superimposed relapses c) primary progressive multiple sclerosis (PPMS) - characterised by progressive disease from onset, with a gradual accumulation of neurological deficit or disability, without relapse or remission and accounts for about 10%-15% of multiple sclerosis. The average age of onset, about 40 years, is later than in RRMS and comparatively more men are affected resulting in an equal male:female ratio.The age of onset and rate of progression is similar to the progressive phase of SPMS d) progressive relapsing multiple sclerosis (PRMS) - progressive disease from onset with superimposed relapses. The progressive disease is predominant and PRMS is considered to be largely similar to PPMS. 10%-15% of people with PPMS will have superimposed relapses

Describe the clinical and pathological features of interstitial lung disease. Outline the common causes and list the differential diagnosis in patients who present with established pulmonary interstitial fibrosis

1) s/s - dyspnoea (often exertional), cough (mom-productive, can be severe). Crackles (end expiratory, basilar, 'velcro'), weight loss, malaise, fatigue 2) pathology - resulting injury from initiating event triggers a pro-inflammatory and pro-fibrotic response that includes an influx of macrophages, fibroblasts, and other inflammatory cells. There is dysregulation of the normal tissue repair process; fibroblastic and myofibroblastic activity persists, resulting in formation of fibroblastic foci, which are a pathological hallmark of the disease, continued deposition of extracellular matrix (ECM) proteins, and progressive fibrosis. May be a role of failed cellular regeneration mediated by shortened telomere length. Pattern of injury is heterogeneous and, at the time of biopsy, fibroblastic foci can be found in various stages of development adjacent to normal alveolated parenchyma 3) aetiology - unknown, unidentified insult causes damage to the alveolar epithelium, endothelium, and basement membrane. Cigarette smoke, organic or metal dust, GORD, diabetes, and infection have each been associated with IPF; however, the exact inciting exposure or exposures remain unknown 4) DD - pneumonia, respiratory bronchiolitis-associated interstitial lung disease, CT disease IPF, drug-related pulmonary fibrosis, asbestosis, hypersensitivity pneumonitis, sarcoidosis, Langerhans' cell histiocytosis, lymphangioleiomyomatosis 5) risk factors - fmx, smoking, inc age, male

Outline the clinical presentation of a patient with OSA, describe the use of sleep studies in its investigation and outline the principles of treatment

1) s/s - excessive daytime sleepiness, apnoeas, episodic gasping, restless sleep, insomnia, chronic snoring, neurocognitive dysfunction eg/ problems with attention, learning, and memory a) risk factors - obesity, male sex, maxillomandibular anomalies eg/ narrowing of jaw, overbite, overjet, malocclusion, CV disease, macroglossia, endocrine disorders eg/ T2DM, metabolic syndrome, polycystic ovary syndrome, hypothyroidism, and acromegaly, fmx, GORD, Down's 2) sleep studies/ polysomnography (PSG) - definitive test a) full night study: patient may return for CPAP titration, split study: diagnosis and CPAP titration performed on the same night b) apnoeas and hypopnoeas are scored and added to determine the AHI, which is used to establish the presence of OSA I) AHI of ≥15 is confirmatory, but ≥5 episodes/hour is sufficient in a symptomatic patient or in a patient with HTN, ischaemic cardiac disease, history of stroke, excessive daytime sleepiness, insomnia, mood disorder, or cognitive dysfunction c) PSG includes EEG, electro-oculographic recording, air flow assessment, EMG, capnography, oesophageal manometry, ECG, and pulse oximetry 3) tx - a) Apnoea-Hypopnoea Index (AHI) or Respiratory Event Index (REI) ≥5 episodes/hr - I) 1st line - CPAP II) 2nd line - oral appliance therapy eg/ mandibular repositioning appliances (MRAs): attach to the upper and lower teeth and effect mandibular protrusion., tongue retaining devices: advance the tongue anteriorly III) 2nd line - implantable hypoglossal neurostimulation IV) 3rd line - upper airway surgery eg/ uvulopalatopharyngoplasty, tonsillectomy, lateral pharyngoplasty, transpalatal advancement pharyngoplasty, expansion sphincter pharyngoplasty, and maxillomandibular advancement (MMA), genioglossus advancement, hyoid suspension, midline glossectomy, tongue suture suspension, epiglottoplasty V) with concurrent obesity - + weight loss ± bariatric surgery VI with persistent hypersomnolence - + modafinil or armodafinil VII) with positional component - + positional therapy: maintains a non-supine sleep position in individuals in whom AHI is low in a non-supine position and polysomnographic documentation of effectiveness is advised b) AHI or REI of ≥5 episodes/hour with discrete anatomical lesions - I) 1st line - upper airway surgery II) as above

Understand the presenting features, investigation and management of Barrett's oesophagus

1) s/s - may have none, otherwise long history of gastro-oesophageal reflux and, occasionally, dysphagia often middle aged, white men, may smoke & be obese 2) ix - a) upper GI endopscopy with biopsy - needs histological corroboration b) when high-grade dysplasia or cancer is found on surveillance endoscopy, endoscopic ultrasound is advisable to evaluate for surgical resectability c) pt with a columnar-lined distal oesophagus without confirmed intestinal metaplasia on biopsy must be followed up with further endoscopies and biopsies 3) mx - a) surveillance with endoscopy every 2 yrs b) 1st line: PPI eg/ omeprazole 20mg c) +/- radiofrequency ablation (used if dysplasia) d) 2nd line: anti-reflux surgery e) oesophagectomy may be considered if high grade dysplasia

List and discuss the symptoms and signs and natural history of an untreated pseudocyst

1) s/s - no specific s/s a) may have - abdominal pain, anorexia, or an abdominal mass after an episode of pancreatitis b) rarely have jaundice or sepsis from an infected pseudocyst c) pleural effusion is also a common finding, may also have fever, scleral icterus 2) natural history - a) most pseudocysts resolve without interference, and patients do well without intervention b) outcome is much worse if develop complications or have the cyst drained c) complications - bleeding, GI obstruction

Indicate the risk factors associated with C. difficile infection and discuss its prevention and treatment

1) s/s - watery diarrhea, fever, nausea, and abdominal pain 2) risks - a) antibiotic use b) PPIs c) hospitalisation d) other health problems e) older age f) elemental diet 3) prevention - a) hand washing b) terminal room cleaning in hospital c) dec abx use d) ?probiotics 4) tx - a) ORT b) abx - metronidazole (10/7) or vancomycin (severe disease) or fidaxomicin NB/ don't sue meds that slow or stop diarrhea eg/ loperamide, as they may worsen C. difficile disease c) consider cholestyramine (slows bowel motility & helps prevent dehydration) d) ?probiotics e) stool transplant aka faecal bacteriotherapy if abx don't work f) surgery - if severe C. difficile colitis, colectomy may improve the outcomes

List the common causes of acute lower gastrointestinal bleeding

1) s/s of lower GI bleeding: a) maroon stools - RHS of colon b) bright red blood per rectum - LHS of colon c) young pt with infectious or noninfectious (idiopathic) colitis - fever, dehydration, abdominal cramps, hematochezia d) older patient with diverticular bleeding or angiodysplasia - painless bleeding & minimal symptoms e) multiple comorbidities - ischemic colitis, abdominal pain, varying degrees of bleeding f) chronic GI bleeds usually present with anaemia 2) causes of lower GI bleeding (adults): a) diverticular disease - diverticulosis/diverticulitis of SI or colon b) IBD - Crohn's, ulcerative colitis c) noninfectious gastroenteritis & colitis d) benign anorectal diseases - haemorrhoids, anal fissure, fistula-in-ano e) neoplasia - malignant neoplasia of SI, colon, rectum, and anus f) coagulopathy g) arteriovenous malformations (AVMs) h) angiodysplasia (vascular malformation in proximal colon) i) ischaemic colitis j) polyps k) hookworm (most common cause worldwide, rare in UK) NB/ massive bleeding from lower GI is rare, usually diverticular or ischaemic colitis NB2/ most acute lower GI bleeds start and stops spontaneously - surgery rarely required. Persistent bleeds should be managed as in upper GI bleeds (stop blood thinners, consider fluids, watch rate+volume, Hb<10) Diagnosis made on history, rectal exam+/- proctoscopy, colonoscopy, angiography (rare)

Specify the symptoms and common causes of acute upper gastrointestinal bleeding

1) s/s of upper GI bleeding: a) hematemesis /melena (bleed proximal to cecum) b) syncope, presyncope (severe intravascular volume depletion) c) dyspepsia d) epigastric pain or diffuse abdominal pain e) heartburn f) dysphagia g) weight loss h) jaundice i) pallor + sweating 2) causes of upper GI bleeding: a) bleeding peptic ulcers - need to eradicate H. Pylori b) Boerhaave - vomiting = lower oesophagus & upper stomach are forcibly inverted - can cause mucosal tear of lower oesophagus or upper stomach c) Mallory-Weiss tear (tear in mucosa of gastric cardia - GOJ). Classically seen after alcoholic "dry heaves" d) acute stress gastritis - mostly pts who have undergone episodes of shock, multiple trauma, ARDS, systemic respiratory distress syndrome, acute renal failure, and sepsis e) Dieulafoy lesions - vascular malformation of proximal stomach f) NSAIDs - cause ulcers g) gastric Ca h) reflux oesophagitis (2-5%) NB/ all pts with significant GI bleed should be seen in hospital, but 85% stop bleeding spontaneously within 48 hours NB2/ Rockall & Blatchford scores assess risk of rebleed and mortality on following criteria: age, co-morbidity, shock, endoscopic Dx, ulcer, chronic liver disease

Describe the procedures that should be followed in taking a sample of blood from a patient and submitting it for cross matching and transfusion

1) sample is required prior to a transfusion to ensure compatibility of blood groups between donor and recipient and, (for red cell transfusion) to screen patients for atypical red cell antibodies which can potentially cause reactions 2) transfusion samples must be labelled with: positive patient identification based on asking the patient (where possible) to state their a) surname and first name b) date of birth, also record: c) hospital number d) sex e) ward f) date and time sample was taken g) check these details match with the patient's wristband and the request

List the underlying diseases or disorders associated with vasospastic changes in the extremities

1) scleroderma 2) systemic lupus erythematosus 3) rheumatoid arthritis 4) Sjögren's syndrome 5) dermatomyositis 6) polymyositis 7) mixed connective tissue disease 8) cold agglutinin disease 9) Ehlers-Danlos syndrome 10) eating disorders 11) haematological conditions eg/ clotting factor mutations, polycythaemia ruba vera, leukaemia 12) hepB + C 13) nervous compression eg/ carpal tunnel, thoracic outlet syndrome 14) vascular causes eg/ thrombosis, atherosclerosis, Buerger's disease 15) Beta-blockers + ergot-containing drugs 16) cytotoxic drugs - particularly chemotherapeutics 17) jobs involving vibration or cold NB/ smoking aggravates

Outline the long -term complications of bacterial meningitis

1) seizures 2) hearing loss 3) brain damage - memory difficulty, learning disabilities 4) gait problems 5) kidney failure 6) septic shock 7) death 8) venous sinus thrombosis 9) cerebral oedema 10) hydrocephalus 11) papilloedema 12) chronic headache NB/ complications more common with bacterial infection NB2/ risk factors for hearing loss include infecting pathogen, older age, females, severity of infection. Seizures in ~20% of patients over 1, ~40% <1 NB2/ mortality varies between pathogens: Pneumococcal 20-30%, Listeria 15-30%, Meningococcal 5-15%, Hib 3-5%, Viral 1%

Outline the features that distinguish seizures from syncope

1) seizures - tongue biting, aura, head turning during spell, unresponsive, unusual posture, limb movement, oramesia of spells, confusion after spells, cyanosis, automatisms, frothing at mouth 2) syncope - lightheaded spells, sweating beforehand, spell with prolonged sitting or standing, pallor

Describe the sources of bias that can arise in studies of disease

1) selection bias - occurs when select sample or data wrong, usually accidentally working with a specific subset of audience instead of whole, so sample unrepresentative of the whole population. 2) recall bias - respondent doesn't remember things correctly 3) observer bias - researcher subconsciously projects his/her expectations onto the research eg/ influencing participants (during interviews and surveys) or doing some serious cherry picking (focusing on the statistics that support our hypothesis rather than those that don't.) 4) survivorship bias - researcher focuses only on that part of the data set that already went through some kind of pre-selection process - and missing those data-points, that fell off during this process (because they are not visible anymore). 5) reporting bias - skew in the availability of data, such that observations of a certain kind are more likely to be reported 6) analytical bias - from way results are evaluated 7) exclusion bias - systematic exclusion of certain individuals from the study 8) attrition bias - loss of participants e.g. loss to follow up during a study 9) publication bias - occurs in published academic research, when the outcome of an experiment or research study influences the decision whether to publish or otherwise distribute it

Describe how lead time bias, duration bias and selection bias affect the assessment of screening

1) selection bias - study population is not representative of the larger population, possibly because of a poor sampling process, or because of a lot of individuals are lost to follow-up 2) lead time (information bias) - the systematic error of apparent inc survival from detecting disease in an early stage 3) duration/length time bias - screening initiative is more likely to detect slow-growing disease

Calculate sensible and insensible fluid and electrolyte losses in routine post-operative care

1) sensible fluid loss = urine, ~1500 mL/day (30 mL/hr is considered adequate urine output) 2) insensible = vaporization from lungs - 400 mL/day and skin - 600 mL/day: assists in regulating body temperature, inc with accelerated body metabolism (also includes faeces) 3) abnormal losses include those from drains, stomas, NG tubes, vomit and fistulae 4) normal daily requirement of Na is 0.7-1mmol/kg, and K is 0.7-1mmol/kg 5) fluid replacement should match total output

Define sensitivity, specificity, predictive value and likelihood ratio, and discuss their inter-relationships, the effect of changes in disease prevalence, and the effects of combining tests in series or in parallel

1) sensitivity of test: proportion of those with a disease which it correctly identifies 2) specificity of test: proportion of those without the disease which it correctly identifies 3) positive predictive value: probability that a subject has the disease given that they have a positive result - determined by prevalence as well as by sensitivity & specificity 4) negative predictive value: probability that a subject does not have the disease given that they have a negative result 5) likelihood ratio - likelihood that a positive test would be expected in a patient with the disease compared to the likelihood in a patient without eg/ if likelihood pt with coeliac disease +ve = 97%, likelihood patient without disease +ve = 3%, likelihood ratio= 97/3 - much better measure of how good a test is: high LR (double figures) means good test, sensitivity & specificity can still be quite inaccurate at high %s eg/ 90% specific/sensitive in population of 1000 = 100 false +ve, 100 false -ve NB/ false positive - test is positive, but pt doesn't have disease NB2/ false negative - test is negative but pt has disease

List the clinical findings that characterise each kind of shock

1) septic - a) acutely ill - pyrexia + rigors, N+V b) signs of hypoperfusion (hypotension) c) tachypnea and tachycardia (bounding pulse) d) pallor/grayish/mottled skin = signs of poor tissue perfusion seen in septic shock e) petechiae or purpura associated with DIC - ominous sign f) altered mental status g) signs depend on area of infection eg/ CNS = signs of meningism, head & neck = cervical lymphadenopathy, chest & pulmonary = dullness on percussion, cardiac = new murmur, abdo & GI = localised tenderness, pelvic & GU = pelvic tenderness, bone & soft-tissue = tenderness, skin = petechiae/purpura 2) anaphylactic - a) may have anxiety, tremor, sensation of cold b) signs of profound vasodilation: warm peripheries; low BP; tachycardia c) severe angioedema of tongue + lips may obstruct airflow. Laryngeal oedema (stridor or severe air hunger). Loss of voice, hoarseness, and/or dysphonia may occur d) hypotension (and resultant loss of consciousness). Cardiovascular collapse and shock can occur immediately, without any other findings e) if hypoperfusion or hypoxia occurs, it can cause altered mentation f) urticaria (ie, hives) - often palms, soles, and inner thighs g) N+V, diarrhoea, and abdominal distension are frequently observed 3) cardiogenic - a) appear ashen or cyanotic and have cool skin and mottled extremities b) peripheral pulses rapid and faint, may be irregular if arrhythmias are present c) signs of myocardial failure - inc JVP, pulsus alternans, gallop rhythm, crackles in lungs; peripheral/pulmonary oedema, heart sounds distant - 3rd + 4th heart sounds present d) pulse pressure may be low, pts usually tachycardic e) signs of hypoperfusion eg/ altered mental status and decreased urine output f) systolic murmur heard in patients with acute mitral regurgitation or ventricular septal rupture - systolic murmur is louder upon Valsalva and prompt standing 4) hypovolaemic - a) class I hemorrhage (loss of 0-15%) - minimal tachycardia, delay in capillary refill b) class II (15-30%) - tachycardia (>100 bpm), tachypnea, dec in pulse pressure, cool clammy skin, delayed capillary refill, and slight anxiety c) class III (30-40%) - marked tachypnea and tachycardia, dec systolic BP, oliguria, and significant changes in mental status, such as confusion or agitation d) class IV (>40%) - marked tachycardia, dec systolic BP, narrowed pulse pressure (or immeasurable diastolic pressure), markedly dec urinary output, depressed mental status (or loss of consciousness), and cold and pale skin e) 4 areas in which life-threatening haemorrhage can occur: chest, abdo, thighs, outside body

List the types of shock and the causes for each type of shock

1) septic -(give Abx within first hour eg/ Tazocin) a) bacterial infection (most commonly gram -ve) 2) anaphylactic - (type I, IgE mediated hypersensitivity reaction - release of histamine etc) a) drugs eg/ penicillin + contrast media in radiology, latex, stings, eggs, fish, peanuts, strawberries (allergies!) 3) cardiogenic - inadequate tissue perfusion due to cardiac insufficiency a) MI, arrhythmias, PE, tension pneumothorax, cardiac tamponade, myocarditis, myocardial depression (drugs, hypoxia, acidosis, sepsis), valve destruction (endocarditis), aortic dissection, VT/VF 4) hypovolaemic - losing ~1/5 or more of the normal amount of blood in the body, subset = haemorrhagic a) blood loss, burns, diarrhoea, excessive perspiration, vomiting, dehydration 5) heat exposure 6) neurological a) spinal injury

Describe the complications of pneumonia including systemic sepsis, lung abscess and empyema

1) septicaemia - bacterial spread from lung parenchyma into blood, may cause metastatic infection eg/ infective endocarditis 2) lung abscess - cavitating area of localised suppurative infection in lung, often with a fluid level a) inadequate treatment of pneumonia (Klebsiella and Staph aureus) b) features - swinging fever, cough, purulent foul-smelling sputum, pleuritic chest pain, haemoptysis, malaise, weight loss, finger clubbing, anaemia, crepitations 3) empyema - pus in pleural space - suspect if pt with resolving pneumonia develops recurrent fever - if CXR confirms then drain - aspirated fluid is yellow & turbid with pH <7.2 a) from bacterial spread or abscess rupture b) bronchoscopy will rule out carcinoma and foreign bodies 4) pleural effusion - inflammation of pleura by adjacent pneumonia - if large, symptomatic or infected then drain 5) respiratory failure - normally type 1 (PaO2<8kPa), give 60% O2 - careful in COPD (monitor ABGs) 6) brain abscess 7) pericarditis + myocarditis 8) cholestatic jaundice - secondary to Abx or sepsis 9) hypotension - dehydration, vasodilation due to sepsis - give IV fluid if systolic BP <90mmHg

Describe the laboratory and radiological investigation of a patient presenting with obstructive jaundice

1) serum bilirubin level elevated (>2-3mg/dL (30-50umol/L)) a) extrahepatic obstruction = big elevation in conjugated, less in unconjugated b) intrahepatic obstruction = inc conjugated & unconjugated bilirubin 2) AP level - markedly elevated in persons with biliary obstruction, but high levels are not specific to cholestasis - to determine if enzyme is likely to be of hepatic origin, measure gamma-glutamyl transpeptidase (GGT) a) extrahepatic obstruction - elevated >3x upper limit b) intrahepatic obstruction - usually elevated <3x upper limit 3) levels of serum transaminases - moderately elevated in pts with cholestasis but occasionally may be markedly increased, especially if cholangitis is present a) extrahepatic obstruction - AST values may quickly rise to > 10x normal value then fall after ~72hrs - over time see elevation in AST levels b) intrahepatic obstruction - most elevations due to intrahepatic disease 4) GGT levels - elevated with diseases of liver, biliary tract & pancreas when biliary tract is obstructed 5) prothrombin time (PT) - may be prolonged because of malabsorption of vitK 6) hepatitis serology 7) urine bilirubin levels - only conjugated bilirubin can be passed into urine (dark) 8) USS - differentiates extrahepatic from intrahepatic causes of jaundice 9) CT scanning - more accurate than US for determining specific cause and level of obstruction. Limited value in diagnosing CBD stones 10) magnetic resonance cholangiopancreatography (MRCP) - visualize hepatobiliary tree

Outline the indications for transplantation in patients with chronic renal failure

1) severe metabolic acidosis 2) hyperkalemia 3) pericarditis 4) encephalopathy 5) intractable volume overload 6) failure to thrive and malnutrition 7) peripheral neuropathy 8) intractable gastrointestinal symptoms 9) in asymptomatic adult patients an eGF of 5-9 ml/min/1.73 m², irrespective of the cause of CKD or if they have other comorbidities 10) refusal of further dialysis

Describe how to use inhalers

1) shake well before use 2) remove cap 3) breath out fully 4) bring inhaler to mouth, make a tight seal 5) start to breathe in slowly - press top once and keep breathing in slowly until you have a full breath 6) remove inhaler from mouth, hold breath for 10s, breathe out 7) teach how to use peak flow meter to monitor PEF 2x day NB/ important not to just spray inhaler into mouth NB2/ with steroid inhaler pt should be counselled to rinse mouth out after use

Describe the clinical features & systemic complications associated with chronic renal failure

1) signs/symptoms: a) early disease = asymptomatic b) as nephrons are lost, surviving nephrons show compensatory hypertrophy (+ constriction of efferent arteriole) so no longer have "down time" - function continuously inc level of wear and tear on nephron & sclerosis develops over time c) skin - pallor of anaemia, yellow tinge of uraemia, gum hypertrophy (ciclosporin), cushingoid appearance (steroids), purpura, excoriations d) abdomen - catheter (or scar), ballotable polycystic kidneys (fluid overload) + liver e) signs of diabetic neuropathy, retinopathy, CV or PVD f) malaise & lethargy, anorexia, N+V and insomnia g) severe - severe uraemia, encephalopathy, seizures or coma 2) systemic complications: a) dec GFR, loss of tubular ability + dec kidney mass = systemic effects (aside from an unconcentrated oligouria): b) oedema c) restless legs/cramps (myoclonic twitches) d) dec GFR + tubular resorption = retention of Na, K and H2O = hypertension (cardiac failure, dec renal perfusion) e) dec bicarbonate resorption & H+ excretion = metabolic acidosis f) dec renal mass leads = dec EPO = anaemia (normochromic/cytic) g) retention of phosphate + loss of renal mass = dec vit D production & hypocalcaemia, causing secondary hyperparathyroidism and renal osteodystrophy h) fibrinous + uraemic exudates develop - may cause pericarditis or pneumonitis i) haemorrhagic ulcers in GI tract, immunological depression & other system alteration may occur NB/ symptoms become common when serum urea >40mM, although urea not directly toxic. Uraemia >50-60mM, pts become "cloudy of the mind"

Describe the two different positions a patient may adopt to undergo a lumbar puncture, and advantages of each with respect to ease of success and measuring opening pressure

1) sitting - a) generally makes it easier to identify anatomical landmarks and planes, especially if difficult to find landmarks eg obese b) sit the patient on a firm surface c) pt should lean forward, and can hug a pillow on a table at the right height to flex (not extend) the back d) lift pt's legs by putting their feet on a stool or chair and flex the hips to above 90 degrees e) if pressure measuring is required, lie the patient on their side (after successful needle insertion) with assistance and then open the tap to measure the pressure - otherwise get overestimation of OP 2) lying down - a) if pt too ill to sit up for LP b) pt should be lying in the lateral decubitus position c) ensure the vertical plane of the patients back is perpendicular to the bed d) flex knees and hips so that knees are close to the chest e) while flexion of the neck is often taught as important, evidence suggests that this has no effect of the size of the interspinous opening and may be uncomfortable for the patient

Define hiatus hernia with regard to anatomical type (sliding and para-oesophageal)

1) sliding - 80%, where gastro-oesophageal junction slides up into chest, acid reflux as lower oesophageal sphincter becomes less competent 2) rolling/para-oesophageal - 20%, where gastro-oesophageal junction remains in abdomen but a bulge of stomach herniates into chest alongside oesophagus, as GO junction remains intact no reflux a) clinical features - 30% pts over 50, obese women, 50% have symptomatic GO reflux b) imaging - barium swallow is best diagnostic test, upper GI endoscopy visualises mucosa but cannot reliably exclude hiatus hernia c) treatment - lose weight, treat reflux symptoms, surgery indications - intractable symptoms despite aggressive medical therapy, advised to repair prophylactically even if asymptomatic as may strangulate - needs surgical repair (high mortality & morbidity) NB/ Bochdalek & Morgagni are congenital hernias due to diaphragmatic malformation

List the common causes and the associated pathology of intestinal obstruction

1) small bowel - a) adhesions - abdo surgery, or inflammation eg/ ruptured appendix b) hernias - protrusion of viscus through defect in wall of its containing cavity into abdominal position 2) large bowel - a) cancer of colon b) constipation c) diverticular stricture d) volvulus - sigmoid or caecal - twisting of structure around itself 3) rare - a) Crohn's stricture (IBD) b) gallstone ileus c) intussusception d) TB e) foreign body 4) acute obstruction - good peristalsis followed immediately by immobility & distension a) secretions inc & absorption dec - bacteria flourish b) stagnation of bowel distal to obstruction (not being flushed out) = bacteria multiply freely 5) subacute obstruction - time for proximal SM to hypertrophy & overcome blockage a) eventually, muscle hypertrophy can't compensate - dilation of bowel with accumulation of swallowed gas and secretions

Describe the risk factors in lung cancer

1) smoking 2) asbestos 3) chromium 4) arsenic 5) iron oxides 6) radiation (radon gas) 7) living in an urban area 8) petroleum products, coal tar + multiple other occupational exposures 9) family history (genetics) 10) fibrosis

Coronary artery disease risk factors

1) smoking 2) sedentary lifestyle 3) obesity 4) hypertension 5) diabetes 6) hyperlipidaemia/hypercholesterolemia 7) inc age 8) male gender 9) family history of IHD 10) use of recreational drugs eg/ cocaine 11) socioeconomic factors 12) type A personality 13) haemostatic factors

Outline the treatment of NS including the need for diuretics and a low-salt diet

1) sodium and fluid restriction 2) high-dose diuretics for oedema - furosemide (1mg/kg/day) +/- spironolactone (2mg/kg/day), need regular weight monitoring to check is working 3) corticosteroids in kids, possibly in adult-onset minimal change disease 4) diet with adequate calorific intake and sufficient protein content (1-2 g/kg daily) 5) in some adults may consider immunomodulatory drugs eg/ cyclophosphamide, ciclosporin, tacrolimus and levamisole 6) some new drugs eg/ rituximab, galactose and antifibrotic agents, are under investigation for the treatment of idiopathic focal and segmental glomerulosclerosis in adults

Name the common surgical conditions leading to disseminated intravascular coagulation (DIC)

1) solid tumors and blood cancers (esp acute promyelocytic leukemia) 2) obstetric complications: abruptio placentae, pre-eclampsia or eclampsia, amniotic fluid embolism, retained intrauterine fetal demise, septic abortion, post partum haemorrhage 3) massive tissue injury: severe trauma, burns, hyperthermia, rhabdomyolysis, extensive surgery 4) sepsis or severe infection: bacterial (Gram-ve & +ve sepsis), viral, fungal, or protozoan infections 5) transfusion reactions (ABO incompatibility haemolytic reactions) 6) severe allergic or toxic reactions (i.e. snake venom) 7) giant haemangiomas (Kasabach-Merritt syndrome) 8) large aortic aneurysms 9) liver disease, HELLP syndrome, thrombotic thrombocytopenic purpura/HUS, and malignant hypertension may mimic DIC but do not occur via the same pathways

Describe the complications that may be associated with the passage of a nasogastric tube; discuss their recognition and management

1) some degree of patient discomfort is common - generous lubrication, topical anaesthetic, and a gentle technique may reduce the patient's level of discomfort 2) throat irritation may- be reduced with administration of anaesthetic lozenges (eg/ benzocaine lozenges) prior to the procedure 3) epistaxis - may be prevented by generously lubricating the tube tip and using a gentle technique 4) respiratory tree intubation and oesophageal perforation - stop if any difficulty breathing, pain more than expected, or any signs of bleeding

Outline the complications of this procedure and the need for immunosuppressive therapy following surgery

1) some forms of kidney disease may return after transplant 2) rejection of the donor organ 3) kidney transplant surgery carries a risk of significant complications, including: a) blood clots b) bleeding c) leaking from or blockage of ureter d) infection e) failure of the donated kidney f) an infection or cancer that can be transmitted with the donated kidney g) death, heart attack and stroke 4) need to take immunosuppressant for rest if life to prevent organ rejection

Specify and explain the defining clinical characteristics of Raynaud's disease or phenomenon

1) spasm of digital arteries causing discoloration of the fingers, and toes (white-blue-red) - vasospasm can be so severe that the fingertips or toes develop ulcerations or gangrene due to lack of circulation 2) more common in women than men 3) Raynaud's disease if a cause cannot be found 4) numbness, burning and severe pain may occur as the fingers warm up 5) attacks may last for hours 6) in chronic, severe disease infarction may occur 7) smoking + caffeine worsen frequency and intensity of attacks, seems to be a hormonal component 8) avoid triggers, smoking, caffeine, calcium channel blockers (nifedipine) or diltiazem, sympathectomy or prostacyclin infusion in severe disease

Outline the investigations of a patient with suspected bronchiectasis

1) sputum culture - determines organisms 2) CXR - cystic shadows, thickened bronchial walls 3) HRCT chest - show bronchial wall thickening, bronchial dilation and cysts at the end of bronchioles with a sensitivity of 97%. 4) spirometry 5) bronchoscopy - locate site of haemoptysis, exclude obstruction, obtain samples for culture 6) serum Ig - 10% adults have antibody deficiency, mainly IgA 7) sinus x-rays may be useful, 30% have concomitant rhinosinusitis 8) test sweat electrolytes if CF suspected

Recognise the various phases of wound healing by evaluating the surgical and traumatic wound

1) stages of wound healing proceed in an organised way and follow four processes: haemostasis, inflammation, proliferation and maturation. Although the stages of wound healing are linear, wounds can progress backward or forward depending on internal and external patient conditions 2) haemostasis phase - haemostasis is the process of the wound being closed by clotting. It starts when blood leaks out of the body so blood vessels constrict to restrict the blood flow. Next, platelets stick together to seal the break in the wall of the blood vessel. Finally, coagulation occurs and reinforces the platelet plug with threads of fibrin. The haemostasis stage of wound healing happens very quickly. The platelets adhere to the sub-endothelium surface within seconds of the rupture of a blood vessel's epithelial wall. After that, the first fibrin strands begin to adhere in about sixty seconds. As the fibrin mesh begins, the blood is transformed from liquid to gel through pro-coagulants and the release of prothrombin. The formation of a thrombus or clot keeps the platelets and blood cells trapped in the wound area 3) inflammatory phase - begins right after the injury when the injured blood vessels leak transudate causing localised swelling. Inflammation controls bleeding & prevents infection. The fluid engorgement allows healing and repair cells to move to the site of the wound. During the inflammatory phase, damaged cells, pathogens, and bacteria are removed from the wound area. These white blood cells, growth factors, nutrients and enzymes create the swelling, heat, pain and redness commonly seen during this stage of wound healing 4) proliferative phase - wound is rebuilt with new tissue made up of collagen and extracellular matrix. The wound contracts as new tissues are built & a new network of blood vessels must be constructed so that the granulation tissue can be healthy and receive sufficient oxygen and nutrients. Myofibroblasts cause the wound to contract 5) maturation phase aka remodeling stage of wound healing - collagen is remodeled from type III to type I and the wound fully closes. The cells that had been used to repair the wound but which are no longer needed are removed by apoptosis. When collagen is laid down during the proliferative phase, it is disorganized and the wound is thick. During the maturation phase, collagen is aligned along tension lines and water is reabsorbed so the collagen fibers can lie closer together and cross-link. Cross-linking of collagen reduces scar thickness and also makes the skin area of the wound stronger. Remodeling begins about 21 days after an injury and can continue for a year or more. Even with cross-linking, healed wound areas continue to be weaker than uninjured skin, generally only having 80% of the tensile strength of unwounded skin NB/ recognise the appearance of a wound that has healed by epithelialisation

List the treatment options for an oesophageal malignancy. Discuss staging for oesophageal malignancy

1) staging - a) stage 0: T0 N0 M0 b) stage I: T1 N0 M0 c) stage IIA: T2 N0 M0 d) stage IIB: T3 N0 M0 e) stage IIIA: T0-2 N2 M0 f) stage IIIB: T3 N2 M0 g) stage IIIC: T4a N1-2 M0 h) stage IV: any T, any N, M1 2) mx - a) stage 0 and stage IA - I) 1st line: endoscopic resection +/- ablation II) 2nd line: oesophagectomy b) stage IB and stage IIA - I) 1st line: oesophagectomy if can tolerate surgery, otherwsie chemo or radiotherapy II) 2nd line: endoscopic ablation +/- stenting +/- brachytherapy for symptom relief c) stage IIB and stage III - I) 1st line: oesophagectomy if can tolerate surgery II) + chemo III) 2nd line: endoscopic ablation +/- stenting +/- brachytherapy for symptom relief d) stage IV - I) 1st line: chemo II) +/- radiotherapy III) +/- endoscopic ablation +/- stenting for symptom relief NB/ not all pts will be fit for surgery

Describe the blood gas abnormalities associated with acute severe asthma

1) start of attack - PaO2 & PaCO2 fall + pH rises (7.5) 2) as asthma attack gets worse - low PaCO2 & high pH start to move back to normal values (the fish - pH at top, pCO2 on bottom) 3) eventually - lungs unable to blow off CO2 - PaCO2 rises + pH & PaO2 fall 4) also - PEF (<50% = severe, <33% = life-threatening), spirometry (dec FEV1/FVC), CXR NB/ normal pCO2 with hypoxaemia suggests current deterioration

Describe pharmacological methods of secondary prevention

1) statins 2) BP meds 3) aspirin 4) antiplatelets eg/ clopidogrel 5) warfarin or NOACs 6) ACEIs or angiotensin II receptor blockers (ARB) 7) B-blockers 8) anti-anginal medicines - nitrates

Discuss the methods relieving common bile duct obstruction

1) stones may be removed using an endoscope during an ERCP 2) some may need surgery to bypass the blockage 3) gallbladder will usually be surgically removed if the blockage is caused by gallstones 4) if due to malignancy the duct may need to be widened by endoscopic or percutaneous dilation, tube may need to be placed to allow drainage.

Explain the following terms, with specific reference to time course; Stroke; Transient Ischaemic Attack; Amaurosis Fugax

1) stroke - focal neurological deficit lasting >24hrs, no apparent cause other than a vascular one 2) TIA - focal neurological deficit lasting <24hrs, generally <1hr, complete recovery 3) amaurosis fugax - painless temporary loss of vision in 1 or both eyes, found prior to carotid system TIA

List the alternatives for bladder drainage when urethral catheterisation is contraindicated including suprapubic catheterisation or ultrasound guided drainage

1) suprapubic - drain the bladder by passing through the anterior abdominal wall into the dome of the bladder 2) ultrasound guided - form of image guided procedure, allowing minimally invasive treatment of collections that are accessible by ultrasound study

Outline the management of trauma to the urethra

1) suprapubic catheter for bladder drainage and subsequent delayed repair 2) ultimate repair of the posterior urethral injury can be performed 6-12 weeks after the event, after the pelvic hematoma has resolved and the patient's orthopedic injuries have stabilized - repair consists of mobilizing the urethra distally to allow a direct anastomosis after excision of the stricture 3) early endoscopic realignment (within 1 week postinjury) using a combined transurethral and percutaneous transvesical approach may be safe 4) trauma to the anterior urethra is often from straddle injuries - sharp blow to the perineum 5) trauma to the posterior urethra almost always results from a severe injury - in males, posterior urethral trauma may tear the urethra completely away below the prostate, wounds can form scar tissue that slows or blocks the urine flow 6) for females, urethral injuries are rare, always linked to pelvic fractures or cuts, tears, or direct trauma to body near vagina

Describe the management of temporal arteritis

1) suspected GCA - a) no visual or neurological symptoms or signs I) 1st line - prednisolone: 1 mg/kg/day orally for 4 weeks, initiated immediately if index of suspicion for GCA is high (don't wait for ix results) b) visual or neurological symptoms or signs - I) 1st line - methylprednisolone pulse therapy: 1 g IV once daily for 3 days, then prednisolone 2) confirmed GCA - a) 1st line - prednisolone: 1 mg/kg/day orally for 4 weeks, gradually taper dose over 6-12 months, most patients require glucocorticoids for at least 1 to 2 years or longer b) + aspirin 75 mg orally once daily - lowers risk of vision loss without increasing risk of bleeding complications c) +/- osteoporosis prevention eg/ calcium carbonate: 1000-1500 mg daily + ergocalciferol: 800 units daily + alendronic acid: 5 mg daily d) recurrent or relapsing disease or severe glucocorticoid adverse effects - + methotrexate 7.5 mg once weekly

List common presenting features, including epigastric pain and/or a lump in the epigastrium

1) swelling with pressure on abdomen by moving or doing any activity 2) symptoms minor in beginning but can progress - death if other organs in stomach get infected (can also get through the peritoneum) 3) noticeable bump in stomach felt when person is either bending to get or do something or when have a bowel movement. Also when coughing, crying and laughing 4) slight change in colour of skin - area effected blackish-blue in colour (strangulation) - can also cause diarrhoea, abdominal swelling, vomiting and pain 5) severe or unbearable pain in epigastrium 6) fever if complicated hernia - if N+V immediate medical attention suggested NB/ epigastric hernias - variety of ventral hernia occurring in linea alba above level of umbilicus, consisting of 1+ small protrusions through which extraperitoneal fat herniates

Describe the symptoms and complications of haemorrhoids

1) symptoms - a) bright red rectal bleeding (most common cause), often coating stool, on tissue, or dripping into pan after defecation b) may be mucus discharge & pruritis ani c) may have severe anaemia d) pain due to strangulation or thrombosis rare 2) complications (from surgery) - a) excessive bleeding b) infection c) faecal incontinence - rare, can sometimes be corrected with another operation d) anal fistula

Describe the benefits, and outline the steps involved, in a systematic review and meta-analysis

1) systematic reviews - literature review that uses systematic methods to collect secondary data, critically appraise research studies, and synthesize studies, designed to provide a complete, exhaustive summary of current evidence relevant to a research question a) systematic reviews of RCTs are key to the practice of evidence-based medicine b) steps - I) defining a question & agreeing an objective method II) search for relevant data from research that matches certain criteria eg/ only using research that is good quality and answers the defined question III) 'extraction' of relevant data eg/ method, participants, how it was paid for (for example funding sources), outcomes IV) assessing quality of data by judging it against criteria identified at the first stage V) analyse and combine the data (using complex statistical methods) which give an overall result from all of the data c) benefits - strongest form of medical evidence as collates data from good quality research 2) meta-analysis - statistical analysis that combines the results of multiple scientific studies, then uses approaches from statistics to derive a pooled estimate - gives a weighted average from the results of the individual studies a) steps - I) formulation of the research question eg/ using PICO model (Population, Intervention, Comparison, Outcome) II) search of literature III) selection of studies ('incorporation criteria') - based on quality criteria IV) decide which dependent variables or summary measures are allowed etc V) selection of a meta-analysis model, e.g. fixed effect or random effects meta-analysis VI) examine sources of between-study heterogeneity, e.g. using subgroup analysis or meta-regression b) benefits - results can be generalized to a larger population, precision and accuracy of estimates can be improved as more data is used, inconsistency of results across studies can be quantified and analyzed, presence of publication bias can be investigated

Describe the clinical features and differential diagnosis of pulmonary oedema

1) tachycardia 2) gallop (or triple) rhythm 3) bilateral basal crackles (fine) 4) distressed & pale 5) dyspnoea/orthopnoea 6) sweaty 7) tachypnoae 8) pink frothy sputum 9) pulsus alterans 10) inc JVP (heart failure) 11) wheeze (cardiac asthma) discuss the differential diagnosis of acute pulmonary oedema 12) DD - a) chest infection eg/ pneumonia - less likely to give pink frothy sputum and breathlessness b) pulmonary embolism - typically presents with pleuritic chest pain, cough, SOB c) adult respiratory distress syndrome - if no cardiac cause d) asthma/COPD

Discuss the pathology, clinical presenting features, diagnosis and management of torsion of the testis and epididymo-orchitis

1) testicular torsion - a) pathology - occurs when spermatic cord (from which the testicle is suspended) twists, cutting off the testicle's blood supply (ischemia). Most common underlying cause is a congenital malformation "bell-clapper deformity" - testis is inadequately affixed to the scrotum so moves freely on its axis and susceptible to induced twisting of the cord and vessels. Generally occurs within tunica vaginalis. Recurrent torsion may lead to atrophy b) presentation - rapid onset of testicular pain (can radiate to abdomen). May be associated vomiting + scrotal/inguinal swelling. Testis may retract & cremaster reflex will be absent. Most common in men <20 c) diagnosis - made on the presenting s/s & urgent USS which shows fluid and the whirlpool sign. Irreversible ischemia begins ~6hrs after onset & emergency diagnosis and treatment required within this time to minimize the risk of testicle loss. Urine dip will exclude infection. Torsion of hyatid of Morgagni may present with similar pain history, but less physical signs d) mx - I) 1st line - immediate urological consultation for emergency scrotal exploration II) + supportive care: morphine sulfate + ondansetron III) 2nd line - manual de-torsion followed by scrotal exploration 2) epididymo-orchitis - a) pathology - discomfort or pain of epididymis, a curved structure at the back of the testicle in which sperm matures and is stored. Inflammation spreads to testicles. Most frequent cause of acute onset scrotal pain in adults. Most commonly Chlamydia and Gonorrhoea in younger men, in more elderly men E.Coli & UTI-causing pathogens b) presentation - dysuria + pain, acute or chronic: if acute, the onset of testicular pain is often accompanied by inflammation, redness, and warmth in the scrotum; if chronic, pain may be the only symptom. In either form, testicular pain in one or both testes can vary from mild to severe, and one or both epididymides may noticeably swell (inflame) and/ or harden (indurate). The pain is often cyclical and may last from less than an hour to several days. Most common in men >20. In contrast with testicular torsion, the cremasteric reflex (elevation of testicle in response to stroking the upper inner thigh) is not altered. Swelling may stretch along inguinal canal and may appear similar hernia (particularly inguinal hernia) - distinguishing: inguinal hernias generally not painful + swelling often disappears upon lying down. Neither is the case with epididymitis. Also, hernias are not episodic, whereas epididymitis frequently is c) diagnosis - if diagnosis is not entirely clear from pt's history + physical exam, a Doppler USS can confirm inc flow of blood to affected epididymis (torsion characterized by ischemia), urine dipstick & MC&S NB/ orchitis causes swelling - most common pathogen is paramyxovirus (mumps), which, if bilateral, may lead to infertility d) mx if likely to be sexually transmitted - I) 1st line - empirical ceftriaxone + doxycycline II) + supportive measures: paracetamol or naproxen or ibuprofen, bed rest and scrotal elevation, if pt is systemically ill with signs of sepsis, IV fluid replacement and initial IV abx may be indicated III) with possible enteric infection: +/- quinolone: ofloxacin or levofloxacin e) mx if likely to be not sexually transmitted - I) 1st line - empirical quinolone: ofloxacin or levofloxacin; + supportive measures as above

List the risk factors for the development of venous sinus thromboses

1) thrombophilia - tendency to develop blood clots due to abnormalities in coagulation eg/ factor V leiden, deficiency of protein or S or antithrombin, lupus, antiphospholipid syndrome, antithrombin III deficiency 2) nephrotic syndrome - kidney problem causing protein loss in the urine 3) chronic inflammatory disease eg/ IBD, lupus and behçet's disease 4) pregnancy and puerperium (the period after giving birth) 5) particular blood disorders eg/ polycythemia vera and paroxysmal nocturnal hemoglobinuria 6) use of oestrogen-containing forms of hormonal contraception 7) meningitis and infections of ENT eg/ mastoiditis and sinusitis 8) direct injury to the venous sinuses 9) medical procedures in the head and neck area 10) sickle cell anaemia 11) dehydration, primarily in infants and children 12) homocystinuria 13) intracranial hypotension 14) obesity 15) cancer 16) chronic haemolytic anaemia 17) Beta-thalassemia major 18) heart disease — either congenital or acquired 19) iron deficiency

Differentiate embolic and thrombotic occlusion

1) thrombus - abnormal clot formed in a vessel, usually due to atherosclerosis (most commonly affected - middle cerebral artery in head, deep veins of legs. Incomplete ischaemia (collaterals). Onset in hours, and previously symptoms will have been present. Artery will feel calcified & bruit from already developed collaterals will be present. Other leg will have reduced pulses 2) embolus - floating clot that lodges somewhere (usually break off from thrombi). Complete ischaemia characterised by parasthesia and paralysis. Onset in minutes & more common in AF pts (source of embolus). No history of ischaemic symptoms previously. Artery will feel soft & no bruit. Other leg will be fine 3) pale infarcts = thrombi. Form right in the vessel, and over time, occlude the vessel, preventing blood from getting through, causing a pale (bloodless) infarct 4) red infarcts = emboli. Once lodged, the body sometimes is able to dissolve these emboli; and sometimes they move farther downstream. Then the damaged vessel gets blood going through it again (reperfusion) and blood leaks out through the damaged area

Causes of Romberg's sign

1) tool to diagnose sensory ataxia (gait disturbance with abnormal proprioception of joints), also measures degree of disequilibrium caused by central vertigo, peripheral vertigo and head trauma 2) pt must close eyes as otherwise can cheat and use vestibular function and vision 3) method - pt removes shoes & stands with feet together with arms at sides or crossed in front of the body; closes eyes & tries to maintain balance 4) Romberg test is positive when the patient is unable to maintain balance with their eyes closed - inc body sway, placing one foot in the direction of the fall, or even falling 5) +ve Romberg's test = sensory ataxia eg/ a) vitamin deficiencies such as Vitamin B12 b) conditions affecting the dorsal columns of the spinal cord, such as tabes dorsalis (neurosyphilis), in which it was first described c) conditions affecting the sensory nerves (sensory peripheral neuropathies), such as chronic inflammatory demyelinating polyradiculoneuropathy (CIDP) d) Friedreich's ataxia e) Ménière's disease

Describe Toxbase and the function of the National Poisons Information Services

1) toxbase = primary clinical toxicology database of the NPIS 2) backbone of the service & 1st-line resource for UK healthcare professionals 3) TOXBASE database provides information about routine diagnosis, tx and mx of pts suffering from exposure to a wide range of pharmaceuticals, chemicals (agricultural, household and industrial), plants and animals 4) TOXBASE is available free to UK National Health Service registered users who will normally be departments or surgeries, rather than individuals 5) all NPIS Units contribute to and authenticate the content of TOXBASE and it represents the most authoritative source of information available

Describe the procedures that should be followed when prescribing blood or blood products for transfusion

1) transfusions of all blood components must be requested from the Blood Transfusion laboratory on an individual named patient basis. Only medical staff may prescribe blood, blood components or blood products. 2) samples labelled as above 3) blood transfusion requests must provide adequate clinical information including: pt's diagnosis and any relevant procedure 3) urgent and out of hours requests - Blood Transfusion Laboratory should be informed by telephone (bleep out of hours) in the event of a major or life-threatening haemorrhage 4) for blood required in 15-60 mins from receipt of sample in the laboratory, lab will provide uncrossmatched blood of the same ABO and RhD group as the patient. 5) for blood required in 60 mins or longer from receipt of sample in the laboratory, lab will provide fully crossmatched blood. 6) patient - a) ensure adequate sized, patient cannula is in place b) take and record baseline observations of temperature, pulse, respiration and blood pressure prior to the transfusion. 7) transfusions - a) must not take place without a wristband attached and verified as correct for the patient which matches identically with the patient identification on the blood compatibility label - positively identify the patient by asking his/her surname, first name and date of birth (whenever possible) and make sure that these patient identification details are the same as on the patient's wristband b) check that the blood group and unit number on the blood bag are identical to those on the compatibility label attached to the blood bag, check the blood - has not passed its expiry date, shows no sign of damage and that there is no evidence of leakage, the blood component complies with any special requirements

Classify causes of a pleural effusion including infective, neoplastic, metabolic, and cardiac causes

1) transudates (clear, straw coloured) - a) inc venous pressure (cardiac failure, constrictive pericarditis, fluid overload) b) hypoproteinaemia (cirrhosis, malabsorption, kidney failure, intestinal failure) c) hypothyroidism d) meig's syndrome (R pleural effusion & ovarian fibroma) 2) exudates (clear, straw-coloured, protein content >30g/L, LDH>200units/L) - inc leakiness of pleural capillaries secondary to infection, inflammation, malignancy - eg/ a) bacterial pneumonia b) TB c) pulmonary infarction d) SLE, RA (connective tissue diseases) e) bronchogenic carcinoma f) malignant metastases + mesothelioma + lymphatic carcinomatosis + lymphoma g) pancreatitis h) sarcoidosis

List the predisposing factors which make a patient vulnerable to developing subdural haemorrhage

1) trauma (can be up to 9 months ago) 2) elderly age 3) falls - epileptics, alcoholics 4) anticoagulants 5) alcoholics (cerebral atrophy) NB/ often extensive due to loose attachment of dura & arachnoid membranes, usually occurs with rupture of small bridging veins where they enter the dura. In at risk groups, bridging veins may become stretched and more susceptible to damage

List the features in a headache which reflect a raised intracranial pressure (with reference to variation with posture, coughing, visual symptoms and diurnal variation) including pertinent exam findings

1) triad of headache + papilloedema + vomiting is generally considered indicative of ICP 2) headache - more worrying when nocturnal, starting when waking (lying down), worse on coughing or moving head and associated with altered mental state. a) early changes in mental state - lethargy, irritability, slow decision making and abnormal social behaviour. Untreated, this can deteriorate to stupor, coma and death 3) vomiting - can progress to projectile with rising ICP 4) eyes - a) pupillary changes - irregularity or dilatation in one eye b) fundoscopy - blurring of disc margins, loss of venous pulsations, disc hyperaemia and flame-shaped haemorrhages. In later stages, obscured disc margins and retinal haemorrhages may be seen c) unilateral ptosis or 3rd & 6th nerve palsies. In later stages, ophthalmoplegia and loss of vestibulo-ocular reflexes 5) late signs - hemiparesis, inc BP, widened pulse pressure and slow irregular pulse 6) acute situations: a) head injury and obtundation: bleeding forms a rapidly expanding haematoma leading to rapidly rising ICP b) syncope, headache and meningismus: abrupt headache with these symptoms suggests ruptured cerebral aneurysm or vascular lesion c) focal deficit then seizures: associated with a mass lesion d) 'talk and deteriorate': patients typically talk recognisably following head injury, then go into coma in the first two days - intracranial haematoma

Differentiate true and false aneurysm

1) true aneurysm involves all 3 layers of the wall of an artery (intima, media and adventitia) a) true aneurysms include atherosclerotic, syphilitic, and congenital aneurysms, ventricular aneurysms that follow transmural MI 2) false aneurysm or pseudoaneurysm is a collection of blood leaking completely out of an artery or vein, but confined next to the vessel by the surrounding tissue a) this blood-filled cavity will eventually either thrombose (clot) enough to seal the leak, or rupture out of the surrounding tissue b) can be caused by trauma that punctures the artery, percutaneous surgical procedures eg/ coronary angiography or arterial grafting

Describe in broad outline treatment changes between 1930 and 1970 and the implications of this for current treatment

1) turning point was the discovery of the bacterium that causes tuberculosis by Robert Koch, unveiled in 1882, so could develop medicine targeted toward the actual characteristics of the disease, rather than simply expressed symptoms 2) before this tx included - clean air, isolation, surgeries to dec lung volume 3) era of effective treatment of tuberculosis was thought to begin in 1946, with the use of streptomycin. This came after the testing of many compounds, most of which proved to be too toxic for use despite their potential to effectively handle the disease 4) TB created a number of difficulties for antimicrobial chemotherapy treatments. First, its capability to lie dormant at low metabolic activity levels means abx can fail to eliminate the presence of the disease. Second, bacterium can be located deep in pulmonary cavities or inside solid material so abx penetration difficult. Third, tuberculosis bacterium takes advantage of a variety of environments inside the body, meaning that antibiotics must be used that address multiple types of infections and infected areas. These challenges led to the current approach, using three to four drug treatments with different drugs targeting extracellular bacteria in sputum, inside necrotic tissue, and persistent strains 5) now have several methods, including potential new vaccines

Distinguish type-2 from type-1 respiratory failure and describe the implications of having a high arterial pCO2

1) type 1 = hypoxia - a) pts without chronic lung disease when there is falling pulse oximetry from 92% saturation or initial pulse oximetry <80% on room air. Severe respiratory failure is diagnosed when arterial blood gas shows PaO2 of <8 kPa (<60 mmHg) on room air b) pts with chronic lung disease may have low pulse oximetry readings and baseline PaO2 values of 6.7 kPa (50 mmHg), making worsening hypoxia difficult to recognise. Decreases of 10% from baseline oxygenation can indicate impending respiratory failure in patients with chronic lung disease 2) type 2 = hypercapnia - a) pts without chronic lung disease when there is hypoxia and acute elevation of arterial PaCO2 >6.0 to 6.7 kPa (>45 to 50 mmHg) and associated acidosis (pH <7.35) b) pts with chronic lung disease can usually tolerate PaCO2 levels of up to 10.7 kPa (80 mmHg) with secondary renal compensation. However, increasing acidosis (decreasing pH) in these patients indicates respiratory failure 3) if CO2 inc shows pts breathing less - sign they can no longer compensate (bad sign) due to tiredness

Describe the typical history and common causes of pulmonary oedema

1) typical history: a) dramatic clinical features of left heart failure - sudden onset of extreme breathlessness, anxiety, feelings of drowning & profuse diaphoresis b) dyspnoea on exertion, orthopnoea (pillows), and paroxysmal nocturnal dyspnoea (suggests cardiac cause - pt wakes up in the middle of the night totally out of breath due to fluid shift from interstitium to vascular compartments) c) cough - pink, frothy sputum in pts with severe disease d) chest pain may indicate acute MI/ischaemia or aortic dissection with acute aortic regurgitation, as the precipitant of pulmonary oedema e) Cheynes-stokes respiratory cycles may occur due to impaired response of central chemoreceptors to CO2 levels 2) causes: a) cardiac - coronary artery disease, pulmonary hypertension, cardiomyopathy, valve problems, L ventricle failure b) non-cardiac - ARDS (causes including: pneumonia, aspiration, sepsis, severe trauma), high altitude, PE, drugs eg/ morphine overdose, acute and chronic kidney disease, hypoalbuminaemia, tumour

Describe the indications for, and risks of, ultrasound scanning, transhepatic cholangiography and endoscopic retrograde cholangiopancreatography (ERCP)

1) ultrasound scanning gallbladder - a) indications - used to diagnose gallstones, US can also diagnose: abdominal pain or distention (enlargement), abnormal liver function, organomegaly, kidney stones, AAA b) risks - no known harmful effects on humans 2) transhepatic cholangiography - a) indications - cholestatic jaundice, to exclude extra hepatic bile duct obstruction, prior to biliary drainage procedure b) risks - percutaneous biliary procedures have higher complication rates than therapeutic ERCP. Complications encountered include infection, bleeding and bile leaks 3) endoscopic retrograde cholangiopancreatography (ERCP) - a) indications - procedure that combines upper gastrointestinal (GI) endoscopy and x-rays to treat problems of the bile and pancreatic ducts, doctors perform ERCP when your bile or pancreatic ducts have become narrowed or blocked eg/ due to gallstones, infection, acute or chronic pancreatitis, trauma or surgical complications in your bile or pancreatic ducts, pancreatic pseudocysts, tumors or cancers of the bile ducts or pancreas b) risks - pancreatitis, infection of the bile ducts or gallbladder, haemorrhage, abnormal reaction to the sedative, including respiratory or cardiac problems, perforation in the bile or pancreatic ducts, or in the duodenum near the opening where the bile and pancreatic ducts empty into it, tissue damage from x-ray exposure, death, although rare

Discuss the general treatment measures and suitable antibiotic regimens for treatment of UTI

1) uncomplicated (acute cystitis, not pregnant) - oral: a) nitrofurantoin for 5 days or, b) trimethoprim/sulfamethoxazole for 3 days or, c) secondary options - fosfomycin (single dose) 2) complicated suitable for outpatient therapy (not pregnant) - oral: a) ciprofloxacin for 7 days or, b) levofloxacin for 5 days or, c) trimethoprim/sulfamethoxazole for 7-14 days or, d) secondary option - amoxicillin/clavulanate for 7 days NB/ complicated UTIs - patients with functional or structural impairments that reduce the efficacy of antimicrobial therapy; pyelonephritis; UTI occurring in pregnancy 3) complicated requiring inpatient therapy (not pregnant) - IV: a) gentamicin for 7-14 days or, b) ceftriaxone for 10-14 days or, c) ampicillin for 10-14 days + gentamicin for 7-14 days or, d) ceftriaxone for 10-14 days + gentamicin for 7-14 days or, e) ciprofloxacin or levofloxacin or meropenem or piperacillin/tazobactam NB/ hospitalisation and parenteral abx if - fever, elevated WBC count, emesis, or volume depletion + typical UTI symptoms 4) if underlying abnormality then this needs tx eg/ if pt has hydronephrosis, find aetiology & tx; if pt has a renal abscess, drain & tx 5) complicated requiring inpatient therapy (pregnant) - IV: a) gentamicin for 7-14 days or, b) ceftriaxone for 10-14 days or, c) ampicillin for 10-14 days + gentamicin for 7-14 days or, d) ceftriaxone for 10-14 days + gentamicin for 7-14 days or, e) meropenem or piperacillin/tazobactam

Discuss the management of bladder infections

1) uncomplicated UTI - a) 1st line - abx: nitrofurantoin (5 days) or trimethoprim/sulfamethoxazole (3 days) or trimethoprim (3 days) or Fosfomycin or pivmecillinam b) +/- symptomatic relief of dysuria: phenazopyridine (2 days when required) 2) complicated UTI - a) 1st line - abx: levofloxacin (7-14 days) or ciprofloxacin (7-14 days) or norfloxacin (7-14 days) b) +/- symptomatic relief of dysuria: phenazopyridine (2 days when required) 3) uncomplicated or complicated UTI: pregnant - a) 1st line - abx: nitrofurantoin (7 days) or Fosfomycin (single dose) 4) at risk of chronic recurrent UTI - a) 1st line - abx prophylaxis ± cranberry products: trimethoprim/sulfamethoxazole or nitrofurantoin b) +/- patient-initiated therapy

Describe the clinical signs which point to neuromuscular ventilatory compromise

1) unexplained hypercapnia 2) dyspnoea 3) sleep disruption 4) recurrent pneumonia 5) tachypnoea 6) signs of diaphragmatic weakness eg/ orthopnoea, paradoxical abdominal wall movement, respiratory alternans, REM associated hypoventilation 7) signs of abdominal muscle weakness eg/ dec cough effort 8) signs of bulbar weakness eg/ dysarthria, dysphonia, dysphagia, aspiration, OSA

Describe the current laws dictating epilepsy and driving

1) unprovoked seizure - a) patient must cease driving immediately & tell the DVLA. b) may be allowed to start driving again after six months if no further seizures and there are no clinical factors (such as a scar on the brain) or results from investigations (such as an EEG) which suggest a high risk of having another seizure 2) if established epilepsy - must have been completely free of seizures for one year, with or without taking anti-epileptic drugs (AEDs)

Describe the natural history of treated and untreated acute arterial occlusion

1) untreated will lead to amputation/life-threatening illness 2) surgery - high morbidity & mortality 3) risk of reperfusion syndrome - tissue damage caused when blood returns to tissue after a period of ischemia or lack of oxygen - results in inflammation and oxidative damage through the induction of oxidative stress rather than restoration of normal function. Cell leakage leads to acidosis and hyperkalaemia 4) compartment syndrome may occur in tight fascial plane 5) acute limb ischaemia may lead to peripheral nerve damage and chronic pain syndromes

List the criteria for endoscopic, surgical or radiological intervention

1) upper GI bleed - a) peptic ulcer disease - endoscopic therapies (all equivalent effectiveness) b) variceal bleed - endoscopic therapies include banding and sclerotherapy I) if endoscopic treatment is unsuccessful: TIPS (transjugular intrahepatic portosystemic shunt) 2) lower GI bleeding - a) upper GI endoscopy to rule out an upper GI source b) proctosigmoidoscopy (e.g. haemorrhoids) c) colonoscopy (may not be very helpful acutely, can be used to treat lesions if bleeding is minor) d) rapid bleeding: angioembolisation OR surgery e) mild bleeding: 99Tc-RBC scan -> angioembolisation OR surgery surgical intervention may be indicated for - diverticular disease, unmanageable polyps and malignancies, other lesions (e.g. AVMs or inflammatory bowel disease) that are not amenable to endoscopic management

Describe the investigation of a patient with a suspected UTI

1) urine dipstick -treat empirically if nitrites or leukocytes +ve whilst waiting for results of MSU culture (pure growth of >10^5 organisms/mL = diagnostic) 2) blood tests - FBC, U&E, CRP, blood cultures, lactate if systemically unwell (urosepsis) 3) USS and referral to urology for assessment if child, male, failure to respond to treatment, recurrent UTI, pyelonephritis, unusual organism, persistent haematuria 4) renal tuberculosis - high suspicion if sterile pyuria + risk factors (esp if HIV +ve), look for high ESR/CRP, ask about past TB. May detect on culture

Outline the investigation necessary to confirm the diagnosis of NS

1) urine dipstick analysis: proteinuria and check for microscopic haematuria 2) MSU for MC&S to exclude UTI 3) quantify proteinuria using an early morning urinary protein:creatinine ratio or albumin:creatinine ratio 4) FBC and coagulation screen 5) renal function tests 6) LFTs (to exclude liver pathology); bone profile (calcium, phosphate, alkaline phosphatase) 7) check for other systemic diseases and causes of nephrotic syndrome: a) ESR and CRP. b) fasting glucose c) immunoglobulins, serum and urine electrophoresis d) autoimmune screen if an underlying autoimmune disease is suspected: autoantibodies and complement levels e) hep B & C; HIV 8) CXR and abdominal or renal ultrasound scan (esp if renal function abnormal): check for pleural effusion or ascites, presence of two kidneys, size & shape of kidneys, any urinary tract obstruction 9) consider complications: a) lipids - hyperlipidaemia b) doppler ultrasound of leg veins in suspected DVT c) abdominal ultrasound, renal vein Doppler scan, venography of the inferior vena cava, CT and MRI scanning of the abdomen if renal vein thrombosis suspected d) ventilation-perfusion scan - 'VQ' nuclear medicine lung scan; CT, pulmonary angiography for pulmonary embolism 10) renal biopsy under ultrasound; renal biopsy may be helpful to guide diagnosis and treatment but is not indicated in all patients with nephrotic syndrome

Describe the methods of evaluating diabetic control

1) urine testing - good as an idea of glucose levels, but relationship between urine glucose levels & blood glucose levels not exact - changes in urine glucose lag behind blood changes, mean renal threshold is 10mM, but range is wide, urine tests can give no guidance on glucose levels below renal threshold (hypos go unchecked) 2) self-monitored blood glucose testing - fingerprick glucose if on insulin - advises on dose, best day-day assessment 3) measurement of HBA1C - mean glucose levels over previous 8 weeks - complications inc as HbA1C inc, but target values depend on pt situations (eg/ elderly more at risk of fall when hypoglycaemic = less tight control), normally 48-57mmol/L NB/ figure misleading in anaemia, thalassaemia, haemoglobinopathy or pregnancy - shorter-term fructosamine test may be used 4) continuous glucose monitoring (ask about hypos)

Discuss the clinical use of thrombolysis, including monitoring and complications

1) use - lysis of blood clots formed in blood vessels, eg/ in STEMI, stroke, and very large PEs eg/ Streptokinase, Anistreplase, Recombinant tissue plasminogen activators: alteplase, reteplase, Tenecteplase 2) monitoring - need to monitor: GCS, BP, pulse, temperature > 37.5 degrees centigrade, O2 sats, RR - if any change greatly or fall out of normal range needs medical attention 3) complications - haemorrhagic stroke is a rare but serious complication of thrombolytic therapy. If a patient has had thrombolysis before, an allergy against the thrombolytic drug may have developed (especially after streptokinase). If the symptoms are mild, the infusion is stopped and the patient is commenced on an antihistamine before infusion is recommenced. Anaphylaxis generally requires immediate cessation of thrombolysis a) signs of intracranial haemorrhage - dec GCS, any neurological deterioration (any limb weakness, speech disturbance), new onset headache, N/V, inc BP b) signs of anaphylaxis - marked hypotension, acute SOB, facial swelling, new onset of wheeze

Describe the routine care of central venous lines

1) use an aseptic technique, wear sterile gloves when carrying out dressing changes and when accessing the catheter 2) monitor temperature, pulse, BP, RR, and O2 sats at least 12 hourly to detect infection 3) do not allow air to enter the catheter eg/ from syringes 4) cap off the catheter with a needle-free access device when not in use 5) monitor for signs of systemic or local infection 6) prophylactic abx when required 7) always secure the catheter firmly to the skin away from the exit site with tape or with a dedicated device such as 'Statlock' for patient's comfort, to prevent tension or accidental dislodgement, and to reduce 'to and fro' motion which increases the risk of catheter related sepsis 8) before it is used for administering drugs or fluids, the patency and correct functioning of the catheter should be established 9) signs of catheter occlusion, partial or complete, should be escalated

Discuss the associated electrolyte abnormalities and describe the management of life threatening hyperkalaemia

1) values - a) significant hyperkalaemia = serum potassium value >6.0 mmol/L (>6.0 mEq/L) b) moderate = 5.0-6.0 mmol/L (5.0-6.0 mEq/L) 2) mostly due to high intake of potassium in the setting of decreased renal excretion or to extracellular redistribution of potassium from intracellular locations 3) common acute manifestations of significant hyperkalaemia - muscle weakness & ECG changes that can potential to progress to a life-threatening arrhythmia 4) ECG - loss of P waves, tall tented T waves, broad QRS 5) mx - a) assess ABCDE including use of ECG (needs constant monitoring to pick up changes) b) get help & ensure pt is on HDU etc c) calcium chloride (10% in 10mls stat IV) - repeat ECG and consider further dose after 5 min if ECG changes persists (protects the heart) d) insulin-glucose IV infusion if severe hyperkalaemia (50mls 50% + 10 units insulin all IV) e) salbutamol 10-20 mg neb if severe hyperkalaemia f) if no improvement consider dialysis - seek advice from Renal or ICU team g) consider cause of hyperkalaemia, prevent further rise and recurrence - stop all nephrotoxic meds including ACEI, angiotensin II receptor blockers, potassium-sparing diuretics, NSAIDS and assess diet

Valvular heart disease - outline the common symptoms

1) valve disease symptoms can occur suddenly. If it advances slowly heart may adjust & may not notice onset of any symptoms. Severity of symptoms does not necessarily correlate to severity of valve disease 2) many of symptoms are similar to those associated with CCF eg/ SOB + wheezing after limited physical exertion, peripheral oedema. Other symptoms include: 3) palpitations, chest pain (may be mild) 4) fatigue 5) dizziness or fainting (with aortic stenosis) 6) fever (with bacterial endocarditis) 7) rapid weight gain 8) mitral stenosis: no symptoms if mild, or progressive severe dyspnoea, palpitations, cough with frothy sputum, atrial fibrillation, pulmonary hypertension, chest infections, systemic emboli 9) mitral regurgitation: none if mild. Inc stroke volume after cardiac enlargement may be felt as palpitation, dyspnoea, orthopnoea, fatigue, atrial fibrillation. Emboli less common 10) aortic stenosis: no symptoms until disease is moderately severe. Then - exercise induced syncope, angina and dyspnoea develop 11) aortic regurgitation: symptoms occur late, and only when LV failure occurs - SOB, fatigue, syncope, orthopnoae, oedema, palpitations, arrhythmias, angina on exertion

Describe the clinical manifestations of portal hypertension

1) varices within esophagus, stomach, rectum, or umbilical area - varices can rupture and bleed, resulting in potentially life-threatening complications 2) hepatosplenomegaly 3) ascites 4) hepatorenal syndrome 5) encephalopathy 6) fetor hepaticus (pear drops) NB/ prognosis related to bilirubin levels, albumin levels, ascites, encephalopathy & nutrition by Pugh-Child score

Recognise varicose veins and describe their anatomical distribution and potential complications

1) varicose veins are tortuous, dilated veins, commonly in the lower limbs - extremely common in Western countries 2) classified as primary (the majority) - idiopathic in nature and caused by an underlying valve defect, or secondary - with an underlying cause eg/ pelvic mass occluding venous return, previous DVT, arteriovenous fistulae or Klippel-Trénaunay syndrome 3) VV more common in the long saphenous vein. "Saphena varix", dilation of the saphenofemoral junction may disappear on lying down and has a bluish tint - not to be mistaken for a femoral hernia, difficult as both show positive cough impulse varicose veins can cause complications because they stop your blood flowing properly: 1) immediate cosmetic implication = varicosities (dilated veins) - may be only "symptom" 2) bleeding - vv near surface of skin can be cut. Bleeding difficult to stop. Minor haemorrhage into subcutaneous tissue, or major haemorrhage at high pressure venous junction 3) blood clots - if in superficial veins can lead to: a) thrombophlebitis - swelling (inflammation) of the veins in your leg (can cause pain) b) DVT -can cause pain & swelling in leg, may lead to serious complications eg/ PE 4) chronic venous insufficiency - if blood in veins doesn't flow properly, it can interfere with the way your skin exchanges oxygen, nutrients and waste products with your blood. Can lead to other conditions: a) varicose eczema - skin becomes red, scaly, flaky b) lipodermatosclerosis -causes skin, usually around calf area, to become hardened and tight, may turn a red or brown colour (due to haemosiderin as static blood leads to RBC breakdown). Chronic deposition with fibrin may lead to an inverted champagne bottle calf c) venous leg ulcers -develops when there's increased pressure in veins of your lower leg - may eventually cause an ulcer

Outline less common causes of Parkinsonism including drugs and the Parkinson-plus syndromes

1) vascular parkinsonism - form of atypical parkinsonism in which parkinsonian symptoms (slow movements, tremor, difficulty with walking and balance, stiffness and rigidity) are produced by 1+ small strokes, rather than gradual loss of nerve cells a) predominant involvement of legs ("lower-body parkinsonism") with gait & balance problems, no tremor, poor response to levodopa (different to P), CT or MRI brain scans showing multiple minute or more extensive strokes, bilateral (different to P), falls, voice changes, cognitive deficits early (different to P) 2) drug induced parkinsonism - Parkinson symptoms due to dopamine antagonists which block dopamine pathways eg/ antipsychotics and anti-emetics - Risperidone, Olanzapine, Metoclopramide 3) Lewy body dementia - cause unknown, buildup of Lewy bodies, clumps of a-synuclein protein in neurons a) worsens over time, dementia comes before parkinsonism, may have visual hallucinations, fluctuations in cognition, slow mvmt, trouble walking, and rigidity, excess mvmt during sleep, mood changes eg/ depression 4) Parkinson's plus syndrome a) progressive supranuclear palsy - deterioration of cells due to tau protein in areas of brain controlling body movement and thinking (midbrain & frontal) I) loss of balance while walking, tendency to fall backward, inability to aim eyes properly, stiffness and awkward movements b) multi-system atrophy - progressive neurodegenerative disorder affecting ANS + mvmt, due to accumulation of protein a-synuclein in glia I) orthostatic hypotension, incontinence, difficult thermoregulation, cerebellar signs: affects HR, erectile dysfunction, motor - tremor, rigidity, loss of muscle coordination, gait, difficulties with speech c) corticobasal degeneration - rare, progressive neurodegenerative disease of cerebral cortex & basal ganglia I) marked disorders in mvmt & cognitive dysfunction, apraxia, aphasia, alien hand syndrome

Outline the clinical types of syncope

1) vasovagal syncope (aka cardio-neurogenic syncope) - most common type of syncope, caused by a sudden drop in BP, which causes a drop in blood flow to the brain. When you stand up, gravity causes blood to settle in the lower part of your body, below your diaphragm. When that happens, the heart and autonomic nervous system (ANS) work to keep your blood pressure stable 2) orthostatic hypotension - type of vasovagal, keeps the blood vessels from getting smaller (as they should) when the patient stands. This causes blood to collect in the legs and leads to a quick drop in blood pressure 3) situational syncope - type of vasovagal, happens only during certain situations that affect the nervous system and lead to syncope eg/ dehydration, intense stress, anxiety, fear, pain, hunger, alcohol or drugs, hyperventilation, coughing forcefully, turning the neck, or wearing a tight collar, urinating 4) postural syncope (postural hypotension) - sudden drop in BP due to a quick change in position, such as from lying down to standing. Certain medications and dehydration can lead to this condition. BP drops by at least 20 mmHg (systolic) and at least 10 mmHg (diastolic) when they stand 5) cardiac syncope - caused by a heart or blood vessel condition that affects blood flow to the brain. Can include arrhythmia, obstructed blood flow in the heart due to structural heart disease, myocardial ischemia, valve disease, aortic stenosis, blood clot, or heart failure 6) neurologic syncope - caused by a neurological condition such as seizure, stroke or TIA. Others - migraines and normal pressure hydrocephalus 7) Postural Orthostatic Tachycardia Syndrome (POTS) - caused by tachycardia when a person stands after sitting or lying down. HR can speed up by 30 bpm or more. Inc happens within 10 minutes of standing. Mostly women 8) 1/3rd have unknown cause

Discuss the role of platelets in the pathophysiology of vascular disease including vascular thrombosis and platelet emboli

1) venous thrombosis - Virchow's triad: endothelial injury + hypercoagulability + stasis predispose to formation of venous thrombosis. 2) endothelial Injury - a) thrombi consisted mainly of fibrin + platelets b) thought stimulus for endothelial injury is hypoxia, causing inc expression of P-selectin on endothelial cells c) P-selectin may contribute to thrombosis by various mechanisms including promoting platelet-endothelial and platelet-leukocyte-endothelial interactions. Endothelial P-selectin surface expression is associated with a variety of inflammatory stimuli, and local inflammation has also been postulated to contribute to the initiation of deep venous thrombosis 3) hypercoagulability - a) hypercoagulable state = conditions that favour a procoagulant state due to imbalances in the haemostatic mechanisms, resulting in an unusual tendency toward thrombosis b) activation of coagulation factors plays a major role in venous thrombosis by concentrating activated factors in regions of reduced flow, such as the valve pockets 4) stasis - a) potential mechanism by which stasis predisposes to venous thrombosis is by inducing local hypoxia and stimulating endothelial release of P-selectin b) in addition to P-selectin, hypoxia-induced exocytosis of Weibel-Palade bodies results in increased release of vWF, which may promote platelet recruitment

List criteria to help differentiate leg ulcers

1) venous ulceration - a) large b) superficial c) gaiter area d) signs of chronic venous disease eg/ varicose veins, lipodermatosclerosis, atrophic blanche, chronic oedema, venous eczema, haemosiderin colour e) may or may not be painful 2) arterial ulceration - a) typically painful b) often small, punched out ulcers c) tend to be deep (necrotic) d) occur over bony prominences & where there are end arteries e) usually dry base without granulation tissue f) symptoms of arterial disease eg/ hairless skin, dec or absent pulses & doppler flow measurement, cold leg, poor capillary return

Outline the physiological absorption of vitamin B12 and folate

1) vitB12 - in meat, fish & dairy, protein-bound & released during digestion, then binds to intrinsic factor in stomach (made by parietal cells) - this complex is then absorbed by terminal ileum. In B12 deficiency, synthesis of thymidine, hence DNA is impaired so RBC production is dec. Stored in liver, store takes time to wear down 2) folate - needed for production & maintenance of cells for DNA + RNA synthesis through methylation, & for preventing changes to DNA - prevents cancer, absorbed in SI + colon with dec absorptive gradient from jejunum to colon. Folates present in food as polyglutamates in dihydrofolate or tetrahydrofolate forms. Polyglutamates broken down to monoglutamates in upper GI tract & during absorptive process are converted to methyl tetrahydrofolate monoglutamate. Folate in green vegetables and offal, absorbed in jejunum. Stores take less time to wear down

Discuss the normal control of serum calcium and outline the actions of PTH, vitamin D and calcitonin

1) vitD & calcitriol - vit D hydrolysed in liver to 25-hydroxy-vitD and in kidney to 1, 25-dihydroxy-vit D (calcitriol - biologically active) calcitriol production stimulated by dec Ca2+, PO4^3- & PTH actions: inc Ca2+ & PO4^3- absorption from gut + kidney, inhibition of PTH release, enhanced bone turnover disordered regulation of calcitriol = familial normocalcaemic hypercalciuria 2) PTH - inc Ca2+ & dec PO4^3 - secreted by chief cells of parathyroid glands - triggered by dec serum ionized Ca2+ (calcium sensing receptors on plasma membrane), controlled by -ve feedback loop inc osteoclast activity releasing Ca2+ & PO4^3 from bones, inc Ca2+ & dec PO4^3 reabsorption in kidney + intestines, inc renal production of calcitriol 3) calcitonin - secreted by C-cells of thyroid causing dec resorption of Ca2+ & PO4^3- (inhibits osteoclast activity, inc renal excretion of Ca2+ & PO4^3. Calcitonin dec serum Ca2+ levels - released in response to hypercalcaemia) used as a marker of recurrence or metastasis in medullary carcinoma of thyroid

Describe the symptoms and signs in a patient with intestinal obstruction

1) vomiting + absolute constipation = cardinal symptoms 2) colicky pain (abdominal) 3) distention 4) nausea 5) anorexia 6) tinkling bowel sounds 7) marked tenderness suggests strangulation

Describe the laboratory assessment of pituitary function and outline the radiological techniques used to investigate pituitary disease

1) want to detect excess or deficient hormone production, determine the cause, and evaluate the severity of the condition 2) measure hormones that the pituitary produces eg/ TSH, and the hormones of other endocrine glands that the pituitary is responsible for stimulating eg/ thyroid hormone thyroxine 3) concentrations of some hormones (eg/ TSH) are relatively constant in the blood, whilst others vary over the course of a day, (GH), or over a cycle (such as FSH and LH). The concentration of a hormone may also be dependent on specific situations. For example, prolactin is raised in women that are breast feeding a) sometimes, because of these variations, 1 blood test does not give enough information b) tests which involve the patient taking meds to either suppress or stimulate hormone production can be used to detect hormonal excess or deficiency respectively 4) complete anterior pituitary function test includes: a) gonadotrophin-releasing hormone (GNRH) stimulation aka luteinising-hormone-releasing hormone stimulation b) thyrotrophin-releasing hormone (TRH) stimulation c) insulin tolerance test (or alternative method of stimulating growth hormone and adrenocorticotropic hormone (ACTH) secretion) 5) growth hormone - varies over day: a) low baseline IGF-1 &2, and insulin-like growth factor-binding protein 3 (IGFBP-3) suggest growth hormone deficiency. IGF-1 is most commonly measured b) insulin tolerance test (aka insulin stress test) is used to stimulate growth hormone and ACTH secretion I) blood is taken for growth hormone, cortisol and glucose and then fast-acting insulin is injected IV. Bloods repeated at 30, 45, 60, 90 and 120 minutes 6) ACTH - a) serum cortisol level is measured at 9 am. If low, ACTH deficiency is likely b) Synacthen test: measures adrenal response to ACTH I) Synacthen (ACTH analogue) is administered as IV bolus with measurements of cortisol (& 17-hydroxyprogesterone if possible congenital adrenal hyperplasia) at 0 minutes, 30 minutes and 60 minutes II) adrenal insufficiency excluded by an incremental rise in cortisol of >200 nmol/L and a 30-minute value >600 nmol/L. In ACTH deficiency the response to the short test may be normal or reduced c) can also do insulin tolerance/stress test (see above): adequate cortisol response is rise to >550 nmol/L 7) gonadotrophin deficiency - FSH & LH, and either estradiol or testosterone (as appropriate for sex can be measured a) GnRH testing: samples measuring LH and FSH levels at baseline and at 20 minutes and 60 minutes after giving a short-acting GnRH analogue. Samples for testosterone (males) and estradiol (females) are also taken at the baseline I) FSH & LH should at least double at 20 minutes and LH response is usually greater 8)TSH - thyrotrophin-releasing hormone (TRH) stimulation test: a) small amount of TRH given IV with blood tests for TSH at 0, and consecutively I) should have inc TSH following TRH injection 9) prolactin - levels taken on 2+ occasions and after relaxation, however 1 prolactin measurement sufficient if the value is >200 μg/L 10) ACTH - growth hormone as above eg/ IGF 1 11) posterior pituitary - ADH & oxytocin (stimulates uterine contraction during birth and ejection of milk during lactation) a) if suspected diabetes insipidus assess by a water deprivation test b) initial tests: plasma glucose, U&Es; plasma and urine osmolality 12) non-laboratory tests - MRI or CT

List the different oral anticoagulant options, including any key advantages and disadvantages

1) warfarin pros - a) regular INR test so can monitor tx b) can use interacting meds as regular INR measurements c) reversible - vitamin K d) experience (50+yrs) e) many indications f) cheaper g) longer half-life (gd if forget) 2) warfarin cons - a) need to regularly measure INR b) interactions - P450, cranberry juice c) reputation (rat poison) 3) DOACS pros - don't need to monitor INR 4) DOAC cons - a) not many indications b) more expensive c) shorter half life d) can't use any drugs that interact e) don't know INR levels - can't monitor tx f) not reversible except dabigatran

Describe the basic principles of prevention of infection in hospitals (isolation, cohort nursing) and in the community (notification of infectious diseases, vaccination and other prophylactic measures)

1) wash hands with warm soap and water vigorously for at least 20 seconds, before drinking, eating, providing care and between caring for patients 2) identify contagions ASAP eg/ C. diff, then initiate isolation if appropriate (staff may need: waterproof gowns, gloves, shoe covers, face shields and masks) 3) use gloves when there's any risk of infection eg/ in contact with bodily fluids 4) disinfect and keep surfaces clean 5) change linens daily and when dirty 6) make sure foods are kept at proper temperatures 7) notification of infectious diseases to public health England when appropriate 8) vaccination - herd vaccination to try and dec spread of certain diseases

List the main endogenous factors that affect renal control of sodium and water excretion

1) water balance - a) kidneys conserve water by producing concentrated urine, or rid the body of excess water by producing dilute urine relative to plasma b) direct control of water excretion in kidneys by vasopressin (ADH) - peptide hormone secreted by hypothalamus. ADH causes insertion of aquaporins into cells lining collecting ducts = water reabsorption. Without ADH, little water is reabsorbed in collecting ducts & dilute urine is excreted c) ADH secretion is influenced by several factors (NB/ anything that stimulates ADH secretion also stimulates thirst): special receptors in hypothalamus that stimuate ADH secretion with inc plasma osmolarity (when plasma gets too concn), stretch receptors in atria of heart - activated by a larger than normal volume of blood returning to heart fromveins - inhibit ADH, stretch receptors in aorta + carotid arteries - stimulated when BP falls - stimulate ADH secretion 2) sodium balance - a) regulation of osmolarity = balancing intake + excretion of sodium & water. (Sodium = major solute in extracellular fluids, so determines osmolarity of extracellular fluids) b) ADH lowers osmolarity (dec sodium concn) by inc water reabsorption in kidneys, diluting bodily fluids. To prevent osmolarity dec below normal, kidneys have a mechanism for reabsorbing sodium in distal nephron, controlled by aldosterone - steroid hormone produced by adrenal cortex. Aldosterone secretion control: I) adrenal cortex directly senses plasma osmolarity. When it inc above normal, aldosterone secretion is inhibited so less sodium is reabsorbed in the distal tubule II) kidneys sense low BP (lower filtration rates & flow through tubule) - triggers response to raise BP and conserve volume. Juxtaglomerular cells in afferent + efferent arterioles produce renin - peptide hormone that initiates hormonal cascade producing angiotensin II, which stimulates the adrenal cortex to produce aldosterone NB/ all involves RAAS - initially stimulated by baroreceptors and osmoreceptors, ADH, ANP, Hypothalamus (thirst)

List the water soluble and fat-soluble vitamins

1) water soluble - vitamins B + C, 2) fat soluble - vitamins A, D, E, K, travel in lymphatics of SI & general circulation of blood via chylomicrons

List at least four parameters obtained from a patient's medical history that might indicate the presence of malnutrition. List appropriate anthropomorphic measurements

1) weight loss - a) unintentionally lose 5-10% of their body weight within 3-6 months b) BMI <18.5 2) getting ill often and taking a long time to recover 3) wounds taking a long time to heal 4) low mood or depression 5) other diseases affecting cognition and concentration eg/ dementia 6) anthropometric measureemnts - non-invasive, quantitative body measurements used to assess growth, development, and health parameters: a) height/length b) weight c) BMI d) head circumference

Outline epidemiological links between cholesterol and cardiovascular risk

1) when there is too much cholesterol in the blood it builds up in the walls of arteries, causing atherosclerosis. Arteries become narrowed and blood flow to the heart muscle is slowed down or blocked. Research is now showing that high levels of triglycerides may also be linked to heart disease 2) most men with LDL receptor defects die before 60, a majority due to coronary artery disease. 3) serum cholesterol has been graphed to deaths per 1000 men; 4mM - 2, 5mM - 4.5, 6mM - 8, 7mM - 14 4) targets: total cholesterol < 5mM, LDL < 3mM

List the common causes of splenomegaly including portal hypertension, lympho-reticular disease and chronic infection

1) with fever - infection (malaria, hepatitis, EBV, TB, CMV, HIV), sarcoid, malignancy 2) with lymphadenopathy - glandular fever, leukamieas, sjogren's syndrome 3) with purpura - septicaemia, typhus, amyloid, meningococcaemia 4) with arthritis - sjorgren's, RA, infection (eg/ lyme), vasculitis/behcet's 5) with ascites - carcinoma, portal hypertension 6) with murmur - SBE/IE, rheumatic fever, hypereosinophilia, amyloid 7) with anaemia - sickle cell, thalasseamia, leishmaniasis, leukaemia, pernicious anaemia, poems syndrome 8) with dec weight & CNS signs - cancer, lymphoma, TB, arsenic poisoning, paraptoteinaemia 9) massive splenomegaly - chronic malaria, myelofibrosis, CML, gaucher's, visceral leishmaniasis (kala-azar) NB/ spleen responsible for removal of damaged RBCs from circulation, cleaning blood of debris, destroying encapsulated pathogens (Strep pneumo, Hib, meningococcus) NB2/ splenomegaly leads to hypersplenism = pancytopenia + BM hyperplasia

Explain the term CSF xanthochromia

1) yellowish appearance of cerebrospinal fluid that occurs several hours after bleeding into the subarachnoid space 2) most commonly subarachnoid hemorrhage from a ruptured cerebral aneurysm 3) CSF fills the SA space between arachnoid & pia mater surrounding the brain, normally clear and colorless a) with bleeding into SA space, the initial appearance of CSF can range from barely tinged with blood to frankly bloody, depending on the extent of bleeding b) within several hours, RBC in cerebrospinal fluid are destroyed, releasing haem, which is metabolized by enzymes to bilirubin, a yellow pigment 4) usually do CT head for suspected SA hemorrhage, but it detects only 98% of cases in the first 12 hours after the onset of symptoms, so LP can confirm if pts have s/s eg/ thunderclap headache, vomiting, dizziness, new-onset seizures, confusion, dec LOC or coma, neck stiffness or other signs of meningismus, and signs of sudden inc intracranial pressure), but no blood is visible on the CT scan

Describe the natural history of asymptomatic gallstones

1) young females more affected (2x inc risk compared to men) 2) smokers 3) obese or have lost a lot of weight in a short amount of time 4) diabetic or family history 5) can happen if have diet extremely low in fat 6) factors that can dec activity in the gallbladder, possibly leading to gallstone formation = cirrhosis, OCP, HRT, pregnancy 7) cholesterol drugs NB/ may be picked up during abdominal imaging undertaken for another reason

Discuss the microvascular complications of diabetes affecting the eyes, and outline their relationship to diabetic control and disease duration

1) ~1/3 young diabetics will suffer from retinal damage, 5% blind after 30 years 2) retinopathy, cataracts & external ocular palsies occur more frequently and earlier in diabetics - progression of retinopathy faster in T1DM than T2DM - good diabetic + BP control slows progression 3) background retinopathy: microaneurysms around posterior pole, may be some superficial haemorrhage, cotton wool spots = microinfarcts within retina - cleared by macrophages within 3-6 months a) proliferative retinopathy preceded by widespread non-perfusion b) ischaemia = VEGF release + development of intraretinal + preretinal new vessels (no symptoms) - prone to bleed & give rise to preretinal haemorrhage (on inferior half of retina) c) with further bleeding, blood seeps into vitreous fluid & obscures vision d) when new vessels grown older vessels undergo collagenous change - give rise to fibrotic traction bands e) bands may pull on retina = further haemorrhage & possible retinal detachment f) in T2DM microaneurysms inc in number & leak = fluid accumulation within retina g) if aneurysms are localized, they are associated with fat & protein deposition h) if oedema extends into macula = loss of central vision i) final outcome (after new vessel development & possible vitreous bleeds) = large exudative plaque in the central macular area

List the cranial nerve nuclei in each constituent part of the brainstem (midbrain, pons, medulla)

10 of the 12 pairs of CN either target or are sourced from the brainstem 1) midbrain - nuclei of the oculomotor nerve (III) and trochlear nerve (IV) 2) pons - nuclei of the trigeminal nerve (V), abducens nerve (VI), facial nerve (VII) and vestibulocochlear nerve (VIII) 3) medulla - nuclei of the glossopharyngeal nerve (IX), vagus nerve (X), accessory nerve (XI) and hypoglossal nerve (XII)

List the electrolyte composition of Hartmann's

131 mEq/L Na 111 mEq/L Cl 5 mEq/L K 29 mEq/L lactate 2 mEq/L Ca pH 5-7 osmolality 278

List the electrolyte composition of 5% glucose in water

1L of 5% Dextrose Injection contains dextrose, hydrous 50 g in water for injection. Caloric value is 170 kcal/L. Osmolarity is 252 mOsmol/L - slightly hypotonic pH is 4.3 (3.2 to 6.5) has 278mmol/L glucose (50g/L) NB/ dextrose readily metabolised by liver, leaving only water which expands total body water - only 1/12th remains in extracellular volume

Outline the haematological abnormalities correctable by splenectomy

3 types of congenital disorders associated with hemolytic anemia and may warrant splenectomy - disorders of erythrocyte membranes, hemoglobinopathies, erythrocyte enzyme deficiencies 1) autoimmune haemolytic anemia may be idiopathic or develop in the course of other disorders eg/ malignant lymphomas, chronic lymphatic leukemia, Iupus erythematosus and others 2) hereditary spherocytosis 3) sickle cell anemia 4) thalassemia 5) Idiopathic Thrombocytopenic Purpura 6) Thrombotic Thrombocytopenic Purpura (TTP) 7) hypersplenism 8) Hodgkin's disease (lymphogranulomatosis) and non-Hodgkin's lymphomas 9) myeloproliferative disorders eg/ acute and chronic leukemias, polycythemia vera, osteomyelofibrosis and essential thrombocythemia 10) Felty's syndrome - advanced RA + splenomegaly + neutropenia, inc risk of infections 11) Gaucher's disease

Classify pancreatitis on the basis of the severity of organ injury

3+ positive factors detected within 48hrs of onset suggests severe pancreatitis & should prompt transfer to ITU/HDU. Mnemonic = pancreas: PaO2: <8kPa Age: >55yrs Neutrophilia: WBC >15 x10^9/L Calcium: <2mmol/L Renal function: urea >16mmol/L Enzymes: LDH >600iu/L; AST >200iu/L Albumin: <32g/L (serum) Sugar: blood glucose >10mmol/L NB/ validated for pancreatitis caused by gallstone + alcohol NB/ pancreatitis may be acute or chronic, dependent on time scale, and the recurrence and persistence of inflammation. Acute pancreatitis ranked by severity into mild and severe using the APACHE II, Ranson or Modified Glasgow criteria

Describe the clinical features and management principles of other overdoses that present commonly to the Emergency Department, including cocaine and aspirin

4) cocaine - a) s/s - I) tachycardia - can be as high as 200 bpm II) HTN III) hyperthermia IV) agitation V) mydriasis VI) diaphoresis b) mx - I) 1st line - observation and monitoring II) agitation or HTN or seizures: + benzodiazepine - diazepam: 5-10 mg IV every 3-5 minutes until symptom control achieved III) volume depletion: + isotonic saline - 20-40 mL/kg IV as rapidly as tolerated until euvolaemia is restored IV) hyperthermia, temperature 39-41.2°C without severe shivering, agitation, or muscle rigidity: + external cooling and sedation - diazepam: 5-10 mg IV every 3-5 minutes until symptom control achieved V) arrhythmias: + anti-arrhythmic therapy or defibrillation - defibrillation or sodium bicarbonate: 1 mEq/kg intravenous infusion 5) aspirin (salicylate) - a) s/s - I) ingestion of >150 mg/kg, or >6.5 g of aspirin or aspirin-equivalent II) abnormal behaviour - unexplained delirium or confusion III) N&V, haematemesis, epigastric pain IV) fever and diaphoresis, and malaise V) SOB VI) tachypnoea, hyperpnoea, Kussmaul's respirations VII) tinnitus and/or deafness/and or dizziness VIII) movement disorders, asterixis, stupor IX) seizures, coma and/or papilloedema b) mx if asymptomatic - monitoring + GI tract decontamination - activated charcoal: 50-100 g orally as a single dose c) mx if symptomatic - I) mild 1st line - supportive care - airway patency and adequate breathing and circulation, correction of electrolyte abnormalities, 2 IV cannula inserted to begin adequate rehydration and to initiate alkalinisation if needed II) salicylate levels measured every 2 to 4 hours until concentration peaks, and the salicylate level falls below 30 mg/dL III) +/- GI tract decontamination - activated charcoal: 50-100 g orally as a single dose, or 25 g orally every 2 hours if multiple doses are required IV) moderate or severe - ICU admission + supportive care - may need intubation and mechanical ventilation, seizures terminated with benzos V) + serum and urinary alkalinisation - sodium bicarbonate VI) +/- GI tract decontamination - as above VII) +/- emergency haemodialysis

Quantify the prognosis for gastric cancer

5-year relative survival rates (excluding deaths from other causes) for patients with gastric cancer vary by stage 71% for patients with stage IA cancer 56% for stage IB 37% for stage II 18% for stage IIIA 11% for stage IIIB 5% for stage IV

List the electrolyte composition of Sodium chloride (0.9%)

9 g/L Sodium Chloride with an osmolarity of 308 mOsmol/L 154 mEq/L Sodium and Chloride NB/ expands extracellular compartment, so 75% goes to interstitial and 25% to intravascular. For plasma expanding purposes, 4x as much crystalloid must be given compared to colloid

Describe Dukes staging system

A = tumour confined to mucosa - 95% 5-yr survival B1 = tumour growth into muscularis propria - 80% B2 = tumour growth into muscularis propria + serosa (full thickness) - 60% C1 = tumour spread to 1-4 regional LN - 30% C2 = tumour spread to >4 regional LN - 30% D = distant metastases (liver, lung, bone) - <1%

Administer a VTE risk assessment using a recognized risk scoring system

ACCP VTE risk assessment for medical pts 1) assess for the following risk factors: a) congestive heart failure b) severe respiratory disease eg/ COPD or ILD c) immobility (confined to bed or needs assistance to ambulate) + at least 1 of: acute ischaemic stroke, active cancer, sepsis, acute neurological disease, IBD, prior venous thromboembolism or thrombophilia 2) none of the above present - VTE prophylaxis not required 3) any risk factor present - assess for the following contraindications: a) acute bleeding b) at risk of bleeding (eg/ brain lesion, frequent falls) c) recent (within 1 month) serious bleeding d) coagulopathy (INR >1.5 or APTT >40 seconds) e) thrombocytopenia (platelets <75) 4) no contraindications - start VTE prophylaxis eg/ enoxaparin 40 mg subcut OD, or fondaparinux 2.5 mg subcut OD 5) if any contraindications - mechanical VTE prophylaxis eg/ anti-embolic compression (TED) stockings, intermittent pneumatic compression device (Flotrons)

Describe the clinical presentation of motor neurone disease

Amyotrophic lateral sclerosis (ALS) 1) presence of risk factors - fmx, age >40 years 2) upper extremity weakness - due to UMN or LMN weakness 3) stiffness, with poor co-ordination and balance (UMN) 4) spastic, unsteady gait (UMN) 5) painful muscle spasms (LMN) 6) difficulties in arising from chairs and climbing stairs (LMN) 7) foot drop (LMN) 8) stiffness and decreased balance with impact on gait (UMN) 9) head drop (LMN) 10) progressive difficulties in maintaining erect posture, with stooping (LMN) 11) muscle atrophy (LMN) 12) inc lumbar lordosis and tendency for abdominal protuberance (LMN) 13) hyper-reflexia (UMN), pathological reflexes: Babinski's sign, cross adductors, jaw jerk. 14) dyspnoea (progressive diaphragmatic weakness) 15) coughing and choking on liquids (including secretions) and eventually on food (UMN - lack of co-ordination of the tongue and pharyngeal constrictor muscles; LMN - actual weakness of tongue & pharyngeal muscles) 16) strained, slow speech (UMN) 17) slurred, nasal, and, at times, dysphonic speech (LMN) 18) propensity for falls (UMN or LMN weakness) 19) sialorrhoea and drooling 20) inappropriate bursts of crying or laughing (pseudobulbar affect)

Determine hypermobility syndrome using a modified 9 point Beighton score

Beighton score is a simple system to quantify joint laxity and hypermobility (inc score = inc laxity) threshold for joint laxity in a young adult ranges from 4-6, so >6 = hypermobility 1) right/left little (fifth) finger - passive dorsiflexion beyond 90° = 1 point each side 2) right/left thumb - passive dorsiflexion to the flexor aspect of the forearm = 1 point each side 3) right/left elbow - hyperextends beyonds 10° = 1 point each side 4) right/left knee - hyperextends beyonds 10° = 1 point each side 5) forward flexion of trunk with knees full extended - palms and hands can rest flat on the floor = 1 point

Describe the assessment of CKD using estimated glomerular filtration rate (eGFR) and the five stages of CKD

CKD = kidney damage or GFR<60mL/min/1.73m^2 for 3+ months. Kidney damage = pathological abnormalities or markers of damage, including abnormalities in blood, urine and imaging studies Stage 1: GFR >90 mL/min - normal or inc GFR with other evidence of renal damage Stage 2: GFR 60-89 - slight dec in GFR with evidence of renal damage Stage 3: A = GFR 45-59, B = 30-44 - moderate dec in GFR with or without evidence of renal damage Stage 4: GFR 15-29 - severe dec in GFR with or without evidence of renal damage Stage 5: GFR <15 - established renal failure NB/ other evidence of renal damage - proteinuria, haematuria, evidence of abnormal anatomy or systemic disease

Outline the clinical presentation of venous sinus thrombosis and how it differs from the presentation of arterial stroke

Cerebral venous sinus thrombosis (CVST) occurs when a blood clot forms in the brain's venous sinuses, preventing blood from draining out of the brain. As a result, blood cells may break and leak blood into the brain tissues, forming a haemorrhage 1) almost all have a headache - worsens over several days, but may also develop suddenly (thunderclap headache). The headache may be the only symptom 2) many have symptoms of stroke: inability to move one or more limbs, weakness on one side of the face or difficulty speaking, but doesn't always affect one side of the body as in the more common "arterial" stroke 3) some have seizures - more common in women who develop sinus thrombosis peripartum, seizures mostly affect only one part of the body and unilateral, but occasionally seizures are generalised and rarely lead to status epilepticus 4) in the elderly, many of the aforementioned symptoms may not occur. Common symptoms in the elderly are otherwise unexplained changes in mental status & dec LOC 5) pressure around the brain may rise, causing papilledema which may be experienced as visual obscurations. In severely raised intracranial pressure, have dec LOC, inc BP, dec HR and patient assumes an abnormal posture

Understand the aetiological classification of diabetes mellitus including the usual presentations of type 1 and type 2 diabetes

DM = group of metabolic diseases in which there are high blood sugar levels over a prolonged period 1) T1DM: usually adolescent but can start at any age. Pancreas fails to produce enough insulin - autoimmune destruction of pancreatic B cells. Pts must have insulin & prone to ketoacidosis & weight loss (also LADA) a) weight loss b) persistant hyperglycaemia despite diet & medications c) autoantibodies - islet cell antibodies & anti-glutamic acid decarboxylase antibodies d) ketonuria on urine dipstick/ketoacidosis e) onset over weeks 2) T2DM: higher prevelance in Asians, men, elderly. Mostly >40 but now teenagers. Begins with insulin resistance (cells fail to respond to insulin properly - impaired glucose tolerance/fasting glucose) - as it progresses a lack of insulin secretion may also develop. Most common cause is excessive body weight and not enough exercise 3) gestational diabetes - occurs when pregnant without a previous history of diabetes 4) MODY = rare autosomal dominant form of T2DM (5-10% pts) affecting young people with a +ve family history

Framingham criteria

Framingham criteria - 2 majors or 1 major + 2 minors 1) major - a) paroxysmal nocturnal dyspnoea a) crepitations b) S3 gallop c) cardiomegaly (cardiothoracic ration >50% on CXR) d) inc central venous pressure (>16cmH2O in RA) e) weight loss (>4.5kg in 5 days in response to treatment) f) neck vein distension (inc JVP) g) acute pulmonary oedema h) hepatojugular reflex 2) minor - a) bilateral ankle oedema (also ascites) b) dyspnoea on normal exertion c) tachycardia (>120bpm) d) dec vital capacity e) nocturnal cough f) hepatomegaly g) pleural effusion

Describe the scoring system of the GCS, including the individual grades of each of the three domains

GCS - looks at eye activity, verbal and motor responses, and assigns points for each to give a composite score, 3 being deeply unconscious and 15 being fully conscious: 1) motor (/6) - movement in response to command = 6 localizes pain = 5 withdraws from pain = 4 flexes in response to pain = 3 extends in response to pain = 2 no response = 1 2) verbal (/5) - fully orientated = 5 confused = 4 inappropriate words = 3 incomprehensible sounds = 2 no response = 1 3) eyes (/4)- eyes open spontaneously = 4 eyes open to command = 3 eyes open in response to pain = 2 eyes remain closed = 1 NB/ need to be able to apply GCS in the assessment and monitoring of a semi-conscious pt

Outline the clinical management of over -anticoagulation with warfarin

INR > therapeutic range but <5, no bleeding = lower warfarin dose or omit next dose, resume when INR lower INR 5-9, no bleeding - stop warfarin, if bleeding risk high give vitamin K1 (1-2mg PO or 0.5-1mg IV), resume warfarin when INR therapeutic INR >9, no bleeding - cease warfarin, give 2.5-5mg vitamin K1 PO or 1mg IV, resume when INR <5, if high risk of bleeding give prothombinex-HT & FFP If any significant bleeding stop warfarin, give 5-10mg vitamin K1 IV + prothombinex-HT 25-50IU/kg + FFP (150-300mL) until INR <5

List the objective parameters that reflect a patient's nutritional state and their drawbacks

MUST screening - Malnutrition Universal Screening Tool: everyone on admission then weekly 1) step 1 = BMI score: >20 = 0, 18.5-20 = 1, <18.5 = 2 + 2) step 2 = unplanned weight loss in last 3-6 months - <5% = 0, 5-10% = 1, >10% = 2 + 3) step 3 = acute disease effect score - acutely unwell or likely no nutritional intake for > 5 days = 2 4) step 4 - add scores together: 0 = low risk - repeat weekly in hospital, care homes monthly, community annually 1 = medium - observe (document intake for 3 days, if adequate review as low risk, if inadequate follow local policy) 2+ = high risk - treat (refer to dietician, set goals, improve & inc overall nutritional score, monitor & review) 5) overall aim of tx = maintain nutritional status, inc weight or improve nutritional status 6) oral - always try food first, little and often, choose full fat & sugary options, nourishing drinks, food enrichment, multivitamin 7) if nutritional requirements cannot be met - oral nutritional supplement, enteral & parental 8) pts unable to take any/sufficient nutrition orally but GI tract functioning, feed enterally 9) parenteral nutrition (PN) = provision of nutrients IV, only when can't supply nutrition using GI tract (ie intestinal failure)

Outline the difficulties of defining hypertension......... and outline the levels of blood pressure defined as normal, borderline or raised including the need to confirm with a 3+ measurements

Stage 1 hypertension - clinic BP is 140/90 mmHg or higher AND Ambulatory BP monitoring (ABPM) daytime average OR Home BP monitoring (HBPM) average BP is 135/85 mmHg or higher Stage 2 hypertension - clinic BP is 160/100 mmHg or higher AND ABPM daytime average or HBPM average BP is 150/95 mmHg higher Severe hypertension - clinic systolic BP is 180 mmHg or higher or clinic diastolic BP is 110 mmHg or higher BP targets - <80 years =< 140/90 mmHg, > 80 years = <150/90 mmHg lower BP targets recommended for diabetics - T2DM = <140/80mmHg, or <135/75mmHg if microalbuminuria or proteinuria is present, T1DM = <135/85mmHg, or <130/80mmHg with nephropathy NB/ diagnosis of hypertension should be based on 3+ readings NB2/ automated machines not used for BP measurements from pts with irregular pulses. Take readings from both arms (within 20mmHg?) - record from arm with higher values. If clinic BP >140/90, ABPM/HBPM to confirm diagnosis. ABPM needs 2 readings/hr during regular ambulatory hours - use average of at least 14 values. HBPM requires two consecutive readings, at least one min apart, with the person seated. BP recorded at least twice a day (ideally morning and evening) for at least 4 days (optimally 7). Discard measurements from 1st day & average the rest NB/ hypertension is a major risk factor for stroke, MI, heart failure, chronic kidney disease, cognitive decline and premature death each 2mmHg rise in systolic BP is associated with a 7% inc in risk of mortality from IHD and 10% inc in risk of mortality from stroke

In a patient with breast carcinoma, clinically stage the disease as appropriate

TX: primary tumour cannot be assessed T0: no evidence of primary tumour Tis: carcinoma in situ T1: tumour ≤2 cm in greatest dimension T1mic: micro-invasion ≤0.1 cm in greatest dimension T1a: tumour >0.1 cm but not >0.5 cm in greatest dimension T1b: tumour >0.5 cm but not >1 cm in greatest dimension T1c: tumour >1 cm but not >2 cm in greatest dimension T2: tumour >2 cm but not >5 cm in greatest dimension T3: tumour >5 cm in greatest dimension T4: tumour of any size with direct extension to a) chest wall or b) skin, only as described below: T4a: extension to chest wall T4b: oedema (including peau d'orange) or ulceration of the breast skin, or satellite skin nodules confined to the same breast T4c: both (T4a and T4b) T4d: inflammatory carcinoma Regional lymph nodes - clinical classification (N) NX: regional lymph nodes cannot be assessed (e.g., previously removed) N0: no regional lymph node metastases N1: metastasis to movable ipsilateral axillary lymph nodes(s) N2: metastasis to ipsilateral axillary lymph node(s) fixed or matted, or in clinically apparent ipsilateral internal mammary nodes in the absence of evident axillary node metastases N2a: metastasis to ipsilateral axillary lymph node(s) fixed to one another (matted) or to other structures N2b: metastasis only in clinically apparent (as detected by imaging studies [excluding lymphoscintigraphy] or by clinical examination or grossly visible pathologically) ipsilateral internal mammary nodes in the absence of evident axillary node metastases N3: metastasis to ipsilateral infra-clavicular lymph node(s) with or without clinically evident axillary lymph nodes, or in clinically apparent ipsilateral internal mammary lymph node(s) and in the presence of clinically evident axillary lymph node metastases, or metastasis in ipsilateral supra-clavicular lymph nodes with or without axillary or internal mammary nodal involvement N3a: metastasis to ipsilateral infra-clavicular lymph node(s) N3b: metastasis to ipsilateral internal mammary lymph node(s) and clinically apparent axillary lymph nodes N3c: metastasis in ipsilateral supra-clavicular lymph nodes with or without axillary or internal mammary nodal involvement.

Describe TMN staging system

Tis - In situ carcinoma; tumor involves only muscularis mucosa T1 - Cancer has grown through muscularis mucosa and extends into submucosa T2 - Cancer has grown through submucosa and extends into muscularis propria T3 - Cancer has grown through muscularis propria and into outermost layers of colon but not through them; it has not reached any nearby organs or tissues T4a - Cancer has grown through serosa (visceral peritoneum) T4b - Cancer has grown through wall of colon and is attached to or invades nearby tissues or organs N0 - No cancer in nearby lymph nodes N1a - Cancer cells found in 1 nearby lymph node N1b - Cancer cells found in 2-3 nearby lymph nodes N1c - Small deposits of cancer cells found in areas of fat near lymph nodes, but not in lymph nodes themselves N2a - Cancer cells found in 4-6 nearby lymph nodes N2b - Cancer cells found in 7 or more nearby lymph nodes M0 - No distant spread seen M1a - Cancer has spread to 1 distant organ or set of distant lymph nodes M1b - Cancer has spread to more than 1 distant organ or set of distant lymph nodes, or has spread to distant parts of the peritoneum NB T/Nx = no description due to incomplete info

Define shock

acute circulatory failure resulting in inadequate or inappropriately distributed organ/tissue perfusion - low BP (S<90mmHg) or low MAP (<65mmHg) with evidence of tissue hypoperfusion (cell hypoxia or inability to use O2 correctly) eg/ mottled skin, urine output <0.5mL/kg for 1hr, serum lactate >2mmol/L

Discuss the principles underlying the use of prophylactic antibiotics in surgery

administration of abx before surgery to help prevent surgical site infections. The use of antibiotic prophylaxis is just one of many actions taken to help reduce the rate of surgical site infections

Define the acute abdomen

any pt who is acutely unwell and in whom symptoms and signs are chiefly related to abdomen - prompt laparotomy sometimes essential - repeated examination is key to making the decision pt hospitalised within a few hours of onset of pain

Describe the symptoms of patients with incisional hernia

asymptomatic, otherwise: 1) fever 2) infection 3) bulging (palpable lump and/or mass on surgical scar) + visible protrusion 4) pain 5) aching 6) swelling 7) foul-smelling drainage 8) redness and/or red streaks (sign of infection) 9) bowel obstruction (strangulation of intestines) 10) symptoms related to compromised organ (digestive disorders, jaundice due to protrusion of liver)

Describe and recognize the clinical triad of parkinsonism

bradykinesia + rigidity/tremor + postural instability 1) 1 core symptom + 1 other: bradykinesia + muscle rigidity or resting tremor or postural instability with no other cause 2) supportive features - a) unilateral onset & persistent asymmetry affecting side of onset most b) progressive clinical course of 10+ years c) excellent response to levodopa for ~5yrs d) anosmia e) incontinence, constipation f) depression, expressionless face, quiet dysarthric voice, micrographia g) dementia (advanced disease) 3) movement - a) gait problems/falls b) slow to initiate c) shuffling, festination - short steps, gets faster

Describe ECG features of left and right bundle branch

bundle branch block - QRS >120ms in broadest lead 1) normally - QRS -ve in leads V1-3, QRS +ve in leads V4-6 2) RBBB - final deflection of QRS is +ve in V1 a) V1 = M, V6 = W = MARROW b) can be normal variant, otherwise - PE, lung disease, pulmonary HTN, IHD 3) LBBB - final deflection of QRS is -ve in V1 a) V1 = W, V6 = M = WILLIAM b) STEMI, LV hypertrophy (anything that causes this), ischaemic or dilated cardiomyopathy, HTN NB/ can't comment on ST or T wave changes if BBB

Identify aspects of a particular service that could be assessed for audit purposes

can measure any area to find out how the present provision compares with the desired standard eg/ structure of care - eg, the availability of a smoking cessation clinic in a locality process of care - eg, waiting times for an appointment at the smoking cessation clinic outcome of care - eg, the number of smokers who quit smoking for one year

Outline the reasons for performing right and left heart catheterisation

cardiac catheterisation with a venous or arterial long-line catheter allows: injection of radio-opaque dye for angiography, measurement of intracardiac pressures and O2 sats, passage of electrophysiological instruments, passage of angioplasty and valvuloplasty balloons catheter is manipulated under fluoroscopic guidance & pt awake and on a cardiac monitor 1) left heart catheterisation - performed via the arterial route eg/ femoral artery, brachial artery, radial artery a) diagnostic uses: assessment of - I) LV function II) weverity of mitral and aortic valve disease III) outflow tract obstruction IV) extent & severity of coronary artery disease (coronary angiography is most common diagnostic study) V) LV biopsies may be taken (eg/ in cardiomyopathies) VI) electrophysiological provocation studies can be performed (eg/ for ventricular tachycardia) b) therapeutic interventions - I) percutaneous transluminal coronary angioplasty (PTCA) II) PTCA and stenting III) tx of ACS, acute MI IV) balloon valvuloplasty V) septal infarction by alcohol injection for hypertrophic obstructive cardiomyopathy (HOCM) 2) right heart catheterisation - performed by venous route, via femoral, internal jugular, subclavian or forearm veins a) diagnostic uses - I) measurement of cardiac output, LV filling pressure and pulmonary artery wedge pressure II) measurement of right heart oxygen sats (eg/ for septal defects) III) assessment of pulmonary hypertension (for example, prior to cardiac transplantation). IV) electrophysiological provocation studies b) therapeutic interventions - I) right-sided valvuloplasties II) radiofrequency ablation eg/ of accessory pathway in Wolff-Parkinson-White syndrome III) direct thrombolysis into pulmonary artery for massive pulmonary embolism IV) insertion of electrodes for cardiac pacemaker devices V) in critically ill pt may be used for acute monitoring of L & R ventricular function, to guide treatment and monitor effects of intervention eg/ shock, respiratory distress, complicated MI, monitoring effects of drugs (eg/ cardiac inotropes), assessing fluid requirements in pts

Council a patient with angina on the importance of reducing cardiac risk factors

certain risk factors inc risk of more fatty patches or plaques (atheroma) forming, which can make angina worse 1) smoking 2) high BP, BMI, cholesterol 3) inactivity - aim for moderate physical activity on most days of the week for at least 30 mins 4) diet - at least 5 portions of fruit & veg per day, dec fatty foods, 2-3 portions of oily fish per week, dec red meat, dec salt 5) alcohol - max of 14 units of alcohol per week, spread out through the week with at least two alcohol-free days a week

Stable angina

chest pain or discomfort that most often occurs with activity or emotional stress, which is relieved by rest. Angina is due to poor blood flow through the blood vessels in the heart 1) retrosternal chest discomfort (pressure, heaviness, squeezing, burning, or choking sensation) rather than frank pain 2) pain may be localized primarily in the epigastrium, back, neck, jaw, or shoulders 3) typical locations for radiation of pain - arms, shoulders, and neck 4) pain precipitated by exertion, eating, exposure to cold, or emotional stress, relieved by rest 5) lasts ~1-5 minutes & relieved by rest or nitroglycerin. Chest pain lasting only a few seconds is not usually angina pectoris 6) may provoke sweating, angor animi, and SOB 7) intensity of angina does not change with respiration, cough, or change in position 8) angina decubitus - variant of angina pectoris that occurs at night while the patient is recumbent

Classify intrahepatic and extrahepatic causes of obstructive jaundice and outline underlying pathology

cholestasis = dec in bile flow due to impaired secretion by hepatocytes or to obstruction of bile flow through intra-or extrahepatic bile ducts 1) intrahepatic - impairment of bile formation/secretion occurs: hepatitis, tumours, cirrhosis, drugs, alcohol, pregnancy, idiopathic, congenital 2) extrahepatic - impedance to bile flow occurs after it is formed: gall stones, carcinoma head of pancreas, biliary stricture, pancreatic pseudocyst, and sclerosing cholangitis NB/ prehepatic jaundice = saturation of bilirubin conjugation process. Bilirubin unconjugated and does not appear in the urine

Define fistula in ano

chronic abnormal communication between the epithelialised surface of the anal canal and (usually) the perianal skin - narrow tunnel with its internal opening in the anal canal and its external opening in the skin near the anus, can form when anal abscesses do not heal properly (usually with perianal or perirectal abscesses; rarely with intersphincteric or supralevator abscesses)

Common reasons why hypoglycaemia occurs

commonest cause = insulin or sulfonylurea treatment in a known diabetic eg/ with inc activity, missed meal, overdose, also alcohol non diabetics = explain 1) Ex..ogenous drugs eg/ insulin, oral hypoglycaemics, alcohol, aspirin poisoning, ACEI, B-blockers, pentamidine, insulin-like growth factor 2) P..ituitary insufficiency 3) L..iver failure (+ some rare inherited enzyme defects) 4) A..ddison's disease 5) I..slet cell tumours (insulinoma) + immune hypoglycaemia (eg/ anti-insulin receptor antibodies in Hodgkin's disease) 6) N..on-pancreatic neoplasms eg/ fibrosarcomas, haemangiopericytomas

Describe the features of diabetic sensorimotor neuropathy and associated risks

dec ability to move or sensation from nerve damage - nerve cells, axons, or myelin sheath 1) s/s - a) difficulty walking - loss of proprioception b) pain, burning, tingling, or abnormal feeling in any area of the body (neuralgia) c) loss of reflexes - first in toes, then extends upward d) glove-stocking distribution of numbness, sensory loss, dysesthesia and night time pain 2) risks - a) ulcers on feet & legs b) infections on feet & legs - can lead to amputation c) get multiple fractures of the knee, ankle or foot, and develop a Charcot joint d) loss of motor function results in dorsiflexion, contractures of the toes, loss of the interosseous muscle function that leads to contraction of the digits (hammer toes)

Summarise the clinical and biochemical features of Addison's disease

destruction of adrenal cortex = glucocorticoid & mineralocorticoid deficiency - signs are capricious (infection? anorexia nervosa?) - primary hypoadrenocorticalism = dec cortisol, aldosterone & sex hormones 1) hyperpigmentation (face, neck, palmar creases) 2) lean 3) tired + fatigued 4) dizzy 5) depression 6) nausea/vomiting 7) diarrhea/constipation 8) abdominal pain 9) postural hypotension (syncope as can't maintain BP) inc: adrenal antibodies, K+ dec: ACTH, aldosterone, glucose, Na+ 10) diagnosis - short ACTH stimulation test - normally tetracosactide (Synacthen) stimulates cortisol >600 at 30 minutes - won't occur with hypoadrenocorticolism NB/ secondary disease (changes in ACTH release) - aldosterone levels maintained as stimulated by angiotensin II rather than ACTH NB2/ adrenalitis often due to autoimmunity - 21-hydroxylase = autoantigen. Other causes of adrenal cortex destruction include TB, surgery, Ca and HIV/AIDS NB3/ Addisonian crisis may be precipitated by illness, trauma or surgery

Outline the complications of diverticulosis

diverticulitis which may lead to: 1) an abscess, which occurs when pus collects in pouch outside LI 2) blockage in colon or SI caused by scarring 3) fistula between sections of bowel or bowel + bladder or vagina 4) peritonitis - can occur if infected or inflamed pouch ruptures, spilling intestinal contents into abdominal cavity, medical emergency and requires immediate care 5) bleeding - usually painless & resolves 6) urinary problems - dysuria, frequency, air in urine

Discuss the potential adverse consequences associated with splenectomy and recommendations for preventing overwhelming post-splenectomy sepsis

don't need to spleen to survive, but it plays a crucial role in the body's ability to fight off bacteria, so pts more likely to develop infections, especially dangerous ones such as Streptococcus pneumoniae, Neisseria meningitidis, and Haemophilus influenzae. 1) need vaccinations to cover S. pneumonia, N. meningitidis and H. influenzae 2 weeks before planned surgery or 2 weeks after emergency surgery, may also have other vaccines 2) infections after spleen removal usually develop quickly = overwhelming post-splenectomy infections, cause death in 50% of cases, <5yo or splenectomy in last 2 years are most at risk 3) blood clot in hepatic portal vein 4) hernia at the incision site 5) infection at the incision site 6) pancreatitis 7) lung collapse 8) injury to the pancreas, stomach, and colon 9) preventing infections - kids may have daily abx, adults usually do not need this, unless they become sick or there is a chance they could become sick 10) need stash of abx if plan on traveling out of the country or to a place where medical help is not available

Describe the emergency treatment of a tension pneumothorax

emergency: 1) remove air - insert large-bore needle with syringe, partially filled with 0.9% saline into 2nd ICS in midclavicular line on side of suspected pneumothorax 2) remove plunger to allow trapped air to bubble through syringe until chest tube inserted 3) alternatively insert large-bore venflon in same location 4) alternatively, insert a cannula at the same location, and allow air to flow out

Define cirrhosis in pathological terms

end-stage of many hepatic diseases - may be defined as "a diffuse process characterised by fibrosis and conversion of the normal liver architecture into abnormal nodules surrounded by fibrotic scaffold (without portal triads)." stellate cells (store fat + fat soluble vitamins) found in Space of Dissé are activated & transformed into myofibroblast-like cells which begin to synthesise collagen initiating factor = continuing hepatocellular damage

Calculate sensible and insensible fluid and electrolyte losses in a febrile patient (Temp 40C)

for every +1oC, a patient will lose 10% more water (insensibly)

Describe the clinical triad reflective of normal pressure hydrocephalus

gradually progressive classic triad of: 1) gait disturbance - due to distortion of corona radiata by dilated ventricles (contains sacral motor fibres than innervate the legs) a) mvmt is slow, broad-based and shuffling (looks like Parkinson's, but rigidity and tremor are less marked and there is no response to carbidopa/levodopa) b) freezing episodes can also occur c) true ataxia and weakness are absent and the gait disturbance is referred to as gait apraxia 2) sphincter disturbance - also due to involvement of sacral nerve supply a) urinary incontinence predominant although bowel incontinence can occur 3) dementia - due to distortion of periventricular limbic system a) prominent features - memory loss, inattention, inertia and bradyphrenia (slowness of thought), dementia progresses less rapidly than Alzheimer's

Given a patient with shock, interpret the cardiac output, central venous pressure, left atrial (wedge) pressure, blood pressure pulse and urine output and, using these values determine the category of shock

haemorrhagic/hypovolaemic shock - 1) stage I: loss of <750mls blood (15% volume), HR <100, BP normal, RR normal, UO >30 mL/hr, slightly anxious 2) stage II: loss of 750-1500 mL (15-30%), HR 100-120, BP normal, RR 20-30, UO 20-30, mildly anxious 3) stage III: loss of 1500-2000 mL (30-40%), HR 120-140, BP dec, RR 3-40, UO 5-15, pt anxious/confused 4) stage IV: loss of >2000 (>40%), HR >140, BP dec RR >35, UO small, confused/lethargic NB/ recognise and initiate appropriate tx for: hypoxia, oliguria, hypotension

Outline the clinical presentation of acute and chronic hepatitis including relevant features in the medical history

hep A - a) risk factors eg/ living in an endemic area, close personal contact with an infected person, men who have sex with men, travel to an endemic region, and exposure to a known food-borne outbreak b) fever, malaise, N&V, fatigue c) jaundice & dark urine & pruritus d) hepatomegaly & RUQ pain e) clay-coloured stools f) fatigue & headache 1) hep B - a) risk factors eg/ perinatal exposure, multiple sexual partners, men who have sex with men, injection drug users, family history of HBV or hepatocellular carcinoma, household contact with an infected individual, and Asian or African ancestry b) normal physical examination c) others - jaundice, hepatomegaly & RUQ pain, ascites, fever/chills, malaise, maculopapular or urticarial rash etc 2) hep C - a) risk factors eg/ unsafe medical practices, IV or intranasal drug use, blood transfusion or organ transplant b) others - constitutional symptoms (usually asymptomatic, but may have fatigue, myalgia, or arthralgia), jaundice, ascites, hepatic encephalopathy (confusion, altered consciousness, or coma), extrahepatic manifestations (vasculitis, renal complications, skin manifestations such as porphyria cutanea tarda) 3) hep D - a) anorexia, N&V b) tiredness c) RUQ pain d) muscle and joint pain e) jaundice, dark urine and pale-coloured faeces 4) hep E - a) fever b) fatigue c) anorexia, N&V d) abdo pain e) jaundice, dark urine, clay-colored stool f) joint pain 5) alcoholic - a) if mild may not experience any symptoms. However, as more damage occurs, you may begin to experience: b) anorexia, N&V, weight loss c) dry mouth d) pain or swelling in the abdomen e) jaundice f) fever g) changes in your mental state, including confusion h) fatigue i) easy bleeding or bruising 6) autoimmune - a) may be asymptomatic, otherwise: b) hepatomegaly & RUQ pain c) spider angiomas d) abdominal distention e) dark urine & pale-colored stools & jaundice f) pruritus g) anorexia, N&V

Describe the management of asymptomatic diverticulae of the colon

herniation of mucosa and submucosa through the muscular layer of the colonic wall and may be the result of colonic smooth muscle overactivity, asymptomatic diverticulosis discovered incidentally requires no treatment. There is weak evidence to suggest that these patients might benefit from increasing dietary fibre, including fruit and vegetables, adequate water

Discuss a diagnosis of hypertension with a patient in lay terms

high BP (hypertension) happens when the force on the walls of blood vessels from the blood within them is more than normal - means heart has to work harder & blood vessels under more strain, so major risk factor for heart disease, stroke and other serious conditions

Describe the symptoms and signs, investigation and treatment of hypocalcaemia

hypocalcaemia = adusted calcium <2.2mmol/L 1) s/s - mnemonic "CATs go numb" - convulsions, arrhythmias, tetany, and numbness in the hands and feet and around the mouth a) petechiae & purpura (larger bruised areas, usually in dependent regions of the body) b) oral, perioral and acral paresthesias, tingling or 'pins and needles' sensation in and around the mouth and lips, and in the extremities of the hands and feet. Early symptom c) carpopedal and generalized tetany d) latent tetany - Trousseau sign of latent tetany (eliciting carpal spasm by inflating BP cuff and maintaining the cuff pressure above systolic), Chvostek's sign (tapping inferior portion of the cheekbone will produce facial spasms) e) tendon reflexes are hyperactive f) life-threatening complications - laryngospasm, cardiac arrhythmias g) heart - dec HR & dec contractility; ECG - intermittent QT prolongation, or intermittent prolongation of the QTc (corrected QT interval) - high risk of torsades de pointes 2) ix & mx - a) adjusted calcium - I) if <1.8mmol/L +/- symptoms, or <2mmol/L + symptoms need urgent referral to hospital for IV calcium gluconate or chloride (unless chronic disease) II) if 1.8-2.2 without symptoms check: PTH + Mg + U&E + vit D, to find cause b) PTH high = vit D deficiency - check vit D levels; tx - high dose vit D eg/ 100,000 units c) PTH low (& Mg normal) = hypoparathyroidism; tx - endocrinology referral as many causes eg/ autoimmune, infiltration, post-surgery, congenital d) Mg low = Mg deficiency; tx - oral replacement, unless <0.4mmol/L then IV

Describe the process of infection by the tubercle bacillus together with the route of spread

initial infection = asymptomatic, shown by tuberculin test immunological phenomenon eg/ erythema nodosum may accompany tuberculin 1) transmission - carried in airborne particles - droplet nuclei, of 1- 5 microns in diameter a) infectious droplet nuclei generated when persons who have pulmonary or laryngeal TB disease cough, sneeze, shout, or sing - tiny particles can remain suspended in air for several hours NB/ not spread by surface contact b) transmission occurs when person inhales droplet nuclei containing M. tuberculosis & droplet nuclei traverse mouth or nasal passages, URT, and bronchi to reach alveoli of lungs 2) pathogenesis - infection when person inhales droplet nuclei containing tubercle bacilli that reach alveoli of lungs a) tubercle bacilli ingested by alveolar macrophages - majority of bacilli are destroyed or inhibited, others cause local consolidation - forms Ghon focus, & transport TB to nodes (then known as primary complex) b) released when macrophages die c) bacilli spread in lymph or blood to more distant tissues & organs (including areas of body in which TB disease is most likely to develop: regional LN, apex of lung, kidneys, brain, bone), in bronchi = bronchpneumonia d) within 2-8 weeks macrophages ingest & surround tubercle bacilli - cells form a barrier shell = granuloma, that keeps bacilli contained & under control (LTBI) e) if immune system can't keep tubercle bacilli under control they multiply rapidly (TB disease) - can occur in different areas in the body, such as the lungs, kidneys, brain, or bone - 5% pts suffer clinical disease, most likely in 1yr 3) reactivation = inc cellular response associated with specific immunity a) cavitations & fibrosis marked in upper zones b) in most pts, lesions undergo fibrosis, calcification & heal c) secondary infection - cellular response = caseation d) distant foci may be obliterated once immune response has been mounted, but some latent organisms may persist

Describe the features of common peroneal nerve entrapment (foot drop)

irritation and entrapment of the common peroneal nerve crossing the fibular head can be due to scar tissue, trauma to knee, fracture of fibula, crossing legs regularly etc 1) dec sensation, numbness, or tingling in top of foot or outer part of the upper or lower leg 2) foot drop 3) "slapping" gait (walking pattern in which each step makes a slapping noise) 4) toes drag while walking 5) walking problems 6) weakness of the ankles or feet 7) loss of muscle mass as nerves aren't stimulating the muscles

Diabetic ketoacidosis (DKA) - outline the metabolic pathways that underlie it and understand the common reasons for its development

ketoacidosis - compensates for starvation 1) in the fasting state the body changes from metabolising carbs to fat oxidation 2) FFA produced in adipocytes & transported to liver bound to albumin, then with carnitine into mitochondria - metabolised to Acetyl CoA, which is polymerised to acetoacetate (ketone) - may produce Acetone + B-hydroxybutyrate (ketones) 3) ketoacids exported from liver to peripheral tissues (brain and muscle) where they can be oxidized, and blood = metabolic acidosis (results in K+ movement from cells into blood where may be lost in urine) NB/ acetone gives ketotic patients typical smell - described as 'fruity' 4) DKA = derangement of mechanism as despite vast amounts of circulating glucose it isn't used due to a lack of insulin - ketogenic pathways maximally "turned on", supply of ketones exceeds peripheral utilisation, and ketosis results 4) common reasons for development of DKA a) infection eg/ UTI b) surgery c) MI d) pancreatitis e) chemotherapy f) antipsychotics g) wrong insulin dose/non-compliance NB/ may develop in previously undiagnosed diabetes, interruption or improper insulin therapy, stress of concurrent illness

List the common somatic tumours which metastasise to the brain

lung, breast, melanoma (skin cancer), colon and kidney cancers commonly spread to the brain

List the potential complications of raised intracranial pressure (acute or chronic)

main complications are vision loss (due to optic nerve swelling/papilloedema) which can be quick or slow, and pain (headache)

Describe the variable clinical presentations of a patient with a Meckel's diverticulum

may have no s/s, otherwise - 1) 50% <2yo, otherwise <8 yo 2) haematochezia - acute, episodic, painless 3) intractable constipation 4) male 5) N+V 6) abdominal cramps

Outline the common methods for imaging the adrenal glands including ultrasound, CT, and isotope scanning and outline the role of surgery in adrenal disorders

modalities of choice in the evaluation of an adrenal mass are computed tomography (CT) scanning, magnetic resonance imaging (MRI), and positron emission tomography (PET) scanning. Ultrasonography has a role in the evaluation of a potential adrenal mass in infants, but no appearance is specific for benign adrenal adenoma. CT should be without contrast unless large mass depending on cause of disease, may need adrenal removed surgically - pts without adrenals or having them removed will require steroid cover to prevent Addisonian crisis

List the irreversible and reversible risk factors leading toward the development of ischemic stroke

modifiable: 1) smoking 2) hypertension 3) DM 4) heart disease (valvular, ischaemic, AF, carotid stenosis, endocarditis) 5) PVD 6) carotid bruit 7) compined pill 8) syphilis (including neurosyphilis) 9) drugs of abuse eg/ cocaine + alcohol 10) hypercholesterolemia + obesity 11) sleep apnoea 12) COX-2 inhibitors Non-modifiable: 1) male gender 2) inc age 3) family + personal history (stoke + TIA) 4) race 5) haemoglobinopathies 6) hyperviscosity/thrombophilia/thrombopenia 7) SLE + amyloidosis 8) migraine 9) hyperhomocysteinaemia

Name and briefly describe the monitoring techniques that help in the diagnosis and management of shock

monitor & manage: 1) O2 levels 2) BP + HR 3) pressure close to the heart - catheters in large veins in the neck, chest, arm, or groin 4) Foley catheter in bladder to measure urine output

Describe the morphology and pathological consequences of acute and chronic hepatitis

morphology: 1) gross appearance - liver may be enlarged and slightly reddened 2) histological appearance - hepatocytes respond to injury by: cytoplasmic vacuolation ("Ballooning Degeneration"), cellular rupture, or simply apoptosis. In severe cases, areas of hepatic tissue may undergo necrosis. The hepatic tissue displays striking inflammation in acute viral hepatitis which manifests with proliferation of kupffer cells as well as parenchymal infiltration with mononuclear cells, especially lymphocytes. Within this inflammatory war zone, areas of hepatocyte regeneration can be observed. Finally, in the case of HBV infection, the cytoplasm of hepatocytes may appear full of granules, giving them a characteristic "Ground-Glass Appearance", representing dense granules of HBV Surface Antigen (HBsAg) pathological consequences: 1) complications of acute viral hepatitis largely depend on identity of the infectious agent along with immune response of the host 2) chronic viral hepatitis - subsets of individuals may go on to develop chronic infections, hepA = never, hepB = occurs infrequently in adults but frequently in children and immunocompromised, hepC = occurs in majority of the cases 3) fulminant hepatitis - rare but catastrophic complication of acute viral infection and occurs when the immune response becomes over-exuberant - hepA = rare but possible, hepB = occurs in a small minority of cases but inc risk during concomitant hepD Virus, hepC = almost never occurs 4) type III hypersensitivity - especially prominent during HBV infection where infected hepatocytes can shed enormous amounts of HBsAg (Viral Surface Antigen). Shed HBsAg can form immune complexes that can then deposit in multiple tissues causing certain glomerular pathologies, skin rash, and arthralgia NB/ acute - In pre-icteric phase, pts have non-specific symptoms, with RUQ discomfort. Progression to liver failure, fulminant liver disease, is defined as development of hepatic encephalopathy within 8 weeks of symptoms or 2 weeks of jaundice. In this early phase, the liver is congested, hepatocytes swell and apoptose. In the later phase, the liver shrinks and may become pale due to immune cell infiltration. Hepatocytes show mitotic activity. Pathophysiology stems from canuliculi interruption, necrosis of hepatocytes and diminished production of proteins

List risk factors for the development of primary hepatocellular carcinoma

most common malignant tumour worldwide - in UK only accounts for 2% 3 types: solitary, multifocal, diffuse - cells grow in columns resembling normal liver tissue & associated with bile secretion, fat formation + glycogen storage 1) HBV infection 2) HCV infection 3) AIH 4) cirrhosis = 80% (alcohol, haemochromatosis) 5) non-alcoholic fatty liver 6) aflatoxin (produced by Aspergillus flavus on unprotected crop) 7) anabolic steroids 8) alpha-1 anti-trypsin deficiency 9) age + male gender

Describe the nature of primary diseases of muscle

muscular dystrophies are a group of hereditary disorders characterized by progressive muscular atrophy and weakness, in most cases muscles of the limb girdles (pelvic and shoulder muscles) are involved

Outline the clinical presentation and treatment of myxoma

myxoma is the most common cardiac tumour and constitutes 50% of all benign cardiac tumours in adults (15% in children), about 75% of primary cardiac tumours are benign, 75% are found in the LA 1) s/s - a) dyspnoea, fatigue, pallor b) syncope, dizziness c) CHF/pulmonary oedema d) embolic manifestations e) systolic or diastolic murmur f) weight loss g) fever h) arthralgia i) Raynaud's phenomenon j) loud first heart sound k) opening snap 2) mx - a) 1st line: myxoma resection (atriotomy) b) +/- valve repair/replacement and/or CABG c) postoperative aspirin d) +/- tx for dysrhythmia, embolisation, and/or heart failure eg/ B-blockers, ACEI, furosemide

Describe the basis on which antibiotics are chosen for varying infections

need to decide: type of organism, route, duration 1) Amoxicillin = chest infections, not if EBV 2) Co-amoxiclav (amoxicillin + clavulanic acid) = broad-spectrum against staph & some anaerobes 3) Benzyl penicillin (IV, pen G) = primary care meningitis (meningococcal & pneumococcal), strep 4) Penicillin V (phenoxymethylpenicillin) = strep, oral 5) Flucloxacillin = staph, strep, narrow spectrum 6) Tazocin = v. broad spectrum 7) Metronidazole = anaerobes, surgery, H. pylori association, C. diff tx, can't drink alcohol 8) Cephalosporins = cross-sensitivity with penicillin, associated with C. difficile so avoid, different generations cover slightly different things, all broad spectrum 9) Doxycycline = atypicals + chest infections + STIs, can cause photosensitivity (sunscreen), not if pregnant or breast-feeding or child (tetracycline), don't give at same time as supplements 10) Nitrofurantoin = UTIs, doesn't work if dec eGFR 11) Trimethoprim = UTIs, never use with methotrexate, , not for pregnant women in 1st trimester 12) Vancomycin = gram +ve infections including MRSA, narrow therapeutic window so need levels 2-3x/week, need to give slowly to prevent red man syndrome, normally IV, only give PO for C. difficile 13) Rifampicin = TB, tears & sweat & urine stained orange/red, P450 inducer so many interactions, can be used as an adjunct for staph (never by itself as resistance likely) 14) Ciprofloxacin = quinolone, not 1st line as risk of C. difficile, broad spectrum - atypicals + gram -ves, other quinolones broader spectrum, not in kids, can have tendon rupture (especially with steroids), can cause anxiety, depression, dec threshold for seizures, active against pseudomonas 15) Clarithromycin = atypicals + chest infections (especially if penicillin allergy), P450 inhibitor (statins need to be stopped, monitor warfarin closely etc), H. pylori eradication

Screening prior to surgery

need to optimise condition, screen for other pathology, assess respiratory status, check for organ dysfunction due to iron overload, CV status, need basic bloods - ensure Hb in normal range (or a little below), otherwise may need transfusion pre-operatively

Describe the functions of a dressing and demonstrate them on actual wounds

no single dressing is suitable for all types of wounds - often several types of dressings will be used during the healing process of a single wound. Dressings should perform one or more of the following functions: 1) assurance of cleanliness - provide bacterial protection 2) environmental control - keep a moist environment at the wound/dressing interface 3) reduction of oedema 4) elimination of space 5) tissue immobilisation 6) minimisation of scar formation 7) enhancement of epithelialisation - absorb excess exudate without leakage to the surface of the dressing, provide thermal insulation and mechanical protection, allow gaseous and fluid exchange

Explain the Bamford classification of stroke, describing the prognostic difference between each stroke type

oxford (Bamford) classification of stroke - diagnosis on stroke territory: 1) Total Anterior Circulation Stroke (TACS): 20%, all of: a) higher dysfunction (eg/ dysphasia, LOC, visuospatial neglect, sterognosis or apraxia) + b) homonymous hemianopia + c) motor/sensory deficit (>2/3 of face/arm/leg) 2) Partial Anterior Circulation Stroke (PACS): 35%, a) either 2/3 TACS (higher dysfunction, homonymous hemianopia, motor/sensory deficit - >2/3 of face/arm/leg) b) or higher dysfunction alone 3) Lacunar Stroke (LACS): 20%, small & deep vessels around BG, internal capsule, thalamus and pons, any of: a) pure motor (>2/3 of face/arm/leg) b) pure sensory (>2/3 of face/arm/leg) c) sensory-motor (>2/3 of face/arm/leg) d) ataxic hemiparesis with NO: new dysphasia, new visuospatial problem, proprioceptive sensory loss only, no vertebrobasilar features 4) Posterior Circulation Stroke (POCS): 25%, any of a) cranial nerve palsy + contralateral motor/sensory deficit b) bilateral motor or sensory deficit c) conjugate eye movement problems d) cerebellar dysfunction e) isolated homonymous hemianopia 5) coding last letter: (S) — Syndrome: (prior to imaging (eg/ TACS)), (I) — Infarct (eg/ TACI), (H) — Haemorrhage (eg/ TACH) 6) TACS = worst outcome, LACS & POCS best eg/ 1yr death rate of 60% compared to 10% % 20% respectively

Define anal fissure

painful tear (mostly 6 o'clock) in sensitive, skin-lined (squamous) lower anal canal distal to dentate line - often, if chronic with a 'sentinel pile' or mucosal tag at external aspect - 90% posterior - hard stools most common cause, anterior = childbirth causes pain on defecation (distal to dentate line) bleeding may occur alongside the pain Crohn's can produce anterolateral fissures

Define pseudocyst and discuss mechanisms of their formation

pancreatic pseudocysts are best defined as localized fluid collections that are rich in amylase and other pancreatic enzymes, that have a nonepithelialized wall consisting of fibrous and granulation tissue, and that usually appear several weeks after the onset of pancreatitis. They are to be distinguished from acute fluid collections, organized necrosis, and abscesses pathogenesis of pseudocysts seems to stem from disruptions of the pancreatic duct due to pancreatitis and extravasation of the enzymatic material. Cause of pseudocysts parallels the cause of acute pancreatitis; 75-85% of cases are caused by alcohol or gallstone disease-related pancreatitis. In children, pseudocysts and trauma are frequently associated.

Describe the typical autonomic and neuroglycopaenic symptoms of hypoglycaemia

plasma glucose <3mmol/L 1) autonomic: sweating, anxiety, hunger, tremor, palpitations, dizziness 2) neuroglycopenic - confusion, drowsiness, visual troubles, seizures, coma 3) most likely before meals & at night, with irregular eating, unusual exertion & alcohol excess 4) physical signs: pallor & cold sweat NB/ pts with long-standing diabetes may lose warning signs - greater risk of severe hypoglycaemia. Pale, drowsy or detached. Behaviour may be clumsy and inappropriate, pts may become aggressive. Others slip rapidly into coma, with or without convulsions

Describe the pathology of acute lobar pneumonia and bronchopneumonia

pneumonia = acute exudative inflammation secondary to airborne infection with bacteria, viruses or mycoplasma 1) acute lobar pneumonia - entire lobe is affected, if not treated evolves in 4 stages - common to all stages = enlargement of affected lobe with loss of spongy appearance a) stage 1 - congestion (day 1-2), affected lung parenchyma partially consolidated, red-purple, partially aerated microscopy: alveolar lumen contains serous exudate, bacteria + rare leucocytes b) stage 2 - red hepatization (day3-4), pulmonary lobe is consolidated, red-brown, dry, firm, with liver-like consistency, cut surface is dry, rough microscopy: characteristic aspect = accumulation in alveolar spaces of exudate rich in fibrin (mainly), with bacteria, leucocytes & erythrocytes. Alveolar walls thickened due to capillary congestion & oedema c) stage 3 - gray hepatization (day 5-7), affected lobe has liver-like consistency, with uniform gray colour, on cut surface, a grayish purulent liquid drains because alveolar lumens are filled with leukocytic (suppurative) exudate (neutrophils + macrophages to remove fibrin). Capillary congestion & oedema still present so alveolar walls are thick d) resolution stage - starts day 8 for 3 weeks (uncomplicated cases), while exudate within alveolar spaces drained through lymphatics & airways ("productive" cough) with gradually aeration of the affected segment 2) bronchopneumonia - affects bronchioles & adjacent alveoli, characterized by foci of consolidation surrounded by normal parenchyma a) affects 1+ lobes, frequently bilateral & basal b) macroscopically - can identify multiple foci of condensation (1-3 cm diameter), white-yellowish, imprecisely circumscribed, centered by bronchiole, separated by normal lung parenchyma c) microscopy : foci of inflammatory condensation centered by bronchiole with acute bronchiolitis (suppurative exudate rich in neutrophils in lumen, foci of ulceration of epithelium & parietal inflammation) d) alveolar lumens surrounding bronchia filled with neutrophils ("leukocytic alveolitis") e) capillaries in alveolar walls show congestion f) inflammatory foci separated by normal, aerated parenchyma

Classify the causes of hypothyroidism and outline the pathological features of Hashimoto's thyroiditis

primary disease: (most common, prevalence in UK of 1% in women + 0.1% in men) 1) autoimmune thyroid disease: a) primary atrophic hypothyroidism - common, associated with auto-antibodies leading to lymphoid infiltration & fibrosis. Found alongside autoimmune conditions eg/ pernicious anaemia, vitiligo. Leads to atrophy (hence no goitre) b) Hashimoto's thyroiditis - goitre due to lymphocytic & plasma cell infiltration, common in women 60-70, may be hypothyroid or euthyroid, autoantibody titres very high (antithyroid peroxidase + antithyroglobulin antibodies). Transient hyperthyroidism as inflammation damages thyroid acini = release, en masse, of stored thyroid hormone. After this period = euthyroidism, then hypothyroidism 2) infective eg/ TB, Waterhouse Friedrichson syndrome 3) iodine deficiency 4) congenital secondary disease: 1) hypopituitarism 2) peripheral resistance to thyroid hormone 3) drug induced eg/ amiodarone, carbimazole others: 1) post thyroidectomy - only in pts who have previously experienced euthyroidism. Hypothyroidism in ~10% after a year. Annual screening may be advantageous 2) post-radioactive iodine treatment - occurs in majority of RAI pts over 20 years post-treatment. 75% rendered euthyroid in short term. Long term monitoring necessary. NB/ chief cause = iodine deficiency

Define evidence based clinical practice and the actions involved in its execution

process of systematically reviewing, appraising & using clinical research findings to aid the delivery of optimum clinical care to patients importance: alternative to basing our decisions on best available evidence is basing them on something else - arguably anything else is more likely to be wrong EBM has been a powerful movement for over 20 years and will be important in careers 1) formulate a clear clinical question from a patient's problem 2) search literature for relevant clinical articles 3) evaluate (critically appraise) evidence for validity & usefulness 4) implement useful findings in clinical practice

Outline the main groups of disease leading to pulmonary fibrosis & clinical restrictive lung disease & relate this to occupational history

pulmonary fibrosis 1) chronic inflammatory processes (sarcoidosis, Wegener's granulomatosis) 2) infections (viral) 3) environmental agents (asbestos, silica, exposure to certain gases) - occupation 4) exposure to ionizing radiation (such as radiation therapy to treat tumors of the chest) 5) chronic conditions (lupus, rheumatoid arthritis) 6) certain medications 7) hypersensitivity pneumonitis - fibrosis of lung after heightened immune reaction to inhaled organic dusts or occupational chemicals (inhaling dust contaminated with bacterial, fungal, or animal products) - occupation 8) idiopathic 9) systemic sclerosis, TB, berylliosis clinical restrictive lung disease 1) interstitial lung disease eg/ idiopathic pulmonary fibrosis 2) sarcoidosis 3) obesity, including obesity hypoventilation syndrome 4) scoliosis 5) neuromuscular disease eg/ muscular dystrophy or amyotrophic lateral sclerosis (ALS) 6) intrinsic lung diseases 7) collagen-vascular diseases eg/ scleroderma, polymyositis, dermatomyositis, SLE 8) some drugs eg/ nitrofurantoin, amiodarone, phenytoin 9) inorganic dust exposure eg/ silicosis, asbestosis, talc, pneumoconiosis, berylliosis, hard metal fibrosis, coal worker's pneumoconiosis - occupation 10) organic dust exposure eg/ farmer's lung, bird fancier's lung, bagassosis, and mushroom worker's lung (all cause hypersensitivity pneumonitis) - occupation NB/ physical signs of localised fibrosis = dec chest wall mvmt on affected side, mediastinal displacement towards lesion, dull percussion, bronchial breath sounds, inc vocal resonance, coarse crackles NB2/ generalised fibrosis = dec global expansion, inc vocal resonance & fine crackles, localised fibrosis = streaky shadowing on CXR, whereas generalised fibrosis gives a honeycomb lung picture

Discuss the possible clinical presentation of a phaeochromocytoma and be aware of the syndromes of which it is a component

rare catecholamine-producing tumour, usually in adrenal medulla, signs are episodic and vague classic triad = episodic headache, sweating, tachycardia (+inc WCC) suspect if BP hard to control, accelerating, or episodic 1) heart - inc pulse, palpitations, faints, MI/LVF, HTN but orthostatic hypotension 2) CNS - visual disorder (hypertensive retinopathy), tremor, fits 3) psychological - anxiety, panic, episodic psychosis 4) gut - D&V, abdominal pain over tumour, mesenteric vasoconstriction 5) others - sweat/flushes, heat intolerance, pallor, impaired glucose tolerance/diabetes mellitus 6) syndromes - a) multiple endocrine neoplasia (MEN) syndrome type 2A and B - lifetime risk is 50% b) Von Hippel-Lindau (VHL) disease - lifetime risk is 10% to 20% c) succinate dehydrogenase (SDH) d) neurofibromatosis type 1 (NF1) - 1% chance

Summarise the clinical and biochemical features of Conn's syndrome

solitary aldosterone-producing adenoma - excess aldosterone, indepenant of RAAS = inc sodium and water retention & dec renin release 1) s/s - consider if following present & not on diuretics a) hypertension b) weakness, cramps, paraesthesia c) polyuria, polydipsia, nocturia d) age 20 to 70 years e) lethargy f) mood disturbance (irritability, anxiety, depression) g) difficulty concentrating 2) biochemical - alkalosis + hypokalaemia

Name the bedside respiratory test of most use in monitoring neuromuscular ventilatory function

spirometry - neuromuscular disorders typically result in a restrictive lung defect on simple spirometry (FEV1/FVC ratio > 0.7). A decline in vital capacity >15%-20% in the supine position indicates diaphragmatic weakness. Spirometry can be performed at the bedside using simple handheld spirometers in acutely ill patients and in the lung function laboratory in stable patients. Serial bedside spirometry measurement is particularly useful in the acute setting for patients with rapidly progressing neurology, such as Guillain- Barré syndrome, to monitor for deterioration. A value of less than 15 mL/kg should prompt urgent intensive care referral for endotracheal intubation in anticipation of potential respiratory arrest

List the types of surgical infections

surgical site infections (SSIs) occur in 1-3% pts , usually within 30/7 of surgery 1) superficial incisional SSI - infection occurs just in the area of the skin where the incision was made 2) deep incisional SSI - infection beneath the incision area in muscle and the tissues surrounding the muscles 3) organ or space SSI - infection in any area of the body other than skin, muscle, and surrounding tissue that was involved in the surgery eg/ organ or a space between organs

Provide a definition of dementia

syndrome caused by brain disorders that lead to memory loss, dec in some aspects of cognition, and difficulties with ADL

Describe the main components of the health needs assessment process

systematic process of: identifying unmet health & healthcare need, distribution in population, making changes to meet those needs helps address health inequities & inequalities, provides practical information for change eg/ what's, for how many people, expected benefits & at what cost? provides opportunity for community/users to "have a say" 1) identify health problem, population affected & stakeholders 2) construct a picture for size & nature of problem by reviewing existing info (as complete and as accurate) 3) identify & profile existing services (quantity/quality, effectiveness/efficiency) 4) make list of interventions recommended by scientific literature 5) obtain views of users & providers 6) analysis & interpretation of results - identify resource implications 7) develop realistic, evidence based & well-costed plan for change in your population 8) test your plan, monitor and evaluate if it has lead to appropriate change


Ensembles d'études connexes

Number the Stars Chapters 1 and 2

View Set

ABEKA: American Government Appendix Quiz H

View Set

POLS EXAM 2 Chapter 7 & 10, GSU POLS 1101 CH 13 Study Guide, POLS 1101: American Government Chapter 12 study guide, POLS Exam 2, Ch. 11 Study Guide

View Set

Psychology Chapter 1-4 Study Guide

View Set

Introduction to Probability and Statistics

View Set